Sei sulla pagina 1di 140

Psiquiatra

Con la colaboracin de

Juan Vega Villar


Especialista en Neurologa por el Hospital Universitario Central de Asturias.
Residente de Radiodiagnstico en el Hospital Universitario de Getafe.

Jon Garca Ormaza


Mdico Psiquiatra. Hospital de Cruces. Osakidetza.
Departamento de Neurociencias de la Universidad del Pas Vasco.
Reservados todos los derechos. Ninguna parte de esta publicacin puede ser reproducida,
transmitida en forma o medio alguno, electrnico o mecnico, incluyendo las fotocopias,
grabaciones o cualquier sistema de recuperacin de almacenaje de informacin sin el
permiso escrito del titular del copyright.

Este libro es de entrega gratuita entre los alumnos del Curso MIR Asturias en su
modalidad Presencial. Su utilizacin es imprescindible para el seguimiento del Curso.

2015. CURSO INTENSIVO MIR ASTURIAS, SL


l.S.B.N.: 978-84-606-6128-3
Depsito Legal: AS ll 1620l5
Imprime: I. Gofer
Mtodo de estudio recomendado
Importancia en el MIR de Psiquiatra

o En cada examen MIR, se pueden encontrar unas 9-10 preguntas directas de esta especialidad, no obstante, con conocimientos de
esta materia, pueden contestarse otras 4 5 preguntas mas, con lo que el nmero total es de unas 15 preguntas anuales.
I La psiquiatra no es una especialidad difcil, necesita ser comprendida. Slo conociendo los conceptos fundamentales de cada
trastorno y los conceptos repeMIR se puede contestar correctamente mas de la mitad de las preguntas.

Tiempo que debes dedicar

o EI tiempo gue debes dedicar a esta asignatura es de 3 das en la primera vuelta y 2 en la segunda. Te recomiendo aproximadamente
12 horas diarias: 3 horas de clase, 7 horas de estudio y 2 horas de test.
o Si no te da tiempo (es normal, hay temas infernales como farmacologa que acaban con la moral de cualquiera) te recomiendo
restrselo al estudio y completar los test y venir a clase. En las modalidades no presenciales te recomiendo ver Ia parte
correspondiente de Ia clase de repaso de Ia tercera vuelta del MIR 2014 (es una clase de casi 3 horas con toda la teora condensada,
as que si ests en primera vuelta sera ver unos 45 min de clase diaria), y complementarlo con preguntas MIR de Psiquiatra por
internet para poder leer el comentario.

Organizacin del estudio

o Lo primero que debes hacer, es mirar Ia importancia de cada captulo, te dars cuenta, de que hay temas que son preguntados todos
los aos.
I El obietivo del estudio es que ENTIENDAS la materia. La psiquiatra puede ser comprendida y recordada sin grandes dificultades. Se
trata de leer cada tema detenidamente, afortunadamente pocas cosas son de memorizar (los ISRS hay que rezarlos cada maana, es
probablemente Ia lista mas rentable del MIR).
o La distribucin recomendada para Ia teora, es Ia siguiente:

Primera vuelta Segunda vuelta

Da Materia Da Materia
I Temas'l,2,3y4 I Temasl,2,3,4,5,y7
Temas 5, 6,7y8 2 Temas 8, 9, IOy'IT
3 Temas9,'IOy'l'l
Psiquiatra

1. Importancia de la Psiquiatra en el MIR

1.1,? Evolucin en los ltimos 35 aos (41 examenes)

16
15
A
14 14 14
13
12
11 11 11 11 11 11
10 10 10 10 10 1010101010 101010 10
9 9 9 9 9

80 81 82 83 84 85 86 87 88 89 90 91 92 93 94 95f 95 96f 96 97f 97 98f 98 99f 99 Of 00.01. 02. 03. 04. 05. 06. 07. 08 09.10.11.12 13 14

2. Nmero de preguntas de MIR en los distintos captulos

A. En Ia historia del MIR (desde 1980)

Introduccin 19

Psicofa rm acolog a 46

Trastornos mentales
33
org nicos

Psiquiatra infantil 7

Esquizofrenia y Tr. por


71
ideas delirantes
Trastornos de la
14
personalidad
Trastornos no orgnicos del
sueo

Toxicoma nas y alcoholismo 61

Tra stornos de a ns ieda d,


. . . 51
som atomorfos y CIISOCIO1IVOS
Trastornos de la conducta
14
alimentaria
V.i.P - e
MlR VIlR

CLASIFICACIN POR IMPORTANCIA


l IMPORTANCIA Todas las asignaturas
i
Clasicacion de todas las asignaturas por el nmero de preguntas MIR incluidas (directas y relacionadas)
0 10 20 30 MJ 50 50 70 80 90 100

Neumologa 100
lnfeca'usas 87
Digestivo 82
Farmacologia 80
Medidmpzeventiva w _ _ 76
Nefrologia w 74

Pediatria m 77.
Endocrino 57
Ginecnltgiayohs. w 56

Cardiologa 52

Neumlugia M 50

Onmloga M 50
Reumatologa 44

Hemainlogia m 43

Tramamlogia w 34

Psiquiatria _ 32

marino M 24

Demahdoga _ 22

Inmuno m 21
Oftalmolagla w 18

APatdgca w 17
Otras... _ 12

Genetica _ 11
Gtian 11

CVascular _ 7
Paliativos _ 7

Imestesia _ 3

Geriatra _ 3

Amtomia l 1

Fisiologla I 1

Importancia

0 20 40 60 80 100 120

09.Trastornos afectivos 100

10.Trastornos de ansiedad, somatomorfos y disociativos

03.Trastornos mentales organicos

05. Esquizofrenia. T. Esquizotipico y T. de ideas delirantes

02. Psicofarmacologia

08.Trastornos mentales debido al consumo de sustancias.

06.Trastornos de la personalidad

01.]ntroduccion

11.Trastornos de la conducta alimentaria

04. Psiquiatria infantil

O7.Trastornos no organicos del sueo


'
expriMIR
*9
CLASIFICACIN POR RENTABILIDAD
RENTABILIDAD Todas las asignaturas

Clasificacion de asignaturassegn Ia rentabilidad del estudio de cada pgina de su libro


0 10 20 30 40 50 60 70 80 90 100

Onoologl a 100
4
Fa rma 98
Derma tologa 85
Nefrologia 64
Gestion w 51
Pediatra _ 60
Genetica m 56
Hematologa 55
Medicina preventiva 54
lnfeCCOSBS 52
Geriatra m 50
Reumatologa h 50
Neumologia 50
Digestivo h 46
Cardiologa 44
Psiquatria 44
Neurologa 44

Endocrino _ 43
A.Patolgim 42
CVascuIar _ 41
Otorrino 37
Inmuno 32
Glnecologiayobs. 32

Traumatologa _ 32
Oftalmologa _ 26
hlatvos 25

Anestesia 19

Psiquiatra

05. Esq ulzofrena. T. Esq uizotipico y T. d e ideas delirantes 00

03. Trasto rnos mental eso rganoos 95

06. Trastornos d e la persoralidad 82

09. Trastornos afectivos 70

10. Trastornos de ansiedad, somatomorfos y d isociativos G2

04. Psiq uiatria infantil 57

01. Introduccion 49

11. Trastornos dela conducta dimentaria 48

02. Psicofarmaoologia 45

08. Trasto rnos mentales d ebido al consumo de sustancias


23
psicotmpioas

O7. Trastornos no o rganicos delsueo 11


NDICE
Mtodo de estudio ..................................................................................................................................................... 3
Importancia en el MIR ................................................................................................................................................ 4
Evolucin en los ltimos 35 aos ................................................................................................................................................. 4
Nmero de preguntas MIR en los distintos captulos ............................................................................................................................. 4
Clasificacin por importancia ............................................................................................................................................................. 5
Clasificacin por rentabilidad ............................................................................................................................................................. 6
ndice ................................................................. . ...................................................................................................... 7

Capitulo l: Introduccin .............................................................................................................................................. 9


i. Clasificacin de los trastornos mentales y del comportamiento ....................................................................................................... lO
2. Psicosis vs. Neurosis ..................................................................................................................................................................... lO
3. Epidemiologa .............................................................................................................................................................................. lO
4. Semiologa ................................................................................................................................................................................... 10
5. Psicoanlisis .................................................................. . .............................................................................................................. 12
6. Miscelnea .................................................................................................................................................................................. 12
7. Normas generales ........................................................................................................................................................................ 13

Captulo Il: Psicofarmacologa ...................... . ...... . .............................. ............... ........ . ...... . ....... ........... 14
1. Ansiolticos ...... , ................................ . ...................... . ........... . ...... .. .............................................................................. t ................. 15
2. Antipsicticos (APS) ...................... . ................................................. 16
3. Antidepresivos .............................................................................................................................................................................. 21
4. Litio .................. . .................................................................... . .................... . ......................................... . ................. , .................... 24
5. Frmacos antidemencia ............................................................................................................................................................... 27
6. Terapia electroconvulsiva ............ . ...................................................................................................... t .......................................... 27
Captulo lll: Trastornos mentales orgnicos ..... . ....... .. .............................. . ...... . .......... ........... .......... ..... 33
'I, Delirium ............................. . ..... . .......................................................... . ....................................................................................... 34
2. Sndrome amnsico orgnico ........................................................................................................................................................ 35
3. Demencias .............................................................................................................................................................. 36
4. Enfermedad de alzheimer ...................................................................................................................... 37
5. Otros tipos de demencias .................................. . ............................................................................. 38
6. Demencias trasmisibles .......................................................................................... .. .................... 40
7. Pseudodemencia ............ ...................................................................................................... . .................................................... 40

Captulo IV: Psiquiatria infantil ............ . ...... . .................... . ........ . ........ . ........ . ...... .......................... . ........... . ........... 44
1. Retraso mental ....................................................................... 44
2. Trastornos generalizados del desarrollo .......................................................... 45
3. Trastorno por deficit de atencion y/o heperactividad (TDAH) .................................. t ....................................................................... 46
4. Trastornos de la eliminacin ......................................................................................................................................................... 47

Captulo V: Esquizofrenia. Tr. Esquizotipico y Tr. por ideas delirantes ........................... . ..................................... . ......... 48
1. Esquizofrenia ................................................................................................................................................................................ 49
2. Trastorno por ideas delirantes ............................ 52
3. Trastorno esquizoafectivo ............................................................................................................................................................. 54
4. Otros trastornos psicticos ............................................................................................................................................................. 54

Captulo VI: Trastornos de la personalidad ................................................................................................................. 57


l. Introduccin .................................................................................................................................................................................. 58
2. Trastornos de la personalidad tipo A.... ...... 58
3. Trastornos de la personalidad tipo B ............................................................................................................................................. 59
4. Trastornos de la personalidad tipo C ............................................................................................................................................. 61
5. Otros trastornos de la personalidad no especificados .................................................................................................................... l

Captulo Vll: Trastornos no organicos del sueo ......................................................................................................... 65


l. Sueo normal .......................................... . ................................................................................................................................... 66
2. Disomnias .................................................................................................................................................................................... 67

Captulo VIII: Trastornos mentales debido a sustancias psicotropas .............................................................................. 70


i. Generalidades ............................................................................................................................................................................. 71
2. Opiceos ...................................................................................... 72
3. Cocana ................................................................................................................................................... 74
4. Otros estimulantes .................................................................................................................................... 76
5. Cannabis marihuana ..................................................................................................... 76
. Resumen drogas .......................................................................................................................................................................... 77

7
7. Trastornos psiquitricos inducidos por sustancias ........................................................................................................................... 77
8. Deteccion de drogas en orina ....................................................................................................................................................... 77
9. Alcohol ........................................................................................................................................................................................ 77

Captulo IX: Trastornos afectivos .......................................... .. .............. ................................................................. 91


1. Clasificacin de los trastornos del humor ...................................................................................................................................... 92
2. Epidemiologa .............................................................................................................................................................................. 92
3. Trastornos depresivos ................................................................................................................................................................... 92
4. Distimia ....................................................................................................................................................................................... 97
5. Depresin doble ........................................................................................................................................................................... 97
6. Diferencia depresin endgena/distimia ....................................................................................................................................... 98
7. Trastornos depresivos puerperales ................................................................................................................................................ 98
8. Episodio maniaco ......................................................................................................................................................................... 98
9. Episodio hipomaniaco .................................................................................................................................................................. 99
10, Episodio mixto ....................................... . .................................................................................................................................... 99
1 1. Trastorno bipolar ....................................................................................................................................................................... 99
12. Suicidio .................................................................................................................................................................................... 102

Captulo X: Trastornos de ansiedad, somatomorfos y disocatvos .............. ...................... . ..................................... 109


l. Concepto y epidemiologa .......................................................................................................................................................... 110
2. Trastornos de ansiedad fbica .................................................................................................................................................... 1 10
3. Ataques de pnico (crisis de angustia) ......................................................................................................................................... 'l 12
4. Trastorno de pnico (trastorno de angustia) ................................................................................................................................ l 13
5. Trastorno de ansiedad generalizada ........................................................................................................................................... 113
. Trastorno obsesivo-compulsivo (TOC) ......................................................................................................................................... l 13
7. Reacciones a estrs grave y trastornos de adaptacin .................................................................................................................. l 15
8. Trastorno de ansiedad secundario .............................................................................................................................................. l 17
9. Trastornos somatomortos y trastornos disocativos ....................................................................................................................... l l 7

Captulo XI: Trastornos de Ia conducta alimentaria ................................................................................................. 125


1. Trastornos cualitativos ................................................................................................................................................................ 125
2. Trastornos cuantitativos ......................................................................................................................... 'I 25
3. Potomania ................................................................................................................................................................................. 129

Repaso Relacional .................................................................................................................................................. 132


1. Epidemiologa ............................................................................................................................................................................ 132
2. Etiopatogenia ............................................................................................................................................................................. 133
3. Fisiopatologa ....................................... ....134
4. Anatoma patolgica .................................................................................................................................................................. 134
5. Clnica ....................................................................................................................................................................................... 134
. Mtodos complementarios de diagnstico .......... l 35
7. Tratamiento ...................................................... "l 35

Indice Temtico ...................................................................................................................................................... 'l37


b
CURSO INTENSIVO MlRASTURlAS

Introduccin
Nmero de preguntas del captulo en el MIR

l ill

ll
90 91 92 93 94 95f 95 96f 96 97f 97 98f 98 99f 99 OOf 00. Oi. 02. 03. 04. 05. 06. 07. 08. 09. 10 il 12 13 14

Nmero de preguntas de cada. terna

Trastornos mentales y del


comportamiento

Epidemiologa

Semiologa

Mecanismos de defensa

Miscelanea

lmprescindible
o EI tema de introduccin es un tema intermedio tanto en nmero de preguntas como en rentabilidad.
o EI concepto mas rentable es conocer las patologas que pueden presentar semiologa psictica y aquellas que pertenecen al espec-
tro de las neurosis donde predomina la ansiedad (3MIR):
o En las neurosis (trastornos caracterizados por ansiedad) el uicio de la realidad est conservado, no hay ideas delirantes ni
alucinaciones. Patologas neurticas: crisis de angustia, trastorno por angustia, agorafobia, trastorno de ansiedad generaliza-
da, trastorno de adaptacin, trastorno de ansiedad fbica, trastorno obsesivo-compulsivo (TOC), trastorno por estrs pos-
traumtico, trastornos somatomorfos y trastornos disociativos.
o Las psicosis cursan con alteracin del iuicio de realidad, y suelen presentar delirios y alucinaciones. Pueden presentar sntomas
psicticos: esquizofrenia, trastorno esquizotpico, tras-torno por ideas- delirantes, trastorno esquizoafectivo, trastornos psicticos
agudos y transitorios. Ademas pueden presentar este tipo de sntomas trastornos afectivos (depresiones mayores con sntomas
psicticos , mana con sntomas psicticos), trastornos organicos o txicos.
c El grupo de enfermedades ms prevalente en nuestro medioes el de trastornos de ansiedad. El trastorno mas prevalente el trastor-
no depresivo mayor. La patologa psiquitrica ms prevalente en muieres es la ansiedad (4-8%) y la depresin (53-10%), mientras
que en hombres es el alcoholismo y la depresin (3-6%) (QMIR). La prevalencia de la esquizofrenia y del trastorno bipolar est en
torno al 1%.
o Es muy rentable conocer la diferencia entre una idea delirante y una idea obsesiva (psicosis-paranoia) (2MIR):
o Una idea delirante es una creencia errnea, pobremente estructurada, irreductible. Invade la personalidad.
o Una idea obsesiva es una idea intrusa y repetitiva, que el individuo reconoce como producto de su mente (no es delirante).
Causan malestar o ansiedad y se intentan neutralizar con pensamientos o acciones (compulsiones).
o EI tratamiento de los trastornos psiquitricos suele incluir una fase aguda para hacer que los sntomas remitan (en general de un
ao de duracin salvo trastornos mas leves), y una fase de mantenimiento dirigida a evitar la reaparicin de stos (aos).
5
.E

Q
1

131.11.13.
l. INTRODUCCIN

l. Clasificacin de los trastornos MIR 13 (10221): Todas las siguientes son complicaciones habi-
tuales del trastorno de ansiedad generalizada no tratado EXCEP-
mentales y del comportamiento Consumo de alcohol y abuso de drogas.
o Comenzamos la asignatura de Psiquiatra presentando las Trastornos psicosomticos.
dos clasificaciones descriptivos y multiaxiales existentes: Trastornos depresivos.
o CIE-IO: Clasificacin Internacional de Enfermeda- Trastornos psicticos. *
des, 10g edicin (Organizacin Mundial de la Sa- Conductas suicidas.
lud).
o DSM-IV: Manual Diagnstico y Estadstico de los
Trastornos Mentales, 4-J edicin, Texto Revisado 3. Epidemiologa
(Asociacin Americana de Psiquiatria).
PREVALENCIA a lo largo de la vida en poblacin general:
CIEJO DSM-IV 0 Esguizofrenia: 1% (3MIR)
3eies 5 eies Psicosis: 3 5%
Eje I: diagnsticos clnicos Eie I: trastorno psiquitrico clini- Trastorno Afectivo Bipolar (TAB): 1%
(psiquitricos y somticos) co Trastornos de ansiedad: 4-8%
Eie II: trastorno de la personali- o Sobre todo muieres venes
dad y retraso mental o Trastorno psiquitrico ms frecuente
Eie III: enfermedades somticas o Disminuye con Ia edad
Depresin mayor: 3-6% en varones y 540% en muieres
Eie Il: discapacidad Eie V: evaluacin de la actividad (sintOmatologa depresivo inespecfica: 9 al 20%).
global El suicidio aumenta con la edad (MIR).
Por sexo, los trastornos mas frecuentes son (MIR):
Eie III: acontecimientos vita- Eie IV: problemas psicosociales y
les ambientales
O Varones: depresin y alcoholismo.
Muieres: ansiedad y depresin.

2. Psicosis vs Neurosis (3mm)


2.1 . Psicosis ESQUIZOFRENIA:
Prevalencia 1%
Trastornos mentales que cursan con JUICIO DE REALIDAD
ALTERADA (MIR): el paciente no distingue lo real de Io ima-
ginario.
Delirios, alucinaciones, lenguaie y/o comportamiento des-
organizados.
Deterioro grave del funcionamiento personal y social.
Enfermedades: esquizofrenia, trastorno esquizotpico, tras-
torno por ideas delirantes, trastorno esquizoafectivo, tras-
tornos psicticos agudos y transitorios, depresiones mayores
con sntomas psicticos , mania con sintomas psicticos.
Los sntomas psicticos pueden aparecer en otros grupos de MIR 00 (6729) Segn la mayoria de los estudios epidemiolgi-
enfermedades (trastornos afectivos, consumo de txicos, en cos, la prevalencia de la esquizofrenia en Ia poblacin general
fermedades neurolgicas...). en todas las culturas es de:
I por 100000 habitantes
2.2. Neurosis 1 por 100 habitantes"
JUICIO DE REALIDAD CONSERVADO: la diferencia fundamental Hay variabilidad dependiendo de factores climticos
entre neurosis y psicosis es el iuicio de realidad (MIR). 5 por 100000 habitantes
o Trastornos crnicos o recurrentes caracterizados por ansie- ShPPNT' Hay variabilidad dependiendo de la clase social
dad que se puede manifestar como: sintomas somticos,
obsesiones, compulsiones, fobias o disfunciones sexuales. MIR 05 (8176): Los trastornos psiquitricos que con ms fre-
No ideas delirantes ni alucinaciones (MIR). cuencia se diagnostican en Atencin Primaria son:
No deterioro tan grave del funcionamiento personal y social Trastornos depresivos.
como psicosis. Trastornos de ansiedad.
No se puede demostrar etiologa orgnica. Alcoholismo.
Tipos: crisis de angustia, trastorno por angustia, agorafobia, Psicosis.
trastorno de ansiedad generalizada, trastorno de adapta- .UPS'JNT' Demencia
cin, trastorno de ansiedad fbica, trastorno obsesivo-
compulsivo (TOC), trastorno por estrs postraumtico, tras-
tornos somatomorfos y trastornos disociativos (MIR). 4. Semiolo-a
MIR 00 (6724): es causa de delirio: 4.1. Alteraciones de la percepcin
. Una intoxicacin.
PERCEPCIONES: Actividades sensoriales que nos informan de la
Un sndrome de abstinencia.
REALIDAD. Si no es real, se tratar de una ALTERACION de la
Un proceso tumoral.
percepcin:
Una infeccin cerebral.
P199353 Un trastorno de ansiedad.*
PERCEPCIN CON OBJETO EXTERNO REAL:
o Ilusin: percepcin distorsionada.
o Sinestesia: una sensacin provoca otra sensacin
(una meloda musical hace ver).
o Pareidolia: un estmulo inespecfico es percibido
como algo conocido (reconocer una forma con-
creta en una nube).

no

MIR
CURSO INTENSIVO MIR ASTURIAS

o PERCEPCIONES SIN OBJETO EXTERNO REAL: ces dentro de la cabeza).


o ALUCINACIN (MIR): percepcin sensorial falsa. o ALUCINOSIS: con conciencia de irrealidad. Tpica
Tpicamente auditiva en Ia esquizofrenia ("or voces del alcohlico: Aydome, GluciHO: oigo y veo bi-
perseCUtorias"). chos.
o PSEUDOALUCINACIN: alucinacin (quien Ia pa-
dece cree que es real) en mundo interno (or vo-

ALUCINACIN PSEUDOALUCINACIN ALUCINOSIS


Localizacin EXTERIOR INTERIOR EXTERIOR
Cree que es real? SI Sl NO
' -Auditivas: ESQUIZOFRENIA
Trastorno -VIsuaIes: SINDROME CONFU- Alcoholismo organicidad '
ESQUIZOFRENIA
SIONAL, DELIRIUM TREMENS drogas (MIR)

o PARALOGIA (MIR): falso razonamiento, no se trata de un


4'-2 AlteraCIones del pensamlenfo sntoma deticitario, sino de uno productivo (por tanto, posi-
ALTERACIONES DEL CONTENIDO tivo).
o IDEAS SOBREVALORADAS: Ideas muy cargadas afectiva-
u
mente que rigen gran parte de Ia vida del suieto (p.ei. fana
tism o) Curso Intensivo MIR Asturias
o IDEAS OBSESIVAS (MIR): Ideas intrusas y repetitivas, gene-
ran ansiedad y a las que oponemos resistencia y con ritua-
Ies (en el TOC).
o IDEAS DELIRANTES
O Idea delirante primaria: es el delirio esguizo-
trnco. Creencia errnea, pobremente estruc-
turada, irreductible. Invade Ia personalidad y
supone una ruptura biografica (cambio de
personalidad) (MIR).
o Idea delirante secundaria odeliroide: creencia
falsa que aparece sobre personalidad previa
susceptible (no hay ruptura biogrfico) como
consecuencia de conmocin afectiva. Carcter
interpretativo (MIR). Bien sistematizado (MIR).
No es vivido como impuesto. Tpico del HE
torno por ideas delirantes (paranoia).
o Delirio de los trastornos orgnicos (MIR): tpico del delirum o
episodio confusional agudo (es lo mismo). EI delirio se ca-
racteriza por ser desestructurado, tluctuante en el tiempo,
oscilar entre contenido pobre y rico (onrico), y entremez-
clarse con alucinaciones.
o ALOGIA (MIR); Empobrecimiento del pensamiento, que se
caracteriza por contenido vaco y curso poco fluido. Es sn- Delirio celotIpco (idea delirante secundaria)
toma negativo de Ia esquizofrenia.

ALTERACIONES DE LA FORMA (RITMO, CONTINUIDAD, PRODUCTIVIDAD Y POSESION) (MIR)


TAQUIPSIQUIA Aceleracin del pensamiento. Mana.
Alteraciones del ,i
RITMO BRADIPSIQUIA Enlentecimiento del pensamiento. Depresin.
BLOQUEO v. I ' 1 Parada del pensamiento. Esquizofrenia.
'
e , Taquipsiquia + prdida de idea directriz (falta de sentido global pero comprensbilidad
FUGADEIDFAS 1 parcial) + asociacin de ideas casual (rimas) + distraibilidad (influencia externa) +
N
aceleracin del ritmo de la expresin verbal. Mania.
' ii h '7
Alteraciones de Ia 7 Prdida de idea directriz (falta de sentido global pero comprensibilidad parcial) + ruptu-
IENSAMIENTQIDIS- . . .
CONTINUIDAD _ -' ra de las aSOCIaCIones normales (sorprendentes, unIIateraIes, .IncomprenSIbIes)
. .
EqIZO-
'l GREGAD'
> r , -.: frenia. (MIR 2014)
PENSAMIENTOIN- 't'; Prdida de la idea directriz y sin conexin significativa lgica entre las palabras: total
PCOHERENTE ' ncomprensibilidad (ensalada de palabras). Psicosis exgenas.
Prdida de Ia capacidad para diferenciar entre Io esencial y lo accesorio (muchas expli-
caciones triviales).
Alteraciones de Ia Obseslvos, epIIptIcos.
PRODUCTIVIDAD . . . . . .
IncapaCIdad para cambiar de un tema a otro, repItIendo Siempre el mIsmo pensamien-
7 to.

Alteraciones de la . .
., EqIzofrenIa.
noseSIon ,7 r4:- gs:

gm,
l. INTRODUCCIN

MIR 07 (8696): Uno de los siguientes sntomas caractersticos del 5.2. Mecanismos de defensa
sndrome esquizofrnico puede considerarse un sntoma
negativo: Se trata de actividades inconscientes del Yo para eliminar la
l . Anhedonia, ansiedad generada por ciertos pensamientos o deseos. Se agru-
2. Alogia. pan en 4 tipos dependiendo de su grado de madurez:
3, Frialdad emocional. Defensas narcisistas (ms primitivas; en nios y en pacientes
4. Paralogia.* psicticos):
5. Abulia. o Distorsin: alteracin de la realidad externa para cumplir
las necesidades o deseos internos.
MIR 'l4 (10390): En el transcurso de la entrevista de un paciente, o NEGACIN: rechazo inconsciente a aceptar o reconocer la
usted cae en la cuenta de que no est entendiendo lo que el realidad externa.
paciente le dice. Decide centrar su atencin en el discurso y se o PROYECCIN: atribuir a otro los pensamientos, motivacio-
da cuenta de que ste no tiene una idea directriz a pesar de que nes y sentimientos inaceptables que son propios de uno
fragmentos concretos del mismo resultan comprensibles. Esta mismo.
alteracin del lenguaie-pensamiento, tpica por otro lado de la Defensas inmaduras (en adolescentes y en algunos pacientes no
esquizofrenia, es lo que en psicopatologa se conoce como: psicticos):
Disociacin del pensamiento. o Regresin: vuelta a un estado madurativamente previo de
Fuga de ideas. funcionamiento emocional.
Lenguaje perseverante. o lntroyeccin: la transposicin de obietos externos y sus cua-
Desorqanizacin del pensamiento.* lidades en el Yo. Lo contrario a la proyeccin.
.UPPNT' Bloqueo del pensamiento. o Otros: acting out, bloqueo, hipocondra, introyeccin, so-
matizacin, comportamiento pasivo-agresivo, fantasa, ais-
4.3. Alteraciones del nivel de conciencia lamiento.
mm; (en adultos baio estrs, en pacientes obsesi-
(Tema de Neurologa). Un paciente con disminucin del nivel de
vos, histeria, fobias, ...:)
conciencia que responde solo a estmulos vigorosos se dice que
o REPRESIN: el mecanismo de defensa ms bsico v el ms
est estuporoso. Si responde a estmulos ms leves decimos que
comnmente utilizado. Las ideas, impulsos o sentimientos
esta obnubilado o somnoliento. Si no despierta ante ningn
inaceptables para el suieto son automticamente expulsa-
estmulo estar en coma. Hay situaciones en algunas patologas
dos del consciente y desterrados al inconsciente. En: histeria
psiquitricas que puedensimulari el estupor de las afectaciones (MIR).
neurolgicas, como en la depresin melanclica. Puede haber 0 DESPLAZAMIENTO: transferencia de las ideas y emociones
una menor reactividad a estmulos en la catatona que acompa- asociadas a un obieto a otro sustituto ms aceptable. Tpi-
a a algunos trastornos psicticos o afectivos. co de los neurticos fbicos (MIR).
o RACIONALIZACIN: ustificacin racional, lgica o acepta-
MIR 'I3 (10223): Cual de estas afirmaciones es FALSA respecto
ble de ideas, sentimientos o conductas inaceptables.
al estupor?
0 CONVERSIN: transformacin de una carga emocional en
'I. Es una alteracin de la conciencia.
un sntoma somtico.
2. Puede darse en la melancola.
3. Supone un estado reversible por estmulos moderados. * o Formacin reactiva: el comportamiento o Ia actitud es diri-
gido de tal forma que se opone completamente a lOs impul-
4. Se utiliza el trmino en estados de mutsmo y reduccin de
sos inaceptables subyacentes del suieto.
la actividad motora.
5. En Neurologa es un estado que precede al coma. o Aislamiento: separacin de una idea de su afecto original
asociado.
n Otros: disociacin, inhibicin, externalizacin, intelectuali-
5. Psicoanlisis zacin.
Defensas maduras
5.1. Fundamentos o SUBLIMACIN: los instintos y motivaciones socialmente
inaceptables son desviados a actividades socialmente apro-
Algunos principios del pensamiento de Freud siguen siendo piadas.
fundamentales en psiquiatra: las nociones de determinismo o SUPRESIN: eliminacin consciente o semiconsciente de
psquico, el inconsciente o la experiencia infantil en la estructura- impulsos inaceptables o conflictos del consciente.
cin de la personalidad. Fenmenos bscos del psicoanlisis: . Otros: altruismo, anticipacin, ascetismo, humor, irona.
Asociacin libre: verbalizar lo que viene a la mente.
Resistencia: incapacidad para asociar libremente. MIR 02 (7392): Atendemos a un paciente tremendamente
Transferencia: el paciente desplaza sentimientos y deseos rela- pesado, siempre con queias diferentes y del que adems
cionados con figuras del pasado al terapeuta. tenemos la sensacin de que no sigue en absoluto nuestras
Contratransferencia: lo que el paciente hace sentir al terapeuta. recomendaciones. Los sentimientos subietivos que el paciente
Etapas del desarrollo libidinal infantil: despierta en nosotros se conocen como:
Oral Anal Flico-edpica Latencia. Desconfianza.
Transferencia.
Mente consciente Contratransferencia*
Discurso.
PPI"? Mundo interno.

6. Miscelnea
6.1. Criterios de derivacin a psiquiatra
El sistema de atencin sanitaria actual requiere que el mdico de
atencin primaria asuma la responsabilidad del diagnstico y
tratamiento inicial de los trastornos mentales ms frecuentes:
ansiedad o depresin leve. Se derivar al especialista en:
0 Trastornos mentales graves: trastornos psicticos, toxicoma-
nas, patologa dual (trastorno mental + toxicomana).
O Gravedad o atipicidad del cuadro clnico.
0 Evolucin crnica o mala respuesta al tratamiento.
CURSO INTENSIVO MIR ASTURIAS

0 Riesgo suicida u homicida.


0 Marcados y graves antecedentes personales o familiares.
o Tratamientos especiales.
0 Ausencia de soporte familiar y/o social.

MIR 05 (8174): Uno de Ios siguientes trastornos NQ implica


obligatoriamente Ia derivacin desde Atencin Primaria aI Servi-
cio de Salud Mental para tratamiento especializado:
'I. Varn de 25 aos con trastorno de pnico y agorafobia de
aos de evolucin.
2. Mujer de 30 aos con cuadro psictco secundario a consu-
mo de cocana.
3. Varn de 59 aos con ideas delirantes de celos de iO aos
de evolucin.
4. Varn de 27 aos que presenta intento de suicido tras ruptu-
ra sentimental.
5. Muier de 48 aos con trastorno adaptativo tipo depresivo,
secundario a conflictividad IaboraI.*

6.2. Criterios de hospitalizacin


Incluso en contra de Ia voluntad del paciente (internamiento
involuntario con autorizacin udicia|) se ingresarn los pacientes
graves en las unidades de psiquiatra de los hospitales generales
(MIR):
o Peligrosidad para otros o para s.
o Incapacidad de cuidar de s mismo.
0 Necesidad de tratamiento especial.

7. Normas oenerales
ETIOPATOGENIA:
Modelo BIO-PSICO-SOCIAL/AMBIENTAL o de ESTRS-DITESIS:
o Factores biolgicos que confieren predisposicin o vulnera-
bilidad especfica para padecer un trastorno mental: DITE
SIS.
o Factores psicosociales y ambientales que actan como de
sencadenantes o precipitantes sobre esa vulnerabilidad bio-
lgica del su'Leto, dando lugar a Ia aparicin del trastorno
mental: ESTRES.

CLNICA: en el DSM-IV, que es Ia gua que usamos para el


diagnstico siempre veremos estos 2 criterios diagnsticos:
o Los sntomas producen malestar clnicamente significativo o
disfuncin socio-laboral.
o No se debe a drogas, frmacos u otras enfermedades.

CURSO: ,
HABITUALMENTE CRONICO.
Normalmente se alternan perodos de enfermedad activa (epi-
sodios o brotes) con perodos de remisin ms o menos comple-
ta.

TRATAMIENTO:
o Psicofarmacolgico.
o Si aparece en Ia pregunta: psicofarmacolgico + psicotera-
pia.
o En casi todos los trastornos se pueden diferenciar estos dos
tipos de tratamiento psicofarmacolgico:
o Agudo
o Mantenimiento

TRATAMIENTO AGUDO MANTENIMIENTO


REMISIN:
desaparicin de los Preventivo: EVITAR RECI-
sntomas y vuelta al DIVAS (nuevos episodios
funcionamiento de Ia enfermedad).
premrbido.
I ao (puede ser
meses en depresio- Desde 5 aos a de por
nesy trastornos de vida.
ansiedad leves).
Ms de 2 episodios pre-
Primer episodio de vios, historia familiar
Ia enfermedad. importante, intentos gra-
ves de suicidio previos.
ll. PSICOFARMACOLOGA

Psicofarmacologa n
Nmero de preguntas del captulo en el MIR

90 91 92 93 94 95f 95 96f 96 97t 97 98t 98 99t 99 OOf 00. O'l. 02. 03. 04. 05. 06. O7. 08. O9. 10 ll 'I2 13 'l4

Nmero de preguntas de cada tema

Clasificacin de los trastornos mentales y del


comportamiento

Ansiolticos 1

Neurolpticos 5

Antidepresivos

Litio 2

Antidemencia

TEC

@ Imprescindible
o Este tema es el segundo ms importante en cuanto a nmero de preguntas. Lo mas rentable es conocer los efectos adversos de los
frmacos que se utilizan en psiquiatra. Se pueden contestar muchas preguntas si conocemos el grupo farmacolgico del frmaco
por el que preguntan.
o Las benzodiacepinas son el tratamiento de eleccin de la ansiedad aguda, del insomnio y se utilizan para el tratamiento de situa-
ciones de agitacin psicomotriz. Sus principales efectos secundarios son la sedacin (extremada precaucin en ancianos y pacientes
con dicultades respiratorias) y la dependencia. El antagonista que se utiliza en la intoxiacin por Benzodiazepinas es el Flumazenilo
endovenoso.
o Los Antipsicticos atpicos o de segunda generacin son el tratamiento de eleccin de la esquizofrenia (3MIR). Los principales efec-
tos secundarios de los neurolpticos, sobre todo en los clsicos o tpicos, son los extrapiramidales (parkinsonismo, discinesias, dis-
tonas) (3MIR). La Clozapina es el que menos produce (unto con la Quetiapina), aunque requiere vigilancia del Hemograma por
riesgo de agranulocitosis. En un paciente que tras iniciar tratamiento con neurolpticos presenta fiebre, rigidez y. elevacin de CK
hay que pensar en un sindrome neurolptico maligno.
o Los ISRS (Fluoxetina, Paroxetina, Sertralina, Fluvoxamina, Citalopram, Escitalopram) son los antidepresivos de eleccin.
o Los principales efectos secundarios de los antidepresivos triciclicos son: alteraciones cognitivas (riesgo en ancianos), nauseas vmi-
tos y estreimiento, y cambios en el QT (3MIR): la prolongacin del intervalo QT y del QRS, y el aplanamiento de ondas T
o El litio es el tratamiento de eleccin en el trastorno bipolar. Para el trastorno bipolar tambin se utilizan Valproato y antipsicticos
(MlR).
o Los efectos adversos del litio estn relacionados con su concentracin (concentracin ideal 0,5-1 ,2mEq/l) y su uso requiere realizar
controles de niveles del frmaco, pruebas tiroideas y funcin renal, sodio y potasio (lOMlR).
CURSO INTENSIVO MIR ASTURIAS

i
l. Ansiolticos
1.1. BZD: Benzodiazepinas (4MIR)
Suelen clasificarse en funcin de su ACClN
___y en de su VIDA MEDIA de eliminacin (ti/2).
ACCION tl/2 lar-a >20 h tl/2 intermedia {-6 20 h tl/2 corta <6 h
Lorazepam
Alprazolam
Diazepam
Bromazepam
ANSIOLTICAS Clorazepam
Ketazolam
Clordiazepxido
Oxazepam
Temazepam
Midazolam
Brotizolam
Flurazepam Lormetazepam _
' TrIazolam
HIPNTICAS Nitrazepam Flunltrazepam ,
Zaleplon *
Quazepam Loprazolam
Zolpidem*
Zopiclona
*: frmacos hipnticos NO benzodiacepnicos que se unen al receptor de BZD.

o Aumento de la fase ll o sueo superficial y disminucin


1.2. Farmacodinamia de BZD del sueo delta y REM.
DESINTOXICACIN DE ALCOHOL Y OTRAS SUSTANCIAS
o Moduladores alostricos positivos de la neurotransmisin
inhibidora del GABA (principal neurotransmisor inhibidor o CONTROL Y TRATAMIENTO DEL SNDROME DE ABS-
del SNC) en los receptores GABAA. TINENCIA Y DELIRIUM TREMENS: diazepam o cloraze-
o Se fiian al receptor de BZD y potencian la capacidad pam (oral, IM o IV).
G Tratamiento de la acatisia secundaria a antipsicticos (in-
del GABA de aumentar la conductanca del Cl- a travs
de su canal, una vez que ste se ha fiiado a su receptor quietud motora, es un efecto adverso extrapiramidal de mu-
GABAA.
chos antipsicticos).
0 AGITACIN PSICOMOTRIZ.
o 5 ACCIONES: ansiolisis, sedacin, induccin de sueo,
0 Clonazepam: empleado en Trastorno Bipolar.
anticonvulsivante y miorrelaiante.
Precaucn: en ancianos empezar el tratamiento con dosis baias
y aumentar lentamente. Pueden provocar confusin y caidas.
1.3. Farmacocintica de BZD
ABSORCIN Y DISTRIBUCIN 1.5. Efectos adversos de BZD
o Oral: se absorben inalteradamente en el tubo digestivo Seguros y bien tolerados pero CUIDADO al mezclarlos con
0 Mayor rapidez de absorcin e inicio de accin: diace- otros frmacos depresores o alcohol!
pam, alprazolam, lorazepam.
Efectos adversos:
o La absorcin IM es errtica.
o Algunas son eficaces lV. SOMNOLENCIA, SEDACIN.
o Atraviesan la barrera hematoenceflica y se acumulan en Alteraciones de la marcha, ataxia, cadas.
cerebro y teiido gra-so. Confusin.
Agitacin paradiica.
METABOLISMO
Alteraciones cognitivas: disminucin de atencin, amnesia
La mayora por el CYP450 3A4. antergrada (especialmente si elevada potencia: alprazo-
' Dos vias metablicas: lam, triazolam, lorazepam).
o Procesos oxidativos (hidroxilacin, desmetilacin, des- DEPENDENCIA FlSlCA Y PSlQUlCA (MIR): mayor si elevada
alquilacin) que dan lugar a metabolitos activos. potencia.
O Procesos de glucuroconiugacin sin metabolitos activos: o Abstinencia si se retira bruscamente:
indicados en ancianos, insuficiencia heptica o renal. Exacerbacin o reaparicin de la ansiedad o in-
Interacciones farmacolgicas: somnio inicial, cefaleas, irritabilidad, temblor, su-
o Eritromicina, ketoconazol o antidepresivos ISRS son in- dacin, mareo, espasmos musculares.
hibidores del CYP450 3A4 y aumentan la concentra- 0 Sntomas de abstinencia verdadera: nuseas,
cin de las BZD. prdida de apetito, despersonalizacin, desrealiza
o Las BZD aumentan las concentraciones de fenitona y cin, depresin, aumento de la percepcin senso-
digoxina. rial, percepcin anormal o sensacin de movi-
miento.
1.4. Indicaciones de BZD Tolerancia: necesidad de mayores dosis para el mismo
efecto.
TRATAMIENTO AGUDO DE LAANSIEDAD (2MIR) Los hipnticos NO benzodiazepnicos presentan un perfil dife-
De forma AGUDA (< 3 meses, incluyendo l mes de retirada), rencial de efectos adversos: gusto metlico, sequedad de boca,
aunque en la prctica muchas veces crnicamente. nuseas, cefaleas, aturdimiento.
o Reaccin a estrs agudo.
o Crisis de angustia.
CONTRAlNDlCAClONES: (usar con precaucin)
o Trastorno adaptativo ansioso.
Apnea del sueo, EPOC.
o Ansiedad secundaria.
Glaucoma de ngulo cerrado.
Trastornos disociativos o de conversin
Miastenia gravis (MIR).
Coadyuvante al inicio del tratamiento con antidepresivos en
Abuso de sustancias.
ansiedad o depresin.
' TRATAMIENTO AGUDO DEL INSOMNIO (2 meses, inclu-
Enfermedad heptica o renal.
yendo l mes de retirada). Porfirio.
Deterioro cognitivo.
o Mayor eficiencia del sueo: aumento del tiempo de
sueo, disminucin del nmero de despertares y de la
vigilia nocturna.
II. PSICOFARMACOLOGA

MIR 03 (7663): Cual de los siguientes es el principal efecto


negativo de los tratamientos con benzodiacepinas en pacientes
I.7. Otros ansiolticos (no BZD)
con trastornos de ansiedad3: BETABLOQUEANTES:
Adiccin. tiles en el tratamiento de los sntomas vegetativos de la
Hipotensin.
ansiedad.
Nuseas. Los ms usados: PROPANOLOL y atenolol.
Agitacin paradiica. i Contraindicados en asma bronquial.
919 0507- main-
ANULADA AGONISTAS ALFA-2: Clonidina.
o Empleo: deshabituacin a sustancias, acatisia.
1.6. Intoxicacin aguda por BZD
CLOMETIAZOL:
Las BZD son frmacos seguros, el riesgo es baio salvo que se
' Hipntico derivado de Ia vitamina BI (tiamina). Suele utili-
asocien con otros depresores del SNC (alcohol, otros depreso
res). zarse en:
o Insomnio en ancianos.
SINTOMAS DE INTOXICACIN: disminucin del nivel de con- o DESINTOXICACIN ALCOHLICA para el control y
ciencia -desde somnolencia a estupor-, ataxia, disartria, miosis, tratamiento del sndrome de abstinencia.
relaiacin muscular. Produce dependencia y riesgo de hemorragias gastrointesti-
TRATAMIENTO DE INTOXICACIN POR BZD: nales.
Medidas generales: establecer va respiratoria e IV. BUSPIRONA:
Administracin IV del antaqonista del receptor de BZD: ' Agonista parcial del receptor 5-HTIA pre y postsinptico.
FLUMACENIL (MIR). Dosis mxima acumulada: 3 mg. o Disminuye la actividad de 5HT en caso de exceso
efecto ansioltico- y la aumenta en caso de dficit -
efecto antidepresivo-.
Aumenta ligeramente Ia actividad NA y DA.
Indicado en trastorno de ansiedad generalizada.
Bien tolerado.
BARBITRICOS :
Su uso clinico en psiquiatra NO est ustificado
Riesgo letal en sobredosis (peligro de suicidio).

2. Anti-sicticos neuroI-ticos) (18MIR)


2.1. Clasificacin
Butirofenonas: HALOPERIDOL, Droperidol INCISIVOS (POTENTES)
o Elevada potencia antipsictica (el que mas haloperidol)
Tioxantenos: Zuclopentixol
. Eticaces en SNTOMAS Posmvos
Benzamidas: Sulpiride, Tiapride o NO EFICACES EN SINTOMAS NEGATIVOS
o Muchos EFECTOS EXTRAPIRAMIDALES (EPS)
Fenotiazinas: Clorpromazina, Levomepromazina, SEDATIVOS
Tioridazina, Flutenazina, Pertenazina o Baia potencia APS
o Sedacin (el que ms la Ievomepromazina)
o EPS: ++
o Hipotensin ortosttica
Efectos anticolinrgicos
CLQZAPINA . POTENTES ,
. RISPERIDONA, PALIPERIDONA o EFICACES EN SINTOMAS POSITIVOS Y NEGATI-
OLANZAPINA VOS (menos en snt. -)
ZIPRASIDONA o MENOS EPS (el que mas risperidona, el que menos la clo-
AMISULPIRIDE zapina)
, QUETIAPINA a Clozapina:
SERTINDOL o Util en REFRACTARIOS
o Agranulocitosis
ARIPIPRAZOL (nico agonista parcial: 3 Genera
. cin)
CURSO INTENSIVO MIRASTURIAS

USO CLNICO
2.2. Farmacodinamia
. ELECCINzATPICOS
TiPICOS o Esquizofrenia resistente (resistente a 2 tratamientos con
o ANTAGONISTAS D2 en las 4 vias dopaminrgicas: atpicos: Clozapina
o Mesolmbica: el antagonismo meora delirios y alucina-
ciones (ecaces en sntomas positivos). CLOZAPINA
o Mesocortical: el antagonismo D2 empeora los sntomas - Riesgo de agranulocitosis (1%) NO debe usarse en pacien
negativos y cognitivos. te con un recuento leucocitario <3.500, antecedentes de
o Nigroestriatal: el antagonismo D2 produce efectos ad- trastornos de mdula sea o agranulocitosis.
versos extrapiramidales. o Puede producir leucocitoss benigna, leucopena,
o Tuberoinfundibular: pueden producir hiperprolactine- eosinotilia y elevar V.S.G.
mia por bloqueo D2. o Vigilancia hematolgica estrecha (MIR):
o Antagonstas adems de receptores o Hemograma basal semanal 18 semanas, quince-
o Colinrgicos muscarnicos Mi nal I ao y luego mensual.
o Alta-I -adrenrgicos o Interrumpir si leucocitos <3000 mm3.
o Histaminrgicos H1 o Riesgo de embolismo pulmonar, miocarditis, cardiomiopat-
IO.
ATIPICOS o Disminuye el riesgo de suicidio en la esquizofrenia.
. ANTAGONISMO D2 y 5HT2A
MEDICACIN DE LIBERACIN RETARDADA
o En menor medida antagonismo de:
o Di, D3, D4 o Indicado en suietos con mal cumplimiento teraputico. Ad-
o 5-HT ministracin IM cada 2 4 semanas.
O Mi o Atpicos: risperidona, olanzapina.
o Alfa-l y alfa-2 o Tpicos: flufenacina, zuclopentixol.
O H'I

2.3. Farmacocintica 2.5. Efectos adversos


ANTAGONISMO RECEPTORES D2
A. ABSORCIN Y DISTRIBUCIN o VA NIGROESTRIATAL: se producen EFECTOS EX-
o Oral o intramuscular. TRAPIRAMIDALES como acatisia, distona aguda,
o Unin a protenas plasmticas 80-90% (activa la parte Ii- parkinsonismo y discinesia tarda.
bre).
o VA TUBEROINFUNDIBULAR: HIPERPROLACTINE-
o Atraviesan la barrera hematoencetlica.
MIA (ginecomastia, amenorrea, disminucin de li-
B. METABOLISMO CITOCROMO P450 bido, infertilidad, riesgo de cncer de mama).
o ANTAGONISMO RECEPTORES MUSCARNICOS MI
CYP450 APS Interaccin o Bloqueo central: convulsiones, pirexia.
1A2 DESMETILA- Clozapina lnductor: tabaco o Bloqueo perifrico: visin borrosa, retencin urina-
CIN Olanzapina Inhibidor: fluvoxamina ria, estreimiento, reduccin de secreciones (se-
2D Clozapina Inhibidores: fluoxetina, pa- quedad de boca), alteraciones cognitivas (amne-
HIDROXILACIN Olanzapina roxetina, sertralina sia), taquicardia sinusal.
Risperdona o . ANTAGONISMO RECEPTORES ALFA-I-ADRENRGICOS
Paliperidona o Hipotensin ortosttica, mareos, taquicardia
Sertindol o ANTAGONISMO RECEPTORES HISTAMINRGICOS HT
3A4 Clozapina Inductor: carbamacepina o Sedacin, aumento de peso.
Quetiapina Inhibidor: ketoconazol, o BLOQUEO RECEPTORES 5- HT2
Ziprasidona eritromicina, inhibidores de o 5-HT2A: disfuncin sexual,
proteasa, tluvoxamina, 5-HT2C: aumento de peso.
tluoxetina, nefazodona

2.4. Indicaciones
. TRASTORNOS PSICTICOS
o ESQUIZOFRENIA
o Otros: trastorno esquizoatectivo, esquizofreniforme, por
ideas delirantes, psicosis txica.
o Fase aguda.
o Prevencin de nuevos episodios.
o TRASTORNO BIPOLAR
o Tratamiento agudo de mana con sntomas
psicticos.
o Prevencin de recidivas.
o ALTERACIONES GRAVES DEL COMPORTAMIENTO
o En cuadros orgnicos (sndrome confusional o
demencia cursan con agitacin o psicosis).
o Tratamiento sintomtico de sntomas psicticos
o Depresin grave con sntomas psicticos.
o Alteraciones del movimiento: tics, corea de Hunttington,
Guilles de Ia Tourette.
Il. PSICOFARMACOLOGA

EFECTOS EXTRAPIRAMIDALES (8MIR)


0 Por BLOQUEO DE LOS RECEPTORES D2 EN LAVA DOPAMINRGICA NIGROESTRIATAL.
a Tpicos > risperidona > resto de atpicos (clozapina lar que menos MIR-l. Dosis dependiente.

CUADRO CLINICO CARACTERISTICAS TRATAMlENTO


DISToNAAGuDA (MIR) Prevalencia: en el 10%. Ms en: -ANTICOLINRGICOS (Bipe-
- varones jvenes (raro en ancianos) rideno, Trihexifenidilo)
- primera vez tomando APS Va IM en situacin aguda
- en los APS de elevada potencia grave
Favorecido por: hipoparatiroidismo o hipocalcemia. (respuesta a los 10-20 min).
m: aguda: en horas; Despus va oral.
Clnica: ESPASMOS MUSCULARES DE CARA Y CUELLO
(crisis culo-giras, oios tios mirando hacia arriba, tortcolis,
espasmo de taringe, dificultad para hablar, tragar). Ms
infrecuente en tronco y extremidades (incluso opstotonos).

Prevalencia: 20%. Ms en: -DISMINUIR LA DOSIS.


- muieres ancianas -CAMBIAR A APS ATPICO.
- dao neurolgico previo ANTICOLINRGICO
m: tras dassemanas o tras aos de uso (Biperideno, Trihexitenidlo).
Clnica:
TEMBLOR-RIGIDEZ
- BRADIClNESIA
- BRADIFRENIA
- SIALORREA

Prevalencia: 25% -D|SN\|NUIR LA DOSIS.


ACATISlA IMIR) Mg: tras horas-semanas. -CAMBIAR A APS ATIPlCO
Clnica: INQUIETUD SUBJETIVA CON NECESIDAD IMPE- -BENZODIACEPINAS (lorace-
RIOSA DE MOVERSE pam, clonacepam).
- movimiento de pies -PROPANOLOL.
- cruzar y descruzar piernas -C|onidina.
Se ha asociado a suicidio y a heteroagresividad.
No confundir con agitacin psictica. Anticolinrgicos poco efecto.

Prevalencia: 5% de pacientes por cada ao de exposicin a -SUSPENDER ANTICOLINR-


APS. GlCOS Sl PRESCRITOS
Factores de riesgo: -DISMINUIR DOSIS DE APS
- altas dosis, cambios de APs, uso de anticolinrgicos -Tetrabenazina, agonistas
- mujeres ancianas DA, antiepilpticos
- comorbilidad con trastornos orgnicos, psiquitricos u -CAMBIAR A OTRO APS AT-
otros etectos extrapiramidales PICO
- casos raros con ISRS, litio,.. Clozapina: el APS que
Inicio: tras meses-aos mas probabilidades tiene de
Clinica: amplia variedad de MOVlMlENTOS ANORMALES resolucin
OROFACIALES. Generalmente:
- chupeteo
- protusin de la lengua
- movimientos coreitormes de las manos (tocar piano, pill
rolling)
Los movimientos se agravan con el estrs
Reversible slo en el 50% de los casos
CURSO INTENSIVO MIR ASTURIAS

OTROS EFECTOS ADVERSOS y


SINDROME NEUROLPTICO Incidencia: <I% en tto con APS tpicos (menos URGENCIA MDICA
MALIGNO (4 MIR): con atpicos). Raramente con otros frmacos
Raro pero potencialmente fatal (eutimizantes, antidepresivos, BZD) y cocaina. Retirada del frmaco.
(modalidad de hasta el 20%). Etiopatogenia: HIPERACTIVIDAD SIMPTICA Monitorizacin intensiva (ECG, TA, funcin
como consecuencia del antagonismo dopaminr- renal). Rehidratacin
gico. Relaiantes musculares (Dantroleno y Benzo-
Factores de riesgo: APS tpicos de elevada poten diazepinas).
cia, aumento o disminucin de dosis reciente o Se pueden utilizar agonistas dopaminrgicos
rpido, retirada brusca de anticolinrgicos, agita- (Bromocriptina).
cin, deshidratacin, hipertiroidismo, enfermedad En casos refractarios: Terapia electroconvulsi-
de Parkinson, trastorno cerebral orgnico, alco- va.
holismo, psicosis. Ms frecuente en varones y Muy importante la deteccin temprana y
venes. evitar la exposicin.
Clnica: por Io general los sntomas evolucionan
durante 2472 horas
- FIEBRE ELEVADA, RIGIDEZ EXTRAPIRAMIDAL EN
TUBO DE PLOMO, CONFUSIN, NIVEL DE
CONCIENCIA FLUCTUANTE
- difuncin autonmica: diaforesis, TA fluctuante,
taquicardia
Analtica: LEUCOCITOSIS, CK ELEVADA, fun-
cin heptica alterada
Incidencia: menor con atpicos. -Reducir dosis APS.
HI PE RPROLACTINEMIA Etiopatogenia: por bloqueo D2 en va tuberoin- -Cambiar a APS de menor incidencia.
fundibular. Descartar tumor. -Amantadina o Bromocriptina (puede empeo-
Tpicos > risperidona yamilsulpik Clnica: generalmente asintomtica. rar Ia psicosis).
ride > otros atpicos - disfuncin sexual.
- amenorrea, infertilidad.
- ginecomastia, galactorrea.
- osteoporosis.
Incidencia: 1% con clozapina. En menor medida CONTROLES HEMATOLGICOS.
ALTERACIONES HEMATOLGI- otros APS (0.05%).
CAS Ms en: Retirada del frmaco si necesario.
- edad aVanzada
Riesgo de agranulocitosis con - muieres Considerar TEC para posterior tratamiento.
CIozapina (MIR) Inicio: en los primeros mess.
Clnica: disminucin del recuento absoluto de
neutrfilos hasta una cifra <500/mm3'
Se desarrolla tolerancia. Aumentar gradualmente Ia dosis.
HIPOTENSIN' 1:1. QRTSTTI A_ Evitacin de cafena y alcohol.
(MIR) Aumento de consumo de Na, lquidos.
Tpicos sedantes? "rompi-12;, 5-; Y
quetiapina >"-resto de otpico
Alteraciones ECG inespecficas. ECG, vigilancia.
ALTERACIONES CARpAcAs Prolonqacin clnicamente sianiticativa del QTc.
Tpicos sedantes .> ziprasidonah Arritmias malignas: tioridacina (retirada del mer-
s'ertiridol >. restogf' cado porque produca retinitis pigmentaria).
Clnica MONITORIZACIN (cada 3 meses):
SINDROME METABLICO - OBESIDAD ABDOMEN, AUMENTO PESO. - permetro abdominal, IMC.
CIozapina, oIanzapina, perfena- - HIPERGLUCEMIA POR RESISTENCIA INSULI- - glucosa, HbA1c, perfil Iipdico.
cina, clorpromacina > quetiapina NICA. - Tensin Arterial.
, risperidona > resto - DISLIPEMIAS.
HIPERTENSIN ARTERIAL.
Clinica: perifricos: visin borrosa, retencin
EFECTOS ANTICOLINRGICOS urinaria,.estreimiento, reduccin de secreciones
Tpicos > CIozapina > oIanzapina (sequedad de boca); centrales: alteraciones cog-
> resto atpicos nitivas (amnesia), taquicardia sinusal.
CIozapina > tpicos sedantes > atpic05 > tpicos Precaucin si antecedentes de convulsiones o
DISMINUCIN a DEL UMBRAL incisivos se va a administrar TEC.
couVULSIVO
Retinitis pigmentaria: tioridacina (retirada).
Otros Ictericia colosttica: clorpromacina.
Fotosensibilidad: clorpromacina.
Disfuncin sexual: todos.
Sedacin: tpicos, CIozapina, oIanzapina, quetia
pina.
O
ll. PSICOFARMACOLOGA

MIR 02 (7393): Un paciente esquizofrnico de 25 aos lleva 2


2.6. Sobredosis semanas tomando 15 mg/dia de Haloperidol. Los sntomas
Clnica: los sntomas de sobredosis de un antagonista de los psicticos han meiorado ostensiblemente. El paciente refiere
receptores de DA son: encontrarse muy inquieto, no puede permanecer sentado mucho
o SNTOMAS EXTRAPIRAMIDALES. rato y se tiene que poner a caminar sin rumbo. CUl, entre las
MIDRIASIS. siguientes, sera una medida INCORRECTA3:
DISMINUCIN DE REFLEJOS TENDINOSOS PROFUNDOS. Aadir i mg de Lorazepan dos veces al da.
HIPOTENSIN Y TAQUICARDIA. Aadir 40 mg de Propanolol una vez al dia.
Sntomas graves: Sustituir el Haloperidol por 2 mq de Risperidona.
- DELIRIUM
Disminuir la dosis de Haloperidol a lO mg/da.
' COMA 019 3507- Tranquilizar al paciente de Io transitorio de esta sintomato-
- DEPRESIN RESPIRATORIA logia.
' CONVULSIONES
Evolucin: suele ser favorable (con la excepcin de la tioridacina MIR 05 (8183): Un paciente psictico, en tratamiento con medi-
y ziprasidona resgo cardaco-). cacin neurolptica, refiere sensacin subietiva de inquietud. A la
Tratamiento: sintomtico. exploracin, se ve que es incapaz de relaiarse, va y viene por la
o Carbn activado, si es posible lavado gstrico (cuando consulta alterna entre sentarse y levantarse, y cuando est de
paciente est consciente). pie, se balanceo de pierna a pierna. El cuadro descrito seria
compatible con el siguiente trastorno inducido por frmacos:
o Si convulsiones: diazepam lV o fenitona.
Parkinsonismo.
o Hipotensin: noradrenalna o dopamina (no adrenalina),
Sndrome neurolptico maligno.
Sndrome de piernas inquietos.
MIR 97 (5335): De los siguientes frmacos antipsicticos Cul
Distona aguda.
de ellos exige en su protocolo teraputico la realizacin de con-
919 3537- Acatisia.
troles de hemograma por riesgo de agronulocitosis:
Decanooto de Flufenacina.
MIR 07 (8601) FARMACOLOGIA: Cal de los siguientes frma-
Trifluoperacino.
cos cree que tiene ms posibilidades de producir un parkinso-
Clozagina.*
nismo yatrgeno?
Levomepromacina.
Omeprazol.
SDPPNT Clotiapina.
Cisapride.
Risperidona.
MIR 99 FAMILIA (6113): Un ioven de 27 aos llega a Urgencias
Clozopina.
con protrusin de lengua y espasmo de torsin de cuello. Un
9950.? Quetiapina.
familiar que le acompaa slo sabe decir que recientemente tuvo
un breve ingreso psiquitrico y le han puesto un tratamiento
inyectable. En esta situacin, el diagnstico ms probable es:
Tetania.
Distona oauda inducida por neurolpticos.*
Trastorno por ansiedad aguda.
Corea de Huttington.
919935). Trastorno por simulacin.

MIR 00 (6722): Respecto de denominado sndrome neurolptico


maligno, una de las siguientes afirmaciones es cierta:
l. Se trata de una reaccin idiosincrsica cuya incidencia se
estima en torno al 0,5% de los pacientes tratados con frma-
cos antipsictcos.
2. Respecto de sus manifestaciones clnicas destacan entre otros
Ia rigidez extrapiramidal en tubo de plomo, la hipertermia,
alteraciones neurovegetativa (hipertensin diastlica entre
otras) y alteracin del nivel de conciencia.
3. Su mortalidad, segn diversos estudios, puede llegar a ser
superior al 20% de los casos descritos.
4. Desde un punto de vista etiopatoqnico el nico qrupo far-
macolgico vinculado a su desarrollo ha sido el qrupo de
los antigsicticos, y por ello su nombre de sndrome neu-
rolptico" maligno. *
54 El tratamiento es de sostn y sintomtico de las manifesta-
ciones clrnicas descritas y habitualmente se efecta en uni
dades de cuidados intensivos.
CURSO INTENSIVO MIR ASTURIAS

3. Antide . resivos (I 8MIR)


3.1. Clasificacin
Existen mltiples antidepresivos que se clasifican en funcin de su accin sobre los sistemas de neurotransmisin.
NEURO-TRANSMISORES CLASE
IMPLICADOS NOMBRES (MIR)
INHIBIDORES SELECTIVOS DE LA RECAPTACIN DE SEROTONINA (5HT): PAROXETINA
|.S.R.S. CITALOPRAM
ESCITALOPRAM
FLUOXETINA
SERTRALINA
FLUVOXAMINA

Inhibidores selectivos de la recaptacin de noradrenalina (NA): I.S.R.N. REBOXETINA


Bloqueo 5-HT2A e inhibicin de recaptacin de 5HT: |.R.A.S. TRAZODONA
Inhibicin de recaptacin de noradrenalina (NA) y dopamina (DA): I.R.N.D. BUPROPION
Inhibicin selectiva de Ia recaptacin de 5HT y NA VENIAFAXINA
A DULOXETINA
Inhibicin no selectiva de la recaptacin de 5-HT y NA: ANTIDEPRESIVOS AMITRIPTILINA
TRICCLICOS CLOMIPRAMINA
IMIPRAMINA
NORTRIPTILINA
DESIMIPRAMINA
DOXEPINA
2 TRIMIPRAMINA
INHIBIDORES DE IA MONOAMINOXIDASA (I.M.A.O.) IPRONIACIDA
ISOCARBOXACIDA
TRANILCIPROMINA
FENELCINA

RIMA (inhibidores reversibles de Ia monoaminoxidasa A) Moclobemida


Alta-Z-antagonistas (aumentan los niveles de NA y 5HT) Mirtazapna
Mianserina
Estimula receptores melatoninrgicos MTi y MT2 y bloquea 5HT2C AGOMEIATINA

Cumn Imenxivo MIR Asturias 2005

3.2. Indicaciones PRIMER EPISODIO:


TRASTORNOS DEPRESIVOS TRATAMIENTO AGUDO: para la desaparicin de los
o ELECCIN: ISRS. sntomas y vuelta al estado normal (remisin).
o Tratamiento agudo. MANTENIMIENTO: 6 MESES para lograr Ia recuperacin
o Tratamiento de mantenimiento. (remisin durante meses).
TRASTORNOS DE ANSIEDAD CON EPISODIOS PREVIOS (RECADAS): mayor duracin del
O ELECCION: ISRS. tratamiento, en ocasiones de por vida.
TRASTORNOS DE IA CONDUCTA ALIMENTARIA
o ELECCIN: ISRS.
TRASTORNOS POR DOLOR
o Amitriptilina,duloxetina.
i Insomnio: Trazodona, mirtazapina.
' Enuresis: Imipramina.
' Prevencin de migraa y cefalea tensional.
Sndrome disfrico premenstrual.
Acciones
uso CLNICO: antidepresivos Acciones anti-TOC
Los antidepresivos de 1 ELECCIN ante un episodio de-
presivo son los
Tardan hasta 3 4 semanas en hacer efecto (tiempo de
latencia del efecto teraputico).
La DURACIN DEL TRATAMIENTO depender de si se trata
de un primer episodio o si ya han existido episodios previos
(MIR):
Acciones anti-pnico Acciones anti-bulimia
II. PSICOFARMACOLOGIA

REMISIN:
TRATAMIENTO AGUDO: - Cese de sntomas
- Hasta la remisin - Vuelta al funcionamiento normal
OBLIGATORIO EN 'TODOS LOS ,
PACIENTES . v TRATAMIENTO DE CONTINUACION-
MANTENIMIENTO: RECUPERACIN:
Hasta la recuperacin Remisn mantenida > meses (si no
- MESES hablamos de recada)
- MISMA DOSIS que el tratamiento agudo
EN PACIENTES CON EPISODIOS
TRATAMIENTO DE MANTENIMIENTO - PROFILAXIS: EVITAR RECIDIVAS O RECURRENCIAS
PREVIOS - Duracon y dosis individual (a veces de por vida)

MIR 09 (9205): (146) En el tratamiento de la depresin en una


3.3. SRS (ama) paciente anciana con insuficiencia cardiaca avanzada,
Antidepresivos de ELECCIN. hipertensin pulmonar severa, anemia e insuficiencia renal
Inhibidores selectivos de la recaptacin de serotonina: crnica, qu frmaco elegira?:
o El ms serotonrgco es escitalopram. o Amitriptilina.
o El ms desinhbidor es fluoxetina. Clorpromazina.
o El ms sedante es la fluvoxamina. Mi
Carbonato de litio.
Efectos secundarios: OOOO Imipramina.
o Insomnio/somnolencia.
o Disfuncin sexual MIR IO (9441): Muier 58 aos. Presenta desde hace semanas
o Ansiedad/agitacin. tristeza y desmotivacin con lloros inmotivados. Pensamientos
o Molestias gastrointestinales: diarreas, naseas, vmitos. negativos sobre el futuro, preocupacin por pequeos
o Efectos anticolinrgicos: menos que tricclicos. Menos la problemas, apata general y abandono de sus actividades
sertralina (eleccin en ancianos). Ms paroxetina y flu habituales. Los familiares explican que hace aos sufri algo
voxamina. parecido y le estuvieron dando unas pastillas que no recuerdan.
o Extrapramidales: paroxetina puede producir acatisia Descartada cualquier patologa orgnica cul es, entre los
(MIR). siguientes, el tratamiento psicofarmacolgico mas adecuado?
Citalopram 20 nda.*
Precauciones: antecedentes de hemorragia gastrointestinal, Risperidona 6 mg/da.
perodo de lavado si IMAO.
Carbamazepina 600 mg/da.
Contraindicaciones absolutas: NINGUNA (MIR). Dazepan 5 mg/noche.
.UFPNT' Metilfenidato 18 mg/da.
3.4. IRSN
MIR 'I4 (155) Acude a la consulta un hombre de 67 aos de
tiles en casos refractarios a los ISRS, pueden ser de eleccin si
edad acompaado de dos de sus hiios que comentan que su
no estn contraindicados. '
padre viene presentando los ltimos dos meses unas prdidas de
Inhibicin selectiva de la recaptacin de 5HT y NA:
memoria cada vez mayores. Previamente a ello pas por una
o Venlafaxina
temporada en la que presentaba estado de nimo baio. A lo
o Duloxetna
lardo de los ltimos meses adems ha comenzado a dar un "no
Efectos secundarios: se" o no me importa como contestacin a la mayora de las
o Hipertensin arterial. preguntas que se le formulan al tuempo que han aumentado sus
o Efectos serotonrgicos. manifestaciones de queia y malestar por sus olvidos, especial-
o Efectos anticolinrgicos. mente por las maanas. A pesar de todo ello parece desenvol-
verse con relativa comodidad en el da a da. Uno de los si-
Contraindicaciones: guientes sera el frmaco ms indicado para el tratamiento de
o Cardipatas graves. ste paciente. Selelo.
o HTA de difcil control. Quetiapina
Lamotrigina
MIR 04 (7918): En relacin al tratamiento de la depresin, una Tacrina
de las siguientes afirmaciones es INCORRECTA: Donepezilo
I. La terapia electroconvulsiva no tiene indicacin en las depre- PPSN. Sertralina
siones neurticas.
2. EI litio se utiliza en los cuadros bipolares.
3. Los IMAOS estn indicados en depresiones atpicas. 3.5. Inhibidores no selectivos de
4 Los inhibidores de la recaptacin de Ia serotonina son mejor recaptacin de 5HT y Na: tricclicos (12MIR)
tolerados que los antidepresivos tricclicos.
5. El tiempo medio de mantenimiento de la medicacin anti- tiles en depresiones graves y refractarios a ISRS.
degresiva es de semanas.*
Efectos secundarios:
o Antagonista receptores HI:
MIR 07 (8760): Seale la asociacin correcta frmaco
o Aumento de peso
antidepresivo-mecanismo de accin:
o Somnolencia
Sertralina inhibidor MAO.
Bupropin inhibidor selectivo de la recaptacin de 5HT. 0 Antagonista receptores MI: efectos anticolinrgicos
(ms amitriptilina, menos desipramina)
Iproniacida inhibidor selectivo de la recaptacin de 5HT.
o Perifricos: estreimiento (MIR), visin borrosa,
Citalopram inhibidor selectivo de la recaptacin de 5HT.*
sequedad de boca, somnolencia, retencin
PPPNT Duloxetna inhibidor de MAO.
III. .
urinaria.
o Centrales: alteraciones cognitivas (amnesia),
taquicardia sinusal.
CURSO INTENSIVO MIR ASTURIAS

o Antagonista receptores alfa-laclrenrgicos: MIR 04 (7923) Una muier de 22 aos de edad es trada al servi-
o Hipotensin postural, mareos. cio de urgencias en estado comatoso despus de una crisis con-
o Somnolencia. vulsiva, con TA de 80/40 y PA 148 I/m. En el ECG, la duracin
o Alteraciones en el ECG (MIR): del QRS es 280 mseg. Haba estado deprimida y comenzado a
o Prolongacin del QT. tomar nortriptilina 2 semanas antes. Cal de los siguientes es el
o Aplanamiento onda T. tratamienteo inicial ms apropiado?:
o Depresin del segmento ST. I. Coger va venosa y administrar bicarbonato sdico intave-
o Nuseas y vmitos (MIR). noso.
2. Coger via venosa, lavado gstrco, y diazepam para contro-
Contraindicaciones absolutas y relativos (MIR):
lar las'convulsiones.
o Cardipatas graves. 3. Control de la va area, coger va venosa y administrar
Riesgo de suicidio por elevada toxicidad. carbn activado por sonda nasogstrica.
Glaucoma de ngulo cerrado. 4. Control de la va aerea y ventilacin mecnica, coqerva
Hipertrofia de prstata (MIR). venosa y administrar bicarbonato sdico intravenosa. *
Primer trimestre de embarazo. 5. Revertir los efectos antimuscarnicos con administracin
Frmacos: fenitona, quinidina, antihistamnicos, anti- intravenosa de fisostigmina.
colinrgicos, antihpertensores centrales (nitroglicerina,
clonidina, reserpina, metildopa). 3.6. IMAOS (2MIR)
Sobredosis por antidepresivos TRICCLICOS: NO suelen ser de primera eleccin.
Sospecha en paciente psiquitrico ante:
o Arritmias
INDICACIONES:
o Depresin RESISTENTE.
o ECG: alargamiento del QRS (signo temprano)
o Alteraciones neurolgicas: delirium, convulsiones. - Depresin ATPICA, Depresin BIPOIAR,
o Sntomas anticolinrgicos: parlisis intestinal y urinaria, - CICLACIN RPIDA.
descompensacin de la TA y de la temperatura, y mi- uso CLNICO:
driasis.
Tratamiento de intoxicacin por triciclicos (MIR):
. Arm de instaurar un IMAO, PERODO DE LAVADO tras
otros antidepresivos de 1-2 semanas (fluoxetina 56 se-
o Provocar vmito no si las convulsiones son inminentes-,
manas).
carbn activado y/o lavado gstrico.
o Despus de IMAO al pasar a otro AD: 2 semanas.
Mantener la va area e ingestin de lquidos.
o DIETA: EVITAR ALIMENTOS RICOS EN TIRAMINA
Administrar bicarbonato sdico via IV rpida (0.5-2 mEq/I) o Quesos fermentados, pats, ahumados, caviar, em-
peridicamente para mantener pH sanguneo >7.45 como butidos, escabeches, conservas.
medida bsica del tratamiento de las arritmias. o Habas, espinacas, chocolate, pltanos, aguacates,
- Diacegam para controlar los efectos sobre el SNC (agita- uvas pasas, higos.
cin, refleios hiperactivos, rigidez muscular, convulsiones).
o Sopas de sobre, caza, cerveza, vino.
Slo si los sntomas persisten se deber administrar salcilato de Por riesgo de crisis adrenrgica (reaccin del queso), simi-
fisostigmina (IV lento) para revertir las manifestaciones del SNC y
Iar aI sndrome serotoninrgico:
cardiovascular de Ia sobredositicacin (2 mg con repeticin de 1 . Inicialmente: cefalea, rigidez de cuello, su
a 4 mg segn necesidad a intervalos de 20-60 minutos). dacin, nuseas, vmitos.
' Despus: hipertensin, hipertermia, taqui-
cardia, diaforesis, temblor, arritmias.

Fiebre

v...
Temblores

Hipertensin

Sntomas
anticolinrgicos

Arritmia cardiaca
(Alteraciones neurolgicas)
Convulsiones
Crisis adrenrgica
EFECTOS ADVERSOS:
Hipotensin ortosttca.
Efectos anticolinrgicos.
Aumento de peso, edema.
Hepatotoxicidad.
Crisis hipertensivas.
o Nerviosismo, insomnio
(nm mmm-n MIR Mmmm
CONTRAINDICACIONES:
c Hepatopatia grave.
o Diabetes.
o Insuficiencia cardiaca.
o Enfermedad hipertensiva complicada.
II. PSICOFARMACOLOGA

3.7. Efectos adversos de todos los METABOLISMO Y ELIMINACIN:


o No se metabolza.
antidepresivos o Vida media de eliminacin de 24 horas.
-DISMINUCIN DEL UMBRAL CONVULSIVO. o Se elimina como in, el 95% por la orina y el 5% restante
por el sudor, heces y saliva.
-SNDROME DE DISCONTINUACIN: o Se tiltra a nivel glomerular y el 80% se reabsorve con
Todos los AD tienen el potencial de producirlo por interrupcin Na y HQO en el tbulo proximal, el resto pasa al tbulo
brusca, especialmente en los de semivida ms corta (paroxetina, distal y se elimina con Ia orina.
venlataxina). La reabsorcin competitiva con el sodio signica mayor posibili-
Incidencia: I/3 de los pacientes. dad de intoxicacin en hiponatremia: dietas hiposdicas, des-
o Ms frecuente en nios y adolescentes hidratacin, sudoracin, tratamiento con diurticos tiazdicos,
o En los que experimentaros sntomas de ansiedad al inicio alteraciones de la funcin renal (MIR).
del tratamiento AD o En obesos y en embarazo Ia tasa de aclaramiento aumenta
ILci_o: a los 5 das de interrumpir el AD. por Io que se requieren dosis mayores.
Clnica: sintomas variados leves y autoIimitados: o Se excreta por leche materna eI 50% de Ia Iitemia materna,
O Somticos: mareos, vrtigo, ataxia, naseas, vmitos, tati- produciendo en el nio letargia, cianosis y hepatomegalia.
ga, mialgia, parestesias, temblor, auras migraosas, insom-
nio. INTERACCIONES FARMACOLGICAS:
0 Psguicos: ansiedad, agitacin, irritabilidad, despersonaliza- AUMENTAN DISMINUYEN COMBINACIONES
cin, talta de concentracin, problemas de memoria, sueos LITEMIA LITEMIA SEGURAS
vividos, nimo deprimido, confusin. -Diurticos aho- -Diurticos osmti- -ANTIDEPRESIVOS
Tratamiento: Se resuelve solo a las 2-3 semanas. rradores de pota- cos -ANTIPSICTICOS
sio -Inhibidores de la BZD
-SINDROME SEROTONRGICO: y tiazdicos (los anhidrasa carbnica -ANTIEPILPTICOS
Por interaccin tarmacodinmica al administrar simultneamen-
del asa menos) -Xantinas
te 2 antidepresivos de accin serotonrgica o su asociacin con
-AnaIgsicos
L-tripttano (precursor de serotonina). -AINEs Vitaminas
Clnica: -IECAs -Mucha sal Anticonceptivos
o Inquietud, confusin, coma, convulsiones. -Poca sal orales
o Hipertermia, diaforesis, diarrea, taquicardia, hipo -Vmitos, diarrea, Antidabticos
hipertensin. sudor
o Temblor, rigidez, mioclonos, hipercnesa, hiperretlexia de
EEII. 4.2. Mecanismo de accin
o Mortalidad <O.I%.
DIAGNSTICO DIFERENCIAL: con Sndrome Neurolptico Ma- Su mecanismo de accin es mltiple, acta a nivel de membra-
ligno: toma de antidepresivos, inicio y progresin mas lento en na e intracelularmente.
dias- e hipercinesia.
Tratamiento: 4.3. Indicaciones (3MIR)
o Ventilacin, va IV. o TRASTORNO BIPOIAR: DE ELECCIN
o Vigilar riesgo de rabdomiolisis. o Tratamiento de la mana aguda (mas lento que los an-
o Frmacos: BZD y antagonistas de serotonina (ciprohep- tipsicticos)
tadina). o Tratamiento de Ia depresin aguda en:
o Resolucin en horas. o MMM prevencin de recadas.
SD. DE SECRECCIN INADECUADA DE ADH: o Trastorno depresivo recurrente.
I o Trastorno esquizoatectivo.
Factores de riesgo:
o Edad avanzada, muier, baio peso corporal. o Trastornos psicticos episdicos.
o Tratamientos con diurticos, carbamacepina, quimioterapia. o Potenciacin de antidepresivos o antipsicticos.
o Fallo en la funcin renal. o Trastornos de conducta, comportamiento agresivo, automu-
o Comorbilidad (hipotiroidismo, diabetes, EPOC, HTA, tilaciones (MIR).
cncer).
o Verano. 4.4. Uso clnico
Clnica: HIPONATREMIA. _y o ANTES de iniciar el tratamiento con litio:
0 Mareos, nuseas, letargia, confusin, calambres, convul o. ECG.
siones.
o Bioqumica sangre: funcin renal y tiroidea, ionogra-
Tratamiento: Retirada inmediata del AD, luego cambiar a otra
ma, glucemia basal, hemograma.
clase de AD o considerar TEC.
o TA, peso.
o Descartar embarazo (es teratgeno).
4. Litio (IOMIR) y o CONTROLES POSTERIORES:
o Litemas semanales durante el I mes, mensuales du
En el organismo humano el Litio se halla en Concentraciones rante meses y luego trimestrales (MIR)
sricos de 10-40 mcg/L, se desconoce su funcin fisiolgica. En o Funcin renal: urea y electrolitos cada 3 meses, creati-
patologa psiquitrica eI litio es eticaz por s slo (no hay dficit nina anual.
de litio). Funcin tiroidea: cada 612 meses.
ECG (MIR): anual
4.1 . Farmacocintica OOO Hemograma y bioqumica de sangre: anual.
ABSORCIN Y DISTRIBUCIN: o Peso y TA con cada Iitemia.
o Oral. Se recomienda tomar despus de comer con agua o DOSIS INICIAL: 400-600 mg/d.
para evitar Ia accin laxante, disminuir nauseas y temblores o DOSIS MANTENIMIENTO: 900-2400 mg/d.
y meiorar Ia absorcin. o Menor en ancianos y enfermedad renal:
o Se absorbe rpido a nivel gastrointestinal . NIVEL PTIMO ENTRE 0.6-1 .2
o No se une a protenas plasmticas.
Tto. agudo 0.8 1.5 mEq/I Tto. agudo
o Atraviesa lentamente Ia barrera hematoencetlica y Ia pla-
centa.
Mantenimiento 0.6-] .2 mEq/l Mantenimiento
0 Adquiere niveles estables en sangre a los 57 das. Niveles txicos >I .5 mEq/I Niveles txicos
I
CURSO INTENSIVO MIR ASTURIAS

. DURAC|N DEL TRATAMIENTO:


o Esperar al menos 3-5 aos de estabilidad antes de n-
tentar la retirada.
o Frecuentemente el tratamiento es de por vida.

La reabsorcin del Litio es competitiva con el Na en el tbulo


proximal

4.5. Efectos adversos del litio (MIR)


o DOSIS-DEPENDIENTES: Tienden a estar directamente relacionados con las concentraciones plasmticas de Li. Menor frecuencia
con niveles <1 mEq/l.
o Puede agravar los sntomas del Parkinson, por lo que en casos de Parkinson se utilizara otro eutimizante (valproato o los nuevos
antiepilpticos).

EFECTO ADVERSO TRATAMIENTO


MOLESTIAS GASTROINTESTINALES: dispepsia, nuseas, vmitos, Tomar con comidas
diarrea.
POLlURlAPOLIDIPSIA. Dosis nica, tiazida
TEMBLOR. Propanolol
Dieta
AUMENTO DE PESO (retencin hdrica), edema.
Fatiga.
Boca seca.
Sedacin-somnolencia.
Antibitico, retinoico.
Dermatologa: agrava el acn y la psoriasis, alopecia.
Trastornos de conduccin: disfuncin del ndulo sinusal (mareos, Generalmente benignas, sin
sincope), contraindicado si fallo cardiaco o arritmia o enfermedad significacin clnica.
nodaL
- Alteraciones ECG: aplanamiento benigno de onda T (ms raro
inversin).
-Miocarditis (raro).
i
Leucocitosis, neutrofilia, linfopenia

L - Disminucin leve de la tasa de ltrado glomerular, fibrasis intersti- Disminuir la dosis de Li


cial, atroa tubular, esclerosis glomerular. Aadir amiloride
- 1% Insuciencia renal irreversible (tras 10 aos).
- Alteracin de concentracin renal: diabetes inspda nefrognica: nica contraindicacin absoluta
por interferencia del Li con la ADH en el tbulo contorneado distal y del Li
colector.
-Hipotiroidismo clnico o subclnico (5-35%) (2MIR). Mayor riesgo en Levotiroxna (no retirar Li si es
mueres de mediana edad, cicladoros rpidas, tras 6-18 meses tra- efectivo)
tamiento Control de niveles de TSH
-Bocio: poco frecuente, ms en zonas endmicas (MIR)
- Hiperparatiroidismo Suspender tto con litio
- Afectacin de tolerancia a la glucosa
- Trastornos cognitivos leves: prdida de memoria, dificultad de
concentracin, incoordinacin, temblor no(MlR 2011), sedacin
- Sndrome de Ebstein (malformacin de vlvula tricspide) en el 3% Si baio riesgo de recada retirar
de los fetos expuestos en primer trimestre. ANTES de embarazo.
-Polidramnios, prematuridad, problemas tiroides, diabetes insipida,
"floppy baby sindrome".
II. PSICOFARMACOLOGA

4.6. Intoxicacin por litio


Dado que el litio penetra en el interior de las clulas lentamente pueden transcurrir 12 horas antes de que se manifiesten los signos y
sntomas de intoxicacin a pesar de litemias txicas.

LEVE - MODERADA MODERADA - GRAVE GRAVE


1.5 2 mEo/I 2 2.5 mEo/l > 2.5 mE o /l
,- 4---.=_srmomas GASTROINTESFINALES Convulsiones generalizadas
Vmitos Nuseas y vmitos persistentes Oliguria, insuficiencia renal
Dolor abdominal Muerte
Diarrea
'SntomasiNEUROLGICOS
Ataxia Visin borrosa
Temblor grosero, de baia frecuen- Fasciculaciones musculares
cia Movimientos clnicos extremidades
Disartria Hiperretlexia tendinosa
Nistagmo Convulsiones
Letargia o excitacin Cambios ECG
Debilidad mUSCUIar Insuficiencia circulatoria (hipotensin, arrit-
Mareas mias)
Estupor, coma

TRATAMIENTO:
o Si no han pasado ms de 3-4 horas: lavado y aspiracin gstrica.
o Mantener hidratacin y corregir las alteraciones electrolticas.
Diuresis forzada en casos leves (<2 mEq/l).
Hemodilisis si litemia >2 mEq/l.

MIR FAMILIA 00 (6247): Cul de las siguientes medicaci- MIR 07 (8699): Sealar cul de las siguientes afirmaciones
nes reduce el aclaramiento de litio y eleva las concentracio- es FALSA en relacin a los efectos adversos de las sales de
nes hasta niveles potencialmente peligrosos: litio en el tratamiento del trastorno bipolar:
lbuproteno. * I. El litio puede ocasionar alteraciones de la funcin renal.
Furosemida. 2. El litio puede causar alteraciones de la funcin electrofi-
Tetraciclina. siolgica cardiaca.
Teotilina. 3 El litio puede provocar exacerbacin de la psoriasis.
PFPNT' Carbamacepina. 4. El litio puede provocar insuficiencia Respiratoria.*
5 El litio puede producir hipotiroidismo.
MIR 98 (5845): Todas las situaciones siguientes excepto una
suponen un peligro de que los niveles de litio sean ms altos MIR 08 (8960): En los pacientes con trastorno bipolar, trata-
de lo esperado. Selela: dos con sales de litio, es necesario el control de:
Uso de diurticos tiazdicos. La presin arterial.
Dieta hipersdica.* La funcin tiroidea.
Dieta hiposdica. El ttulo de anticuerpos.
Deshidratacin. El volumen de eyeccin ventricular.
.V'JPNT' Alteracin de la funcin renal. PPPN. El segmento ST.

MIR 04 (7915) Cul de los siguientes aspectos es ca- MIR 'l l (9690) El litio constituye el tratamiento protilctico de
racterstico del tratamiento con sales de litio?: primera eleccin en el Trastorno Bipolar tipo I. cual de los
Precisa para su control de determinaciones de litemia. siguientes efectos secundarios es caracterstico de este
Es eficaz en el tratamiento del episodio manaco. trmaco?:
Eleva los niveles de uricemia.* I. Temblor postural tino.*
Es eficaz en la profilaxis del trastorno bipolar. 2. Cefaleas de predominio frontal.
.UPPNT' Puede aparecer, a largo plazo, toxicidad tiroidea. 3. Atracones bulimicos especialmente de hidratos de car
bono. .
MIR 05 (8175): Cul de las siguientes afirmaciones seria 4. Hiperlipemia (aumento de triglicridos en sangre).
lNCORRECTA en el seguimiento de un paciente tratado con 5 Puede inducir depresin con riesgo autoltico especial
sales de litio?: mente importante.
'I . Precisa para su control de determinaciones de litemia.
2. Si el paciente desarrollase hipertensin arterial la pri- MIR 12 (9930): Seale cul de los siguientes psicotrmacos
mera medida a tomar sera Ia supresin gradual del li M requiere control de los niveles en sangre:
tio y la introduccin de otro trmaco lp. ei. Lamotriqi- l. Clozapina.
na .* Carbonato de litio.
3A Es preciso controlar la funcin tiroidea y renal. cido valproico.
4. El litio puede producir efectos antiagresvos independien- Imipramna.
tes de su accin estabilizante. 9553. Carbamacepina.
5. Ninguna evidencia respalda una teoria de deficiencia de
litio en el trastorno bipolar. MIR 'l4 (10384): (150) Qu anlisis deben ser controlados
en los tratamientos de mantenimiento con sales de litio?
Enzimas hepticas.
Anticuerpos antinucleares.
Ferritina y sideremia.
Funcin tiroidea y renal.*
.UFPN. Tirocalcitonina.

am.
MIR
CURSO INTENSIVO MIR ASTURIAS

4.7. Otros estabilizadores del nimo (son


antiepilpticos) . Terapia electroconvulsiva (TEC)
(MIR)
Se trata de frmacos empleados en neurologa para el trata-
miento de la epilepsia. Provoca sobre el SNC crisis comiciales generalizadas de tipo
Se utilizan en TAB como estabilizadores del humor. tnica-clnico, con una duracin total para toda la serie de
aproximadamente 20 sg.
VALPROATO/CIDO VALPROICO:
La terapia electroconvulsiva es una tcnica eficaz y segura, pero
me; suele reservarse a los casos ms graves y refractarios a otros
Mana.
tratamientos (MIR).
Depresin bipolar.
Mantenimiento. A. MECANISMO DE ACCIN
Eficaz en cicladores rapidos. La TEC acta sobre:
Tiene que estar entre 50-I25 mmol/L. Puede producir molestias o Liberacin de neurotransmisores: GABA, 5-HT, opioides y
gastrointestinaless (dispepsia, nauseas, diarrea), disfuncin catecolaminas (DA, NA).
heptica, temblor, sedacin en funcin de dosis. Fluio y metabolismo cerebral: aumento. Tras la crisis dismi-
Efectos dosis-independientes y benignos: leucopenia, tromboci nuye el fluio cerebral.
topenia, alopecia, obesidad. Neurofisiologa: trazado enlentecido y asimtrica que se
Efectos adversos graves: fallo heptico, pancreatitis, agranuloci- normaliza durante el mes siguiente.
tosis, ovario poliqustico-hiperandrogenismo. o Se le relaciona con el efecto teraputico.
La TEC N_Oproduce dao cerebral estructural.
CARBAMACEPINA: B. INDICACIONES:
M Trastornos afectivos:
o Mana.
o Depresin grave con sntomas psicticos.
o Depresin bipolar.
o Depresin con elevado riesgo de suicidio (MIR).
o Mantenimiento.
o Depresin resistente.
Potente inductor enzimtica.
o
Estados de inhibicin o agitacin intensa.
Riesgo de hiponatremia, agranulocitosis, anemia, fallo heptico,
o
Cuando estn contraindicados los AD: embarazo, an-
dermatitis exfoliante -Sd Steven Jonson, pancreatitis.
cianos.
LAMOTRlGlNA: o Cuando fallen otros tratamientos (MIR).
Usos: o Tratamiento profilctico en trastorno depresivo recu-
o Prevencin de episodios depresivos en pacientes bipola- rrente, que respondi a TEC.
res. Trastornos psicticos:
Frecuentemente produce erupcin cutnea (raramente Sd Steven o Catatona o agitacin psicomotriz que no cede.
Jonson). o Confusin grave no orgnica.
o Episodios con agitacin importante.
OTROS: o Sndrome neurolptico maligno.
OXCARBAMACEPINA, TOPIRAMATO, GABAPENTINA, PREGA- o Casos resistentes, sobre todo esquizofrenia hebefrnica
BALINA.
o desorganizada
C. CONTRAINDICACIONES
5. Frmacos antidemencia o NO existen contraindicaciones absolutas.
o Relativas:
Estimulan la neurotransmisin colinrgca, con lo cual meioran o o ACV: esperar semanas o meses.
al menos enlentecen el deterioro de la memoria en la Enferme- o Hipertensin intracraneal.
dad de Alzheimer (agentes promnsicos). Tambin meoran los o IAM: esperar 3 meses.
sntomas comportamentales de la EA (desinhibicin, apata, o HTA grave.
vagabundeo) 0 Suspender tratamiento con litio (aumenta confusin y amne-
Dos tigos: sia).
ANTICOLINESTERSICOS: Revisar la insulina y los antidiabticos orales, por los efectos
0 Son: sobre la glucemia de la TEC.
0 DONEPEZILO D. EFECTOS ADVERSOS
GAIANTAMINA Mortalidad <0.0I%
RIVASTIGMI NA Cardiovasculares: arritmias.
Inhibidores de la acetilcolinesterasa (AChE), enzima que Neuropsiquitricas (ms frecuentes): convulsiones prolon-
degrada la acetilcolina (ACh). gadas, confusin (5-1 0%), amnesias ante o retrgradas que
Indicados para enfermedad de Alzheimer leve o moderada. se recuperan tras semanas, cefaleas y dolores musculares.
Efectos secundarios gastrointestinales (nauseas, diarrea, Los de la anestesia general.
vmitos, disminucin apetito, dispepsia), mareos, cefaleas,
fatiga, astenia y sudacin MIR 07 (8705): En cul de los siguientes casos la terapia elec-
troconvulsivante es un tratamiento de PRIMERA ELECCION32
ANTAGONISTAS RECEPTORES NMDA (receptor del glutamato): Esquizofrenia paranoide.
0 Es la MEMANTINA. Psicosis reactiva.
0 Bloqueo el exceso de glutamato en la EA. Depresin mayor con sntomas melanclicos.
Indicados para enfermedad de Alzheimer moderada. Trastorno equizotpico de la personalidad.
Efectos secundarios: mareos, cefaleas, estreimiento. SWF-WN. Trastorno delirante crnico.
ANUIADA
II. PSICOFARMACOLOGA

RESUMEN DE PSICOFRMACOS
1. ANSIOLTICOS: BENZODIACEPINAS (BZD)
Farmacodinamia: moduladores alostricos positivos de la neurotransmisin inhibidora del GABA principal neurotransmisor inhibidor-
en los receptores GABAA (actan potenciando la capacidad del GABA de aumentar Ia conductancia del CI a travs de su canal).
Clasificacin segn su vida media (ti/2):
o Larga: ASL: Clordiacepxido, Cloracepato, Diacepam; HPN: Fluracepam, Nitracepam, Quazepam.
o Intermedia: ASL: Alprazolam, Bromacepam, Ketazolam, Loracepam, Oxacepam, Temacepam; HPN: Flunitracepam, Loprazo-
lam, Lormetazepam.
I Corta: HPN: Brotizolam, Midazolam, Triazolam.
ASL: ansolticas; HPN: hipnticas
o TODAS tienen 5 acciones: ansiolisis, hipnosis, anticonvulsivante, miorrelaiante y sedacin.

1.1. INDICACIONES:

o Ansiedad aguda (MIR): crisis de angustia, trastorno por estrs agudo, ansiedad secundaria a enfermedades orgnicas
o En los trastornos de ansiedad crnicos, como coadyunvantes (duracin mxima 3 meses, incluyendo l mes de retirada
progresiva): trastorno de ansiedad generalizada, trastorno por estrs postraumtco...
o Trastornos disociatvos conversin.
Insomnio (duracin maxima 2 meses, incluyendo l mes de retirada progresiva).
Sndrome de abstinencia a alcohol y otros txicos.
Acatisia secundaria a antipsicticos.
Agitacin psicomotriz.
Trastorno bipolar I (clonazepam).

'I .2. EFECTOS ADVERSOS Y CONTRAINDICACIONES:

o Tolerancia, dependencia fsica y psquica: TODAS (mayor a mayor potencia y menor tl /2: alprazolam) (MIR)
o Enlentecimento psicomotor (> con tI/2 larga), amnesia antergrada
o En caso de intoxicacin: nico pscofrmaco que tiene antagonista: Flumacenil (MIR)
o Contraindicadas en enfermedades que cursan con debilidad muscular como Ia miastenia gravis o sndrome de apnea del
sueo (MIR).
Otros FRMACOS ANSIOLTICOS
o ANSIOLTICOS: Betabloqueantes, agonistas alfa-2 adrenrgicos (clonidina) y buspirona.
HIPNTICOS: zopiclona, zoIpidem, zaleplon.
o Actan sobre el mismo receptor BZD.
o Todos ellos de vida media corta.

2. ANTIPSICTICOS

Butirofenonas: HALOPERIDOL, Droperidol INCISIVOS (POTENTES)


Tioxantenos: Zuclopentixol c Elevada potencia antipsictica (el que mas haloper-
Benzamidas: Sulpiride, Tiapride M
z - Eficaces en SNTOMAS POSITIVOS
9 . NO EFICACES EN SINTOMAS NEGATIVOS
o Muchos EFECTOS EXTRAPIRAMIDALES (EPS)
u; Lu Fenotiazinas: Clorpromazina, Levomepromazina, Tioridazi- SEDATlVOS
8 E na, Flufenazina, Perfenazina o Baia potencia APS
E :9 o Sedacin (el que ms Ia levomepromazina)
l- '_ o EPS: ++
CLOZAPINA o POTENTES
RISPERIDONA, PALIPERIDONA o EFICACES EN SNTOMAS POSITIVOS Y
OIANZAPINA NEGATIVOS (menos en snt. -)
6 ZIPRASIDONA o MENOS EPS (el que ms risperidona, el que menos
AMISULPIRIDE Ia clozapina)
QUETIAPINA o Clozapina:
ws
o % SERTINDOL o Util en REFRACTARIOS
0% (5' , o Agranulocitosis
ARIPIPRAZOL (Unico agonista parcial: 3 Generacin)
E m

2.1. MECANISMO DE ACCIN

TPICOS:
o Antagonistas dopaminrgicos: bloqueo de los receptores D_2_ en las 4 vas dopaminrgicas
< o Va mesolmbica: efecto antipsictco sobre sntomas positivos (delirios y alucinaciones).
o Va mesocortical: empeoran sntomas negativos y cognitivos,
5 o Va nigroestriatal: efectos extrapiramidales.
S o Va tuberoinfundibular: hiperprolactinemia.
a
- Antagonistas de los receptores Ml , alfa-l -adrenrgico y H'l .

wLMR
{m
CURSO INTENSIVO MIR ASTURIAS

ATPICOS:
o Antagonistas dopaminrgicos-serotoninrgicos: bloqueo simultneo de los receptores D2 y 5HT2A en las 4 vas dopamingi-
cas.
o Antagonistas de otros receptores: DI , D3, D4, varios 5HT, MI , HI , alfal , alfaZ.

2.2. INDICACIONES

o Trastornos psicticos (trastornos esquizofrnicos, de ideas delirantes, breve).


o Fase aguda.
o Mantenimiento.
o Trastorno bipolar
o Tratamiento aguda de mana.
o Mantenimiento.
0 Alteraciones del comportamiento, agresividad, agitacin.
Depresin grave con sntomas psicticos.
o Alteraciones del movimiento: tics, corea de Hunttington, Guilles de la Tourette.

2.3. EFECTOS ADVERSOS

EXTRAPIRAMIDALES: Por bloqueo de los receptores D2 en la va nigroestriatal. Tpicos incisivos > tpicos sedativos > atpicos.
o Distona aguda: (MIR) Tratamiento con antiparkinsonianos anticolinrgicos (biperideno, trihexifenidilo).
o Parkinsonismo: (MIR) temblor, rigidez, acinesia. EI efecto secundario mas importante de los neurolpticos. Tratamiento: anti-
colnrgicos (biperideno, trihexifenidilo).
o Acatisia: (MIR) inquietud psicomotriz. Tto: cambiar a antipsictico atpico, disminuir dosis, aadir propanolol o aadir benzo-
diacepinas (loracepam).
o Discinesia tarda: (MIR) Movimientos periorales y Iinguales. Incapacitantes y de difcil tratamiento, el efecto extrapiramidal ms
grave.
SINDROME NEUROLPTICO MALIGNO (MIR): reaccin idiosincrsica en el 0,5% de los pacientes tratados con antipsicticos. Pue-
de ser producido tambin por: cocana, litio, carbamacepina, fluoxetina, triciclicos o benzodiacepinas.
o Sntomas y signos: Rigidez muscular extrema, hipertermia (41), taquicardia, hipo o hipertensin, alteraciones variables del
nivel de conciencia (puede llegar al coma).
o Pruebas de laboratorio: Leucocitoss y aumento importante de CK, enzimas hepticas y moglobina Plasmtica.
o Tratamiento: Emergencia mdica UCI-: Control constantes vitales, electrolitos, balance hdrico; retirada del antagonista do-
paminrgico; agonsta dopaminrgico (bromocrptina, L-dopa) y relaiante muscular (dantrolene).
o Mortalidad: puede superar el 20%.
OTROS EFEQTOS 2:
o Dermatolgicos: Fotosensibildad por clorpromacina.
o Hematolgicos: Agranulocitosis por clozapna controles hematolgicos peridicos- (MIR).
o Efectos anticolinrgicos: Sequedad de boca, visin borrosa, estreimiento, retencin urinaria y somnolencia.
0 Hpotensn ortosttica: (MIR)
o Endocrinos: Aumento de la secrecin de prolactina, aumento de peso.
o Oalmolgicos: Coloracin irreversible de la retina (similar a retinitis pigmentaria) por tioridacina. (MIR). Coloracin benigna
de las lentes anteriores oculares y crnea posterior de por clorpromacina.
o Ictericia colostsica: Por clorpromacina.
o Cardiovasculares: Los antipsicticos sedativos son ms cardiotxicos. La ziprasidona se asocia a prolongacin del QTC, Ia
clorpromacina causa prolongacin del QT y PR con aplanamiento de onda T y depresin del ST. La tioridacina se asocia a
arritmias malignas RETIRADA DEL MERCADO-.

3. ANTIDEPRESIVOS

3.1. INHIBIDORES SELECTIVOS DE LA RECAPTACIN DE SEROTONINA ISRS

o Antidepresivos de 1 eleccin (eficacia similar a los ADT, meior tolerancia y seguridad).


o Ctalopram (el ms selectivo no interacciones), Escitalopram, Fluoxetina (ti/2 mas larga, ms desinhibidor), Fluvoxamina
(ms sedante), Paroxetina (ms efectos AntiAch), Sertralina.
Indicaciones:
o Trastornos depresivos: depresin mayor, atpica, bipolar, distimia (tratamiento agudo y de mantenimiento).
o Trastornos de ansiedad: t de ansiedad generalizada (TAG), de estrs postraumtico (TEPT), obsesivo-compulsivo (TOC), de
panico y trastornos adaptativos.
o Trastornos de conducta alimentaria.
Efectos adversos:
o Son los antidepresivos ms seguros; no son cardiotxicos
o Los efectos secundarios se deben a acciones indeseables de la 5HT.
o Los ms frecuentes G-I: nauseas, vmitos, diarrea.
o Otros: insomnio o somnolencia, cefalea, disfuncin sexual, inquietud y acatisia (MIR).
o Sdr de retirada: especialmente en los de ti /2 corta
o No tienen contraindicaciones absolutas (MIR).
o No deben asociarse con IMAOs.
o Periodo de lavado antes de iniciar tto con IMAOs: 1-2 semanas, excepto con Ia fluoxetina 4-5 semanas.

l\_/I_I_R_
ll. PSICOFARMACOLOGA

3.2. INHIBIDORES DE RECAPTACIN DE SEROTONINA Y NORADRENALINA IRSN

o Inhibicin dual y selectiva de la recaptacin de 5HT y NA


o Sinergia farmacolgica: mayor eficacia antidepresiva,
o Venlafaxina y Duloxetina
o Contraindicaciones relativas: HTA de difcil control.

3.3. ANTIDEPRESIVOS TRICICLICOS ADT

o Amitriptilina, Clomipramino, Desimipramina, Doxepina, Imipramina, Nortriptilina, Trimipramina (MIR).


o 5 acciones a nivel receptorial en SNC:
o teraputicas: inhibicin de Ia recaptacin de 5HT y NA (MIR).
o efectos adversos: antagonismo receptores anticolinrgico-muscarnicos MI , antagonismo de receptores alfa-adrenrgicos
y antagonismo receptores histamnicos HI.
Indicaciones:
o Trastornos depresivos: tratamiento agudo y de mantenimiento.
o Trastornos de ansiedad: TOC (slo Ia clorimipramina e| ms serotonrgico-).
o Otros: bulimia, dolor crnico (amitriptilina), enuresis (imipramina), prevencin de migraa.
Efectos adversos:
o Efectos anticolinrgicos:
o Centrales: confusin, trastorno de memoria, temblor, disminucin del umbral convulsivo.
o Perifricos: visin borrosa, midriasis, sequedad de boca, estreimiento (MIR), retencin urinaria (MIR) .
o Efecto anti-alfa-adrenrgico: hipotensin ortosttica central, mareos.
0 Efecto antihistamnico: sedacin, aumento de peso.
o Efectos 2 cardiovasculares:
o Palpitaciones, taquicardia.
o Alteraciones ECG: efecto quinidina-Iike; prolongacin QTC y PR, ensanchamiento QRS, depresin segmento ST, aplana-
miento onda T, aparicin onda U (MIR).
o Taquiarritmias ventriculares.
o En sobredosis: arritmias mortales.
o Otros: convulsiones (raras), reacciones confusionales en ancianos en tto con antiAch, disfuncin sexual, aumento de peso
apetito, temblor, sindrome de retirada, nauseas, vmitos (MIR).
o Ante Ia triada: arritmias, convulsiones y sntomas anticolinrgico , sospechar intoxicacin aguda por tricclicos.
Contraindicaciones:
0 Absoluta: IAM reciente.
o Relativas: glaucoma de angulo cerrado, hipertrofia prosttica, epilepsia, ancianos con parkinson en tto con agentes antico-
linrgicos, primer trimestre del embarazo (MIR).
Interacciones medicamentosas:
. Evitar con antihistamnicos, anticolinrgicos, antihipertensivos, simpaticomimticos, alcohol y metildopa.
o Antidepresivos tricclicos + IMAOS: Evitar en Io posible.
o Paso de IMAOS a antidepresivos tricclicos: esperar 2-3 semanas.
o Paso de antidepresivos tricclicos a IMAOS: Unos das.

3.4. INHIBIDORES DE LA MONOAMINOXIDASA IMAOs

o Mecanismo de accin: inhibicin irreversible de la principal enzima degradadota de las monoaminas cerebrales (MAO, en su
formas Adegrada las monoaminas cerebrales principalmente- y B degrada Ia tiramina-)
o La inhibicin de la MAO alcanza el mximo a los 5-l O das de iniciado el tratamiento
La recuperacin se produce a las 2 semanas de interrrumpir el tratamiento
o Tranilcipramina (MIR)
o Se evitar su uso con alimentos ricos en tiramina por riesgo de crisis hipertensivas (quesos fermentados, ahumados, pats,
embutidos, hgado, caviar; pltanos, aguacates, higos; vino, cerveza) (MIR)
A. Uso clnico
o NO son frmacos de 1 eleccin. Se usan en
o Depresin refractario y atpica, especialmente con sntomas ansiosos, fbicos y obsesivos
o Trastornos de ansiedad: TAS y TPK
B. Efectos adversos _
o Hipotensin ortosttico, anticolinrgicos, nauseas, temblor, aumento de peso, disfuncin sexual, insomnio, parestesias en
piernas, edema en tobillos, mioclono
o Crisis hipertensiva grave SI se ingieren alimentos ricos en tiramina: cefalea occipital, palpitaciones, taqui o bradicardia, hiper-
pirexia, nuseas, vmitos, sudoracin, hemorragia cerebral, IAM
o Reversibles con fentolamina (5-IO mg IV) o nifedipino (20 mg sI)
C. Contraindicaciones
o Hepatopatas severas, diabetes, insuficiencia cardiaca, enf hipertensiva complicada.
D. Interacciones medicamentosas
o Con frmacos simpaticomimticos, antihipertensivos, antihistamnicos, aumento de Ia sensibilidad a la insulina, antidiabticos
orales, morfina y meperidina, barbitricos, fenitoinas.
. Potencia Ia accin de anestsicos, alcohol, benzodiacepinas, anticolinrgicos, opiceos y otros antidepresivos.
S o Antes de instaurar tratamiento con IMAOS deben deiarse 1-2 semans libres de tricclicos, ISRS (excepto fluoxetina: 4-5 sema-
5 nas) e IRSN.
S
g
3

,3

MIR
CURSO INTENSIVO MIR ASTURIAS

4. SALES DE LlTlO
o Absorcin oral. NO sufre metabolismo; se elimina sin cambios por la orina
o La mayor parte es reabsorbido por tbulo proximal compitiendo con el Na, lo que significa ms posibilidades de intoxicacin
en estados de hiponatremia como: dietas hiposdicas, empleo de diurticos tiazdicos, deshidratacin, sudoracin excesiva o
insuficiencia renal (MIR).
o Antes del inicio del tratamiento se recomienda ECG, determinacin de urea, creatinina, y glucemia, asi como pruebas de tun-
cin tiroidea y renal y test de embarazo (MIR).
o MARGEN TERAPUTICO MUY ESTRECHO: Iitemias: tratamiento agudo: 0,5-l,5 mEq/l; de mantenimiento: >0.8 mEq/l; nive-
les txicos: >l.5 mEq/l.
o La dosis diaria habitual 4001600 mg/da. En ancianos no suele necesitarse ms de 900 mg/da (MIR).
o Deben hacerse litemias semanales durante el primer mes, mensuales durante meses y luego trimestrales (MIR).
o Deben pasar 12 horas entre la ltima toma y la determinacin analtica y 5 das entre la modificacin de dosis y la de-
terminacin analtica.
o Bioqumica, T4, TSH, creatinina, urea y orina cada 6-12 meses; ECG anual (MIR).

4.1. INDICACIONES (MIR)


Efectivo en el 4050% de los pacientes
o Tratamiento aaudo del trastorno bipolar (mana, hipomano y depresin)
o NO en episodios mixtos o cicladores rpidos (Carbamacepina)
o Prevencin de recurrencias en los trastornos afectivos (especialmente bipolares)
a Trastornos esquizoatectivos, trastornos psicticos episdicos, trastornos de conducta, comportamiento agresivo

4.2. EFECTOS ADVERSOS

Gastrointestinales: dispepsia, nauseas, vmitos, diarrea...


Aumento de peso: por retencin hdrica
Edema
SNC: temblor, sedacin, disminucin cognicin, incoordinacin
mil (MIR): disminucin de filtrado, fibrosis intersticial, atrofia tubular, esclerosis glomerular, diabetes inspida netrognica
(poliuria con polidipsia secundaria) e incluso insuficiencia renal irreversible (contraindicacn de litio)
Endocrino: a larao plazo hipotiroidismo (MIR) (en el 3%; + en muieres), puede aparecer bocio (MIR)
Cardiovascular: tde la actividad del nodo sinusal: mareos y sncopes (MlR)
Dermatolgicos: erupciones, acn, exacerbacin de la psoriasis, alopecia
Hematolgicos: leucocitosis, neutrotilia, lintopenia
Teratogenicidad: malformaciones cardiovasculares Sdr de Ebstein (3%)

4.3. INTERACCIONES FARMACOLGICAS


AUMENTO DE LA LITEMIA (MIR) DISMINUCIN DE LA LlTEMlA
1. Diurticos ahorradores de potasio l. Diurticos osmticos
2. Diurticos tiazidicos 2. lnhibidores de la anhidrasa carbnica
3. lECAS 3. Xantinas
4. AINES 4. Dieta rica en sal
5. Dieta pobre en sal
6 Vmitos, diarrea o sudoracin excesiva
No alteran la Iitemia la mayora de los otros psicofrmacos ni los anticoncetivos orales (importante porque es potencialmente te-
ratgeno)

4.4. INTOXICACIN
u Litemia 1,5-2 mEn: Sntomas gastrointestinales. Vmitos, dolor abdominal. Sntomas neurolgicos: Ataxia, temblor grosero,
disartria, letargo, debilidad muscular, mareos, nistagmo.
o Litemia 2,0-2,5 mEgzl: Aumento de sint gastrointestinales. Sint neurolgicos: visin borrosa, tasciculaciones, hiperretlexia ten-
dinosa, convulsiones, delirium, cambios en EEG, estupor o coma.
o Litemia > 2,5 mEn: Convulsiones, oliguria e insuficiencia renal, muerte.

4.5. TRATAMIENTO DE IA INTOXICACIN

o Medidas de soporte, ingreso en UCI, suero salino. Dilisis en litemos superiores de 2 mEq/l.

4.6. OTROS ESTABILIZADORES DEL HUMOR

Valproato, Carbamacepina, Oxcarbamacepiena, Lamotrigina, Topramato.

5. FRMACOS ANTIDEMENClA
c Estimulan la neurotransmisin colinrgica, con Io que meoran o al menos enlentecen el deterioro de la memoria en la En-
termeclad de Alzheimer (EA). Tambin meioran los sntomas comportamentales.

5.1. lNDlCAClONES

o Anticolinestersicos: (donepezilo, galantamina y rivostigmina) EA leve o moderadamente grave


o Antagpnistas de receptores NMDA: (memantina) EA moderadamente grave
Il. PSICOFARMACOLOGIA

5.2. EFECTOS ADVERSOS

o AnticoInestersicos: gastrointestinales, mareas, cetaIeas, fatiga, astenia y sudacin


o Antaaonistas de receptores NMDA: mareos, cefaIeas, estreimiento

6. TERAPIA ELECTROCONVULSIVA - TEC


o Provoca sobre eI SNC crisis comiciaIes generaIizadas, de tipo tnico-cInico, con una duracin totaI para toda Ia serie de 20
a 25 sg.

6.1. INDICACIONES

o Trastornos atectivos(es Ia principal indicacin): Depresin con sntomas psicticos, Depresin con elevado riesgo de suicidio
(MIR), Depresin resistente, Estados de inhibicin o agitacin intensa, Cuando estn contraindicados Ios AD, Tratamiento pro-
tilctico en TDR
o Trastornos psicticos: Confusin grave no orgnica, Agitacin importante, Catatona, Casos resistentes a distintos APS

6.2. CONTRAINDICACIONES

No existen contraindicaciones absolutas

6.3. EFECTOS ADVERSOS

o Mortalidad: I [10.000 pacientes (0.01%)


o Cardiovasculares: arritmias
o Neuropsquitricas: convulsiones proIongadas (si >3 m debe tratarse), confusin (5-IO%), amnesias ante o retrgradas que
se recuperan tras semanas, cetaIeas y dolores musculares. -
CURSO INTENSIVO MIR ASTURIAS

Trastornos mentales organicos m


Nmero depreguntas del captulo en el MIR

9o 91 92 93 94 9st 95 9f 96 97f 97 9er 9a 99r 99 oor oo. 01. 02. 03. 04. os. 06. 07. os. o9. 10. 11. 12. 13 14

Nmero de preguntas de cada tema

Delirium-Sd confusional

Sndrome amnsico orgnico

Demencias

Enfermedad de Alzheimer

Otros tipos de demencas

Demencias trasmisibles

Pseudodemencia

C6) Imprescindible
o Impresicindible el concepto de sindrome confusional: el Delirium o sndrome confusional orgnico es una disfuncin cerebral AGU-
DA generalizada, que suele ser secundaria a un problema sistmico subyacente (infecciones, txicosm), y que cursa caracterstica-
mente con una alteracin del NIVEL DE CONCIENCIA y de la capacidad de ATENCION. Es autolmitado y meiora cuando meiora
la causa que lo desencadena. Hay que tratar la causa subyacente y los sntomas suelen requerir tratamiento con Neurolpticos de
alta potencia (9MIR).
. Los sndromes amnsicos organicos suelen afectar a la memoria reciente afectando a Ia capacidad de incorporar nuevos recuerdos.
eCualquier agresin al cerebro puede llegar a causar una demencia, pero la causa ms frecuente en nuestro medio son las demen-
cias degenerativas, en concreto la Enfermedad de Alzheimer.
o Enfermedad de Alzheimer: comienza por fallos de memoria, se sigue de un Ienguaie pobre y fallos aprxicos. En fases avanzadas
se deterioran la marcha, la conduda social y los pacientes se deterioran fsicamente falleciendo por complicaciones sistmicas.
o Demencia frontotemporal: alteraciones de conducta (variantes frontal) o del lenguaie (temporal) precoces.
aDemencia Cuerpos de Lewy: deterioro cognitivo de curso fluctuante, alteraciones de la marcha, alucinaciones visuales precoces
(5MIR).
o Hidrocefalia normotensiva: alteraciones de la marcha, demencia subcortical, incontinencia urinaria. Se diferencia de otras demen-
cias subcorticales por los hallazgos en neuroimagen. Tratamiento: derivacin de LCR.
o Enfermedad de Creuizfeldt Jakob: sospecharlo en individuos con demencia rpidamente progresiva con alteraciones de la marcha y
mioclonas.
o Pseudodemencia: Fallos cognitivos en pacientes con depresin. Inicio rpido, fallos relacionados con la falta de atencin y de cola-
boracin en los test. Suele ser el paciente el que expresa las queias cognitivas (al contrario que en el Alzheimer que suele ser la fa-
milia) y se acompaa de clnica depresivo. Puede meiorar con Antidepresivos.

este
MJ;
III. TRASTORNOS MENTALES ORGANICOS

i. Delirium (sndrome confusional

SNDROME CEREBRAL ORGANICO caracterizado por disfuncin


cerebral AGUDA generalizada, en la cual el hecho mas impor-
tante es el deterioro fluctuante del nivel de conciencia y de la
atencin (7MIR).
0 Alteracin cerebral global:
O Alteraciones de la percepcin: ilusiones y alucinaciones,
sobre todo visuales (4MIR).
ldeacin delirante transitoria (2MIR).
Deterioro del razonamiento y comprensin.
Deterioro de la memoria inmediata y reciente.
Dificultades de atencin (MIR).
OOOOO Desorientacin temporal, y en casos graves espacial y
de persona (falsos reconocimientos) (2MIR).
o Trastorno del ciclo sueo-vigilia (inversin) (MIR).
o Trastornos emocionales: irritabilidad, miedo, euforia, apa-
ta, perpleiidad, ansiedad (MIR)
0 Trastorno psicomotor: hipo o hiperactividad con agitacin,
agresividad (MIR).
o Puede haber signos neurolgicos inespecficos (temblor,
asterixis, nistagmo, falta de coordinacin motora, inconti- Cuiso Intensiva MIR Asturias
nencia urinaria) (MIR).

MIR 00 (6720): Cual de las siguientes caractersticas es ms til 1.1. Epidemiologa


para distinguir el cuadro confusional agudo de Ia demencia32
Disminucin de Ia capacidad de ATENCIN.* o En el 30-40% de los ancianos hospitalizados (MIR).
Desorientacin. o Factores de riesgo: mayores de 60 aos, hospitalizacin,
AIucinaciones. comorbilidad organica, dao cerebral previo (demencia, ACV,
Afectacin de la memoria. tumor), historia previa de delirium.
mewva Alteraciones del ciclo sueo-vigilia.
i .2. Etioioga
MIR 02 (7399): Cul de las siguientes afirmaciones es 0 Orgnica: enfermedades metablicas, infecciosas (sobre todo
INCORRECTA respecto del delirium o sindrome confusional?: infeccin de orina), neurolgicas, intoxicacin o abstinencia de
Su principal caracteristica clinica es la presencia de
agentes txicos o farmacolgicos (4MIR). Relacin temporal
creencias falsas, irreductibles a Ia argumentacin Iaica v
prxima entre Ia enfermedad y el sindrome confusional agu-
que no auardan relacin con el entorno cultural del
do.
paciente.*
2. Su principal caracterstica clinica es Ia alteracin de la 1.3. Curso
conciencia.
3. Suele asociarse a alteraciones globales de las funciones o Inicio repentino (dias u horas).
cognitivas. o Curso breve y fluctuante con empeoramiento vespertino (no-
4. Se acompaa a menudo de alteraciones del humor, la che).
percepcin y el comportamiento. 0 Meiora rpida cuando se identifica y elimina el factor causal
5. No es infrecuente Ia presencia de temblor, asterixis, (MIR).
nistagmo, falta de coordinacin motora e incontinencia o Duracin inferior a 6 meses. Habitualmente dura menos
urinaria. de 'l mes (MIR).
0 Puede evolucionar hacia demencia.
MIR 1 1 (9686) La coexistencia de alucinaciones y de alteraciones
de la conciencia es sugestiva de: 1 .4. Tratamiento
Los estados ansiosos.
ANTIPSICTICOS DE ELEVADA POTENCIA:
Los estados confusionales*.
o A poder ser oral: risperidona (atipico) o haloperidol a dosis
Los estados depresivos.
baias .
Los estados maniacos.
PPS-N." Los estados psicticos. o Si IM de eleccin haloperidol (el ms seguro si existe cardiopa-
tia).
MIR 12 (993i) La existencia de alucinaciones y de alteraciones o Si gran agitacin, suiecin mecnica para evitar auto y hetero-
de la conciencia es propio de: Iesiones (MIR),
Los estados ansiosos.
Los estados confusionales*. MIR 03 (7662): Un hombre de 73 aos con Enfermedad
Los estados depresivos. Pulmonar Obstructiva Crnica esta ingresado en una planta de
Los estados maniacos. Medicina Interna tras ser atendido en Urgencias por una
psicticos. Insuficiencia Respiratoria Global, secundaria a una Infeccin
.UP -EQT' Los estados
Respiratoria. Durante su segunda noche en el hospital, presenta
agitacin, desorientacin temporal y espacial, falsos
reconocimientos, insomnio y agresividad verbal y fisica hacia el
personal cuidador. El paciente se arranca la mascarilla de
oxigeno y las vias de perfusin. Es portador de una prtesis de
cadera derecha. La enfermera de turno le avisa a Vd., que es el
mdico de guardia. Cul de los siguientes comportamientos
asistenciales es correcto en el contexto clinico descrito?:
CURSO INTENSIVO MIR ASTURIAS

I. Invitar al paciente a firmar el Alta Voluntaria, previa Trastorno histrico de la personalidad.


informacin de los riesgos derivados del no tratamiento de Sndrome confusional agudo.
su condicin patolgica. Ictus en territorio de Ia arteria cerebral media derecha.
2. No iniciar ningn procedimiento diagnstico ni teraputico Inicio de demencia.
al tratarse de un problema psiquitrico. PFPNZ Angiopata amiloide.
3. Proceder a Ia suiecin mecanica del paciente para
posibilitar su sedacin mediante la administracin de
cloracepato dipotsico por va LM. 2. Sndrome amnsico onnico
4. Proceder a la suiecin mecnica del paciente para
posibilitar su sedacin mediante Ia administracin de Sndrome caracterizado por deterioro marcado de la memoria
reciente y remota conservando la memoria inmediata (Diagns-
haloperidol por va I.V. y Ia realizacin de los
procedimientos diaqnsticos y teraputicos que se estimen tico diferencial: en demencia la ltima memoria en deteriorarse
indicados.* es Ia remota).
5. Proceder a la suiecin mecanica del paciente, evitando Caractersticas:
sedacin de ningn tipo, aslndolo en una habitacin o Antecedentes o presencia obietiva de lesin o enfermedad
insonorizada. cerebral (MIR).
*ACTUALMENTE NO IV, ADMINISTRAR IM o Trastornos de Ia memoria:
o Amnesia antergrada o de fiiacin: incapacidad
MIR O (8443): Un paciente de 68 aos, hospitalizado por para aprender cosas nuevas.
fractura del cuello del fmur, asegura que es obieto de o Amnesia retrgrada o de evocacin: dificultad para
persecucin y vctima de un complot, y protesta de que la recuperar la informacin.
enfermera no le hace caso, como si estuviera de acuerdo con No afectacin de la memoria inmediata.
sus perseguidores. Ante un cuadro as, Io primero que habra No afectacin de la atencin, conciencia, ni deterioro inte-
que explorar es: Iectual.
Si hay una historia psiquitrica previa. o Puede haber: confabulaciones (invenciones que rellenan
Si el contenido del pensamiento es delirante. vacos de memoria, no hay conciencia de Io errneo o irreal
Si el paciente est orientado y es capaz de fiiar su atencin. del relato), desorientacin temporal, alteraciones emociona-
Si el paciente es capaz de autocrtica. les y falta de conciencia de enfermedad.
PPPIJI" Si el comportamiento de Ia enfermera ha sido adecuado.
2.1. Amnesia global transitoria
MIR 07 (8703): Un seor de 60 aos, empleado de comercio, o Amnesia total sbita, de varias horas de duracin y recupe-
acude a urgencias acompaado de su familia; stos cuentan racin completa (MIR). No otros signos neurolgicos. El com-
que el da anterior, de forma bastante brusca, comenz a decir portamiento es normal o casi normal, salvo repeticin de Ia
cosas raras, a no responder a Io que le preguntaban y a misma pregunta muchas veces (MIR). Durante el periodo sin-
mostrarse confuso incluso a su propio nombre. Efectivamente en tomtico no es capaz de retener nuevos recuerdos, y cuando
Ia exploracin parece no entender lo que se le pregunta, no este periodo termina no sera capaz de recordar Io sucedido.
recuerda nada de lo que ha pasado y no sabe ni el da, ni el o Ms frecuente en mediana edad y ancianos. Suele tener un
lugar en el que est. El diagnstico sera:
desencadenante fsico (sumergirse en agua fra, esfuerzo fsi-
Psicosis breve. co...) o emocional (recibir informacin desagradable, discu-
Delirium.*
siones intensas...). No son raros los antecedentes de migraa
Alzheimer. e hipertensin (MIR).
Esquizofrenia.
,UPPN. Trastorno de ansiedad.
MIR 06 (8439): Una paciente de 62 aos, con antecedentes de
leve hipertensin bien controlada, es trada a urgencias por
MIR 07 (8599) NEUROLOGIA: Acude a urgencias un paciente haber presentado en las ltimas haras, con inicio relativamente
de 80 aos que presenta desde hace 2 das alteracin del nivel
brusco, confusin mental, desorientacin temporal, preguntas
de consciencia de forma fluctuante con lenguae incoherente,
reiteratvas, e incapacidad para recordar nada de lo dicho o
agitacin y desorientacin. La familia refiere que durante Ia sucedido recientemente, sin trastornos motores, sensitivos ni de
ltima semana al paciente se le ha prescrito Iormetazepam por
Ia conciencia. Cuando es explorado, cuatro horas despus del
dificultad para conciliar el sueo. En Ia exploracin fsica el inicio, se encuentra ya mucho meior, y muestra un rendimiento
paciente se encuentra otebril con TA 140/70 y sin focalidad
cognitivo y una exploracin neurolgica normales, pero no re-
neurolgica motora ni sensitiva, con Ienguaie incoherente y
cuerda nada de Io sucedido. Probablemente ha sufrido:
agitado. La frecuencia cardiaca era rtmica a 65 lpm. CUI de
Una demencia aguda transitoria.
las afirmaciones Ie parece correcta?:
Un ataque de histeria conversiva.
I. Se trata de un paciente que ha sufrido un accidente vascular
Una intoxicacin por CO.
cerebral y debe realizarse una TAC cerebral urgente y Una amnesia qlobal transitoria.
administrar benzodiacepinas para controlar la agitacin.
.UPS'JNT' Un cuadro psictico.
2. Se trata de una clnica sugestiva de crisis comicial y debe
realizarse un electroencefalograma urgente y administrar
benzodiacepinas endovenosas.
3. Se trata de un delirium y debe retirarse el Iormetazepan y
administrar un neurolptico para controlar la aqitacin.*
4. Debera realizarse una puncin lumbar para descartar
posible etiologa infecciosa.
5. Se trata de un efecto indeseable del Iormetazepam que
desaparecer al cabo de unos das sin ser necesario retirar
medicacin.

MIR 09 (9125) NEUROLOGIA: Un paciente de 78 aos, previa-


mente sano, presenta una clnica de varias horas de evolucin
de alteracin del nivel de conciencia y de las funciones mentales
superiores, con tendencia a Ia apata y a Ia somnolencia. Tiene
trastornos de la percepcin, con algunas alucinaciones. A su
familia Io que ms les extraa es que el cuadro sea muy uc-
tuante, pues pasa de estar casi dormido a agitarse y vociferar, y
a ratos parece estar lcido. Pensaria:
lll. TRASTORNOS MENTALES ORGNICOS

DEMENCIAS CORTICALES: signos neuropsicolgicos de


3. Demencias lesin cortical (afasia, agnosia, apraxia).
o Las estructuras cerebrales profundas estan in-
3.1. Definicin demnes (se mantienen vigilia, atencin y de-
ambulacin).
Sndrome de deterioro cognitivo global, de CURSO PROGRESI-
o Pueden presentar sndromes lobulares cortica-
VO, preservando el nivel de vigilancia DD con delirium- (MIR).
les (frontal, parietal, temporal) o como una
Interferencia con el rendimiento social y laboral del individuo.
combinacin de stos:
El deterioro cognitivo global incluye afectacin de la memoria, . Alzheimer: fronto-tmporo-parietal.
orientacin y lenguaie (2MIR). ' Pick: fronto-temporal.

Frontal: alteracin de personalidad,


comportamiento social, funciones eecu-
tivas superiores (abstraccin, razn,
solucin de problemas), perseveracin,
abulia, afasia, agrafia, refleo prensin
positivo, hiperfaga, incontinencia de
esfnteres.

Parietal: apraxia, agnosia


visuoespacial, desorienta-
cin topogrfica, anosog-
nosa, sdr Gerstman.

Curso Intensivo MIR Asturias


Tem oral: amnesia, afasia
MIR 04 (8014): En la cuarta edicin del DSM-IV la demencia se progresiva con alexia y acal-
define como: culia.
1 , Una alteracin de la conciencia y de la cognicin por un
breve perodo de tiempo. DEMENCIAS SUBCORTICALES:
2. Un deterioro de mltiples funciones coanoscitivas, incluida Caractersticas: lentitud de procesamiento mental y alteraciones
la alteracin de Ia memoria, pero no de la conciencia. de la marcha. Eiemplos:
Un deterioro progresivo de la memoria que puede o Demencia por cuerpos de Lewy (DCL), demencia vascu-
desarrollarse en ausencia de otros trastornos significativos. lar, hidrocefalia normotensiva, demencia SIDA, esclero-
Un trastorno neuropsiquitrico compleio de ndole orgnica, sis mltiple y las enfermedades de Parkinson y Hunting-
que incluye deterioro, delirio y alucinaciones. ton.
La alteracin neuropsiquatrica presente en una amplia lista
de enfermedades cuyo denominador comn es la 3.5. Pruebas diagnsticas
incapacidad para la integracin familiar y social. Descartar causas tratables de demencia:
Analtica: hemograma, VSG y bioqumica (electrolitos sricos,
3.2. Prevalencia funcin heptica y renal, glucosa).
Aumenta con la edad: Hormonas tiroideas.
1% en menores de 50 aos. cido flico y vitamina 812.
5% entre los mayores de 65 aos (se duplica cada 5 aos a Serologas de Les y VIH.
partir de los 65). Cobre y ceruloglasmina: si sospechamos enfermedad de
Wilson.
3.3. Etiologa Anlisis de orina.
Todas las agresiones del cerebro, sean de causa degenerativa, Rx de trax y electrocardioarama.
txica (alcoholismo, intoxicacin por CO), metablica, traum- : enlentecimiento generalizado, con escasa especificidad.
tica, infecciosa, tumoral o vascular. Evaluacin neuropsicolgica: El test ms usado es el mini
C La causa ms comn de demencia en los pases occidenta- examen cognoscitivo. Otros: Weschler, Camdex, Camcog.
les es la Enfermedad de Alzheimer (EA), demencia degene- TAC craneal.
rativa o senil (45%). Resonancia magntica en funcin de la clnica y del TAC. Apor-
o Demencia por cuerpos de Lewy (DCL). ta datos para el diagnstico de algunas demencias vasculares
0 Demencia Vascular (DV). Cuando se asocia a otra se de- y enfermedades degenerativas (parlisis supranuclear progresi-
nomina demencia mixta. va, atrofia olivo-pontocerebelosa,. . ..)
o EA, DV, DCL y demencia frontotemporal constituyen el 95% PET Y SPECT: no se usan rutinariamente. En EA vemos hi-
de los casos. pometabolismo e hipovascularizacin tmporo-parietal bila-
En nios y venes las principales causas son tumores cerebra- teral.
les, enfermedades degenerativas (Huntington, leucodistrofia), Los estudios genticos NO estn indicados en la evaluacin
drogas, traumatismos, infecciones (SIDA, Creutzfeldt-Jakob, rutinaria de una demencia.
panencefalitis esclerosante subaguda) y alteraciones metabli-
cas (adrenoleucodistrofia, Wilson). MIR 03 (7558): Todas las siguientes pruebas complementarias
Contexto gentico: patrn Autosmco Domnante en Hunting- son recomendadas en la rutina diagnstica de la demencia,
ton y Un porcentaje de Alzheimer, Creutzfeldt-Jakob y demencia MENOS una cul es sta?:
frontotemporal, mientras que la enfermedad de Wilson se 1. Electrolitos sricos.
transmite con patrn Autosmco Recesivo. 2 Serologa Iutica.
3. TAC cerebral.
3.4. Clasificacin anatomo-clnica 4. Genotigo ApoE.*

La distincin entre demencias cortical y subcortical es compro- 5 Funcin tiroidea.


metido:
Cs?
CURSO INTENSNO MIRASTURIAS

'
MIR 08 (8964): Un hombre mayor es llevado a urgencias por la
4' Enfermedad de AIZhelmer polica municipal. Cuentan los policias que deambulaba por la
calle sin poder donde vivia. Est vestido
4 l ' Clnica explicar correctamente
aunque con zapatillas de casa y no es capaz de decur cul es su
. Demencia corcal domicilio. Se muestra algo inquieto y dice que su hiia ir a bus-
fronto-tmporo-parietal _
. Agravamiento nocturno de los sntomas (a! igual que en el carle aunque no recuerda su telfono. Es incapaz de responder
deumy a sencullas edad, el nombre de su o un
comosu hiia
Ipreguntas
No exnsten alteracnones de otro tipo. El diagnostico
FASES INICIALES:
:ZGono.
o La alteracin principal es el trastorno de memoria, que l. Trastorno psictico.
suele ser la manifestacin inicial. 2. Trastorno depresivo.
o Primero la memoria reciente con amnesia antergrada, en 3- Trastorno por abuso de alcohol.
fases avanzadas se deteriora la memoria remota. Se asocia 4. Trastorno bipolar.
con desorientacin progresiva temporal y espacial. 5. Demencia.*
Sndrome afasia-apraxia-agnosia:
Afasia: suele comenzar con dicultades nominativas y cir-
cunloquios, posteriormente se afecta la comprensin y fina-
lemente la fluencia.
o Apraxia y dificultades visuoespaciales: dificultad para reali-
zar tareas motoras secuenciales aprendidas (vestirse, co-
mer, resolver puzzles, copiar figuras geomtricas).
o Agnosia o incapacidad para reconocer e interpretar el signifi-
cado de las experiencias sensoriales:
o Visual: incapacidad para reconocer obietos o
personas.
o Anosognosia: no reconoce la enfermedad.
o
Astereognosa: incapacidad para reconocer ob-
etos mediante el tacto.
o Prosopagnosia: incapacidad para reconocer
caras.
o Somatoagnosia: incapacidad para reconocer
una parte del cuerpo como propia.
- No hay alteracin del nivel de conciencia DD con delirium-.
FASES AVANZADAS
- Alteraciones de personalidad y del comportamiento con
agresividad y agitacin o mutismo y pasividad .
- Puede haber clnica psictica con delirios (normalmente
poco organizados) y alucinaciones. Hasta el 10% sufre el
sndrome de Capgras (piensan que hay mpostores sustitu- 4"3 Hallazgos blOlOglCOS
yendo G familiares/cuidadores). NEUROQUlMlCA:
o Al final los pacientes presentan alteraciones de la marcha , _ . . . _ _
con parkinsonismo, dificultades para la deglucin y suelen ' Disminuan de la achwdad colunrqrca lh5l del 90%) por
fallecer por alteraciones nutricionales e infecciones frecuen. perdida (le neuronas del nucleo basal de Meynert que Sinteti-
temente por broncoaspiracin. zanacenlcoluna. DlsmaCIn de acetllcollna y CAT (colina
o Un lO% presenta crisis comiciales. Son frecuentes las mio- acetlltransferasal. . _
.
clonas multifocales en fases avanzadas. La angiopata ami- Otros: d'sm'mc'o'j de actrvrdad serotonrgllca, noradrenrgi-
lodea suele asociarse a hemorragias lobares corticales. ca y_de somatostatina, sustancua P, vasopresuna Y rS neuro
o El curso hasta el fallecimiento suelen ser 8-10 aos, aunque peptidos.
a veces puede ser ms rapido o incuso sobrevivir ms de ANATOMA PATOLGlCA:
20 anos. Placas y ovillos de deaeneracin neurofibrilar: ms abundantes
4..2 Factores de riesgo que en enveiecimiento normal y localizadas en zonas de asociacin
cortical. Constituidas por agregacin de la protena intraneuronal
o EDAD AVANZADA: El ms importante. tau, lo que provoca la muerte neuronal,
o Sexo femenino. Placas neurticas: placas seniles extracelulares formados por ncleo
o Bao nivel educativo. central de pptido amiloide rodeado de axones y dendritas (neu-
o Traumatismos craneales. ritas) anmalosi Menos especficas.
o Enfermedad arteriosclertica intensa. Mm en lbulos frontal y temporal (tpicamente
. Antecedentes Familiares: en ncleo basal de Meynert o sustancia innominada que provo-
. Demencia (x 3,5); ca prdida de sinopsis).
o 10% genticamente determinada (AD): APP, presenili- ,
nas l y 2. EI inicio de suele ser a edades ms venes 4--4 Pruebas complemenlnGS
' :){p'PPe E lP'El ene "es 'e's (EZ E3 Y NEUROIMAGEN: Tomoqrafa Axial Computarizada y RM
o Su obietivo principal es descartar otras patologas.
o E2 de la apo-E: efecto protector, pero curiosamente
ms amiloidosis cerebral, que se asocia a EA.
o Atrofia cortical generalizada pero de predominio sobre zonas
o de asociacin cortical (unin entre lbulos parietal y tempo-
Genotipo apo-E 3/4 o 4/4: aumenta el riesgo.
ral, y sistema lmbico).
o Down: x 2,7
o Dilatacin de ventrculos laterales (aumento de astas latera-
les) y tercer ventrculo.
LCno es necesario de rutina.
o LCR bsico normal. AB 42 disminuido, tau hiperfosforilado
aumentado.

ms
MIR
lll. TRASTORNOS MENTALES ORGANICOS

MIR 00 (6770): Un paciente de 68 aos, sin antecedentes neuro-


4.4. Tratamiento lgicos, psiquitricos ni tratamientos farmacolgicos previos,
Farmacoteragia: enlentecimiento del deterioro cognitivo progresivo presenta desde hace 8 meses deterioro mental progresivo, fluc-
y meiora de los sntomas comportamentales durante 9-12 meses. tuaciones en su nivel de atencin y rendimiento cognitivo, aluci-
o Anticolinestersicos (inhibidores de la colinesterasa). Frecuen- naciones visuales y en la exploracin neurolgica, signos parkin-
temente producen clicos abdominales por incrementar la acti- sonianos leves. Probablemente sufre una:
vidad colinrgica parasimptica (pueden producir mareos y Enfermedad del Alzheimer.
empeorar los bloqueos AV). Demencia vascular.
o Donepezilo. Demencia con cuerpos de Lewy. *
o Rivastigmina. Enfermedad de Huntington.
o Galantamina. 91.599.? Psicosis hebefrnica.
o Antagonistas de los receptores NMDA (receptores del gluta-
mato): Memantina. Meora unos meses memoria y aprendiza- MIR 'IO (9443): Un hombre de 67 aos acude a la consulta
presentando un deterioro cognitivo progresivo y de rpida
ie.
evolucin (meses) que afecta preferentemente a las funciones
o Tratamiento sintomtico
corticales prefrontales (fluencia verbal, funciones eiecutivas) y
Sntomas psicticos: antipsicticos.
parietales (funcin visual-constructiva). Tambin destaca un
Insomnio: antipsicticos o antidepresivos sedantes.
Depresin: antidepresivos ISRS. dficit en la capacidad atencional, preservacin de la capacidad
mnsica, sintomas psicticos que incluyen alucinaciones visuales
Trastornos de conducta: antipsicticos.
y olfativas e ideas delirantes secundarias, sntomas
Crisis convulsivas: anticonvulsivantes.
extrapiramidales (parkinsonismo acintico) y neurovegetativos,
Alteraciones de la deglucin: sonda nasogstrica.
alteraciones en la marcha y notable fluctuacin de la gravedad
Prevencin de lceras de decbito.
de la clnica de un da para otro. EI cuadro clnico sugiere:
PNPSJP -NT' Conseios a la familia: hacer vida rutinaria, no cambios de
Una demencia tipo Alzheimer.
domicilio...
Una demencia vascular.
Una demenciaSIDA.
MIR OI (7035): Los frmacos anticolinestersicos (donepezilo y
Una demencia por cuerpos de LM
rivastigmina) en Ia enfermedad de Alzheimer:
9.59.? Una demencia de Pick.
I. Son eficaces slo en los pacientes en estadio leve de
demencia.
2. Meioran el rendimiento coqnitivo, los defectos funcionales y 5.2. Demencia vascular
los trastornos de conducta.*
La demencia vascular (DV) debida a infartos cerebrales es la causa
3 Producen meioras escasas en todos los pacientes.
ms frecuente de demencia tras EA y DCL. Aproximadamente el
4. Deben utilizarse en Ia dosis menor posible.
20% de pacientes con ictus desarrollar demencia. Los infartos que
5 Tienen efectos secundarios cardiovasculares potencialmente
provocan demencia suelen ser infartos situados en reas crticas,
graves.
infartos mltiples de tamao grande o pequeo (lacunar) o infartos
que afectan casi exclusivamente a sustancia blanca (enfermedad
de Binswanger). Puede aparecer superpuesta a una EA (clemencia
5. Otras demencias mixta).

5.1. Demencia por cuerpos de Lewy (DCL) CARACTERSTICAS:


o SIGNOS NEUROLGICOS FOCALES: disartria, hemiparesia,
La DCL representa hasta un 30% de los casos inicialmente eti- hemianopsia, ataxia, parlisis seudobulbar, afasia.
quetados de EA. Se denomina as por la presencia de inclusiones o Ms sntomas de ansiedad y depresin.
intracitoplasmticas con positividad para los anticuerpos frente a o
Neuroimagen: presencia > lesiones isqumicas.
sinuclena a en las neuronas de la corteza (cuerpos de Lewy). Los
o La hipertensin es un fuerte factor de riesgo para Ia DV y la
cuerpos de Lewy tambin muestran agregacin en la enferme-
estabilizacin de la TA produce estabilizacin de la clinica y
dad de Parkinson (EP):
meiora funcional.
CARACTERSTICAS: o Inicio agudo y curso fluctuante, con empeoramientos bruscos y
. Alteraciones extrapiramidales leves: cara de mscara, recuperaciones parciales.
bradicinesia y alteracin de la marcha. o La escala de Hachinski tiene por obietivo distinguir entre de-
o Alucinaciones visuales desde fases iniciales (en Alzhei- mencias vascular y degenerativa.
mer y Parkinson en fases avanzadas). FORMAS ESPECIALES:
o Cambios sbitos y fluctuantes de atencin, cognicin y 0 C.A.D.A.S.I.L. (acrnimo de Arteriopatia Cerebral Autosmico
vigilia.
Dominante con Infartos Subcorticales y Leucoencefalopatla).
- Trastorno del comportamiento relacionado con el sue-
Se relaciona con el cromosoma 19 (misma regin que Ia mi-
o REM (aparece en DCL o EP): no muestran parlisis graa hemipliica familiar). Es una demencia familiar vascular
durante esta fase y presentan movimientos actuando en
con engrosamiento de la media y reduplicacin de la elstica
los sueos. interna. No presenta calcificaciones ni depsitos de amiloide.
o Disminucin de acetilcolina y CAT. Cursa con ictus de repeticin y leucoaraiosis.
o Hipersensibilidad a frmacos: 0 AMILOIDOSIS FAMILIAR: en relacin con un gen del cromo-
o Los frmacos usados en EP como L-dopa soma 20. Cursa con crisis convulsivas e ctus de repeticin.
pueden producir alucinaciones.
o Los antipsicticos producen aparicin de
MIR 97 (5327): Un enfermo con hipertensin, cefaleas, mareos,
signos extrapiramidales. con un cuadro de curso fluctuante y bruscos empeoramientos de
Diagnstico: en el EEG alteraciones marcadas con brotes intermi- incontinencia emocional, disfasia, deterioro de la memoria y
tentes de ondas lentas signos neurolgicos focales transitorios, padece muy probable-
TRATAMI ENTO: mente:
Sndrome de Korsakoff.
o Inhibidores de la colinesterasa: estabilizan Ia
funcin cognitiva y pueden reducir las alucina- Demencia de Alzheimer.
ciones durante meses. Demencia por hematoma subdural.
o BZD de accin prolongada ayudan a suprimir el Demencia multinfarto.*
trastorno del comportamiento relacionado con .UPP-NT' Sndrome de Ganseri
el sueo REM.
CURSO INTENSIVO MIR ASTURIAS

5.3. Demencia fronto-temporal (DFT)


Sndrome degenerativo que incluye la enfermedad de Pick y otros
trastornos similares. Causa de hasta 15% de demencias (propor-
cin mayor en menores de 65 aos).
CARACTERSTICAS:
o Inicio ms temprano (media de 53 aos) y curso rpido (mor-
tal en < 4 aos).
o Alteracin del lbulo frontal: trastornos del comportamien-
to, abandono de tareas, desinhibicin, irresponsabilidad,
Iogorrea y estereotipias, posteriormente mutismo. EI con-
trol de los esfnteres se deteriora. Segn avanza la enfer-
medad se pierde fluidez verbal, se descuida Ia higiene
corporal, comen vorazmente y pueden estar irritables y
agresivos:
o Afectacin de Ia parte anterior de lbulo temporal. En fa-
ses iniciales se preservan relativamente las funciones tem-
porales (memoria).
o Afectacin del arco perisilviano del Ienauaie: parafasias,
afasia anmica.
o Lbulos parietales preservados: pueden copiar un dibuio y TRIADA SINTOMTICA DE HIDROCEFALIA NORMOTENSIVA:
no se desorientan. - APRAXIA DE LA MARCHA.
o E atrofia cortical fronto-temporal. - INCONTINENCIA URINARIA.
- DEMENCIA SUBCORTICAL.

DIAGNSTICO:
o La extraccin de 30mL de LCR reduce de forma transito-
ria Ia hidrocefalia. Es la forma ms empleada porque el
resto de pruebas no son especficas. EI LCR tiene pre-
sin, protenas y glucosa normales.
- W
o TAC: dilatacin ventricular, especialmente en
astas frontales. Atrofia inexistente o mnima.
o RM: adems de los hallazgos del TC se obser-
van ngulos del cuerpo calloso mayores de
409, y vacos de seal (iet) a Ia salida del
acuaducto de Silvio.
o Cisternografa isotpica: bloqueo del contraste
en Ias cisternas de Ia base y permanencia del
contraste en los ventrculos mas de 24 horas.
TRATAMIENTO:
o Puncin lumbar evacuadora (30 ml): meiora transitoria.
t Vlvula de derivacin en ventrculo lateral con drenaie de LCR
a trax o cavidad abdominal.
o Complicaciones: hematoma subdural, obstruc-
cin o infeccin de Ia valvula.

4+1; MIR FAMILIA 00 (6530): Paciente de 58 aos operado 3 meses


mm, MIR Asturias antes de aneurisma de Ia comunicante anterior manifestado por
hemorragia subaracnoidea. EI post-operatoria inmediato fue
favorable, pero en el ltimo mes eI paciente desarrolla dificultad
5.4. Hidrocefalia normotensiva progresiva para Ia marcha, apata, torpeza mental e incontinen-
cia urinaria. CuI de las siguientes complicaciones le parece
Enfermedad de Hakin. Demencia potencialmente reversible . ms probablemente Ia responsable del deterioro neurolgico
tardo?:
ETIOLOGA: I. Hidrocefalia comunicante.*
o Idioptica: la mas frecuente y Ia de peor respuesta al trata 2. Vasoespasmo de las arterias cerebrales anteriores.
miento. r 3. Hiponatremia por secrecin inadecuada de hormona anti-
o Antecedentes de hemorragia subaracnoidea, traumatismo diurtica.
craneoencefalico o meningitis. 4. Resangrado por oclusin incompleta del saco aneurismti-
o Defecto en la reabsorcin del LCR por estenosis de acueduc- C0.
to, tumores, patologa de fosa posterior. 5. Demencia vascular.
CLiNiCA:
TRIADA SINTOMTICA: MIR 02 (7302): Cul de las siguientes consideraciones N_O es
o APRAXIA DE LA MARCHA: pies pegados al suelo, tenden cierta sobre la Hidrocefalia a Presin Norma|?:
cia a caer hacia atras sin debilidad en piernas. I. Presentacin clnica en el adulto.
. INCONTINENCIA URINARIA. 2. Triada clnica consistente en alteracin para la marcha,
o DEMENCIA SUBCORTICAL: bradipsiquia, desorientacin, demencia e incontinencia urinaria.
apata y deterioro de Ia memoria que suele evolucionar a 3. EI estudio por Resonancia Maantica Nuclear proporciona
demencia completa. datos Qatognomnicos.*
4. La etiologa es desconocida en la mayora de las ocasiones.
5. El tratamiento consiste en Ia realizacin de derivacin ventr-
culo peritoneal del lquido cefalorraqudeo.
III. TRASTORNOS MENTALES ORGANICOS

MIR 05 (8076): Hombre de 70 aos que consulto por un trastor


no de la marcha y un deterioro cognitivo subagudo. Nos indican
que el diagnstico de presuncin del paciente es hidrocefalia a
presin normal. En este coso, cul de los siguientes datos N_Q
esperara encontrar?:
l. Una hdrocefolia comunicante con acueducto de Silvio per-
meable en la resonancia cerebral.
2. Un trastorno de lo marcha tipo aprxico.
3. Un LCR con leve elevacin de la presin de apertura y con
un aumento de clulas y grotenos.*
4. La realizacin de una puncin lumbar evacuadora (30 ml de
LCR) puede meioror la marcha de! paciente.
5. Ausencia de signos de atrofia cortical cerebral.

. Demencias transmisibles Cum intensivo MIR Asmrias 2003

6.1. Entemedodes prinicos Compleios peridicos de ondas agudas.

Los prones son agentes infecciosos proteicos que carecen de 6.2. Compleio demencia-Sida
cidos nuclecos. La protena prinica (PrP) es el constituyente
predominante de los priones e interviene en sinopsis, sealiza-
CLNICA:
DEMENClA SU BCORTlCAL:
cin intercelulor y transporte del cobre. En enfermedad se pro-
o Deterioro cognitivo: enlentecimiento psquico, deterioro
ducen agregados intraneuronales txicos de isoformos alterados
de memoria y concentracin.
de PrP y la corteza cerebral adopto un aspecto vacuolar o es-
Deterioro conductual.
pongiforme.
Deterioro motor: crisis convulsivas, mioclonas, parapare-
sia espstico.
ENF. DE CREUTZFELDT-JAKOB:
Edades tempranas: 50-64 aos. DlAGNSTICO:
Mortal rpidamente: meses. o TAC/RM: atrofia cerebral con aumento de ventrculos y
Sin tratamiento. alteraciones de ganglios basales.
Demencio progresiva rpida, apata, alteraciones de LCR: pleocitosis linfocitaria leve, bandas oligoclonales,
conducto, alucinaciones, delirios. VIH.
Mioclonos, signos piromdales, extrapiramidales o cere- TRATAMIENTO:
belosos.
o TARGA: tratamiento ontirretroviral de gran actividad.
DIAGNSTICO: (Efavirenz y zidovudina pueden causar psicosis).
_|G_: compleios peridicos de ondas agudos o un pa- Antidepresivos, estimulantes, antipsicticos.
trn de supresin de descargas.
LCR: marcador proteico l43-3 en el 90%, inespecfico 7. Pseudodemencia
(tambin en encefalitis, hipoxia, tumores). Presencia de
protena tau (la misma que en EA). Sintomatologa demencial en el contexto de otros trastornos
psiquitricos, especialmente depresivos.
BlOPSIA CEREBRAL: el diagnstico definitivo es anoto-
Ms en ancianos.
mo-potologco y suele ser postmortem.
Manifestaciones clnicas de demencia subcortical.
TRANSMISIN: NO agravamiento nocturno (diagnstico diferencial con
o Espordica. demencia).
latrgena: transplante de crnea o administracin de Suele revertir al meiorar el cuadro psiquitrico
hormona de crecimiento.
Forma familiar: enfermedad de Gerstmonn-Strassler-
Scheinker (15% del total). Inicio ms temprano, herencia
AD, susceptibilidad por mutacin en gen PrP del cromo-
soma 20.
Enf. CreutzteldtJakob variante: transmisin entre espe-
cies. Afecta o gente mas oven (medio de 26 aos), pre-
domina clnica psiquitrico y sensitiva, lentitud inespeci-
fica del EEG, alteracin del ncleo tolmico pulvinar.

MIR 00 (673i): Paciente de 60 aos, sin ningn antecedente de


inters, que en un perodo de meses desarrolla un cuadro de
intenso deterioro cognitivo. En la exploracin destaco, adems
de la existencia de un sndrome rgido-ocintico, una otaxia de
la marcha y mioclonios. Ante este cuadro, qu diognstico,
entre los siguientes, estamos obligados a plantearnos en primer
lugar?:
Uno leucoencefolopoto multifocal progresiva.
Una panencefolitis esclerosante subogudo.
Una formo espordica de Creutzteldt-Jakob.*
Una otrofio multisistmica.
Uno encefalopata de Wernicke.

eau

Mili
CURSO INTENSIVO MIR ASTURIAS

MIR 01 (7137): A su consulta acude un hombre de 65 aos


acompaado por sus familiares, que refieren haber observado
en el paciente desde hace un mes una prdida sustancial en la
memoria reciente y a largo plazo. El propio enfermo se queia de
la prdida de memoria y resalta dicha discapacidad con gran
angustia, dando Ia impresin de que exagera las propias defi-
ciencias. Usted se plantea un diagnstico diferencial entre cua-
dro demencial y pseudodemencia. CuI de entre las siguientes
caractersticas clnicas le hara sospechar con mayor seguridad el
primero de los cuadros?:
i. Prdida temprana y llamativa de las relaciones sociales.
2 Aaravamiento nocturno de las deficiencias.*
3. Conservacin de la atencin y de la orientacin.
4 Conducta incongruente con a gravedad de Ia disfuncin
cognoscitva.
5. Marcadas diferencias en la realizacin de tareas de dificul-
tad similar.

MIR 'I4 (10389): Acude a la consulta un hombre de 7 aos de


edad acompaado de dos de sus hiios que comentan que su
padre viene presentando los ltimos dos meses unas prdidas de
memoria cada vez mayores. Previamente a ello pas por una
temporada en la que presentaba estado de animo baio. A Io
largo de los ltimos meses adems ha comenzado a dar un "no
se" o un "no me importa como contestacin a Ia mayora de Ias
preguntas que se le formulan al tiempo que han aumentado sus
manifestaciones de queia y malestar por sus olvidos, especial
mente por las maanas. A pesar de todo ello parece desenvol-
verse con relativa comodidad en el da a da. Uno de los si-
guientes sera el frmaco ms indicado para el tratamiento de
ste paciente, selelo.
Quetiapina.
Lamotrigina.
Tacrina.
Donepezilo.
.UFPN? Sertralina.*
III. TRASTORNOS MENTALES ORGANICOS

RESUMEN DE TRASTORNOS MENTALES ORGANICOS


T, DELlRlUM (SlNDROME CONFUSIONAL AGUDO): _
o Disfuncin cerebral aguda generalizada con deterioro fluctuante del nivel de conciencia, que habitualmente dura menos de I
mes (MIR).
o Etiologa: orgnica (enfermedades SNC, metablicas, txicas). Relacin temporal con el trastorno que lo produce y se corrige al
remitir ste (MIR).
o Clnica: deterioro fluctuante del nivel de conciencia (MIR), alucinaciones visuales (MIR), ideacin delirante transitoria (MIR), dete-
rioro de la memoria inmediata y reciente, dificultades de atencin (MIR), desorientacin tmporoespacial y personal (MIR), inver-
sin del ritmo sueo-vigilia (MIR), hipo o hiperactividad con agitacin (MIR), trastornos emocionales (MIR), signos neurolgicos in-
especficos (MIR), empeoramiento vespertino.
o Tratamiento: antipsicticos de elevada potencia. Si gran agitacin suiecin mecnica (MIR).

2. SNDROME AMNSICO ORGNICO:


o Deterioro de las memorias remota y reciente, estando conservada la inmediata. Dficit de la capacidad para aprender informa-
cin nueva (amnesia antergrada) y/o incapacidad para recordar la informacin aprendida previamente (amnesia retrgrada).
o Puede haber confabulaciones, desorientacin temporal.
o NQ trastornos de la atencin, ni de Ia conciencia, ni intelectual.
o Etiologa: orgnica (MIR).
AMNESIA GLOBAL TRANSITORIA (NEUROLOGA) (MIR): Amnesia total sbita, de varias horas de duracin y recuperacin comple-
ta. Comportamiento normal. Ms frecuente en mediana edad, ancianos, migraas e hipertensin.

3. DEMENCIAS:
o Deterioro intelectual global de curso progresivo en el que se conserva el nivel de vigilancia (conciencia) (MIR). Interferencia con
el rendimiento socio-laboral.
o Etiologa: en los pases occidentales la causa ms frecuente es la degenerativa (enfermedad de Alzheimer) (MIR) pero cualquier
agresin al cerebro: txica (alcohol, intoxicacin por CO), metablica, traumtica, carencal (812, flico), tumoral o vascular
puede producir demencia.
o Gentica: el gen de apoE (especialmente un doble aIeIo E4) es un factor predisponente en las demencias de inicio tar-
dio.
o Clnica: deterioro de la memoria: prdida paulatina y clara de Ia reciente, la inmediata puede estar conservada en los estadios
iniciales, la ltima en afectarse es la remota, deterioro de la orientacin y del lenguaie (afasia) (MIR).
o Pruebas diagnsticas: analtica sangunea: hemograma, VSG, bioqumica (iones, funcin heptica y renal, glucosa); hormonas
tiroideas; 312 y cido flico; serologas de les y VIH; Cu y ceruloplasmina si se sospecha enfermedad de Wilson; anlisis de
orina; Rx trax y ECG; EEG; evaluacin neuropsicolgica; neuroimagen: TC o RM craneal (indispensable) PET y SPECT (en oca-
siones)-.
o Los estudios genticos NO estn indicados en la evaluacin de rutina de la demencia (MIR).

4. ENFERMEDAD DE ALZHEIMER:
I Demencia cortical (fronto-temporo-parietal).
EI sntoma de inicio es la prdida de memoria: inicialmente reciente e inmediata y al final tambin Ia remota (MIR).
Desorientacin en relacin a la prdida de memoria.
Afasia: prdida progresiva del lenguaie espontneo, anomia.
Apraxia: incapacidad para realizar tareas especficas.
Agnosia: incapacidad para reconocer e interpretar el signicado de las experiencias sensoriales (MIR).
NO alteracin del nivel de conciencia.
0000000 Otros sntomas: cuadros afectivos en 2/3 de los pacientes, psicticos en el 20-30%, cambios de personalidad y trastornos
del comportamiento. El 10% crisis convulsivas.
o Agravamiento nocturno de los sntomas (MIR)
o En las demencias de inicio precoz, la evolucin es mas rpida.
o Hallazgos biolgicos:
o Neuroqumica: disminucin de la actividad colinrgica, tambin de la serotonrgica, noradrenrgica, y de somatostatina
y otros neuropptidos y aminocidos (MIR).
o AP: placas y ovillos de degeneracin neurobrilar, placas neurticas, prdida de neuronas.
o Neuroimagen: atrofia cortical generalizada, sobre todo temporal. Aumento del grosor de los surcos corticales y ventricu-
lomegalia.
Tratamiento: Frmacos que meioran la transmisin colinrgica:
Inhibidores de Ia colinesterasa (donepezilo, rivastigmina y galantamina).
o Antagonistas no competitivos de los receptores NMDA (memantina) (MIR).

5. OTROS TIPOS DE DEMENCIAS

5.1. DEMENCIA POR CUERPOS DE LEWY

o Deterioro mental progresivo (rpido), sntomas parkinsonianos leves. Se asocia con frecuencia a alucinaciones visuales o auditi-
vas, y secundariamente delirios. Puede haber dficit de atencin y asociacin con estados confusionales (MIR)
o Curso fluctuante (MIR).
'<_T:
3
Q 5.2. DEMENCIA VASCULAR

o
(f . Infartos situados en reas criticas, infartos mltiples de tamao grande o pequeo (lacunar) o infartos que afectan casi exclusivamen-

MR
CURSO INTENSIVO MIR ASTURIAS

te a sustancia blanca (enfermedad de Binswanger).


o lnicio agudo, curso fluctuante con empeoramientos bruscos y recuperaciones parciales (MIR).
o Signos neurolgicos focales trastornos de la marcha, inestabilidad en bipedestacin y cadas frecuentes, disartria, urgencia de
miccin, parlisis pseudobulbar. Frecuentemente, cambios de la personalidad y del humor (MIR).
o Poco probable en ausencia de signos neurolgicos focales y de lesiones cerebrovasculares en TC o RM.

5.3. DEMENCIA FRONTOTEMPORAL

Sndrome degenerativo que incluye la enfermedad de Pick y similares. Causa de hasta 15% de demencias (proporcin
mayor en menores de 65 aos).
Caracteristicas:
o Inicio ms temprano (media de 53 aos) y curso rapido (mortal en < 4 aos).
. Alteracin del lbulo frontal: trastornos del comportamiento, abandono de tareas, desinhibicin, irresponsabilidad,
logorrea y estereotipias, posteriormente mutismo. El control de los esfnteres se deteriora.
o Afectacin de la parte anterior de lbulo temporal.
. Afectacin del arco perisilvano del lenguaie: parafasias, afasia anmica.
o Lbulos parietales preservados.

5.4. HIDROCEFALIA NORMOTENSIVA

o Alteraciones en la macho + incontinencia urinaria + demencia subcortical llo ltimo en aparecer) (SMIR).
o Etiologa: Idioptica ms frecuentemente (peor pronstico), otras con antecedentes de dao cerebral (2MIR).
o Diagnstico: Habitualmente con PL que produce meiora transitoria (LCR presin normal), TAC, cisternografa isotpica.
o Tratamiento: puncin lumbar evacuadora (2MIR); vlvula de derivacin (MIR).

. DEMENCIAS TRASMISlBLES

6.1. ENFERMEDAD DE CREUTZFELDT-JAKOB

. Edades tempranas: 50-64 aos.


o Mortal rapidamente: meses. Sin tratamiento.
o Demencia progresiva rapida, apata, alteraciones de conducto, alucinaciones, delirios, coma.
o Mioclonas, signos piramidales, extrapiramidales o cerebelosos.
Diagnstico:
o EEG: compleios peridicos de ondas agudas o un patrn de supresin de descargas.
o LCR: marcador proteico l 4-3-3 en el 90%, inespecfco (tambin en encefalitis, hipoxia, tumores). Presencia de pro-
tena tau (la misma que en EA).
o BIOPSIA CEREBRAL: el diagnstico definitivo es anatomo-patologco y suele ser postmortem.

6.2. DEMENClA POR HIV

o Demencia subcortical por destruccin multifocal difuso de la sustancia blanca por el virus.
o Diagnstico: TAC/RM: atrofia cerebral con aumento de ventrculos y alteraciones de ganglios basales.
o LCR: pleocitosis linfocitaria leve, bandas oligoclonales, VIH.
o Tratamiento: TARGA.

7. PSEUDODEMENCIA
o Sintomatologa aparentemente demencial en el contexto de otros trastornos psiquitricos (el 75% trastornos afectivos) (2MIR).
o Sintomatologa de demencia subcortical, sin agravamiento nocturno de los sntomas (MIR)
o Mas frecuente en ancianos (MIR). Suele revertir al meiorar el cuadro psiquitrico.

l\_/l_l_R_
IV. PSlQUlATRA INFANTIL

Psiquiatra infantil
Nmero de preguntas del captulo en el MIR

lll
l l l l l l

l l l l l l I l I U I l l I l I I l l

90 91 92 93 94 95f 95 9f 96 97f 97 98f 98 99f 99 OOf 00. 01. 02. 03. 04. 05. 06. O7. 08. O9. 10 11 12 13 14

Nmero de pregunras decada lema

Traslornos generalizados del


desarrollo

Trastornos por dficit de 4


., . . .
alencnon y/o hiperachwdad

Trastornos de la eliminacin

. Imprescindible

o El rraramenro farmacolgico de eleccin en el Dficr de Arencin e hiperaclividad es el Melilfenidalo, un derivado anlieramnico.

1 . Reiraso mental
Desarrollo mental incompleto o detenido, caraclerizado por:
o Deterioro de las funciones concreias de cada poca del desarrollo (fales como las funciones cognitivas, las del lenguaie, las motri-
ces y la socializacin) que conrribuyen al nivel global de inteligencia (medido habitualmente por el cocien'le nfeleciual Cl: media de
100 y desviacin eslndar (DE) de 15; RM: 2 DE por debaio de la media <70)
o Afeclacin de lo adapiocn al ombienle o compeiencia social
NIVEL,
GRADO C'sP'E-YALENQA ' (SOCIAL ; ACADMCO 1 #0995907
CLASE
; RES'DENC'A . ECONQM'A
Y
562; 85% Baia 69 Educable Comunidad Puede irabaiar

Enirenable de manelar
j . 10% Menos baia 29 Medio supervisado Sapaz
. nero para gasfos
Distribucin Muy eslruclurado con Puede utilizar mo-
-> o
3-4> <1 o . . ., .
. normal Dependiente supemston completa nedas y billetes

Dislribucn Muy esirucrurado con Dependencia para


130/o (19 Custodiado
normal supervisin complela manejar dinero
(53
a
D. CAPACIDAD INTELECTUAL LMITE: Cl de 70 a 84
CURSO INTENSIVO MIRASTURIAS

.- 1
I .i . Epidemwloglo - AUT'SMO Sindrome SDR. Trastorno
de Heller ASPERGER de Rett
o 3 2 >
Prevalencia:_l% Edad inicio >_ > 2 aos
o Edad de Inico: antes de los 18 aos. anos anos meses
o 2 VARONES: I MUJER Sexo
o Frecuencia en cuanto a etiologa: (>frecuencia) V arones varon es VG rones n'
l GS
o De causa desconocida: el 75%, suele ser leve (Cl: 50-
Lenguaie Sl Sl No Sl
9l' _ , afectado
o Sindrome Down (trisomia ).
de del.2'l Retraso mental S Si No S
o Tasa de mortalidad elevada (especnalmente en eI grave y
profundo) por las complicaciones de las enfermedades fsi- I nteraccron.
socral deterio- , , ,
cas asociadas i Si Si Si Si
rada
1.2. CllnICO Conductas
Sl Sl Sl Sl
Comorbildad: influye negativamente en eI pronstico restringidas y
repetitivas
o 30-70% al menos otro trastorno mental y del comporta-
miento:
o Lo ms comn: constelacin de sntomas (impulsividad, 2.1. Autismo infantil (Leo Kanner,
irritabilidad, hiperactividad, frustracin, problemas de
(2MRJ I943)
atencin, conductas agresivas, movimientos estereoti-
pados). Desde nios Se trata de un grupo hetereogneo.
o Las esterotipias son movimientos repetitivos, rtmicos, en o 70% con Retraso Mental" retardados sin Ienguaie, absortos
apariencia involuntarios, y que no parecen seguir un enla autoestimulacin.
propsito definido. Pueden interrumpirse con Ia distrac- o 30% con CI normal: Con inteligencia normal, se implican
cin del paciente. Son formas frecuentes los balanceos, en rituales o conductas estructuradas:
aleteos con las manos, morderse en los labios, pero o Autismos de alto funcionamiento: tienen dificulta-
tambin conductas de autogolpeos como cabezazos re- des en Ienguqe,
petidos o autoabofetearse o Sndrome de Asperger: lenguaie normal.
o Hasta en el 25%: depresin, ansiedad. ,
o 10%: trastorno por dficit de atencin con hiperactivi- A' EPlDEMlOlGlA
dad (TDAH). 0 PrevalenCIa: OCZ-0.05%.
o 5%: autismo (pero el 80% de los autistas tienen algn 3'5 VARONES: l MUJER (ZMIRI
de retraso o Clases socnales altas.
grado mental).
B. ETIOLOGA
o Factores genticos (MIR) y biolgicos (trastornos del neuro-
desarrollo, retraso mental, epilepsia, factores nmunolgicos
y bioqumicos).
o Factores psicosociales (estrs).
c. CLNICA
Por definicin ANTES DE LOS 36 MESES.
o Inicio posterior: autismo atpico.
o TRASTORNO DEL DESARROLLO DE IA INTERACCIN
SOCIAL:
o Alteracin del uso de comportamientos no verbales
(contacto visual, expresin facial, gestos, posturas).
o Incapacidad para desarrollar relaciones con compae-
ros.
o Ausencia de tendencia espontnea de compartir con
otros intereses, objetivos.
o Falta de reciprocidad social o emocional.
o TRASTORNO DEL DESARROLLO DE LA COMUNICACIN:
MIR '|4 (10387)- Se denomina es1eropg a o Retraso o ausencia total del desarrollo del Ienguaie
I. Gestos bucolinguales extraos. 0F0l- . . _ .
2_ Repecin reiterada e innecesaria de un acto,* o Falta de capaCIdad para InICIar o mantener una conver-
3. Movimientos muy aparatosos que aumentan la expresividad SGCIn- ' .. . .
de los gestos. o Uso estereotipado y repetitivo del Ienguale, lenguae
4, Agitqciones psiccgs infensgs_ IdIOSInCFSICO, ecolalia, inversin de IOS pronombres (YO
muy
5 Gestos aparatosos acompaados de palabras soeces. 1l-
o Sndrome de Asperger: Meior lenguaje expresivo que
2. Trastornos generalizados del receptivo (MIR).
o TRASTORNO DEL DESARROLLO DEL COMPORTAMIENTO,
desarrollo INTERESES Y ACTIVIDADES:
o Adhesin inflexible a rutinas o rituales especficos.
FW: lO'ZO/l 0000 lnlldUOS- o Manierismos motores estereotipados y repetitivos.
o Preocupacin persistente por partes de obietos (MIR).
CRITERIOS DIAGNSTICOS GENERALES: Otros: cambios de humor repentinos, hipercinesia, agresividad,
DflCll comunicaaon Y Ienguale. autolesiones (umbral doloroso disminuido), habilidades aisladas
' DGIICII de hablllddes SOCIOleS- (autistas sabios: musicales, memoria repetitiva, clculo, hiper-
o Conductas restringidas y/o repetitivas. Iexia), insomnio, enuresis, RM). <_<
FE
D. TRATAMIENTO: 5
o Meiora de la conducta social y comunicacin. g
.
o Multidisciplinar (neuropediatra, psiquiatra):
(est
MIR G
IV. PSIQUIATRA INFANTIL

o Terapia conductual y familiar. aos de edad


o Sintomtico: o Inicio en Ia adolescencia: ningn criterio diagnstico
' Antipsicticos para estereotpias. antes de los 'IO aos
. ISRS para compulsiones.
3.3. Tratamiento
FARMACOLGICO: control de los sntomas nucleares (MIR)
Aislado del resto, no se
interesa en Io que hacen I ESTIMULANTES: DE ELECCIN Y PREVIA REALIZACIN DE
PRUEBAS PERTINENTES (Exploracin fsica, tensin arterial,
pulso, peso y talla).
METILFENIDATO (derivado anfetamnico).
- Efectos secundarios: retraso del crecimiento,
disminucin del apetito, dificultades de concilia
cin del sueo.
o Antidepresivos:
o ATOMOXEI'INA: puede ser de primera eleccin.
Antidepresivo inhibidor de recaptacin de nara-
drenaIina.
o Otros: ISRS, tricclicos.

PSICOTERAPIA: cognitivo-conductual, familiar, APOYO ESCO-


IAR.
Se fiian en partes
concretas de los obietos, MIR FAMILIA 00 (6630): Qu grupo de psicofrmacos estaran
especialmente redondas contraindicados en el tratamiento de un trastorno por dficit de
(las ruedas de los coches, atencin?:
Curso Intensiva MIR Asturias 2006 el tambor de Ia lavadora)
Antidepresivos.
Barbitricos.*
Anticonvulsivantes.
MIR 03 (7661): En relacin con el autismo, cul de las siguien-
Estimulantes.
tes afirmaciones M es correcta32
.UWPJNT' Antipsicticos.
'l. Es frecuente el retraso en el desarrollo intelectual.
2 Hay una interaccin social anmala y restringida.
MIR 02 (7397): Acuden a la consulta unos padres con su hiio de
3. Ms frecuente en sexo masculino.
8 aos. Tanto ellos como los profesores estn preocupados
4 Es habitual Ia fiiacin a obietos.
porque el nio es muy inquieto, no para en clase, siempre est
5 Son frecuentes las conductas desafiantes y provocativas.*
molestando y no se centra ni un momento en sus tareas. Se
diagnostica trastorno por dficit de atencin con hiperactividad.
3. Trastorno por dficit de atencin En Io referente a los psicofrmacos cul sera su eleccinZ:
Un ansioltico de vida media corta.
con hiperactividad (TDAH) Un derivado anfetamnico.*
Un neurolptico incisivo.
Un antidepresivo.
3.1. Epidemiologa PPS-0.? Un antiepilptico.
o Prevalencia: 2-5%
MIR 10 (9439): Tras diagnosticar un trastorno dficit de atencin
o Edad inicio: antes de los 3 aos (no suele identificarse hasta e hiperactividad (TDAH) segn criterios del DSM-IV y antes de
Ia edad escolar) iniciar tratamiento con psicoestimulantes Ia American Academy
o Predisponentes: of Child and Adolescent Psychiatry propone la realizacin de:
o Varones 3-5: 'I muier. I. Electrocardiograma, electroencefalograma TAC. hemAtime-
o Grupos sociales econmicamente baios, orfelinatos, tra y bioqumica.
hacinamiento, familias numerosas. 2. Exploracin fsica, tensin arterial, pulso, peso y talla.*
o FACTORES BIOLGICOS: enfermedades durante em- 3. Exploracin fsica, tensin arterial, pulso y EKG.
barazo, hipermadurez al nacimiento, enfermedades de 4. Exploracin fsica, pulso, hematimetra y bioqumica
primera infancia, consumo de OH/tabaco de Ia madre, hemtica.
FACTORES GENETICOS, complicaciones perinatales. 5. Talla, peso. EKG y EEG.
3.2. Clnica
MIR H (969i) Seale cual de los siguientes tratamientos est
- PRIMERA INFANCIA: A partir de 3 aos puede establecerse indicado para el trastorno por dficit de atencin con hiperacti-
diagnstico. vidad:
o Extraordinariamente sensibles a los estmulos, duermen Fluoxetina.
poco y lloran mucho. Haloperidol.
0 EDAD ESCOLAR (sntomas al menos meses): Sertralina
o HIPERACTIVIDAD (signo mas frecuente). Metilfenidato".
o Dficit de atencin. PPPNT Clozapina.
o Impulsividad.
o Otros: signos neurolgicas menores dficit de coordi-
nacin, deterioro en Ia percepcin motora-, Iabilidad
emocional, baia autoestima, baio umbral a Ia frustra-
cin, dificultades escolares de aprendizaie y de com-
portamiento-, conducta agresiva y desafiante en el 75%
SUBTIPOS
o COMBINADO.
. Predominio de DFICIT DE ATENCIN.
o Predominio de HIPERACTIVIDAD.
o En funcin dela edad de inicio
o Inicio infantil: algn criterio diagnstico antes de los IO
9522?

MB,
CURSO INTENSIVO MIR ASTURIAS

4. Trasrornos de ia eliminacin
La secuencia normal del desarrollo del conTroI del intesno y
veiigo es FN FD VD VN.
1. Connencia fecal nodurna.
2. Continencia fecal diurna.
3. Control de veiiga diurno.
4. Confrol de veiiga nocfurno.
EPIDEMIOLOGlA:
o La prevalencia disminuye con la edad.
CURSO:
o En la mayora de los CGSOS remisin espontnea en Ia ado-
Iescenca.
o Duracin de al menos 3 meses.

ENURESIS
Nocturna: > 5 aos
EDAD
Diurna: > 4 aos
Varones
Sexo .5. frecuencia . . .
( ) DIurno y secundarla: muleres
Subiipos Diurno menos frecuenTe que nocturna

TRATAMIENTO C.'d9' (piP'smm' .


o lmIpramIna: ecaz pero ToleranCIa.

ENCOPRESIS
EDAD > 4 aos
CriIerio temporal Durante ms de 3-6 meses
Sexo (_> frecuencia) Varones
Subfipos Durna + frecuenTe que nocTurna
o Conduduol (hblfos evacuadores).
TRATAMIENTO o Impramna.

Curso Intensivo MIR Astun


V. ESQUIZOFRENIA Y TRASTORNOS POR IDEAS DELIRANTES

Esquizofrenia, paranoia y otras psicosis


Nmero de preguntas del captulo en el MIR

90 91 92 93 94 95f 95 9f 96 97f 97 9Bf 9B 99f 99 OOf OO. Oi. O2. 03. O4. 05. 06. O7. 08. O9. 10 H 12 13 14

Nmero de preguntas de cada tema

Esquizofrenia 32

Trastorno por ideas


delirantes persistentes - 9
(Paranoia)

Trastorno esquizoafectivo

Otros trastornos psicticos 1

(5) lmprescindible
I Tema muy importante con varios conceptos muy rentables y repetidos:
c La prevalencia de la esguizofrenia esta en torno al 1%. Su etiologa es multifactorial, hay factores genticos y ambientales. No hay
un sntoma patognomnico, presentan una combinacin de sntomas positivos (delirios, alucinacioneSWQj' y negativos (aplana
miento afectivo, abulia, apata) (AMIR).
o EI subtipo paranoide es el ms frecuente y predominan las ideas delirantes y las alucinaciones. El subtipo'herbefrnico o desor-
gan-izada es una forma de peor pronstico, con lenguaie y comportamiento muy desorganizados. L'a forma catatnica es ms in-
frecuente, se presenta con importantes sintomas pscomotores (catatoma, rigidez crea, ecolalia y ecopraxiaw) y puede precisar
terapia electroconvulsiva.
I Son de peor pronstico el sexo masculino, el inicio precoz, los sntomas negativos, las formas familiaries y el subtipo herbefrnico,V
asi como un mal auste premrbido [4MIR].
. El tratamiento de eleccin son los antipsicticos atpicos (BMIR). Como hemos visto en el tema de Farmacologa los efectos adver-
sos ms importantes de los antipscticos son los extrapiramidales: Parkinsonismo, distona aguda y discinesias tardas.
o EI trastorno por ideas delirantes (paranoia) suele aparecer de forma mas tarda que la esquizofrenia. Las ideas delirantes suelen
ser menos extraas que en la esquizofrenia, y el funcionamiento de la persona suele ser aceptable fuera del sistema delirante. No
suele haber alucinaciones. Tambin se trata con antipsicticos atpicos (6MIR).
CURSO INTENSIVO MIRASTURIAS

o Concordancia entre gemelos dicigotos: 12%.


FACTORES BIOLGICOS:
Sndrome caracterizado por distorsiones fundamentales y Cualquier alteracin del neurodesarrollo puede elevar Ia suscep-
tpicas de Ia percepcin aIucinaciones-, del pensamiento tibilidad a padecer esquizofrenia o una enfermedad psictica.
delirios-, emociones afectividad inapropiada o embota- Los cambios cerebrales que se producen en los primeros episo-
miento afectivo- y cognicin. Se inicia en Ia adolescencia o dios psicticos parecen tener un efecto neurodegenerativo.
inicio de Ia adultez. Neurotransmisores implicados: DA, 5HT, NA, GABA, gluta-
mato.
Curso crnico.
0 Produce incapacidad personal, laboral y/o socio-familiar. FACTORES AMBIENTALES:
NuIa o escasa conciencia de enfermedad. Infecciones durante el embarazo, complicaciones obstetri-
La caracterstica mas especfica del sndrome esauizofrnico es Ia cas, (parece haber un mayor riesgo en nacimientos en pri-
incapacidad para distinguir entre Ia realidad interior v exterior mavera-invierno).
MI.- Migracin.
1.1. Epidemiologa Ms riesgo en areas urbanas que rurales.
Consumo de sustancias estupefacientes por Ia madre duran-
o Prevalencia a Io largo de Ia vida: O.5-I%. te embarazo.
Incidencia: 15-20/1 00.000 habitantes/ao Clase socioeconmica baia.
Edad inicio pico: 1525 varones; 25-35 en muieres.
a Cuando se inicia despus de los 45 aos se habla de
1.3. Clnica
esquizofrenia de comienzo tardo. NO HAY SNTOMAS PATOGNOMNICOS.
Por sexo: La esquizofrenia es un sndrome cuyo diagnstico se hace por el
'Aunque histricamente se ha defendido una incidencia por clnico en base a Ia presencia de sntomas POSITIVOS, NEGATI-
sexos igual, actualmente sabemos que es algo mas frecuen- VOS, COGNITIVOS y otros.
te en varones (1,4:1). SNTOMAS Posmvos:
En varones: edad de inicio ms precoz, habitualmente
Alteracin o aparicin de una funcin que no existe en normali
peor funcionamiento premrbido y mayor probabilidad
dad.
de sntomas negativos. Todo ello conlleva un peor
o DELIRIOS (ideacin delirante): alteraciones del contenido
pronstico.
del pensamiento. Mas estructurado pero irreductible a Ia ar-
gumentacin lgica. Tpicamente de persecucin.
MIR 00 (6729) Segn Ia mayora de los estudios epidemiolgi-
PSEUDOALUCINACIONES y ALUCINACIONES: percepcin
cos, Ia prevalencia de Ia esquizofrenia en Ia poblacin general
sensorial falsa (sin obieto real externo) En Ia esquizofrenia
en todas las culturas es de:
tpicamente auditivos (pero puede haberlas de cualquier ti-
1 por 100000 habitantes.
po).
1 por IOO habitantes*
LENGUAJE DESORGANIZADO: alteracin del pensamiento
Hay variabilidad dependiendo de factores climticos.
y aparicin de descarrilamiento, tangencialidad, incoheren-
5 por 100000 habitantes.
cia o neologismos en el discurso.
.UPSNT' Hay variabilidad dependiendo de Ia clase social.
AFECTO INAPROPIADO: risas inmotivadas, incoercibles o
vacas, expresin facial risuea, ensoacin.
MIR 09 (9208): Respecto a la esquizofrenia, seale Ia respuesta
FALSA: COMPORTAMIENTO CATATNICO O DESORGANIZA-
'I. En Estados Unidos Ia prevalencia de esquizofrenia a Io DO.
largo de Ia vida es de alrededor del 1%. 0 Catatona: estupor, mutismo, exitacin, negativismo, ri-
2. La esquizofrenia tiene igual prevalencia en varones que en gidez, catalepsia, flexibilidad crea, obediencia auto-
muieres. mtica, ecopraxia, ecolalia (MIR) y otras estereotipias y
3. La evolucin de Ia enfermedad es similar en hombres aue manierismos.
en mu'eres.* ' o Comportamiento: desorganizacin, desinhibicin.
4. Segn el DSM-IV-TR Ia incidencia anual de esquizofrenia SNTOMAS NEGATIVOS:
vara de 0,5 a 5,0 por 100.000. Sntomas negativos o deficitarios: dficit de una cualidad previa.
5. Las tasas de incidencia y prevalencia son aproximadamente Pueden ser primarios (propios de Ia esquizofrenia) o secundarios
iguales en todo el mundo. (debidos al tratamiento antipsictico).
o ApIanamiento o embotamiento afectivo: inmovildad, falta de
MIR IO (9437): Seale cual de las siguientes aseveraciones es respuesta en Ia expresin facial del suieto, contacto visual pobre ,
CORRECTA en relacin con el suicidio en Ia esquizofrenia: reduccin del Ienguaie corporal, actitud ausente.
I. EI porcentaie de personas con esquizofrenia que fallece por Alogia: pobreza del contenido del pensamiento y, conse-
suicidio alcanza el 5%. cuentemente, del len'guae.
2. La presencia de deterioro cognoscitivo se asocia con un Abulia y apata: falta de motivacin para la realizacin de
menor riesgo de suicidio. actividades e incapacidad para iniciar y persistir en activida-
3. EI riesgo de suicidio es mayor en las personas de edad des dirigidas a un fin.
media con varios aos de evolucin de la enfermedad. Otros: disminucin de Ia atencin, anhedona, aislamiento
4. EI riesgo de suicidio es mayor en muieres. sodal, falta de autocuidados.
5 El riesgo de suicidio es mayor en las personas procedentes
de un estatus socio-econmico baio. OTROS SNTOMAS:
ANUIADA o Disfria, depresin, Ansiedad.
Alteraciones del ritmo sueo-vigilia.
I .2. Etiopatogenia Alteraciones de la actividad psicomotora y signos neurolgi-
cos menores: alteraciones de Ia marcha, balanceo, estereo
MULTIFACTORIAL: factores externos actan sobre una suscepti- tipias.
bilidad gentica. No existe un nico gen responsable, sino mlti- Falta de conciencia de enfermedad (falta de insght).
ples genes (que de forma individual tienen escasa importancia). Abuso de alcohol, tabaco y drogas.
FACTORES GENTICOS: Intentos de autolisis o suicidio consumado.
Si padre o madre con esquizofrenia la prevalencia es del
10%.
Ambos (padre y madre) con esquizofrenia: 40%.
Concordancia entre gemelos monocigotos: 47%.

MIR
V. ESQUIZOFRENIA Y TRASTORNOS POR IDEAS DELIRANTES

MIR 91 (2890): Para el diagnstico de esquizofrenia se requiere MIR 07 (8696): Uno de los siguientes sntomas caractersticos del
forzosamente: sndrome esquizofrnico puede considerarse un sntoma negati-
I . Afectividad embotada. VO:

2. gm se produzca durante su curso un deterioro en el aiuste Anhedonia.


social y laboral del paciente.* Alogia.
3. Que Ia alteracin no se prolongue ms all de seis meses. Frialdad emocional.
4 La presencia de dos episodios con recuperacin total entre ellos Paralogia.*
de al menos dos meses. PMRFN." Abulia.
5- Que el paciente tenga una personalidad previa de tipo esqui-
zoide MIR 07 (8702): La caracterstica mas especfica del sndrome esqui-
zofrnico es:
MIR 99 (6363): Un paciente diagnosticado de un primer episodio de I. La existencia de trastornos de la percepcin.
un trastorno esquizofrenforme presenta una buena respuesta al 2. La incapacidad para distinquir entre la realidad interior y la
tratamiento psicofarmacolgico con antipsicticos en el plazo de exterior?k
un mes, obietivndose una remisin total de la sintomatologa 3. EI marcado simbolismo que expresa su conducta.
psictico. Llegados a este punto, la actitud ms recomendable, entre 4. La hipersensibilidad.
las siguientes, es: 5. La incongruencia del pensamiento.
I , Suspender el tratamiento psicofarmacolgico puesto que el
episodio ha cedido y el riesgo de recidiva es baio. MIR 08 (8963): Seale cul de los siguientes sntomas N_O pertene
2. Suspender el tratamiento psicofarmacolgico e iniciar un trata ce a los clasificados como sntomas negativos en la esquizofrenia:
miento psicoteraputico especifico que minimice el riesgo de re- Aplanamiento afectivo.
cidiva a medio o largo plazo. Alogia.
3. Comportamiento desorqanizado.*
Buscar la mnima dosis eficaz de antimStico que mantenaa al
Disminucin de la atencin.
paciente asintomtico v mantener el tratamiento durante un m-
nimo de un ao, momento en el cual se puede plantear su inte- f-"PFNT Abulia.
rrupcin gradual.*
4. Buscar la mnima dosis eficaz de antipsictico que mantenga al MIR 09 (9212): La ecolalia y la ecopraxia se observan en:
El sndrome catatnico.*
paciente asintomtico y mantener el tratamiento de forma nde
El sndrome melanclico.
finida, puesto que la esquizofrenia es una enfermedad crnica y
muy desestructurante. Las neoplasias frontales.
Los estados confusionales.
5. Dados los efectos negativos a largo plazo del uso de antipsic-
.U'PPNT El estrs postraumtico.
ticos (Ei. discinesia tarda) conviene sustituirlos en cuanto sea
posible por benzodiacepinas de vida media larga que manten-
gan el efecto ansioltico deforma indefinida. MIR 14 (10390): En el transcurso de la entrevista de un paciente,
usted cae en la cuenta de que no esta entendiendo lo que el
MIR 00 (6728) Un paciente de 23 aos acude al servicio de urgen- paciente le dice. Decide centrar su atencin en el discurso y se
cias de un hospital general acompaado por sus padres por presen- da cuenta de que ste no tiene una idea directriz a pesar de que
tar, tras su vuelta de un viae a Inglaterra, un cuadro de 3 das de fragmentos concretos del mismo resultan comprensibles. Esta
evolucin de insomnio, irritabilidad, confusin, agitacin psicomo- alteracin del Ienguaie-pensamiento, tpica por otro lado de la
triz, comentarios delirantes de contenido mstico-religioso, persecu- esquizofrenia, es lo que en psicopatologa se conoce como:
torio, sexual, megalomanaco y alucinaciones visuales y auditivos. Disociacin del pensamiento.
Como antecedentes personales relevantes destacan una apendice- Fuga de ideas.
toma a los I7 aos y ocasional consumo de alcohol y de hachs. Se Lenguaie perseverante.
trata de una persona con buen aiuste psicosocial previo, as como Desorqanzacin del pensamiento.*
de un buen estudiante que cursa el 49 curso de ingeniero aeronuti- 91:.957' Bloqueo del pensamiento.
co. No constan antecedentes personales ni familiares de tipo psi-
quitrico. ante esta situacin, una de las siguientes afirmaciones
es correcta:
I .4. Criterios diagnsticos
I. Estamos ante un primer episodio psictico. 0 SINTOMAS CARACTERSTICOS: 22 durante al menos un
2. A partir de este momento conviene efectuar en los das/semanas mes (basta I si es delirio extrao o alucinaciones):
siguientes un despistaie de patologa orgnica concurrente, in- o IDEAS DELIRANTES.
cluyendo el consumo de txicos.
o ALUCINACIONES.
3. A tal efecto es conveniente efectuar de forma protocolizada,
o Lenguaie desorganizado (descarrilamiento, tangenciali-
entre otras, una exploracin fsica, neurolgica, analtica, neu-
dad, incoherencia o neologsmos).
ropsicolgica, as como las pruebas pertinentes de neuroimgen
o Comportamiento catatnico o desorganizado.
estructural del tipo tomografa axial computerizada.
Sntomas negativos (aplanamiento afectivo, alogia,
4. El siguiente paso es informar a los padres de que su hiio pade- '
ce una esguizofrenia paranoide.*
abulia, otros).
5. Se debe iniciar un tratamiento sintomtico de tipo farmacolgico 0 Disfuncin sociolaboral.
a base de antipsicticos y/o ansiolticos. 0 Duracin: signos de la alteracin persisten al menos me-
ses entre prdromos, fase activa y fase residual.
MIR 02 (7394): Cuales son los trastornos perceptivos, de entre los 0 Descartar trastornos esquizoafectivo y trastorno del humor
que se enumeran, ms caractersticos de las esquizofrenias? con sntomas psicticos.
Ilusiones hipnaggicas. 0 No debido a los efectos fisiolgicos directos de una sustan-
Alucinaciones visuales zoomrficas. cia (drogas, frmacos) o de una enfermedad somtica
Alucinaciones o pseudoalucinaciones auditivos"
Paraeidolias.
PPS??? Alucinosis auditivos. 1.5. Subtipos de esquizofrenia
ESQUIZOFRENIA PARANOIDE:
MIR 05 (8177): Muier de 58 aos, vive sola, con antecedentes de
HTA y artritis reumatoide, con buen control farmacolgico. Presenta o La ms frecuente.
alucinaciones auditivos y cenestsicas, con ideas delirantes de o Inicio ms tardo.
periuicio con los vecinos y de contenido mstico-religioso de 4 meses 0 Meior pronstico.
de evolucin. CuI sera el diagnstico ms probable3: Predominio de ideas delirantes persecutorias y pseudoaluci-
Demencia frontal. naciones y alucinaciones (sobre todo auditivos).
Sndrome confusional agudo.
Depresin delirante,
Esguizofrenia de inicio tardo.*
PFPNT Psicosis psicgena.
CURso INTENSIVO MIR ASTURIAS

les hasta los finales, caracterizados por sntomas negativos y


deterioro sin sntomas positivos.
ESQUIZOFRENIA INDIFERENCIADA:
Cuando no cumple criterios de los subtipos anteriores.

1.6. Estudios complementarios


Recordad: el diagnstico es clnico. La presencia de alteraciones
estructurales es de peor pronstico.

NEUROIMAGEN:
o Estructural (TAC, RM):
o Aumento de ventrculos laterales y 39 ventrculo.
o Materia gris reducida, sobre todo en lbulo tempo-
ral.
o Menor peso cerebral: -%.
o Atrofia cortical prefrontal y temporal (compleio
Curso lnten.
amigdala-hipocampo).
ESQUIZOFRENIA HEBEFRNICA O DESORGANIZADA: o Ensanchamiento de surcos corticales.
o Inicio precoz. o Disminucin del tamao de amgdala, hipocampo
o Peor pronstico. y circunvolucin parahipocmpica.
o Inicio insidioso y curso continuo. o Funcional (SPECT, PET):
o Suele haber habido personalidad premrbida tmida y soli- o Hipofrontalidad: disminucin relativa del metabo-
taria. lismo y/o fluio sanguneo cerebral regional del cr-
o Comportamiento desorganizado. tex prefrontal.
o Afectividad superficial e inapropiada (risas inmotvadas, o Menor integridad de materia blanca en tractos
desinhbicin, burlas). frontotemporales.
o Ganglios basales: disfuncin pero pocos cambios
o Pensamiento desorganizado.
estructurales.
PRUEBAS NEUROPSICOLGICAS:
o Dificultades en las funciones eiecutivas: capacidad de resol-
ver problemas (por hipofrontalidad).
o Dificultades en la memoria de trabao y peor atencin sos-
tenida.
EXPLORACIN NEUROLGICA:
Signos neurolgicos menores (MIR):
o Coordinacin pobre.
o Confusin derecha-izquierda.
o Movimientos en espeio.
Neurofisiologa:
0 Sueo: disminucin de la latencia del REM y del tiempo total
de sueo.
Curso Intensivo M . - urias 2005
MIR 99 (6367): CuI de los siguientes sntomas es tpico de
MIR 04 (7917): En la esquizofrenia desorganizada, una de las Ia esquizofrenia paranoide?:
siguientes afirmaciones es INCORRECTA: Ideas delirantes de tipo persecutorio.
I . Es sinnimo de hebefrenia. Respuestas paradjicas.
2. Tiene meior pronstico a Iarqo plazo que Ia esquizofrenia Alucinaciones auditivas en forma de voces.
paranoide.* AmbicIencia afectiva.
3 Existe desorganizacin en el lenguaje. S-FFAN? Alucinaciones visuales en forma de microzoopsias.*
4. La afectividad est alterada y bsicamente es aplanada.
5 Hay ideas delirantes fragmentadas y poco sistematizadas. MIR 08 (8962): En la exploracin fsica del paciente con esquizo
frenia es frecuente encontrar:
ESQUIZOFRENIA CATATNICA: Anomalas endocrinas.
Siqnos neurolqicos menores.*
0 Muy infrecuente.
Alteraciones cardiolgicas.
La de mayor sustrato biolgico. Alteraciones inmunitariass
Terapia electroconvulsiva es eficaz. PIPWNT' Dermopatas crnicas.
Sntomas pscomotores graves: estupor, excitacin, catalep-
sia, negativismo, rigidez, flexibilidad crea y obediencia au- MIR II (9684) Referido a las formas clnicas de Ia esquizofrenia
tomtica. cul de las siguientes afirmaciones es CIERTA?:
0 Ecolalia, ecopraxia. I. La forma CATATONICA se presenta con sintomatologa
pobre, predominio de delirios y suele ser de inicio tempra-
ESQUIZOFRENIA SIMPLE: no.
No muy frecuente. 2. En la forma HEBEFRNICA existe predominio de alteracio-
o Desarrollo insidioso y progresivo. nes psicomotrices, negativismo y conductas esteriotipadas.
o Incapacidad para satisfacer las demandas de Ia vida social y 3. La presencia de estados de exaltacin anmica, maniforme
disminucin del rendimiento. con extravagancias es propia de las formas SIMPLES.
o Escasa respuesta emocional. 4. La forma PARANOIDE presenta predominantemente trastor-
o Sin sntomas positivos claros. nos del pensamiento y trastornos sensoperceptivos.* S
5. Los sintomas obsesivos y compulsivos asociados a disgrega-
ESQUIZOFRENIA RESIDUAL: cin progresiva son habituales en la forma ESQUIZOTPICA.
S
3
Estado crnico final del curso de la esquizofrenia, en el que se Q
cn
ha producido una evolucin progresiva desde los estadios inicia- D.
V. ESQUIZOFRENIAY TRASTORNOS POR IDEAS DELIRANTES

1.7. Pronstico de la esquizofrenia (MIR)


MEJOR PRONSTICO PEOR PRONOSTICO
Buen aiuste premrbido (social, laboral). AJUSTE PREMRBIDO POBRE: El ms importante pero NO
Edad de inicio ms avanzada. modificable.
Sexo femenino. Edad de inicio precoz.
Subtipo paranoide. Sexo masculino.
Factores precipitantes obvios. No existencia de factores precipitantes.
Inicio agudo. Inicio nsidioso.
Sntomas de trastornos del humor. Comportamiento autista, retramiento social.
Casado. Buen soporte sociofamiliar. Soltera, viudedad. Escaso soporte sociofamiliar.
Historia familiar de trastornos del humor. Historia familiar de esquizofrenia.
o Sntomas positivos. Sntomas negativos.
. ADHERENCIA Y CUMPLIMIENTO TERAPETICO: importante y Signos y sntomas neurolgicos.
modificable. Historia de trauma perinatal.
Larga duracin sntomas.
Duracin de psicosis no tratada prolongada.
Muchas recadas.
Historia de abusos, agresiones.
TRATAMIENTO PSICOSOCIAL:
MIR 03 (7655): Todas las siguientes son caractersticas asocia- o Entrenamiento en aptitudes sociales.
das a mal pronstico en la esquizofrenia EXCEPTO una, sele- Tratamiento cognitivo-conductual: entrenamiento en resolu-
cin de problemas, meiora de las distorsiones cognitivas
Existencia de un factor estresante precipitante.* o Terapia familiar.
Inicio progresivo. o Psicoeducacin al paciente y familias.
Antecedentes familiares de esquizofrenia. Tratamientos Asertivos Comunitarios: tratamiento intensivo
Soltero. de los casos graves.
.UFPNT'FT Signos y sntomas neurolgicos presentes.
MIR 93 (3521): Cul de los siguientes sntomas del paciente
1.8. Tratamiento esquizofrnico meiora ms con neurolpticos?:
M"
Ante falta de respuesta al tratamiento o recada, primero evaluar Alogia.
el grado de cumplimiento (MIR). Anhedona.
TRATAMIENTO AGUDO: Dificultad de atencin.
0 Objetivo: eliminar la sintomatologa y retornar al nivel de .UFPN. Abulia y apata.
funcionamiento premrbido.
. Antipsictico: antipsicticos atpicos. En episodios previos, el MIR OI (7131): El tratamiento integral de la esquizofrenia duran-
antipsictico que haya sido eficaz excepto si hay problemas te la fase de mantenimiento incluye varios abordaies. CuI de
de tolerancia. los tratamientos que se mencionan constituye una respuesta
o Meioran ms los sntomas positivos (porque se deben a M3:
exceso de actividad dopaminrgica). i. Empleo continuado de antipsicticos entre dos aos y de por
Puede ser necesaria la suiecin temporal ante casos de agitacin vida, atendiendo a la dosificacin correcta y observando la
psicomotriz y el ingreso involuntario. "adherencia.
2. Tratamiento electro-convulsivo.
TRATAMIENTO de MANTENIMIENTO: 3. Empleo de tcnicas de resolucin cle problemas.
o Obietivo: prevencin de nuevos episodios. 4 Empleo de tcnicas de control de aurosal (o estado de
o 50% de los pacientes esquizofrnicos va a sufrir recidivas alerta).
durante el ao posterior por lo que es recomendable reali 5. Intervenciones pscoeducativas sobre la familia.
zar mantenimiento.
o Antipsctico: preferentemente el que se haya utilizado en Ia MIR 06 (8440): AI evaluar a un paciente que padece una es-
fase aguda. Mnima dosis eficaz. quizofrenia resistente al tratamiento Ia primera medida a consi-
o Verificar si se est cumpliendo el tratamiento y valorar derar ser:
administrar medicacin de liberacin retardado. I. Uso de Ia Terapia Electroconvulsiva.
o Utilizar los tratamientos psicosociales indicados. 2. Pasar el paciente a un antipsictico atpico.
3. Verificar gue el paciente estuviera cumpliendo
DURACIN DEL TRATAMIENTO CON ANTIPSICTICOS el tratamiento prescrito. *
4. Ingresar al paciente en un hospital.
PRIMER episodio I 2 aos 5. Aadir un tratamiento psicoteraputico.

MLTIPLES episodios 5 aos crnico

RIESGO VITAL De forma indefinida (de por vida)


2. Trastorno por ideas delirantes
a Llamada tambin PARANOIA.
TERAPIA ELECTROCONVULSIVA (TEC) o Delirios no extraos:
o Indicada en (MIR): o Situaciones que pueden ocurrir en la vida real.
o Pacientes catatnicos y hebefrnicos. o > i mes de duracin.
o Pacientes con agitacin importante. o Bien organizados y estructurados (crebles).
o Pacientes resistentes a varios antipsicticos. o NO alucinaciones.
o Pacientes que por algn motivo no pueden tomar antipsi o NO disfuncin sociolaboral acusada.
cticos (embarazo, ancianos).
Elevado riesgo de suicidio
o Delirium o confusin grave no orgnica.

6:42.16
CURSO INTENSIVO MIR ASTURIAS

2.1. Epidemiologa o Delirios de falso reconocimiento:


Prevalencia vida: 0.025-0.03%. o Sndrome de Capgras: ilusin de los dobles
Incidencia: 1-3 casos / 100.000 habitantes / ao. (un familiar ha sido reemplazado por un im-
Edad media inicio: 40 aos. postor"). No se reconcoe a un familiar.
o Fenmeno de Frgoli: perseguidores o familia-
Ligero predominio de mueres.
res asumen el disfraz de extraos. Se reconoce
Ms frecuente en clases sociales baias o aisladas (dficits
a un extrao.
sensoriales, inmigrantes).
o Trastorno de negacin, nihilista o Sndrome de Co-
2.2. Clnica tard: delirio de haber muerto o de prdida de los
propios rganos.
Inicio progresivo e nsdoso (se habla de desarrollo en
contraposicin al proceso del delirio de la esquizofrenia). 2.3. Diagnstico
No alucinaciones prominentes o sostenidas (puede haber
alucinaciones tctiles u olfativas en relacin con el delirio). - Ideas delirantes no extraas de >l mes.
Lenguaie ampuloso o circunstancial cuando hablan de los Nunca ha cumplido criterios de esquizofrenia .
delirios. o Excepto por el impacto directo de las ideas delirantes, la
No ruptura biografica. La persona puede mantener un buen actividad psicosocial y el comportamiento estn conserva-
nivel de funcionamiento fuera del sistema delirante. dos.
Ausencia de conciencia de enfermedad o Si ha habido episodios afectivos simultneamente a las
Delirios: sistematizados, factibles, lgicos (Mm! ideas delirantes, su duracin ha sido breve en relacin a la
O Percepciones correctas con interpretacin delirante: de ellas.
interpretativo. o No se debe a los efectos fisiolgicos directos de alguna
TIPOS ms frecuentes (MIRI: sustancia o a enfermedad mdica.
O Erotomanaco o Sndrome de CIrambau/t: mas en
mueres. 2.4. Curso y pronstico
O GrandOSidad o megalomanaco.
O Celotlpico o Sndrome de Otelo: ms en varones. o Curso crnico. El delirio puede manifestarse de modo fluctuan
Muy frecuente en alcoholismo. te, pero nunca deia de estar siempre latente, especialmente el
persecutorio.
Persecutorio: el ms frecuente. Mas en varones.
o Difcil de tratar (no conciencia de enfermedad y mal cumpl-
miento teraputico).
o Factores de buen pronstico:
o Buen nivel de funcionamiento sociolaboral.
Sexo femenino.
Inicio antes de los 30 aos.
Inicio sbito.
Breve duracin de la enfermedad.
OOOOO Factores desencadenantes: estrs, inmigracin, aisla-
miento, conflictos familiares.
o Los de meor pronstico: persecutoro, somtco y erotoma-
naco.

T. DELIRANTE . .7 ESQUIZOFRENIA
o 35-45 aos o 20-30 aos
o Personalidad paranoide o Esquizoide
o Desarrollo o Proceso
o Sin deterioro del YO o Deterioro del YO
Delirio Somtico (o psicosis hipocondraca mono- o Delirio sistematizado, con o No sistematizado, sin pro-
sintomtica): delirios de infestacin de Ekbm, propagacin social pagacin social
dismorfofobia, de mal olor corporal o halitosis. o Poco frecuente o Ms frecuente
Igual frecuencia hombres y muieres.
2.5. Tratamiento
o Antipsicticos atpicos.
o La psicosis hipocondraca monosintomtica clsicamente se
trata con pimocide (tipico).

MIR OI (7132): Uno de los siguientes criterios E corresponde


al diagnstico de Trastorno Paranoide:
l. Es un delirio bien sistematizado.
Presenta una conducta rgida.
Ligero deterioro de Ia personalidad.
Comprobacin de la realidad acusadamente distorsionada.
9390. Adaptacin social frustrada por el sistema delirante.
V. ESQUIZOFRENIA Y TRASTORNOS POR IDEAS DELIRANTES

MIR 03 (7657): Una paciente de 62 aos refiere que un famoso


cantante le manifiesta desde hace aos su amor, a travs de 4. Otros trastornos osicticos
insinuaciones o gestos en sus intervenciones pblicas. Ha trata-
do, sin recibir respuesta, de comunicarse con l, mediante lla-
madas telefnicas, cartas o incluso yendo a su domicilio, por Io
4.1. Trastorno esquizofreniforme
que fue denunciada. No sufre alucinaciones y su capacidad de o Cumple los criterios diagnsticos para esquizofrenia, pero:
iuicio, fuera del tema citado, es totalmente adecuada. CuI o NO disfuncin sociolaboral.
sera Ia primera sospecha de diagnstic03: o Duracin entre I y meses.
Depresin psictica no congruente con el humor. o Recuperacin del funcionamiento anterior una vez
Trastorno de ideas delirantes persistentes.* resuelto el trastorno.
Demencia incipiente.
Disfuncin sexual hiperertica. 4.2. Trastorno psictico agudo y transitorio
.UFPNT Esquizofrenia hebefrnica. Sndrome psictico AGUDO y TRANSITORIO. Puede sobrevenir
en respuesta a un factor de estrs psicosocial grave.
MIR II (9689): Una de las siguientes caractersticas clnicas O Sntomas positivos de Ia esquizofrenia.
corresponde al Trastorno Delirante:
Entre I da y I mes.
'l. Es un delirio estructurado y creble, formado a base de cre-
Recuperacin completa en el plazo de 2-3 meses (a
encas incorrectas acerca de Ia realidadxterna
menudo en pocas semanas o incluso das).
2. Trastornos del curso del pensamiento (bloqueo o disgrega-
cin del curso del pensamiento). 4.3. Trastorno psictico inducido por
3. Sntomas negativos como abulia y aplanamiento afectivo.
4. Alucinaciones auditivas, sustancias
5. Existe una conciencia clara de enfermedad. Relacin temporal clara durante la intoxicacin o la abstinencia
o dentro del mes despus al consumo.
MIR 12 (9937): En el diagnstico diferencial entre paranoia y
esquizofrenia, es FALSO que: 4.4. Trastorno psictico compartido
I. Los temas persecutorios aparecen en ambas entidades.
EI delirio es mas comprensible en la paranoia. Transferencia de delirios de una persona a otra.
2.
3. El Yo est mas desestructurado en la esquizofrenia. o La persona transmisora del delirio suele tener una tras-
4 Ambas entidades aparecen clsicamente en las mismas torno psictico.
edades". La persona a la que le han transmitido el delirio suele
5. Los sntomas disociativos son tpicos de Ia esquizofrenia. ser sugestionable (sin historia previa de trastorno psic-
tico).
Relacin inusitadamente cercana entre ellos y relativamente
3. Trastorno es-uizoafectivo aislados de otra gente.

4.5. Sndrome de Charles Bonnet


3.1. Epidemiologa Cuadro que cursa con alucinaciones visuales.
Prevalencia a Io largo de Ia vida: 0.5-0.8%. Mala respuesta a neurolpticos.
Ms en muieres (hombres inicio ms precoz). Tpicamente precede a deterioro cognitivo en pacientes
neurolgicos o con dficit sensoriales (invidentes).
3.2. Clnica
4.6. Psicosis postparto (puerperal)
Sntomas de esquizofrenia (al menos 2 de los siguientes:
ideas delirantes, alucinaciones, Ienguaie desorganizado, Relacin con el trastorno bipolar y el trastorno depresivo mayor.
comportamiento catatnico o desorganizado, sntomas nega- o Incidencia: 1-2 / 1000 nacimientos.
tivos) 50% primparas.
Sntomas de episodio depresivo mayor, manaco o mixto Recurrencia del 20-25%.
(especificar tipo depresivo o bipolar). A los pocos das del parto (2-3 semanas).
Ideas delirantes o alucinaciones durante >2 semanas en 3 tipos:
ausencia de sntomas prominentes del estado de c'inimo. o Estados confusionales de origen orgnico (infecciones,
Los sntomas afectivos estn presentes durante una parte eclampsia, anemia).
sustancial de toda la evolucin de los periodos activos y resi- o Reacciones psicgenas ligadas a estrs.
dual de la enfermedad. o Ligados al trastorno bipolar.
No se debe a sustancia o enfermedad mdica.
MANIFESTACIONES CLNICAS:
3.3. Tratamiento o Sntomas prodrmicos: fatiga, insomnio, inquietud, Iabili-
Antipsicticos atpicos. dad emocional, depresin.
Dependiendo del subtipo: Ms adelante: suspicacia, confusin, incoherencia, afirma-
o Depresiva: antidepresivos. ciones irracionales, preocupacin excesiva por Ia salud y
Bipolar: estabilizadores. bienestar del beb.
o
Ideas delirantes en el 50% de los casos.
Pensamientos de infanticidio o suicidio.
Alucinaciones en el 25%.
Alteraciones de Ia motilidad: suelen referir incapacidad pa-
rCI moverse, pararse, O CGmInGr.

TRATAMIENTO:
o Antipsicticos.
Litio.
Antidepresivos.

a-u.
CURSO INTENSIVO MIR ASTURIAS

'
RESUMEN DE ESQUIZOFRENIA v marinosP5815055 osuwms-
1. ESQUIZOFRENIA
Distorsiones fundamentales y tpicas de la percepcin a|ucinaciones-, del pensamiento delirios- y de emociones afectividad in-
apropiada o embotamiento afectivo- y cognicin. Curso crnico. Produce incapacidad personal, laboral y/o sociolaboral. Escasa
conciencia de enfermedad.

1.1. EPIDEMIOLOGA

o Prevalencia vida: 1% (2MIR). Edad de inicio: 15-25 aos en varones; 25-35 en mujeres. Si inicio despus de los 45 aos: es-
quizofrenia de inicio tardo (MIR). Varones algo ms que muieres (pero en examen igual habra que contestar que misma pre-
valencia). Muieres: meior pronstico. Muerte por suicidio consumado: 5-1 0%.

'I .2. ETlOPATOGENIA

o Factores genticos (prevalencia en gemelos monozigticos: 50%).


o Factores biolgicos: hiperactividad dopaminrgica.
o Factores psicosociales y ambientales:
o Migracin, consumo de sustancias.
o Alteraciones durante embrazo o parto.

1.3. CLNICA
NO sntomas ni signos patognomnicos.
Sntomas positivos (5MIR) Sntomas negativos
- Ideas delirantes - Alogia
- Alucinaciones (o pseudoalucinaciones): auditivas - Aplanamiento (embotamiento) afectivo
- Lenguaie desorganizado - Anhedona, aislamiento social
- Afecta inapropiado - Abulia-apata
- Comportamiento desorganzado/bizarro - Falta de autocuidados
Meior respuesta al tratamiento Peor respuesta al tratamiento
Otros sntomas y trastornos asociados: humor disfrico, alteraciones del ritmo sueo/vigilia, alteraciones de la actividad psicomotora
(alteraciones de la marcha, balanceo, estereotipas), falta de conciencia de enfermedad (falta de insight). Ansiedad. Abuso de canna-
bis, de tabaco, de cafena. Suicidio.

1.4. CRITERIOS DIAGNSTICOS

Sntomas caractersticos: 2 ms de los siguientes durante al menos un mes, (menos si se ha recibido tratamiento). Vale con uno
slo si delirios o alucinaciones.
o Ideas delirantes.
o Alucinaciones.
o Lenguaje desorganzado (disgregado, incoherente, tangencial, neologismos).
o Comportamiento catatnico o gravemente desorganzado.
o Sntomas negativos (aplanamiento afectivo, alogia, abulia).
Disfuncin socio-laboral.
Duracin: signos de la alteracin
i persisten al menos meses entre prdromos, fase activa de sntomas caractersticos (>'I mes)
y/o fase residual.
Descartar trastorno esquizoafectivo o trastorno del humor con sntomas psicticos.
No debido a los efectos fisiolgicos directos de una sustancia (droga de abuso, frmacos) o de una enfermedad somtica.
1.4.1. Subtipos diagnsticos:
o Paranoide (MIR): la ms frecuente (60-70%), de inicio ms tardo, de meor pronstico. Predominio de ideas delirantes de per-
secucin y pseudoalucinaciones o alucinaciones (auditivos). Respuestas paradiicas. Ambivalencia afectiva.
o Hebefrnica o desorganzado: la de inicio ms precoz y de peor pronstico. Regresin marcada a comportamientos primiti-
vos, desinhibdos y desorganzados (2MIR). Atectividad superficial e inapropiada (risas inmotivadas). Marcado trastorno del
pensamiento (desorganzado) y del lenguaie (incoherente). Pobre contacto con la realidad. Activos pero sin propsito, com-
portamiento tonto o fatuo, errtico y vaco de contenido.
o Catatnica: muy infrecuente. Son caractersticos de esta forma los sntomas motores (MIR): estupor, excitacin (a veces cam
bios rpidos entre ambos), mutismo, negativismo, catalepsia, flexibilidad crea, rigidez, obediencia automtica. La de mayor
sustrato biolgico. Terapia electroconvulsiva es eficaz.
0 lndiferenciada: cumplen los criterios generales para el diagnsticos de esquizofrenia pero no satisfacen los criterios para nin-
guno de los tres subtipos especficos anteriormente descritos.
o Simple: no muy frecuente. Desarrollo progresivo de sntomas negativos caractersticos de la esquizofrenia residual, sin snto-
mas positivos claramente manifiestos. Escasa respuesta emocional (MIR).
o Residual: estado crnico del curso de la enfermedad, caracterizado por la presencia de sntomas negativos y de deterioro
persistentes de al menos l ao de evolucin. Clara evolucin progresiva desde los estadios iniciales (con sntomas psicticos
positivos) hacia los finales.
V. ESQUIZOFRENIAY TRASTORNOS POR IDEAS DELIRANTES

1.5. ESTUDIOS COMPLEMENTARIOS

o Neuroimagen: aumento del tamao ventricular (de los ventrculos laterales y del 39 ventrculo) (MIR) .

1.6. PRONSTICO
lnicio brusco, factores desencadenantes, edad tarda, sexo femenino, apoyo familiar, sntomas depresivos y sobre todo un buen
funcionamiento previo apoyan un buen pronstico. El adecuado cumplimiento teraputico es el principal factor pronstico sobre el
que podemos incidir.

1.7. TRATAMIENTO

Ante falta de respuesta o recada evaluar cumplimiento teraputico (MIR).


Antipsicticos atpicos: son el tratamiento de eleccin (2MIR). Meior respuesta de los sntomas positivos (MIR).
o Criterio de eleccin de un frmaco: buena respuesta previa al mismo. En los primeros episodios: antipsicticos atpicos.
o La duracin del tratamiento del 19 episodio debe ser de al menos un ao. En la enfermedad con recadas se debe mantener
el tratamiento al menos 5 aos antes de intentar la retirada (2MIR).
TEC (terapia electroconvulsiva): cuando fracasan los pscofrmacos. Es de primera eleccin en (MIR):
o Forma catatnica y hebefrnica resistentes.
o Esquizofrenia con alto riesgo de suicidio.
o En gestantes y ancianos con pluripatologa que tengan contraindicados los psicofrmacos.
Tratamiento psicosocial: psicoterapia cognitivo-conductual: tcnicas de control del estado de alerta y resolucin de problemas (MIR).
Psicoterapia familiar: la alta expresividad familiar (hostilidad o crtica) se correlacionan con ms recadas. Psicoeducacin al pa-
ciente y familia (MIR).

2. TRASTORNO POR IDEAS DELIRANTES PERSISTENTES (PARANOIA)


Presencia de l o varias ideas delirantes, relacionadas entre s, no bizarras (de carcter interpretativo), que persisten al menos l
mes, sin deterioro marcado del funcionamiento psicosocial (MIR).
No hay otros sntomas de la fase activa de la esquizofrenia (MIR). Puede haber alucinaciones tctiles u olfatorias relacionadas con el
tema delirante-, no auditivas ni visuales (o muy poco importantes) (MIR).

2.1. EPIDEMIOLOGA
a Raro; prevalencia: 0.025-0.03%. Edad de inicio. 35-55 aos. Ligera preponderancia de muieres. El aislamiento social favore-
ce su aparicin (sordera, inmigrantes).

2.2. CLNICA
o Inicio progresivo e insidioso.
- Delirio: interpretativo, lqico, posible y bien sistematizado (2MIR). Surge de un conflicto psicoafectivo.
o Tipos: celotpico (MIR), grandioso, persecutorio (el mas frecuente y meior pronstico), erotomanaco (MIR), somtico
psicosis hipocondraca monosintomtica (MIR), mixto e inespecfico.

2.3. TRATAMIENTO

o Antipsicticos atpicos.

3. TRASTORNO ESQUIZOAFECTIVO
o Episodio depresivo mayor, manaco o mixto simultneamente con sintomas esquizofrnicos (stos han estado presentes sin
sntomas afectivos durante al menos 2 semanas).
o Tratamiento: antipsicticos y antidepresivos (subtipo depresivo) o eutimizantes (subtipo bipolar).

4. OTROS TRASTORNOS PSICTICOS


o Trastorno esquizofreniforme: equivalente a la esquizofrenia excepto en la duracin: ms de un mes y menos de meses (in-
cluyendo todas las fases: prodrmica, activa y residual). El tratamiento es similar a la esquizofrenia.
o Trastorno psictico agudo y transitorio: presencia de delirios y/o alucinaciones y/o lenguaie desorganizado y/o comporta-
miento catatnico o gravemente desorganizado. Duracin: entre un da y un mes. Suele haber un desencadenante y recupe-
racin completa.
o Trastorno psictico compartido (folle a deux): trastorno de ideas delirantes inducidas.
o Psicosis postparto (puerperal) (MIR): en estrecha relacin con el trastorno bipolar y depresin mayor.
o Clnica: sntomas prodrmicos inespecficos; suspicacia, confusin, afirmaciones irracionales, preocupacin excesiva por
el beb; ideas delirantes en el 50%; alucinaciones en el 25%; alteraciones de la motilidad.
CURSO INTENSIVO MlRASTURIAS

Trastornos de la personalidad
Nmero de preguntas del captulo en el MIR

l l 'l l

90 9'I 92 93 94 95t 95 96f 96 97f 97 98t 98 99f 99 00f 00. 01. 02. 03. 04. 05. 06. 07. 08. 09. 10 l'l 12 13 14

Nmero de preguntas de cada tema

Trastorno de la personalidad
tipo A
3

Trastorno de la personalidad
9
tipo B

Trastorno de la personalidad
tipo C

Trastorno de la personalidad
no especicados

Otros trastornos de la
personalidad

Imprescindible

o Muy rentable el trastorno lmite de la personalidad, de los dems trastornos es importante conocer sus rasgos principales.
o El trastorno de la personalidad lmite se caracteriza por un patrn general de inestabilidad y marcada impulsividad. Frecuente-
mente el suieto realiza esfuerzos para evitar el abandono, tiene relaciones interpersonales inestables e intensas, autoimagen in-
estable, atectividad inestable, sensacin crnica de vaco, impulsividad e incluso intentos autolticos. (8MIR)

S
E

g
D

n:

M151:
gmc.
a
VI. TRASTORNOS DE LA PERSONALIDAD

MIR O7 (8698): La diferencia fundamental entre "rasgo de per-


l . Introduccin sonalidad" y "trastorno de personalidadll es la siguiente:
l. No existen diferencias entre ambos conceptos. Se utilizan de
modo equivalente.
1.1. Rasgos de personalidad 2. El rasgo de personalidad es un patrn persistente de funcio-
namiento cognitivo, afectivo y relacional intlexible, desadap-
o Patrones persistentes y estables de formas de percibir, sentir
tativo y causa malestar.
y relacionarse y pensar sobre el entorno y sobre uno mismo.
El trastorno de Ia personalidad es un patrn persistente de
o Se ponen de maniesto en una amplia gama de contextos funcionamiento coanitivo, afectivo Lrelacional inflexible, de-
sociales y personales consistencia-. sadagtativo y causa malestar. *
El rasgo de personalidad es por definicin patolgico y
1.2. Trastornos de la personalidad como tal debe de tratarse.
El rasgo de personalidad conduce inevitablemente hacia un
o Los rasgos de la personalidad solo constituyen trastornos de
trastorno de la personalidad.
la personalidad cuando son inflexibles y desadaptativos y
causan deterioro funcional significativo o malestar subietivo.
3 grupos fundamentales: A, B y C. 2. Trastornos de personalidad tipo
ClE DSM A
Paranoide Paranoide
Esquizoide Esquizoide A 2.1 . Trastorno paranoide
Esquizotpico
EPlDEMtOLOGA:
Disocial Antisocial
Inestabilidad Emocional Lmite o Prevalencia en poblacin general: 0.5-3%
- tipo impulsivo o FR: familias de esquizofrnicos, varones, minoras, in-
- tipo lmite migrantes y dcits sensitivos.
Histrinico Histrinico CARACTERSTICAS CLNICAS:
Narcisista o Sospechas de que los dems les van a engaar, hacer da-
Ansioso (con evitacin) por Evitacin o, de infidelidad: suspicacia.
Dependiente por Dependencia Obsesivo- C Ven significados ceultos amenazadores en hechos triviales,
Anancstico Compuls. perciben ataques a su persona o reputacin por lo que es-
Otros tn dispuestos a contraatacar.
- narcisista Descontianza en las relaciones interpersonales.
- excntrico Rencorosos, fros atectivamente, racionales, generadores de
- inestable miedo y conflictos.
- inmaduro
- pasivo-agresivo 2.2. Trastorno esquizoide
psiconeurtico
EPIDEMIOLOGA:
Sin especificacin No especificado o Prevalencia en poblacin general: 0.5-7%.
CARACTERSTICAS CLNICAS:
CRlTERlOS DlAGNSTlCOS GENERALES:
No disfruta con las relaciones sociales-familiares.
Patrn permanente, apartado acusadamente de las expecta-
Escaso inters en mantener relaciones sexuales.
tivas de la cultura del suieto, que se manifiesta en al menos
No amigos ntimos o de confianza.
2 de las reas siguientes:
Escoge actividades solitarias.
Cognicin.
Afectividad. Indiferente a los halagos o crticas.
Frialdad afectiva.
Actividad interpersonal.
lntroversin.
Control de impulsos.
Fantasa rica, centrados en su propio mundo.
lntlexible, se manifiesta en una amplia gama de situacio-
nes personales y sociales
Estable y de Ia'rga duracin, y se inicia en la adolescencia 2.3. Trastorno esquizotpico
o al principio de la vida adulta EPIDEMIOLOGA:
Provoca malestar clnicamente significativo o deterioro
o Prevalencia en poblacin general: 0.5-5%.
sociolaboral o de otras reas importantes de la vida del su-
eto o Ms en Familiares de esquizofrnicos.
No se debe a otro trastorno mental ni a efectos directos de CARACTERSTICAS CLNICAS:
una sustancia o enfermedad mdica. Experiencias sensoperceptivas inhabtuales, incluidas las
NORMAS GENERALES DE TRATAMIENTO: ilusiones corporales.
Psicoterapia. Creencias raras o pensamiento mgico que influye en el
Farmacoterapia: sintomtica. comportamiento, ideas de referencia no delirantes. Suspica-
O Eutimizantes, antidepresivos y antipsicticos. cia o ideacin paranoide.
Ingreso en unidad de hospitalizacin de psiquiatra si ries- Comportamiento o apariencia rara, excntrica.
go vital para si o los dems. Atectividad inapropiada o restringida.
Ansiedad social en relacin a los temores.
Lenguaie caracterstico y peculiar.
Desrealizacn, en situaciones de estrs pueden descompen-
sarse y presentar sntomas psicticos breves.
CURSO INTENSIVO MIRASTURIAS

MIR FAMILIA 00 (6632) En el trastorno paranoide de la personali- MIR 09 (9209): Ante un hombre de 32 aos, que desde hace aos
dad es FALSO que: se muestra reservado, tiene escaso contacto con los sucesos coti-
dianos, escaso inters en las relaciones sexuales, suele estar intere-
Exista un exceso de suspcacia.
Se tenqan ideas delirantes de periuicio. sado en actividades solitarias, apenas tiene amigos, y es poco
Se tienda a ser rencoroso. influenciable ante crticas o elogios de Ios dems, se debera pensar
Se confe poco en los dems. en realizar un diagnstico de:
mewwe Se sospeche de Ia fidelidad de Ia pareja. Trastorno de Ia personalidad por evitacin.
Trastorno antisocial de Ia personalidad.
Trastorno paranoide de la personalidad.
Trastorno esquizoide de la personalidad.*
PHPPJNT' Trastorno narcisista de Ia personalidad.

CLUSTER A Raros
'iTrast'o'fno Pafr'anoide , Trastorno Esquizoide Trastorno Esquizotpico

0mm muwnm mui-ma

Distorsiones cognoscitivas o perceptivas,


Desconfanza y suspcacia. Distanciamiento de las relaciones sociales.
excentricidades del comportamiento.
o Introvertidos, incapaces de establecer y mantener relaciones interpersonales.
o Mal socializados, viven aislados, en distintas formas de marginalidad. Vulnerables a patologa psiquitrica.

3. Trastornos de personalidad tipo Embaucadores.


Abuso de sustancias y alcohol.
B Maltratadores.
Suelen tener EEG anormal y signos neurolgicos menores.
3.1. Trastorno antisocial
3.2. Trastorno de inestabilidad emocional
EPIDEMIOLOGA:
Tambin llamado:
o Prevalencia: hombres: 2-3.5%; muieres: 1%.
. LMITE.
o Inicio antes de Ios 15 aos, se estructura como trastorno
0 BORDERLINE.
disocial en Ia adolescencia.
o Mayor en: zonas urbanas pobres, poblaciones carcelarias EPIDEMIOLOGA:
(hasta el 75%), presenta patrn familiar (5 veces ms fre- o Prevalencia en poblacin general: I.5-2%, esta aumentan-
cuente los familiares directos). do mucho.
CARACTERSTICAS CLNICAS: o 2 muieres : l hombre.
o Con eI tiempo alta incidencia de depresin mayor.
Comportamientos ilegales, deshonestidad.
Impulsividad, incapacidad para planificar el futuro. CARACTERSTICAS CLNICAS:
o Irritabilidad y agresividad (peleas, agresiones). o Esfuerzos frenticos para evitar el abandono.
o Despreocupacin imprudente por su seguridad o Ia de los . Relaciones interpersonales inestables e intensas, que alter-
nan idealizacin y devaluacin. >
dems. Irresponsabilidad.
o Falta de remordimientos. o Autoimagen inestable.

I o Impulsividad daina para s en al menos 2 areas (gastos,
sexo abuso de sustancias, etc)
.w o Autoagresiones: desde autolesiones con propsito ansioliti-
co, gestos parasuicidas, intentos autoliticos y suicidio con-
sumado.
o Es el trastorno de personalidad aue mas frecuentemente
cursa con autolesiones.
o Gran reactividad animica e inestabilidad afectiva.
o Sentimientos crnicos de vacio.
o Ira inapropiada e intensa.
- Ideacin paranoide transitoria relacionada con estrs o
sntomas disociativos graves.
Alteraciones del sueo: latencia REM disminuida. .1:
.z
Test de supresin con dexametasona y test de estimulacin <_t
:3
con TRH anormales. g
0 CI normal. W
D.

MIR
Vl. TRASTORNOS DE LA PERSONALIDAD

o Exige admiracin excesiva, trato de favor especial .


lnterpersonalmente explotador, carece de empata.
o Envidia a los dems o cree que los dems le envidian a l.
o Comportamiento arrogante, soberbio.

o de tipo impulsivo: predominio de ausencia de control de


impulsos, explosiones de violencia, comportamiento amena
zante.
o cle tipo lmite: autoimagen, obietivos y preferencias confu-
sos, relaciones interpersonales intensas e inestables, ame-
nazas suicidas y actos autolesivas.

3.3. Trastorno histrinico


EPIDEMIOLOGIA: il
o Prevalencia en poblacin general: 23%.
y
o Ms en muieres. Curso lntensiv Asturias 2005
CARACTERSTICAS CLNICAS:
0 Necesidad de ser el centro de atencin.
Comportamiento Seductor o provocador. Utiliza su aspecto MIR Il (9685) Una muier de 23 aos acude a urgencias por
fsico para atraer la atencin. encontrarse muy nerviosa tras una discusin con su pareia. En su
Expresin emocional superficial y cambiante. historia clnica se refleian varias demandas semeantes en el ao
Dramatzacin, teatralidad, exagerada expresin emocional. anterior, en dos de ellas tras un gesto autoltico. Se constatan
Sugestonable y fcilmente influenciable. tambin conflictos frecuentes en las relaciones cle pareia, cam-
Considera sus relaciones ms ntimas de lo que en realidad bios laborales y discusiones familiares. Dice sentirse incompren-
lo son. dida por todos incluidos los psiquiatras que la atienden. El dia-
o Otros: disfuncin sexual (anorgasmia en las muieres e im- gnstico es:
potencia en los varones), forma de hablar subietiva. Trastorno de la personalidad lmite*.
o En situaciones de estrs su sentido de realidad se deteriora Trastorno de la personalidad histrinico.
fcilmente. Distimia.
Trastorno disociativo.
.UPS'JNT' Trastorno de despersonalizacin.

MIR 04 (7916) Una oven de 23 aos acude al servicio de ur-


gencias con varios cortes superficiales en la cara interna de
ambos antebrazos, y en un estado de somnolencia y torpor que
hace suponer la ingesta reciente de psicofrmacos o substancias
psicoactivas. La paciente alega que estaba muy nerviosa ("a
punto de explotar) y que haba ido tomando tranquilizantes sin
encontrar meiora, hasta que acab autoinflingindose los cortes
para paliar la tensin interna. Un episodio as es frecuente en:
La esquizofrenia.
El trastorno esquizotpico de la personalidad.
El retraso mental.
El trastorno explosivo intermitente.
WPFNI El trastorno lmite de la personalidad. *

MIR 05 (8179): Todas las siguientes, EXCEPTO una, son indica-


ciones de ingreso psiquitrico en pacientes con trastorno de
Personalidad lmite. Seale la INCORRECTA:
l . Peligro inmediato para otros.
2. Impulsividad suicida o intentos suicidas serios.
3. Episodio psictico transitorio con impulsividad.
4. Crisis en el tratamiento con repercusiones en la
Cum Intensivo MiR Asturias 2005 vida y el entorno del paciente.
5. Episodio depresivo.*
3.4. Trastorno narcisista
MIR 07 (8701): Sealar el trastorno cle la personalidad en el que
EPIDEMIOLOGA: es ms frecuentela presencia de autolesones:
0 Trastorno narcisista cle la personalidad.
Prevalencia en poblacin general: <l%.
Trastorno paranoide de la personalidad.
o Ms en varones.
Trastorno antisocial de la personalidad.
CARACTERSTICAS CLNICAS: Trastorno lmite de la personalidad.*
o Autoimportancia. Fantasas de xito ilimitado, poder, belleza. .UIPSNZ Trastorno obsesivo de la personalidad.
Cree que es especial y nico.
CURSO INTENSIVO MIR ASTURIAS

MIR 08 (8958): Paciente de 27 aos que acude por tercera vez


en la semana a la urgencia por cortes superficiales, autoinfling- 4.3. Trastorno obsesivo-compulsivo (anan
dos, en ambos brazos. En la entrevista clnica destacan datos cstico)
como importante mpulsividad, consumo de diferentes txicos
de manera abusiva, inestabilidad en las relaciones y numerosos EPIDEMIOLOGA:
intentos autolticos. Seale el diagnstico ms probable: o Prevalencia: l2%.
Fase maniaca de un trastorno afectivo bipolar. o Ms frecuente en:
Trastorno de personalidad Imite.* o Hombres.
Trastorno de personalidad esquizotpico. o Familiares directos de las personas que sufren el
Trastorno de ansiedad generalizada. trastorno.
.UPFON. Sndrome de Cotard.
CARACTERSTtCAS CLNICAS:
MIR 13 (10218): o Dedicacin excesiva al trabaio con exclusin de actividades
Una muier de 23 aos acude a urgencias por encontrarse muy de ocio.
nerviosa tras una discusin con su pareia. En su historia clnica Obstinado, rgido, inflexible.
se refleian varias demandas semejantes en el ao anterior, en Perfeccionismo que interfiere con la finalizacin de las tareas.
dos de ellas tras un gesto autoltico. Se constatan tambin con- Avaro. ,
flictos frecuentes en las relaciones de pareia, cambios laborales y No sintomatologa alucinatora o delirante.
discusiones familiares. Dice sentirse incomprendida por todos Dificultad para delegar tareas, tirar obietos intiles.
incluidos los psiquiatras que la atienden. El diagnstico sera: Preocupado por el orden, listas, normas, horarios, detalles
hasta el punto de perder de vista, el obieto principal de la ac-
Trastorno de la personalidad lmite. * tividad.
Trastorno de la personalidad histrinico. o Falta de decisin, dudas y precauciones excesivas que refle-
Distimia. an una profunda inseguridad personal; pedantera; capaci-
Trastorno disociativo. dad limitada para expresar emociones.
.UPF-JNT' Trastorno de despersonalizacin. . o Egosintnico, personalidad reconocida como propia (a dife-
rencia de TOC: egodistnico).

4. Trastornos de personalidad tipo 5. Trastornos de personalidad no


C es o ecificados
4.1. Trastorno por evitacin 5.1 . Trastorno pasivo-agresivo
EPIDEMIOLOGIA: o Patrn generalizado de actitudes negativistas y resistencia
o Prevalencia en poblacin general: 0.5-'I%. pasiva a las exigencias de desempeo adecuado.
CARACTERSTICAS CLNICAS:
v Evita contacto interpersonal importante por miedo a las
5. 2. Trastorno depresivo
crticas, por lo que evita trabaios o actividad con personas. o Patrn generalizado cognitivo y de comportamiento depresi-
'
No se implica con la gente si no est seguro de que va a vo.
gustar. Represin en las relaciones ntimas.
O Sentimientos de inferioridad que le hacen que se inhiba en MIR 09 (9204): Sealar en cul de los siguientes trastornos de la
situaciones sociales nuevas. personalidad es ms IMPROBABLE la aparicin de sintomatolo-
o Imagen de si mismo como de inepto socialmente, poco ga alucinatora:
interesante. Trastorno lmite.
0 Muy reacio a correr riesgos. Trastorno esquizotipico.
4.2. Trastorno por dependencia Trastorno obsesivo-compulsivo.*
Trastorno esquizoide.
EPIDEMIOLOGIA: 51 3:9 30? Trastorno paranoide.
Prevalencia: 0.5-5%.
I Mas frecuente en mueres.
CARACTERSTICAS CLNICAS:
o Voluntario para hacer las tareas desagradables para lograr
proteccin y apoyo de los demas.
o Incapaz de iniciar proyectos, hacer las cosas a su manera,
expresar desacuerdo por temor a perder el apoyo. Dificultad
para decidir sobre cuestiones cotidianas.
o lncmodo y desamparado si esta solo; bsqueda urgente de
nueva relacin si termina una relacin importante.
o Otros han de asumir la responsabilidad sobre su vida.
o En el trastorno psictico compartido (folie deux) el miembro
que adopta el sistema delirante suele tener un trastorno de la
personalidad por dependencia. Suelen aguantar durante
tiempo prolongado abusos, infidelidades, malos tratos.
Vl. TRASTORNOS DE LA PERSONALIDAD

., ,Maznifescciones de los distintos frgstdtnas de lagersqmlidod e

CLUSTER B Su'elos lnmaduros"


Trasrorno Anfisocial Trastorno Lmite

Inestabilidad en las relaciones interpersonales, auloimagen, afectivi-


Despreco y violacin de los derechos de los dems.
dad y una notable mpulsvidad.
Nulo arrepentimiento.

emm-
Aufolesiones.
'
foilag . ..

.l:
Cum lmensiva MIR Asturias 2005

Excesiva emotivdad y bsqueda de atencin, Grandiosidad, necesidad de admiracin y falla de empata.


Mal socializaclos, conductas desconlroladas o inconvenientes.
Desaiusfados emocionalmente.

5g
.n_<

Q e
MIR
CURSO INTENSIVO MIRASTURIAS

CLUSTER C (Temerosos)
Trastorno Obsesivo
Trastorno por evitacin Trastorno Dependiente
Compulsivo de la personalidad

Temor a la evaluacin negativa por los Miedo a todo, muy preocupados por el Miedo a la soledad. Hipersensibles a ia des
dems orden. aprobacin.
Aislamiento social Obsesiones egosintnicas (diferencia con Autoestima nula.
el TOC) Se adhieren a otras personas.
o Comparten caractersticas de ansiedad o miedo. Introvertidos y neurticos.
o No solo tienen miedo a determinadas situaciones sino que las estrategias para afrontarlo son disfuncionales.

Z3
g

M111
214

Vl. TRASTORNOS DE LA PERSONALIDAD

RESUMEN DE TRASTORNOS DE IA PERSONALIDAD


1. INTRODUCCIN
Patrn permanente, inflexble y estable, de cognicin, afectividad y comportamiento, que se manifiesta en una amplia gama
de situaciones.
Provoca malestar clnicamente significativo o deterioro en rea importantes de la vida del suieto.
Aparecen en la infancia o adolescencia y persisten en la madurez.
Ms frecuentes en varones: paranoide, antisocial, narcisista, obsesivo-compulsivo.
Ms frecuentes en muieres: limite, histrinico, por dependencia.

2. TRASTORNOS DE PERSONALIDAD CLUSTER A


Introvertidos, incapaces de establecer y mantener relaciones interpersonales.
Mal socializados, viven aislados, en distintas formas de marginalidad.
Desaiustados emocionalmente.
Vulnerables a patologa psiquitrica (sobre todo psicosis).
Tipos: PARANOIDE, ESQUIZOIDE, ESQUIZOTPICO.
PARANOIDE: Suspicacia excesiva.
ESQUIZOIDE: Solitario, no disfruta de relaciones interpersonales.
ESQUIZOTPICO: Creencias extraas, comportamientos extravagantes.

3. TRASTORNOS DE PERSONALIDAD CLUSTER B


Mal socializados, conductas descontroladas o inconvenientes.
Desaiustados emocionalmente.
Tipos: ANTISOCIAL, INESTABILIDAD EMOCIONAL O LMITE, NARCISISTA, HISTRINICO.
ANTISOCIAL: incumplimiento y violacin sistemtica de normas sin arrepentimiento.
INESTABILIDAD EMOCIONAL: inestabilidad en autoimagen, relaciones y emociones. Gran impulsividad y autolesiones fre-
cuentes.
NARCISISTA: grandiosidad, necesidad de admiracin, falta de empata.
HISTRINICO: teatralidad, exageracin.

4. TRASTORNOS DE PERSONALIDAD CLUSTER C


Caracterizados por ansiedad importante.
Tipos: POR EVITACIN, OBSESIVO-COMPULSIVO O ANANCSTICO, POR DEPENDENCIA.
POR EVITACIN: sentimientos de inferioridad que producen incapacidad en habilidades sociales.
OBSESIVO-COMPULSIVO: perfeccionismo excesivo que produce ineficacia y sufrimiento.
POR DEPENDENCIA: dependencia de terceras personas por baia autoestima.

5. TRATAMIENTO
Psicoterapia: muy eficaz.
Farmacoterapia sintomtico. Se utilizan practicamente todos los psicofrmacos porque ninguno es especfico: ansiolticos, an-
tidepresivos, eutimizantes, antipsicticos.
NO TEC.
Ingreso en unidad de hospitalizacin de agudos si riesgo vital para s o los dems (MIR).
CURSO INTENSIVO MIR ASTURIAS

Trastornos del sueo


Nmero de preguntas del captulo en el MIR

I l I I I I I I I r I n l l I I I I I l I I

90 91 92 93 94 95f 95 9f 96 97f 97 98f 98 99f 99 00f 00. 01 02. 03. 04. 05. 06. 07. 08. 09. 10 11 12 13 14

Nmero de preguntas de cada tema

Sueo normal

Dsomnias

Imprescindible
c Tema poco preguntadolos ltimos aos. Lo ms importante son las fases del sueo nomal y conceptos generales del insomnio.
o Durante el sueo normal se producen 4-5 ciclos, cada uno dura entre 70-100 minutos. En cada ciclo se repiten 4 fases de sueo
No REM cada vez ms profundo [|, || y delta (antes III, IV)] y una fase de sueo REM.
o El trastorno mas frecuente es el insomnio. Puede ser secundario (a enfermedades somticas o mentales, frmacos, txicos...) o
primario. EI tratamiento debera ir dirigido a la causa en aquellos que son secundarios, y con medidas no farmacolgicas de ini-
cio. Si precisa tratamiento farmacolgico se utilizan hipntcos benzodacepnicos en pautas cortas (dos meses con retirada pro-
gresva).

g
D


214.3
MIR
W
VII. TRASTORNOS NO ORGANICOS DEL SUEO

o El sueo REM (rapid eye movements) o MOR (movimientos


I . Sueo normal oculares rpidos) aparece aproximadamente a los 90 minu-
tos tras iniciarse el sueo: es lo que se denomina latencia del
o 7-8 horas diarias con grandes variaciones. sueo REM. En la depresin endgeno o melanclica (y en el
o 4-5 ciclos de sueo durante la noche, cada uno dura entre trastorno lmite de personalidad) la latencia del sueo REM
70-100 minutos. En cada ciclo se repiten 4 fases de sueo esta disminuida (40 minutos o menos).
No REM cada vez ms profundo [l, || y delta (antes lll, IV)] y o En esta fase el muestra un ritmo de base Beta (13-22
una fase de sueo REM. Hz) similar al de la vigilia y ondas en diente de sierra.
o Fase l: transicin entre vigilia y sueos o En el MQ se registra atona muscular.
O En el : ondas alfa ytheta. o En el EO_G movimientos oculares rpidos. Es la fase de
las ensoaciones.
o Los primeros episodios REM son de corta duracin, con
aumento progresivo hasta ms de 30 min el ltimo, al
(WWMWWMWNWNW contrario de lo que ocurre con la fase delta, que se acorta
durante la noche.
Adormecimiento, ondas alfa
MIR 91 (2829): El patrn del encefalograma en la fase REM del
o Fase ll (el 50% del tiempo total de sueo) o sueo superficial. sueo corresponde a:
o En el E_EQ: ondas theta, husos de sueo y compleos K. Ondas alfa
o En el M: hipotona progresiva. Ondas beta.*
o Los HUSOS de sueo (spindles) son salvas de ondas si- Ondas delta.
nuosidades 12-14 Hz de 0,5-I .5 sg que se repiten cada Ondas theta.
3-8 minutos. .UFWN. Ondas de baio voltae y espigas (husos del sueo).
o Los compleios K son ondas lentas bfsicas (ondas nega-
tivas escarpadas seguidas de un componente positivo) DURACIN DE LAS DISTINTAS FASES DEL SUEO EN EL
amplitud mnima de 75 microvoltos y al menos 0.5 sg de ADULTO Y RECIN NACIDO:
duracin. Se repiten cada 1-3 minutos. Un estmulo que Adulto:
despierte al suieto en fase | puede producir un compleio o Fase I: 5% del total del tiempo de sueo.
K en fase ll sin que el individuo se despierte o Fase II: 50%.
o Sueo delta: 25%.
o En RN no hay sueo delta. En la infancia (a partir del
primer ao) la duracin del sueo delta es maxima. Esta
fase del sueo va disminuyendo con la edad a medida
que se incrementa la fase II. En la veiez acaba por des-
aparecer Ia fase delta
o Sueo REM: 20-25%.
o En el RN, la fase REM ocupa un 50% del tiempo total de
sueo. El sueo REM permanece constante a lo largo de
toda la vida adulta y su cantidad absoluta se ha relacio-
nado con un adecuado funcionamiento cognitivo.

MIR OI (7037): Cual de estas armaciones sobre el sueo es


FALSA?:
I. El sueo REM ocupa un 20-25% del tiempo del sueo total en
adultos.
2. En ancianos sanos puede estar muy reducida la fase 4 de
sueo no-REM.
3. El sueo REM y no-REM se alternan en ciclos de 90-110
minutos.
4. EI sueo de ondas lentas corresponde a las fases 3 y 4 del
sueo no-REM.
5. El sueo REM M0 menos de un 10% del tiempo de sueo
total en lactantes.*
Husos de sueo, y puntas de vertex, fase 2

o Sueo delta (antiguas fases III y IV), profundo o lento:


o En el ondas deltas baa frecuencia: 0.53 Hz y am-
plio voltaie: 75 microV-.
o En el EM_G: hipotona extrema.
o En el : ausencia de movimientos oculares.

Sueo profundo, fase delta

H
S
MIR
CURSO INTENSIVO MIR AsrumAs

Sueo NREM (NMOR) SUEO REM (MOR)


20-25%
VlGILlA REIAJA FASE l FASE u SUEO DELTA
ACTIVA ClON (oios 5% 50% 25%
(oios abier- cerrados) FASE FASE IV
tos) alto (8- III 13%
beta > 12Hz) occi- 12%
l2Hz tron- pital
tal
EEG beta > alfa (8- theta theta delta (< 4Hz) beta (>12Hz)
12Hz fron- 12Hz) occi- (4:8Hz) husos de ondas en dientes de
tal pital sueo sierra
'(l 2-14
Hz)
compleios
K (ondas
lentas y
bifsicas)
EMG Activo Hipotona + Hipotona progresiva ++ + Atona

EOG Mvtos. Movimientos lentos rotatorios en balancn" Ausentes rpidos, coniugados


rpidos
PROFUN Superficial Media Profundo media
DIDAD
FC, TA, Descenso, estabilidad Aumento de la T.A
FResp taquicardias y apneas
Regulacin PRL
hormonal GH (+) TSH(-) ACTH (-)
Fenmenos ensueos"
fsicos Erecciones peneanas
T"J Homeotermia Poiquilotermia

Parasomnias Bruxismo Sonambulismo Pesadillas,


Terror nocturno T del Sueo REM

En funcin de su DURACIQN, el insomnio puede ser:


2. Disomnias o Transitorio: duracin inferior a l semana. Existencia de
acontecimientos desencadenantes (ietlag, examen). Tra-
Alteracin de la cantidad calidad u horario del sueo. Son: n- tamiento de eleccin: frmacos hipnticos.
somnio, hipersomnio y trastornos del ritmo sueo-vigilia. o De corta duracin: entre l y 3 semanas. Desencadenado
por acontecimientos adversos que desencadenan estrs
2.1. Insomnio no orgnico ms duradero. Tratamiento: hipnticos.
EPIDEMIOLOGA: o De larga duracin o crnico: ms de 3 semanas. Trata-
El trastorno de sueo ms prevalente. miento: etiolgico + medidas de higiene del sueo +
El 10-15% de la poblacin adulta tiene insomnio crnico y hipnticos durante conos periodos de tiempo (mximo 2
el 25-35% transitorio o de corta duracin. meses, incluyendo en ellos l mes de retirada progresiva
Las muieres 1,3 veces ms. para evitar el insomnio de rebote).
Las >65 aos 1,5 veces ms. En funcin de su ETlOLOGlA, el insomnio puede ser:
El secundario a un trastorno mental es el Subtipo ms tre- o Primario o psicosiolgico.
cuente (30-50% de los insomnes). o Secundario:
CLNICA: Trastornos mentales: todos.
Queias de sueo: dicultad de conciliacin, mantenimiento o Enfermedades somticas.
sueo poco reparador. Frmacos y sustancias de abuso.
Frecuencia: al menos 3 veces/semana durante por lo menos OOOO Factores ambientales (ruido, T, altitud).
l mes.
La no satisfactoria cantidad o calidad del sueo produce
marcado malestar o interfiere con las actividades socio-
laborales.
Ausencia de causa organica conocida (neurolgica, somt
ca, medicamentos o uso de sustancias psicoactivas).
En funcin de su NATURALEZA el insomnio puede ser:
0 De conciliacin: tpico de trastornos de ansiedad.
o De mantenimiento: tpico del trastorno por estrs pos-
traumtico.
o Despertar precoz: al menos 2 horas antes de lo habitual.
Tpico de la depresin endgena.
VII. TRASTORNOS NO ORGANICOS DEL SUEO

j EnfermedadesIsorhtCa55u_. 'i' * ,, _
Insomnio familiar fatal: degeneracin espongiforme de ncleos
talamicos que debuta con insomnio, progresa al coma y Ia
muerte.
Artritis reumatoide
Asma
Cncer
Cefaleas
Corea de Huntington, Demenca
Diabetes
Distrofia muscular de Duchene
Encefalopatia heptica
Enfermedad cerebrovascular
EPOC
Epilepsia
Esofagitis
Fibrosis qustica
Hipertiroidismo
Infarto de miocardio
Insuficiencia renal crnica
Menopausia
Parkinson
Refluio gastroesofgco
SIDA
Prolapso vlvula mitras

Frmacos y sustancias de abuso


Abstinencia de alcohol
Alfametildopa
Anticonceptivos orales
Beta bloqueantes, Broncodilatadores
Cortcosteroides
Difenlhidantoina
Hormonas tiroideas
IMAO
Psicoestimulantes
Teofilinas
Tiazidas

TRATAMIENTO:
Evaluacin inicial para descartar causalidad (en cuyo caso tra-
tamiento sintomtico si se puede).
I MEDIDAS NO FARMACOLGICAS:
o Psicoeducacin.
o Medidas de higiene del sueo.
o Tcnicas de relaiacin.
o Reestructuracin cognitiva.

HIGIENE DEL SUEO


o Horario regular de acostarse y levantarse. No permanezca
en Ia cama mas de 8 horas/da.
o Si se despierta a media noche y no logra conciliar el sueo,
Ievntese y realice alguna actividad relaiante hasta que vuel-
va a sentir sueo.
o Evite dormir durante eI dia.
o Abstencin de bebidas estimulantes y alcohol por Ia tarde-
noche.
o No fumar antes de acostarse ni durante Ia noche.
o Efecte ejercicio fsico durante el dia de forma regular pero
nunca en las tres horas antes de acostarse.
o Establezca una rutina relaiante que prepare el sueo.
o Mantenga temperatura confortable en Ia habitacin as como
oscuridad y tranquilidad

. MEDIDAS FARMACOLGICAS:
o Frmacos hipnticos para el tratamiento agudo del in-
somnio.
o Los mas utilizados son los agonistas del receptor de
benzodiacepina (slo 2 meses incluyendo l mes para
Ia retirada del frmaco).
Otros frmacos con capacidad hipntca: antidepresivos sedan
tes (mirtazapina, trazodona), antipsicticos sedantes (clotiapina),
cIometiazoI (til en ancianos).

__M__I_13,
CURSO INTENSIVO MIR ASTURIAS

RESUMEN DE TRASTORNOS NO ORGANICOS DEL SUEO


1. SUEO NORMAL
4-5 ciclos de sueo por noche. Cada ciclo: Fase l, Il, delta (lll y IV) y fase REM.

SUEO
Sueo NREM (NMOR) susio REM (MOR)
VlGlLIA RELAJA FASE l FASE II SUEO DELTA 20-25%
ACT|VA ClON (oios 5% 50% 25%
(oios abier- cerrados) FASE Ill FASE lV
tos) 12% 13%
EEG beta > alfa (8-1 2Hz) theta theta delta (< 4Hz) beta (>12Hz) (MlR)
'lZHz fron- occipital (4-8Hz) husos ondas en "dientes de sierra"
tal compleios K
EMG activo hipotona hipotona atona
EOG mvtos movimientos lentos rotato- ausentes rapidos, coniugados
rpidos rios en balancn"

En el recin nacido la fase REM ocupa un 50% del tiempo total de sueo (MIR), y no hay fase delta. A partir del primer ao la duracin
del sueo DELTA es mxima y luego disminuye hasta desaparecer practicamente en la veiez.

2. DISOMNIAS
Alteracin de la cantidad, calidad u horario de sueo. Son para la CIE-10: insomnio, hipersomnio, t del ritmo sueo-vigilia.

2.1. INSOMNIO NO ORGANICO

EPIDEMIOLOGA:
o , El trastorno del sueo ms prevalerte. Ms en muieres, >65 aos, pacientes psiquitricos.
CLINICA:
0 Queias sobre dificultad para conciliar, mantener o sueo poco reparador.
o Presentes al menos 3v/s durante l mes (DSM-IV: al menos l mes).
0 Marcado malestar o interferencia socio-laboral.
i
o No causa orgnica conocida.
En funcin de la duracin:
o Transitorio: dura <1 semana. Acontecimientos desencadenantes identicables. Hipnticos.
o A corto plazo: entre 1 y 3 semanas. Acontecimientos desencadenantes mas duraderos. Hipnticos.
o De larga duracin o crnico: >3 semanas. Tratamiento: etiolgico + medidas de higiene de sueo + hipntcos por cortos
periodos de tiempo (2 meses).
En funcin de la etiologa:
o Primario, psicofisolgico.
0 Secundario:
o trastornos mentales: todos.
o frmacos y drogas de abuso: corticoides, lMAOs, broncodilatadores, betabloqueontes, derivados tiroideos, drogas
estimulantes.
o enfermedades somticas: demencia, EPOC, asma, insuciencia cardiaca congestiva, enfermedades que cursen con
dolor, refluio gastroesofgico.
o factores ambientales: ruido, T, altitud.
TRATAMIENTO: etiolgico si es posible; el farmacolgico slo por perodos cortos de tiempo.
o Medidas no farmacolgicas: psicoeducacin, higiene del sueo, relaiacin, reestructuracin cognitiva.
o Farmacolgco: benzodiacepinas, zaleplon, zolpiden, zopiclona, antidepresivos sedantes.
V|lt. SUSTANCIAS PSICOTROPAS

Trastornos mentales debido a sustancias


osicotro cas
Nmero de preguntas del captulo en el MIR

90 91 92 93 94 95f 95 9f 9 97f 97 98f 98 99f 99 00f 00. O'l. 02. 03. 04. 05. 06. O7. 08. O9. 'lO H 12 13 'I4

Nmero de preguntas de cada tema

Generalidades l

Opiceos 25

Cocana 5

Otros estimulantes 3

Cannabis o marihuana

Trastornos psiquitricos
inducidos por drogas

Deteccin de drogas en
orina

Alcoholismo 31

(5). Imprescindible
o Uno de los temas ms preguntados, donde lo ms rentable es conocer los sntomas de las intoxicaciones y de cuadros de absti-
nencia a los principales txicos:
o Intoxicacin por OQCGOS: hipoactividad sistema nerviso simptico, miosis, hipotensin, bradicardia, disminucin nivel de concien-
cia, puede provocar parada cardiorrespiratoria. Tratamiento: Naloxona. (4MIR)
o Abstinencia de Opiceos: hiperactividad del sistema nervioso simpatico, midriasis, inquietud, piloereccin, taquicardia, hiperten
sin, diarrea, vmitos.
o Tras la desintoxacin de OQCOS se busca la deshabituacin. Se utiliza Naltrexona (antagonista larga duracin). En casos en los
que no es posible (embarazo, politoxicomanas, antecedentes psiquitricos) se utiliza tratamiento sustituctivo con agonistas opi-
ceos legales (Metadona).
o La cocana tiene efectos simpticomimticos, causando semiologa tal como midriasis reactiva, hipertensin arterial, taquicardia,
inquietud, hipertermia. Las antetaminas pueden producir semiologa similar. (3MIR)
o La encefalopata de Wernicke se suele presentar de forma aguda en suietos alcohlicos, y se caracteriza por confusin, ataxia y
alteraciones de Ia motrica ocular. Se trata con dosis altas de tiamina parenteral, y de forma preventiva debera administrarse a
pacientes con baio nivel de conciencia de causa desconocida. (3MIR)
o EI sndrome de abstinencia alcohlico puede ser mortal y suele alcanzar la mxima intensidad a los 2-4 das de interrumpir la
ingesta. Suele provocar hiperactividad autonmica, ansiedad y clnica psictica (delirio, alucinaciones visuales). Para su preven-
PSIQUIATRA cin y tratamiento se utilizan benzodiaceginas de efecto prolongado.
gsm
e MIR
1 f
CURSO INTENSIVO MIR ASTURIAS

o Para la deshabituacin del alcohol ademas de tratamiento psicosocial se utiliza tratamiento farmacolgico. Para reducir el craving
(deseo de consumir) se utiliza Naltrexona. EI Topiramato ayuda en el control de impulsos. Para aumentar el efecto txico del alco-
hol y as generar rechazo a su ingesta (provocan efectos histaminrgicos muy desagradables) se utiliza Disulfiram y Cianamida.

cia a alcohol o barbitricos puede poner en peligro la vi-


i. Generalidades da del suieto, no as el sndrome de abstinencia a opi-
ceos.
1.1 . Conceptos generales o TARDIO: Dsregulacin del sistema nervioso vegetativo
y de las funciones psquicas basicas que persisten du-
DROGA: Sustancia que al ser introducida en el organismo rante largo tiempo (meses o aos) tras la abstinencia.
produce una alteracin de la conducta y/o el estado psquico.
SNDROME AMOTIVACIONAL:
lNTOXlCAClN: Sndrome reversible que se caracteriza por la o Aparece con todas las sustancias que crean dependencia.
aparicin de cambios psicolgicos o comportamentales des- o Consiste en: astena, apata, falta de inters, reduccin
adaptatvos y fisiolgicos. generalizada de cualquier actividad, disminucin de la ca-
TOLERANClA DlRECTA: Estado de adaptacin caracterizado pacidad de concentracin, disminucin de la atencin.
por la disminucin de los efectos de una sustancia tras la admi- o Suele persistir largo tiempo despus de deiar el txico y si
nistracin repetida de la misma. culta enormemente la deshabitu cin reinsercin social.
Tolerancia inversa: Alcanzar progresivamente idnticos efectos
con dosis progresivamente menores.
Tolerancia cruzada: Disminucin del efecto de una sustancia
como consecuencia del consumo continuado de otra de carac-
tersticas similares.

DEPENDENClA: Presencia de signos y sntomas cognitivos,


comportamentales y fisiolgicos que indican que el individuo ha
perdido el control sobre el uso de sustancias psicoactivas y las
sigue tomando a pesar de sus consecuencias adversas.
Criterios para establecer el diagnstico de dependencia (sirven
para drogas y alcohol):

Tres o ms de los tems siguientes: (deben estar presentes en algn


momento de un periodo continuado de 12 meses).
I. Tolerancia.
2. Abstinencia.
3. La sustancia es tomada con frecuencia en cantidades mayores
o durante un periodo ms largo de Io que en principio se pre-
tenda \//

Cm'so memo MIR mida


4. Existe un deseo persistente o esfuerzos infructuosos de contro-
lar o interrumpir el consumo de esa sustancia. MIR 03 (7656): CuI de las siguientes opciones relacionadas
5. Se empleo mucho tiempo en actividades relacionadas con la con el consumo de sustancias es INCORRECTA?:
obtencin de dicha sustancia. i. La intoxicacin producida por sustancias se define como un
. Reduccin importante de las actividades sociales, laborales o sndrome reversible que se caracteriza fundamentalmente
recreativas debidas al consumo de la sustancia. por la aparicin de cambios psicolgicos o comportamenta-
7. Se continua tomando Ia sustancia a pesar de los problemas les desadaptatvos y fisiolgicos.
psicolgicos o fsicos recdivantes o persistentes, que parecen 2. La abstinencia se define como un sndrome especfico de una
causados o exacerbados por el consumo de la sustancia. sustancia debido al cese o reduccin de su consumo, que
causa un malestar clnicamente significativo y/o un deterioro
ABUSO / CONSUMO PERJUDlClAL: Patrn desadaptativo de Ia actividad laboral y social.
de consumo que no llega a cumplir los criterios diagnsticos de 3. EI abuso o consumo perjudicial se define como un patrn
la dependencia. No se presenta sndrome de abstinencia cuando desadaptativo de consumo que no llega a cumplir los crite
se interrumpe el consumo. rios diagnsticos de la dependencia.
o Decimos que una sustancia produce DEPENDENCIA FSICA 4. El concepto de tolerancia a una sustancia hace referencia al
cuando al suprimir su consumo, produce un estado fisiolgi- hecho de que con el consumo continuado se precisan cada
co alterado (sndrome de abstinencia), que es especfico pa- vez dosis menores para producir el mismo efecto.*
ra cada sustancia. Drogas que producen dependencia fsica: 5. EI sndrome de abstinencia producido por la falta de la dosis
alcohol, opiceos, barbitricos, BZD. de la sustancia consumida tiene caractersticas especficas
o La DEPENDENCIA PSQUICA se traduce no en sntomas para cada sustancia y se alivia con un nuevo consumo de la
fsicos, sino en un estado de ansiedad y nerviosismo, que lle- mismas
va a una necesidad irresistible de consumo (craving, relacio-
nado con el neurotransmisor dopamina). Sustancias que 'l .2. Tratamiento de las drogodependencias
producen dependencia psquica: cocana, anfetaminas, can-
nabis o tabaco. Comprende 3 fases:

SNDROME DE ABSTlNENClA: DESINTOXICACIN:


Sndrome especfico para cada sustancia que aparece despus o Conseguir que el paciente deie de consumir droga.
de interrumpir o disminuir el consumo tras una ingesta crnica. o La desintoxicacin de la mayora de las sustancias se consi-
Causa malestar clnicamente significativo y deterioro de la activi- gue entre 7 y 14 das (incluso menos) sin riesgo para la vida
dad social o laboral. del paciente. Permite al organismo funcionar sin la droga. Se
0 AGUDO: Coniunto de sntomas y signos orgnicos y ofrece un apoyo para evitar el sndrome de abstinencia agu-
psquicos que aparece inmediatamente despus de inte- do.
rrumpir el consumo de una sustancia psicoactiva. Dura 0 No se consigue evitar el sndrome de abstinencia tardo ni la
de 4-10 das. Cede al consumir de nuevo la droga. Es dependencia psquica.
especfico para cada sustancia. El sndrome de abstinen-
Vlll. SUSTANCIAS PSICOTROPAS

DESHABtTUACIN: ., REINSERCIN:
Proceso largo y compleio en el que se pretende que desaparez- o Debe solaparse con la fase 2 (deshabituacin), con la pro-
ca el sndrome de abstinencia tardo mientras el paciente apren- gresiva integracin del individuo en el medio social que le
de una serie de estrategias teraputicas (comportamentales) que corresponde.
le permitirn no recaer en el consumo.
o Dos tendencias:
o Aislar al paciente del contexto social donde se produca el
consumo
v Mantenerlo en el mismo contexto social intentando man-
tener la abstinencia mediante ayuda psicolgica de tipo
comportamental.

2. O-ceos
'
AGONIS'FAS PUROS AGONlSTAS PARClALES AGONISTA/ANTAGONISTAHMIR) ANTAGONISTA (MIR)
Afinidad pr receptores. u, Afinidad porrecept'ores u, no> Antagonistas p, agonistas parcia Afinidad por todos los. recepto-
- . receptoresrxz'y.:a;:"z r por receptoreslc-nt-a,. Ies c y a, res pero sin actividad
Morfina (tu2 = 4-6 h) o Buprenorfina Nalorfna o Naloxona
Herona (tu2 = 3-4 h) Pentazocina o Naltrexona
Metadona (tm = 24 h) Levalorfano o Nalmefene
Meperidina Ciclorfano
Fentanilo (tm = l h) Butorfanol
Codena Nalbufina
Dextropropoxifeno
Propoxifeno
Loperamida
Difenoxilato
Levorfanol
Hidromorfona
Oxicodona
Oximorfona
Tramadol

o OPICEOS: Sustancias derivadas del opio o productos de o Antidiarricos: Codena, loperamida, difenoxilato.
sntesis que se acoplan a los receptores opioides del SNC. o Antitusgenos : Codena, dextropropoxifeno.
Estas sustancias presentan tolerancia cruzada entre ellas.
o Agonistas puros: reproducen los efectos naturales y los 2.2. Efectos del uso de opiceos
de la intoxicacin.
o Agonistas parciales: si se administran solos ocupan el EFECTOS TRAS SU CONSUMO:
receptor sin dar lugar a los efectos de la intoxicacin, Placer intenso con sedacin durante 2-3 horas.
aunque s, en menor grado, a los analgsicos. Son ca- lNTOXlCACIN AGUDA (SOBREDOSIFICACION):
paces de antagonizar los efectos de un agonista puro Causa ms frecuente de muerte en ADVP.
cuando se administran coniuntamente. Hipoactividad simptica:
o Agonistas Z Antagonistas: si se administran solos cierto 0 Euforia.
efecto analgsico. Se comportan como antagonistas al . Rubor cutneo.
ser administrados unto a un agonista puro. o Picores (sobre todo con la morfina).
Antagonistas: poseen afinidad por los receptores pero
Miosis.
carecen de actividad. lmpiden y revierten la accin de
Somnolencia, disminucin del nivel de vigilancia, hasta
los agonistas. Su supresin brusca no produce sndro-
llegar al coma.
me de abstinencia.
o Disminucin de la frecuencia respiratoria, con posible pro-
o La vida media de los opiceos se relaciona inversamente con gresin a parada cardiorrespiratoria y edema agudo de
la intensidad del sndrome de abstinencia y directamente con pulmn, que puede producir la muerte.
la aparicin de la sintomatologa y duracin de la misma
Hipotensin.
(t'l/2 corta: sndrome de abstinencia de aparicin ms pre-
o Bradicardia.
coz, ms intenso y de menor duracin).
o o Disminucin de la temperatura corporal.
SUSTANClAS OPIOIDES: las producidas por el organismo
(encefalinas, endorfinas, dinorfinas) que se acoplan a los TRATAMIENTO de la intoxicacin aguda:
mismos receptores. Naloxona i.v. 0,42 mg. Accin inmediata. Si no meiora repetir
dosis a los 3-5 minutos (hasta 3-4 veces). Tras estabilizacin
2.1. Uso mdico de los opiceos repetir dosis cada 2 horas (duracin de la accin: 2-3 horas).
Cuidado con la reaparicin de los sntomas, ya que la mayora
o Analgesa: dolores intensos agudos (incluidos cnceres ter- de los opiceos tiene una vida media ms larga que la naloxo-
minales) y dolor crnico (morfina). na.
o Parto: meperidina (no reduce motilidad uterina). La meperi- o Herona: mantener naloxona durante horas.
dina no se usa en el tratamiento del dolor crnico porque su
o Metadona: mantener naloxona durante 24 horas.
metabolito activo (normeperidina) produce disforia y excita-
cin del SNC (mioclonias, temblor y convulsiones) y se acu- TRASTORNOS POR CONSUMO CRNICO:
mula durante das despus del inicio o aumento de las dosis Por el propio opiceo:
sobre todo en pacientes con insuficiencia renal. o Estreimiento.
NO se usan en colecistitis aguda pues contraen el esfnter de . Prdida de secreciones, nariz y boca secas.
Oddi. . Depresin de la funcin inmunitario.
Anestesia y transdrmicos para dolor crnico: fentanilo. o Sndrome nefrtico .
Edema aaudo de pulmn cardioqnico: Martina!
su
CURSO INTENSIVO MIR ASTURIAS

Otros: Neuropata perifrica, ambliopa, mielopatia, leucoence-


falopata, infecciones: infeccin de partes blandas por Estaflo- PAUTAS DE ACTUACIN:
coco Aureus (las ms frecuente), hepatitis B, C, delta, SIDA, en- 0 Embarazo: se aconseia tratamiento sustitutivo con METADONA
docarditis derecha por Estafilococo aureus, flebtis sptica, can- (IO-20 mg/da). No se recomienda iniciar desintoxicacin. Mu-
didiasis diseminada, TBC, ETS, artritis por Pseudomona aerugi- chas veces requiere ingreso inmediato.
nosa. o Adicto con enfermedad subyacente: desintoxicacin.
SNDROME DE ABSTINENCA AGUDO A OPICEOS: MIR 94 (3779): Entre los signos que se presentan en la intoxica-
Prcticamente inocuo cuando no existe enfermedad grave sub- cin por opiceos podemos demostrar:
yacente.
Miosis bilateral.*
c Aparicin: 6-12 horas tras ltima dosis. Aumento de la frecuencia respiratoria.
Mximo: 24-48 horas. Hipertermia.
Remisin: 6-10 das. Rinorrea.
SNTOMAS: 9.59%)? Taquicardia.
Controlables por el paciente:
Sintomatologa subietiva de abstinencia. MIR FAMILIA 00 (6621): Cul de los siguientes sntomas se
Deseo de consumir de nuevo el opiceo. asocia a la sobredosis de opiceos? :
Pupilas dilatadas.
Queias.
Edema pulmonar.
Splicas.
Aumento de la temperatura corporal.
Gritos.
Rubor cutaneo.
Llanto.
.UPFN. Crisis comiciales.
Agresividad:
AN U LADA
Sintomatoloaa no controlable por activacin excesiva del siste-
ma noradrenrgico.
MIR 02 (7375): Cul de los siguientes opioides E es adecua-
do para el tratamiento del dolor crnico de etiologa cancero-
SINTOMAS PRECOCES PERIODO DE ESTADO sa?
(14-18 Ii.) (24-48 h.) I. Metadona.
- Bostezos - Inquietud-agitacin 2. Meperidina.*
- Rinorrea - Temblores, calambres 3. Morfina.
- Sudacin profusa - Insomnio 4. Fentanilo.
- Lagrimeo - Dolor abdominal 5. Tramadol.
- Midriasis - Nuseas
Piloereccin -Vmitos MIR 08 (8965): Un hombre de 23 aos de edad es encontrado
- Espasmos musculares - Diarrea por la polica en estado comatoso y llevado al servicio de urgen-
- Anorexia - Hipertermia cias hosptalario. El paciente est hipoventilando a 4 respiracio-
- Hipertensin nes/min, ciantico y con pupilas miticas. Existen mltiples sea-
- Taquicardia les de venopuncin. Tras control de Ia va area, ventilacin y
- Taquipnea circulacin, se administran 2,0 mg de naloxona intravenosa. El
paciente despierta del coma, la frecuencia respiratoria aumenta
a 24/min, est inquieto con diaforesis y midriasis. Cul de las
siguientes acciones es la mas adecuada en este momento?:
1. Alta a su domicilio aconseiando seguimiento en programa
de deshabituacin.
2. Observacin en urqencias con examenes frecuentes del
estado mental durante un mnimo de 4 a horas.*
3. Iniciar infusin de naloxona intravenosa con 2/3 de la dosis
inicial por hora.
4. Administrar IO mg de metadona intramuscular.
5. Suiecin mecnica en prevencin de cuadro de agitacin y
agresin.

MIR 08 (9052): Un paciente varn de unos 40 aos es trado a


urgencias por haber sido hallado cado en la va pblica en una
zona de ocio nocturno. El paciente est poco reactivo, presenta
una frecuencia respiratoria de 7 por minuto, una presin arterial
de 90/60 y frecuencia cardiaca de 58 por minuto. Pupilas miti-
cas. Despus de asegurar la permeabilidad de la va area,
cul sera el paso inmediato mas apropiado a seguir3:
Administrar 0,4 ma de naloxona im o iv."
Obtener un hemograma y una bioqumica bsica.
Solicitar un examen toxicolgico de orina
Realizar un ECG.
9595!"? Solicitar una TC craneal.

o La piloereccin y la fiebre se asocian a la abstinencia grave. 2.3. Tratamiento


Hay una serie de variables que influyen en la intensidad del DESINTOXICACIN
sndrome de abstinencia independientemente del opiceo utili-
zado: TRATAMIENTOS INESPECFICOS:
o Dosis diaria utilizada. o Frmacos:
Va de administracin. o Ansiolticos: diacepam, clorazepato, alprazolam.
Antigedad en el uso del txico. Disminuir dosis a partir de 4 da y abandonar al 12-
Nmero de sndromes de abstinencia padecidos. 14.
Estado fsico del paciente. Hipnticos: midazolan.
Expectativas reales de resolucin del problema. Anlgesicos: Paracetamol, AAS.
VIII. SUSTANCIAS PSICOTROPAS

I Otros: levomepromacina, clotiapina, difenoxilato, me- 2. PROGRAMAS DE MANTENIMIENTO CON AGONISTAS


toclopramida, ondansetrn. OPICEOS:
o Indicaciones: Consisten en la toma crnica de opiceos suministrados
o Primera desintoxicacin. de forma legal para evitar las complicaciones de su uso
o Cuando haya resultado de utilidad en ocasiones ante- ilegal.
riores. o La METADONA es la sustancia de eleccin, se adminis-
o Si existe apoyo familiar y deseo intenso de abandonar tra nicamente por va oral.
drogas. o Buprenorfina+naloxona.
o Consumos de herona 250-750 mg/di'a. 0 Se ha utilizado herona y el LAAM (levoalfacetilmeta-
o Cuando no hay abuso. clol), similar a la metadona pero con vida media de 72
horas (se abandon por problemas cardiolgicos).
o TRATAMIENTO CON AGONISTAS OPICEOS: Indicaciones:
o METADONA: dosis inicial 20-50 mg e ir disminuyendo o Drogodependencias de aos de evolucin.
10-20% al da. c Complicaciones orgnicas graves.
o DEXTROPROPOXIFENO: Menor margen teraputico 0 Embarazadas.
que metadona (dosis superiores a 1200 mg/da pueden o Heroinmanos con patologa psiquitrica.
dar lugar a psicosis txica o crisis comiciales).
o BUPRENORFINA. PROGRAMAS DE REINSERCIN SOCIAL
0 AGONISTAS ALFA-2-ADRENRGICOS:
Programas de ayuda socio-laboral para integrarse plenamente
o Clonidina: disminuye Ia liberacin de NA por parte del en la sociedad.
SNC (locus coeruleus) que aparece aumentada tras su-
presin de opiceos. NOTA: ante un heroinmano que exige al mdico general un
o Ventaias: no tolerancia o dependencia, sedante, per- opiceo de sintesis para paliar su sndrome de abstinencia, la
mite el uso inmediato de antagonistas como naltrexo- actitud debe ser NO prescribir ningn frmaco que no entre
na. dentro del pacto teraputico y enviar a un centro especializado
o Inconvenientes: sedacin, astenia, hipotensin, bradi- (psiquiatra).
cardio.
o Contraindicaciones: hipotensin, IMAOs, arritmias, 2.4. Sndrome de abstinencia en el recin
insuficiencia renal, depresin grave, psicosis grave,
deterioro orgnico grave, embarazo.
nacido
o Lofexidina: Similar a clonidina, menos tendencia a hipo-
tensin Los recin nacidos de madres heroinmonas, Suelen ser
prematuros y de baio peso, con aumento de la morbi
0 PAUTAS DE DESINTOXICACIN CORTAS: mortalidad, pero tienen menor tasa de distrs respiratorio, ya
Mezcla variable de antagonistas opiceos (naloxona/naltrexona) que los opiaceos estimulan la produccin de surfactante.
+ clonidina + frmacos de tratamientos inespecficos. Aparece a las 48 horas de vida (algo mas tarde con metado-
na).
DESHABITUACIN Mortalidad sin tratamiento: 330%.
Clnica: Irritabilidad, temblor, llanto (80%), hiperreflexia,
1. PROGRAMAS CON ANTAGONISTAS DE OPICEOS: NAL- taquipnea, diarrea, hiperactividad (60%), vmitos (40%), es-
TREXONA. tornudos, bostezos, hipo (30%).
Tratamiento: Sedantes (fenobarbital, diacepam) u opiceos
INDICACIONES CONTRAINDICACIONES (metadona, paregrico -soluc. opicea-).
o Breve historia de consu- o Menores de 18 aos
mo de opiceos o Embarazo y lactancia MIR 98 (5839): Qu armacin de las siguientes, respecto a Ia
Pacientes motivados o Hepatopatas naltrexona es correcta?:

Dependientes de opice- o Politoxicmano (cuya princi- l. Es un frmaco sustitutiva de la herona. utilizado en progra-
os que Io soliciten pal droga de abuso no sea mas de mantenimiento.
o Pacientes que salen de el opiceo) 2. Es un agonista opiceo, utilizado en la desintoxicacin i de
otros programesterapu- o Antecedentes de trastorno herona. ,7
ticos (para prevenir nue- psiquitrico grave Es un sedante que se aplica como paliativo
' en el sndrome
l
vo consumo) de abstinencia de opiceos. _
o Pacientes abstinentes que Es un antaqonista de los ooiceos de accin prolongada,
vuelven al medio habitual que se utiliza despus de la cura de desintoxicacin.*
de consumo Es un antagonista de los opiceos de accin rapida, que se
o Pacientes con largos utiliza en el tratamiento de las sobredosis de narcticos.
periodos de abstinencia
que han sufrido recada
reciente
3. Cocana
ESTIMULANTE. Sus efectos euforizantes son breves y van
o Se busca eliminar los efectos placenteros de la droga. Antes seguidos del efecto rebote o crash: disforia, fatiga, depre-
hay que asegurarse de una desintoxicacin completa previa sin, ansiedad, deseo de ms cocana (craving).
para no provocar sndrome de abstinencia: previo al inicio Produce intensa dependencia psquica. Entre los sntomas de
del tratamiento con antagonistas opiceos, se practica un M abstinencia es posible encontrar hipersomnia, aumento del
de Naloxona (se inyectan 0,8 mg de naloxona s.c.), si en I apetito y estado de nimo disfrico.
hora no hay sndrome de abstinencia se puede usar el trata- Su vida media es de aproximadamente l hora.
miento con antagonistas.
c La naltrexona es un antagonista competitivo que se adminis- 3.1. Administracin
tra por va oral. Su metabolizacin es heptica. Su metaboli- Oral: masticando hoias de coca .
to el -betanaltrexol es activo y elimina va renal. Intranasal: esnifar clorhidrato de cocana. La mas frecuente.
o Baio seguimiento (10% a los meses). Intravenosa: slo clorhidrato de cocana o speedball (clor-
o Efectos indeseables: astenia, molestias gstricas, alteracin hidrato de cocana + herona).
del estado de nimo. Fumada: Pasta de coca (infrecuente en Espaa), cocana
base o crack (free-base smoking).
o Inhalar vapores calentados.
CURSO INTENSIVO MIR ASTURIAS

Pueden aparecer sntomas depresivos con ideas o comporta-



o La forma de administracin influye en la velocidad de apari mientos suicidas.
cin de efectos psicoactivos y en el desarrollo de dependen-
cia: fumada, inhalada e inyectada mayor velocidad y mayor 3.6. Complicaciones del uso
poder adictivo que intranasal (esnifada). crnico
3.2. Mecanismo de accin o Perforacin de tabigue nasal.
t Disminucin del fluio sanquineo cerebral demostrado en
o Produce efectos simpaticommticos. Inhibe la recaptacin de
los estudios de SPECT.
catecolaminas (dopamina y noradrenalina) y aumenta su li
o En suietos jvenes y con buena salud el consumo puede
beracin.
asociarse a IAM, palpitaciones, arritmias, muerte sbita
0 Su uso continuado lleva a la deplecin cerebral de dopamina por paro respiratorio o cardiaco y ACV.
y noradrenalina.
o Tos, bronquitis y neumona por irritacin e inflamacin de
o La tolerancia, aparece con el consumo repetido, indepen-
los teiidos de revestimiento del tracto respiratorio.
dientemente de la va de administracin. v Crisis comiciales.
3.3. Uso mdico Necrosis heptica.
o Prdida de peso y malnutricin por los efectos supresores
o Escasos. del apetito.
o Vasoconstrictor y anestsico local. Se ha usado en oftalmo- o Higerprolactinemia (impotencia, ginecomasta, ameno-
loga y ORL. rrea, galactorrea, esterilidad)
o Trastorno paranoide (delirio persecutoriol.
3.4. Intoxicacin aguda . Alucinaciones tctiles y visuales de insectos, con intenso
o Produce un estado de euforia, hiperactividad, inquietud, rascado y lesiones en la piel (formicacin o sndrome de
ansiedad, tensin, alerta, agitacin psicomotrizL sntomas Magnam).
manacos, as como sensacin de grandosidad y agresivi o Cuando est establecida la dependencia se producen una
dad. serie de cambios conductuales o psicolgicos que incluyen:
depresin, irritabilidad, anhedonia, falta de energa y asla
EFECTOS FSICOS: miento social.
Hiperactividad simptica
o Midriasis reactiva. 3.7. Tratamiento farmacolgico de la
o Hipertensin arterial.
o Taquicardia. dependencia
Hipertermia. DESHABITUACIN:
Hiperreflexia.
Sudacin o escalofros. o En Ia actualidad no hay estrategias teraputicas que sean
Nuseas o vmitos. claramente beneficiosas en el tratamiento de la dependencia
Puede haber movimientos estereotipados, dolor en el pe- por cocana.
cho, convulsiones, confusin, discinesias o distonas, len- a Se han propuesto diferentes frmacos con el fin de facilitar la
guaie incoherente o coma. abstinencia: antidepresivos tricclicos (imipramina), ISRS, litio,
a La causa ms frecuente de fallecimiento en intoxicacin algunos estimulantes y agonistas dopaminrgicos (bromo-
aguda seria la parada cardaca producida por arritmias e criptina, amantadina), frmacos anticraving para disminuir
HTA. deseo de consumo (naltrexona), antiimpulsivos (topirama-
o Efectos depresores como bradicardia, descenso de la TA y to)...
disminucin de la actividad psicomotora son menos frecuen- - Abordaje psicosocial terapia individual y grupal .
tes y solo aparecen con el consumo crnico de dosis altas.
MlR 98 FAMIIA (5591): Un hombre de 25 aos es conducido al
EFECTOS PSQUICOS: servicio de urgencias por su familia porque dice que "le persi-
. Delirios: autorreferenciales, de persecucin. guen unos asesinos que van a matarle. En la exploracin fsica
o Alucinaciones audtvas, visuales o tctiles (lo ms tpico se observan pupilas dilatadas, temperatura de 37.8 C, llO lpm
es la sensacin de sentir insectos baio la piel) y TA de 160/95 mmHg, sin otros hallazgos. La familia afirma
formicacin sndrome de Magnam-, con uicio de la que tiene historia de abuso de drogas. La drogaique ms pro-
realidad intacto (alucinosis). bablemente ha producido esta reaccin es:
Si las ideas delirantes o alucinaciones persisten fuera de la Alcohol;
intoxicacin aguda, se diagnostica de trastorno Qsictico indu- Cocana.*
cido por cocana. Diacepam.
Herona.
TRATAMlENTO DE LOS PROBLEMAS FlSlCOS POR lNTOXl-
WPF-N." Fenobarbital.
CACIN AGUDA:
o Convulsones: BZD (diacepam).
MIR 01 (7135) Entre los sntomas mas frecuentes de la intoxica-
Taquicardia: Propranolol.
cin por cocana N_O se encuentra:
Reaccin hipertensva: fentolamina. Midriasis.
Agitacin: antpsictico. Sntomas manacos.
Higotensin arterial.*
3.5. Abstinencia Taquicardia.
Se caracteriza por la presencia de un estado de nimo disf- 91.5 .0353. Agitacin psicomotriz.
rico acompaado de dos o ms de los siguientes cambios
fisiolgicos: MIR 14 (10388): Hombre de 28 aos que es trado a urgencias,
o Fatiga. un sbado en la madrugada, en estado de agitacin pscomotriz
o Sueo vividos y desagradables. y con ideas delirantes. SU acompaante refiere abuso previo, por
o Insomnio o hipersomnia. parte del paciente, de cocana. Qu signo/sntoma NO espe-
o Aumento del apetito. rara encontrar?
o Enlentecimiento o agitacin psicomotriz. Hi90terma.*
Puede haber anhedonia y craving (deseo irresistible de cocai- Taquicardia.
na). Midriasis.
Los sntomas de abstinencia se observan despus del consu- Diaforesis.
mo de dosis altas y repetidas. .UPPQFT' Dolor torcico.
VIII. SUSTANCIAS PSICOTROPAS

4.3. Alucingenos
4. Otros estimulantes
. El prototipo es el LSD (dietilamina del acido lisrgico). Otras
sustancias dentro del grupo de alucingenos son mescalina
4.1. Anfetaminas y psilocibina.
Similares a la cocana. Es ms comn el abuso episdico que la dependencia.
Mecanismo de accin: Liberacin de dopamina y noradrena- Se desarrolla tolerancia rpida (para efectos euforizantes y
lina presinpticas e inhibicin de su recaptacin. psicodlicos, no para los fsicos).
El uso crnico puede dar lugar a un trastorno por deplecin Hay tolerancia cruzada entre ellas.
de las catecolaminas. No hay descrito sndrome de abstinencia (no dependencia
usos MDICOS: fsica).
Narcolepsia. EFECTOS: producen alteracin de las funciones sensopercep-
Trastornos por dficit de atencin (metilfenildato, dextroanfe tivas. Una vez ingeridas producen dos tipos de sntomas:
famina). o Sntomas somticos: midriasis, temblor, febrcula, taqui-
Sndrome de hiperactividad en la infancia (metilfenildato, cardia, sudacin, vrtigo, parestesias, nuseas... (hiper-
dextroanfetamina). actividad simptica).
Depresiones resistentes y seniles. Sntomas psquicos;
EFECTOS DE LAS ANFETAMINAS: Alteraciones del humor, ansiedad, intensificacin subietiva de las
Los efectos de las anfetaminas, tienen relacin con la va de
percepciones, ilusiones, alucinaciones y sinestesias (sonidos
vistos), despersonalizacin, desrealizacin, todo ello en estado
administracin (oral, i.v., intranasal) y con la dosis.
de vigilia y alerta totales. Las alucinaciones producidas por esas
0 A dosis baias: Relaiacin, energa, autoconfianza, disminu-
sustancias reciben el nombre de psicodlicas y pueden acompa
cin de Ia fatiga, del sueo y del hambre, facilitan el apren-
arse de agitacin psicomotriz.
dizaie. Estos efectos aumentan con la dosis hasta un punto a
partir del cual provocan cambios conductuales desadaptati- o Efectos indeseables del LSD y resto de alucinaenos:
vos y patologa psiquitrica: agresividad, violencia, episodios o Crisis de panico o mal viae (el efecto indeseable mas
paranoides. frecuente).
Su uso crnico produce alteracin de la memoria y de la Cuadros depresivos.
atencin, embotamiento afectivo, fatiga, tristeza y retraimien- Episodios paranoides con conductas agresivas.
to social. Inicio de un trastorno psiquitrico de larga duracin (psi-
AI suspender el consumo sostenido de estas sustancias, apa- cosis crnica).
rece un efecto rebote (similar al que ocurre al suspender el Como resultado de una predisposicin individual.
consumo crnico de cocana) consistente en: disforia, ansie- La sintomatologa psquica producida por los alucinge-
dad, irritabilidad, fatiga y depresin. Tratamiento: BZD. nos puede aparecer mucho tiempo despus sin que haya
Intoxicaciones graves producen hiperactividad simptica: consumo, son los "flash-back" (reviviscencias). Aparecen
midriasisl taquicardia, hipertensin, sudacin, escalofros, en el 20% de los consumidores y su duracin es breve, de
nuseas, vmitos y cambios conductuales consistentes en: segundos.
angustia extrema, agitacin psicomotriz, agresividad y vio-
lencia. Pueden producir cuadros psicticos de tipo paranoide. MIR 99 FAMILIA (6111): A un ioven de 19 aos, sin anteceden-
Tratamiento sintomtico. tes de inters, se le lleva a Urgencias en un estado de angustia
Se presenta tolerancia a las anfetaminas. La disminucin del extrema, sudoracin, taquicardia, temblores, febrcula de
apetito y origina cetosis (acidosis) que implica mayor elimi- 37,4C y midriasis. La familia refiere que regres as a casa tras
nacin de anfetaminas. una salida nocturna. Desconocen habitos de consumo de txi
El uso de anfetaminas por va subcutnea se ha asociado a cos. El cuadro clnico orientara el diagnstico hacia:
infecciones por Eikenella corrodens. Intoxicacin por alucingenos.*
Trastorno por angustia con agorafobia.
MIR 91 (2847): La midriasis puede observarse en una de las Intoxicacin por opiceos.
siguientes situaciones clnicas: Intoxicacin patolgica por alcohol.
I. Sobredosis de opiceos. PPI"? Reaccin psictica aguda.
2. Intoxicacin grave por anfetaminas.*
3. Abstinencia tabquica.
4 Consumo diario superior a 20 mg de diacepam por espa- 5. Cannabis-marihuana
cio de meses como mnimo.
5. Tras la administracin de fentanilo. El compuesto activo es el THC (delta-9-tetrahidrocannabinol).
Existen receptores endgenos especficos. Los Iigandos end
4.2. Extasis (MDMA) genos (anandamidas) actan como neurotransmisores.
Formas:
o Derivado anfetamnico (Metletilendioximetanfetamina), psi- Marihuana: 1015% de THC.
coestimulante pseudoalucingeno, entactgeno y empat- Resina seca de Hachs o Kif: I 0% de THC.
geno. Aceite de hachs: 50% de THC.
Dosis oral efectiva: 75150 mg. Ms activo fumado: porro.
Inicio de los efectos: a los 30 min de Ia ingesta, aparecen efectos
simpaticomimticos: ansiedad leve, taquicardia, aumento de la 5.1. Sntomas fsicos
TA, hipertermia, hipercinesia, euforia.
o Los efectos txicos generales son mltiples, describindose: o Irritacin coniuntival.
crisis de hipertermia, deshidratacin, arritmias, colapso car- Taqucardia.
diovascular, crisis convulsivas, CID, rabdomiolsis, insuficien- Aumento del apetito, sequedad de boca.
cia renal y muerte. Se han descrito cuadros gsicticos crni-
c9; por uso prolongado (produce un efecto txico irreversible 5.2. Sntomas psquicos
sobre las neuronas serotoninrgicas del SNC).
Otros frmacos de este grupo: MDA (droga del amor) y Normalmente son leves:
MDEA (MDE o Eva). Relaiacin.
Euforia leve, ansiedad.
Sensacin de lentitud del paso del tiempo.
Deterioro de la capacidad de iuicio.
Aumento de la sociabilidad.

MLB.
CURSO INTENSIVO MIR ASTURIAS

o Disminucin de la capacidad de abstraccin y concentra-


cin, y del rendimiento intelectual. 5'4 consumo CTOIHCO
o Deterioro de la memoria, vigilancia y, rendimiento psicomo- EFECTOS SOMTiCOS:
tor. o Aumenta la frecuencia de tos crnica, sinusitis, faringitis,
o Percepcin distorsionada de las distintas partes del cuerpo, bronquitis, entisema y displasia pulmonar (la marihuana tu-
distorsin temporoespacal. mada contiene mayores cantidades de carcingenos que el
o Puede desencadenar psicosis txica o delirium. tabaco).
o Se puede producir aumento de peso.
5.3 intoxicacin aguda Las alteraciones biolgicas incluyen la supresin de la funcin
o Irrtacin conjuntival, hipertensin, taquicardia, cuadros inmunitaria y la supresin de la secrecin de testosterona y hor
psicticos, flashbacks. Tratamiento sintomtico (BZD). mona luteinizante.
EFECTOS PSQUICOS
o Disminucin de la capacidad de atencin, del rendimiento
intelectual y del aprendizaje.
o Anhedonia, sndrome amotivacional y letargia.
o Pueden producirse crisis de panico y cuadros psicticos.

. Resumen dro-as

. . . intoxicacin Antidoto o . . . . .,
SUSTANCIA , Abstmencra Desmtoxrcacron
Acan Aguda Tto
Agonistas opiceos
, . . . , . (metadona)
OPIACEOS Depresora MIOSIS Naloxona IV FiSica
Antagonistas opiceos (nal-
trexona)
Ansiedad, distaria, 4 , .
ATD ' agen te d
COCANA Estimulante Midriasis Sintomtico depresin (suicidio), T S. opaminergi-
. _ cos, anticravmg
apetito y sueno
ANFETAMINAS Estimulante Midriasis Sintomtico z coca (< intensidad) Benzodiacepinas
DROGAS SNTESIS EU'Fme
y alucmogena
Midriasis Sintomtico No
ALUCINGENOS Alucingena Midriasis Sintomatco No
. Relaiante
CNNABIS y alteraciones lsocricas Sintomtico Psiquica Psicoterapia
sensoperceptivas
BARBITRICOS Depresora Medias fiias Si ntomtico Ansiedad Fenoba rbital
BENZODIACEPINAS Depresora Flumazenl Ansiedad Diacepam

7. Trastornos psi-uitricos inducidos cor sustancias



4 ; . 4 213.12% Trastor- Tri-asian ' Trastorn . . '. "
-=Trast.or-'
SUSTANCIA 2:31:22: noirpsic nos del nos de "Zigzjt'es. nos del
v ''
ticos- . Y ' humor , ansiedad : FPEQ.
Alcohol X X X X X X

X X X
X X X X X
Camina X X X X X X
Mttahnasg X X X X X X
Drogas sintesis: X X X X
' x
X
AIcirtig X X x
Opioicies' X X X X X
Sdcints' X X X X X X X X X

8. Detecan de dro-as en orina 9. Alcohol


DROGA 7': TiEMPO-DE/DECEEIN o EI alcohol etlico se absorbe en un 20% en el estmago y el
Anfetaminos 1_3 das resto en el tracto intestinal. La absorcin aumenta cuando se
Barbitrcos mezcla con bebidas carbnicas o con el estmaqo vaco.
Vida med: com] 12 das o l gramo de alcohol, equivale a 7 Kcal (caloras vacas: sin
Fenobarbital AI menos 4 das nutrientes, ni vitamin08)-
BZD o Se detectan niveles plasmticos a los lO mn de la ingesta
Vida medio corta (triazolom) i da apareciendo el pico mximo a los 60 minutos de la ingesta.
Vida media larga (diacepam) 7 das o Atraviesa la barrera teto-placentaria y hemato-enceflica.
Cnnabis La Mim puede ser:
Consumo semanal 7-10 das o Urinaria
Cnsum dr 2'4 semanas o Pulmonar (por el aire espirado) S
Cocana (benzoilecgonina) 13 das . Sudoracin E
S
Opiceos Z)
Herona 1-3 das 9
La
D.
Metadona 2-4 das
es
MLB
Vlll. SUSTANClAS PSlCOTROPAS

- HGADO, 90%.
o Se produce una oxidacin no microsomal por la alco-
holdeshidrogenasa (consumo de NAD). La oxidacin es
constante entre 8-12 mI/hora, no depende de los nive-
les plasmticos.
o La oxidacin microsomal (microsomal etanol oxydation
system- M.E.O.S.), que se activa solo con concentracio-
nes muy altas, usa NADPH, y presenta autoinduccin
(aumenta un 30% con el consumo repetido).
o Una tercera va es la metabolizacin mediante la cata-
lgg en mitocondrias y peroxisomas, pero es poco im-
portante.
El alcohol potencia efectos de otros depresores: benzodia-
cepnas, barbitricos, ATD, antipsicticos. JAJA
cc (ml) de bebida x graduacin x 0,8 w lnlvn V-l \ll-! 13.. l>

gr. de etanol = EI cdigo de circulacin permite una alcoholemia mxima de 0,5 g/l
100 para la conduccin de vehculos.

9.1 . lntoxicacin La ausencia de signos de afectacin cerebelosa (incoordinacin


motora y ataxia) con niveles de alcohol en sangre por encima de
o Sobre el SNC: Pseudoexcitacin inicial (aumento de la socia l .5 g/l demuestra que existe tolerancia (directa).
bilidad, euforia) y posterior, depresin y coma. Depresin del La gastrectoma reduce Ia tolerancia, porque existe una absor-
centro vasomotor: hipotermia. cin ms rpda.
Sobre aparato cardiovascular: Vasodilatacin perifrica. o Existe tolerancia farmacocintica por activacin del sistema
Sobre aparato respiratorio: Depresor del centro respiratorio M.E.O.S. y formacodinmica en el SNC con reduccin de la
a dosis altas. respuesta neuronal a los efectos del alcohol.
o Sobre el rin: Acta como diurtico, inhibe la liberacin de o La tolerancia aumenta paulatinamente salvo en fases
ADH. finales (cuando existe afectacin heptica importante)
o Sobre el aparato genital: En varones estimula la libido y en las que pequeas dosis consiguen efectos despro-
disminuye la potencia sexual. En muieres produce relaiacin porcionados (tolerancia inversa).
uterina en dosis altas.

9.2. Intoxicacin aguda


Prueba ms especfica para medir consumo reciente de alcohol:
alcoholemia (concentracin de alcohol en sangre).

ETANOL EN'SANGRE , EFECTOS


Embriaguez subclnica a Aumento del tiempo de reac-
< 0.5 g/l cin
< 50 mg/dl o Menor precisin
Embriaguez ligera o a Euforia
excitacin o Locuacidad
0.5-1 .5 g/l o Agresividad
50150 mg/dl
Embriaguez grave o Aumento de los sntomas ante-
l.5-3.0 g/l riores
150-300 mg/dl o Ataxia
o Nistagmo
o Lenguaie farfullante
o Deterioro de la atencin y me-
mora nm lmunqxak? A lunas
. Clnica vegetativo: nuseas,
hipotensin, vmitos
TRATAMIENTO DE lA lNTOXlCACIN AGUDA:
Coma o Evolucin a estupor o coma o Sintomtico: observacin, control de la glucemia (sueros
3-5 g/l glucosados, si se precisan) y de la respiracin.
300-500 mg/dl o Ante la sospecha de alcoholismo crnico administrar tiamina
Zona letal o Puede ser mortal por depresin (vitamina Bl) con el suero glucosado (prevencin de Wernic
> 4 g/l del centro respiratorio o aspiro- keKorsakov).
> 400 mg/dl cin del vmito. o Si hay agitacin: BZD vida media larga (diazepam, clorace-
pato), antipsicticos.
Durante el perodo de embriaguez grave puede producirse una En casos extremos hemodilisis.
amnesia temporal: no recuerda lo ocurrido durante ese perodo.
lNTOXlCACIN ALCOHLICA IDIOSINCRTICA o BORRA-
CHERA PATOLOGlCA:
o Tras ingesta de poca cantidad de alcohol aparece un cuadro
de gran agitacin, clera y estado confusional (despropor-
cin dosis-respuesta) en individuos no dependientes.
o La duracin es de minutos a horas. Cesa con el sueo. Am-
nesia de lo ocurrido.
o Es poco frecuente, ms en abstemios, en personas con tras-
torno cerebral orgnico, epilepsia, ti personalidad, consumo
concomitante de hipnticos y / o sedantes.
CURSO INTENSIVO MIR ASTURIAS

AMNESlA LACUNAR (BLACK OUT O PALlMPESTO): CAGE (Cutting down, Annoyance by criticism, Guilt feelings,
o Amnesia lacunar respecto a un periodo de tiempo en el que Eye-openers).
el suieto estaba bebiendo aun sin mostrar signos de intoxica- 4 preguntas:
cin, la conducta parece normal. l. Ha tenido alguna vez la impresin de que debera beber
menos?
MIR 97 (541 l): Cul de las siguientes afirmaciones es cierta 2. Le ha molestado alguna vez la gente criticndole su forma
en relacin con la intoxicacin etlica aguda? de beber?
l. A partir de los 'IOO mg/dl de etanol en sangre existe riesgo 3. Se ha sentido alguna vez mal o culpable por su costumbre
de coma. de beber?
2. Las alcoholemias superiores a OO mq/dl se consideran 4. Alguna vez lo primero que ha hecho por la maana ha sido
potencialmente letales.* beber para calmar sus nervios o librarse de una resaca?
3. Los efectos del etanol sobre el sistema nervioso no se corre- o Posible alcoholismo: l tem positivo.
lacionan con su concentracin sangunea. o Probable alcoholismo: 2 ms.
4. El coma etlico puede ocasionar hiperventilacin, hiperter- Alcohol Clinical Index.
mia y alcalosis respiratoria. o 17 sntomas clnicos + 13 tems de anamnesis.
5. La broncoaspiracin del contenido gstrico es una compli-
c Probable diagnstico: 4 mas tems positivos en cada grupo.
cacin poco frecuente de este estado.
MALT (Mncher Alcoholismus Test).
o Parte obetiva: 7 tems.
9.3. Alcoholismo crnico Parte subietiva: 26 tems.
ETiOLOGA Puntuacin de 0-5: No alcoholismo
Multifactorial: Puntuacin de l 0: Sospecha de alcoholismo.
o Factores genticos: Puntuacin de l l ms: Alcoholismo confirmado.
Riesgo x 4 en familiares directos de un alcohlico. AUDIT (Alcohol Use Disorders Identification Test).
Gemelos: concordancia en monocigotos de 60% y en dcigotos o lO preguntas.
de 30%. o 2 en muieres 2 8 en varones: bebedor de riesgo (de sufrir
o Factores sociales. problemas derivados del consumo de alcohol o de desarrollar
o Factores psquicos. una dependencia).
o Z 13 (ambos sexos): problemas fsico-psquicos con la bebida
MIR 95 (4372): Indique el enunciado FALSO respecto al alco- y probable dependencia alcohlica.
'
holismo:
1. Parientes cercanos al alcohlico tienen un riesgo aproximado MARCADORES BIOQUMICOS:
de cuatro veces superior para padecer alcoholismo que la CONSUMO CRNICO:
poblacin normal. o Elevacin de enzimas hepticos:
No hay datos aue apoyen una posible influencia qentica.* o ALT/GPT.
coro. En el tratamiento de la abstinencia alcohlica es necesario o AST/GOT (elevaciones superiores a GPT).
una nutricin adecuada. o Cociente GOT/GPT > 2: muy sugestivo de consumo
En el tratamiento de la abstinencia se debe administrar tiami- crnico de alcohol.
na al menos durante una semana. o GGT.
Los benzodiacepinas tienen un margen elevado de seguridad o Se normaliza tras 3-4 semanas de abstinencia
en el tratamiento de la abstinencia alcohlica. (su cada en 5 das es patognomnica). Ms
sensible que VCM.
TlPOS DE ALCOHOLlSMO: o Falsos positivos: enfermedades hepatobiliares,
Primario: (Tipo A de Babor / ll de Cloninger) insuficiencia cardaca, diabetes, obesidad, taba-
- 70 80% de los casos de alcoholismo. quismo, frmacos inductores.
0 Ms frecuente en varones (10% frente 35%).
o Patrones de consumo: - VCM: Macrocitosis (con anemia o sin ella).
0 Mediterrneo (bebedor excesivo regular): Consumo o Elevado en bebedores por dficit de vitaminas del grupo
B as como por los efectos del alcohol sobre la eritropo-
prcticamente diario (carecen de capacidad de abs
yesis.
tinencia y no consiguen deiar de beber completa-
mente nunca). Embriagueces infrecuentes. Dependecia Aumento VCM + Aumento de GGT = Especifcidad
frecuente de alcoholismo del 75%
. Anglosain (bebedor excesivo irregular): Grandes intoxi-
caciones (no tienen capacidad de control cuando beben), c Elevacin de Transferrina Carbohidrato Deficiente (CDT). CDT
separadas por dias de abstinencia. Embriaguez frecuente. parmetro ms especfico.
Dependencia ms infrecuente. o Otras alteraciones:
Secundario: (Tipo B de Babor / l de Cloninger) o Aumento del cido rico
o Existe enfermedad psquica previa (trastornos afectivos, c Aumento de los triglicridos y colesterol por disminucin
ansiedad, esquizofrenia, trastornos de la personalidad), de la glucognesis
o Ms frecuente en muieres.
MONITORIZACIN DE ABSTINENCIA:
9.4. Diagnstico de alcoholismo o Tras 4 semanas de abstinencia deben disminuir o normalizarse:
0 AST/ GOT
El diagnstico de alcoholismo se hace en base a la historia c ini-
o ALT/GPT
ca. El resto de pruebas son de ayuda.
o GGT
UNlDAD DE BEBIDA ESTANDAR (UBE): o VCM
UBE (en Espaa) = 10 g de alcohol puro o La GGT y el VCM, seran las 2 pruebas ms sensibles en el
(1 consumicin de cerveza / vino equivale a l UBE). seguimiento de la abstinencia alcohlica.
Para pasar de UBEs a ml de alcohol multiplicar por 12,5.
LMITE DE RtESGO DE CONSUMO:
CUESTIONARlOS DE DETECClN: Consumo de riesgo: aquel que por su cantidad o circunstancia se
Brief MAST (Brief Michigan Alcoholismus Screening Test). asocia a una probabilidad superior a la sanitaria y socialmente
o lO preguntas. asumible de acarrear consecuencias adversas.
o Probable diagnstico de alcoholismo: ms. o Hombres: 40 g/da.
CBA / KFA (Cuestionario Breve para Alcohlicos). - Mujeres: 24 g/da.
o 22 preguntas.
n Probable diagnstico de alcoholismo: 5 ms.
VIII. SUSTANCIAS PSICOTROPAS

CONSUMO PERIUDICIAL DE ALCOHOL: Corazn de da de fiesta: taquicardia paroxstica tras


Consumo periudicial: ha dado lugar a problemas tsicos o psqui- un exceso de ingesta alcohlica sin otros datos de car-
cos relacionados con el consumo. diopata. Es reversible.
o Hombres: 60 g/da.
o Muieres: 30 g/dia. SISTEMA HEMATOLGICO:
Las muieres toleran peor el alcohol que los hombres por menor Anemia: las anemias pueden ser hemolticas (lesin dire-
presencia de alcohol deshidrogenada en la muier. cta de Ia membrana de los hematies) o megaloblstica
(dficit de Vt BIZ y cido tlico) que cursa con aumento
BEBEDOR MODERADO Y EXCESIVO: del VCM.
o Bebedor moderado: por debaio de los lmites de consumo de Neutropenia.
riesgo Trombopenia.
o Bebedor excesivo: bebe por encima de los lmites de consumo
de riesgo SISTEMA OSTEOMUSCULAR:
Se distinguen bebedor moderado y excesivo por Ia cantidad de Miopatia aguda proximal: se caracteriza por: mialgia
calorias que el alcohol aporta a la dieta. Cuando mayor del aguda, prdida de fuerza proximal y edemasi Puede
20%, se habla de bebedor excesivo. haber rabdomiolisis, hemoglobinuria e insuciencia renal
aguda. La miopata puede ser subclnica, encontrndose
9.5. Criterios diagnsticos un aumento de CPK y LDH.
Osteoporosis: el alcohol es la primera causa de osteopo-
ABUSO DEL ALCOHOL: rosis en varones. Se produce por mala absorcin de cal-
Patrn de consumo de alcohol en el que aparecen uno o ms de cio, vitamina D y otros nutrientes.
los siguientes durante 12 meses:
0 Consumo recurrente de alcohol que da lugar a incumpli-
miento de las obligaciones.
o Consumo recurrente de alcohol en situaciones en las que
hacerlo es fsicamente peligroso.
o Problemas legales por consumo de alcohol.
o Consumo continuado de alcohol a pesar de problemas
sociales continuos o recurrentes o problemas interpersona-
les causados o exacerbados por etectos del alcohol.
DEPENDENCIA DEL ALCOHOL:
o Son los mismos que los vistos en el tema anterior para
otras drogas.
o La presencia de temblor, malestar, o nauseas al levantarse
por Ia maana que desaparecen al beber alcohol, indica
que el diagnstico de dependencia es muy probable.

9.6. Alcoholismo crnico Cum Intensivo MIR Asturias

COMPLICACIONES FISICAS:
Las MS IMPORTANTES: cirrosis heptica, neuropatia perifrica SISTEMA NERVIOSO:
y miocardiopata. Enfermedad de Marchiafava-Bignami:
o Desmielinizocin del cuerpo calloso y la comisura
0 APARATO DIGESTIVO: blanca anterior.
o Hipertrofia parotdea. o Demencia lentamente progresiva, disartria, hipoci-
o Estago: refluio gastroesotgico y pirosis. nesia y descontrol de esfnteres.
Las nauseas y vmitos pueden dar lugar a un Sndrome o Se produce por carencias nutricionales en alcohli-
de Mallory- Weiss cos entre 45-60 aos con cirrosis heptica.
o Estmago: gastritis agudas que pueden evolucionar a Mielinolisis central pontina:
gastritis crnica y metaplasa. o Desmielinizacin de la base de la protuberancia.
o Pancreatits aguda que puede evolucionar a pancreatitis Poco trecuente y de causa desconocida. Obnubila-
crnica si no se detiene el consumo de alcohol. cin, cuadriparesia, paresia tacial y trastornos res-
o La talta de enzimas pancreticos y sales sndrome de ma- piratorios.
labsorcin, con carencia de Vit B, tolatos y grasas. o Mal pronstico.
- En ms del 50% de los alcohlicos aparece esteatosis Ambliopa alcohlica:
heptica. Puede ocurrir tambin hepatitis alcohlica, que 0 Atectacin del nervio ptico con disminucin pro-
puede ser asintomatica, aguda (dolor abdominal, icteri- gresiva de la agudeza visual y escotomas.
cia, Ieucocitosis, hepatomegalia, necrosis centrolobulillar Neuropatas del S.N. parasimptico:
y aumento de las transaminasas) o tulminante (esteatosis o lleo adnmico e impotencia sexual.
masiva, hemlisis e hiperlipidemia Sd de Zieve-). Altera- Degeneracin cerebelosa de predominio vermiano.
cin analtica caracterstica de la hepatitis alcoholica:
Esclerosis laminar de Morel.
GOT/GPT> 2.
o El 15% de los alcohlicos crnicos desarrollan una cirro-
NEUROPATAS PERIFRICAS:
o EI trastorno neurolgico mas frecuente en alcohli-
sis heptica. Entre el 10-50% de los pacientes con cirrosis
cos crnicos.
desarrollan un hepatocarcinoma.
o Es una polineuropata mixta sensitivo-motora de
o SISTEMA CARDIOVASCULAR: predominio sensitivo y en miembros interiores.
o EI etanol tiene etecto antiagregante en pequeas dosis. o Produce clinica de dolor y parestesias.
o La ingesta crnica excesiva de etanol eleva la TA. Se pro- o Abolicin de retleios, sobre todo los aquleos.
duce hipotensin si el consumo es ocasional. METABOLISMO:
o Favorece la fibrilacin auricular y las extrasstoles. o EI alcohol produce hipoglucemia a dosis altas por-
Miocardiopata alcohlica: hipertrofia ventricular izquier- que aumenta la secrecin de insulina.
da. La miocardiopata dilatada aparece con ms de IO
o Inhibe la neoglucognesis heptica (MIR).
aos de abuso.
a Aumenta la cetognesis, disminuye la lipolisis y la
o Los ACV en relacin con el alcohol se producen por: oxidacin de acidos grasos, produciendo hiperlipi-
HTA, arritmias, ICC y alteraciones de la coagulacin.
demias.
0 Trastornos en la sntesis de aminocidos por dficit
CURSO INTENSIVO MIR ASTURIAS

Entre 30-50% de los accidentes de trfico y el 17% de los


@
de vitamina B. laborales con vctimas mortales se atribuyen a la ingesta ex-
o La ingesta excesiva de alcohol aumenta Ia excre- cesiva de alcohol.
cin renal de: Mg, Ca y K. Esto se relaciona con
MIR 94 (3716): Un paciente alcohlico presenta durante Ia
algunos de Ios sntomas:
o Hipomagnesemia con sntomas de la absti-
convalecencia de una neumona pasada en su domicilio (Ia
familia dice que no ha bebido alcohol) un cuadro de confusin
nencia alcohlica.
y ataxia severa. En la exploracin se aprecia nistagmus horizon
0 Hpocalcemia con osteoporosis.
tal en oio derecho en la mirada hacia la derecha. Cul es su
o Hipopotasemia con arritmias cardacas.
opinin?:
INFECCIOSAS Y ONCOLGICAS: I. El paciente debe haber bebido aunque la familia Io nie-
o Efecto inmunosupresor celular y humoral. gue.
o Infecciones mas frecuentes que sufren los alcohli- 2. El cuadro es sugerente de encefaltis Imbica postinfeccio-
cos: Hepatitis B y C, T_B_C_, neumona por neumoco- sa.
cos G (-) y anaerobios, peritonitis, endocarditis y 3. Hay que descartar un hematoma subdural, de entrada.
meningitis bacteriana. 4. La clnica es sUgerente de degeneracin hepatocerebral.
o El alcoholismo es un factor de riesgo para cncer 5. Debe recibir tiamina con urqencia.*
de cavidad oral, faringe, esfago, estmago, hga-
do, colon y recto. TRANSTORNOS MENTALES CONCOMITANTES:
EFECTOS HORMONALES: SNDROM DE WERNICKE-KORSAKOV:
Urogenitales: En el sndrome deZWernicke-Korsakov, hay una fase aguda
o Dosis moderadas de alcohol aumentan el impulso (encefalopata de Wernicke) y un estado residual (Sndrome
sexual en varones pero disminuyen la capacidad de Korsakov), Etiologa: '
de ereccin. o Dficit de tiamina (vitamina BI). -
o Incluso en ausencia de trastorno heptico una mi- - La causa ms frecuente es el alcoholismo crnico.
nora significativa de varones alcohlicos crnicos o El alcohol interfiere con eI transporte activo intestinal de la
puede presentar atrofia testicular irreversible con BI.
reduccin concomitante de los tubos seminferos y o Las reservas de tiamina no superan las 3 sem. -
prdida de clulas espermaticas.
o En muieres puede aparecer amenorrea. Una dieta rica en hidratos de carbono o la administracin de
Endocrinolgicas: suero glucosado en un paciente con dficit Iarvado de BI greci-
o Hipercortisolema, I de ADH y de T3 y T4 (hipo- pita o aqrava la encefalopata de Wernicke-Korsakov.
funcin tiroidea).

SINDROME ALCOHLICO FETAL:


CLNICA de WERNICKE-KORSAKOV:
o Abortos espontneos y efecto teratgeno.
o Incluso la ingesta de cantidades moderadas de etanol
entraa un riesgo de que aparezca un sndrome al-
cohlico fetal que consiste en:
o Retraso del crecimiento prenatal (CIRcrecimiento
intrauterino retardado) y postnatal (talla pequea
persistente).
o Afectacin SNC: microcefalia, retraso mental,
Nistagmus y parlisis de motores
sndrome de hiperactividad y alteraciones de la con
oculares
ordinacin.
o Anomalas faciales: dismorfia facial, labio superior
Ata xi a
delgado con ausenciarde filtrum, nariz corta, hen-
diduras palpebrales pequeas y mandbula pe
quea, defectos en el esmalte dental.
Alte racin del
o Malformaciones cardiovasculares: CIA, CIV.
nivel de
o Malformaciones renales: hipoplasia renal, rin en
conciencia.
herradura.

I- ENCEFALOPATA DE WERNICKE:
o De forma aguda, aparecen:
0 Alteraciones de la motrica ocular extrnseca. La
parlisis cq-motora es complea, pero suele ser
mas precoz en motores oculares externos producin-
dose un estrabismo convergente.
Microcefalia, dismorfia facial, labio superior delgado'con ausencia de Ataxia e inestabilidad en bipedestacin.
filtrum, nariz corta, hendiduras palpebrales pequeas y mandbula pe- Trastornos psicoconductuales con obnubilacin y des-
quea orientacin.
o Puede haber trastornos relacionados con disfuncin
COMPLICACIONES SOCIALES del sistema nervioso autnomo: Taquicardia, hipoten-
El alcoholismo causa un deterioro social es causa de violen- sin, sncopes o hipotermia por afectacin hipotalmi-
cia dentro y fuera de la familia. ca.
Aparecen enfermedades, accidentes o complicaciones psi- o Frecuente asociacin con polineuropata.
quitricas, problemas econmicos.
Aunque la forma de comienzo tpica es brusca, algunos suetos
El comportamiento agresivo, Ia Iabiliclacl emocional, deterioro
desarrollan el dficit de forma insidiosa debido a los efectos re-
de Ia capacidad de uicio y la sexualidad inapropiada que ca-
petidos del txico y a la deficiencia nutricional.
racterizan a la intoxicacin alcohlica afectan a las relaciones
interpersonales.

Mi};
0
VIII. SUSTANCIAS PSICOTROPAS

2- PSICOSIS DE KORSAKOV (PSICOSIS AMNSICA O MIR 07 (8593) NEUROLOGA: En un paciente que presenta un
CONFABULATORIA): cuadro agudo de cuadro confusional, oftalmoparesia por
o Los Sntomas residuales ms frecuentes son: afectacin del sexto par bilateral y ataxia de Ia marcha pen-
o Ataxia residual. sara en:
o Amnesia de evocacin antergrada y amnesia de fiia- Encefalopata hiperglucmica.
cin (la ms afectada es la memoria reciente). Tam- Encefalopata de Korsakoff.
bin puede haber amnesia retrgrada variable. Infarto cerebeloso.
o Fabulacn: el paciente rellena con invenciones sus Administrarle inmediatamente tiamina.*
huecos de memoria como mecanismo compensato- PPNT' Intoxicacin por plomo.
rio.
MIR 14 (10307) NEUROLOGIA: Un varn de 52 aos presen-
KORSAKOV ATAXIA RESIDUAL ta deforma aguda un sindrome confusional, paresia de ambos
msculos rectos externos oculares y ataxia de la marcha pen-
sara en :
Encefalopata hiperglucmica
Encefalopata de Korsakoff
Infarto cerebeloso
Administrar inmediatamente tiamina*
.UPPN. Intoxiacin por plomo

. DEMENCIA:
o EI efecto txico del alcohol puede causar demencia.
o Ms del 60% de los alcohlicos desintoxicados tienen
deterioro cognitivo leve que no se puede considerar
demencia. EI deterioro meiora mucho en los 2 primeros
meses de abstinencia y luego mas lentamente.

. TRASTORNO PSICTICO PERSISTENTE INDUCIDO POR


ALCOHOL:
o Alteraciones sensoperceptivas: alucinaciones (auditivos
sobretodo, visuales, olfativas).
Falsos reconocimientos.
Curso Intensivo MIR Asturias
o Ideacin delirante:
o Paranoide: persecucin y periuicio (paranoia al-

TRATAMIENTO:
MI.
o Celos: el consumo crnico de alcohol es Ia primera
o Administracin de tiamina intravenosa. Inicialmente, 50 mg
causa de celotipia o celopata.
iv y 50 mg im. Seguir con 50 mg/da im hasta que el pa-
o En estos cuadros hay buen nivel de conciencia, los pa-
ciente se alimente bien. Es conveniente aadir sulfato de
cientes estn bien orientados, con memoria normal y
magnesio al tratamiento. sin deterioro de Ia capacidad de uicio.
o Lo que antes revierte son los trastornos de la motilidad ocu-
lar.
o Remisin total del cuadro cuando se trata en momentos ini- .Il
ciales Los sintomas residuales suelen responder peor. Curso Intensivo MIR Asturias
o Fallecen 17%.

MIR 88 (2130): La encefalopata de Wernicke es debida a un


dficit de:
Vitamina C.
Vitamina A.
Vitamina BI.
Vitamina B.
91:59.? Vitamina E.

MIR FAMILIA 00 (6625): Cul de Io's siguientes datos es una


caracteristica principal de Ia encefalopata de Wernicke?:
Inicio agudo.*
Asociacin a crisis comiciales alcohlicas.
Asociacin a amnesia retrgrada.
Tratamiento eficaz con cido flico.
PPPNT Asociacin a alucinaciones visuales.

MIR 07 (8704): Un paciente bebedor habitual de alcohol es


llevado a urgencias con un cuadro de desorientacin, dificul-
tad para recordar hechos de los das anteriores, errores en los
razonamientos, marcha torpe y una desviacin en los oios
divergente que antes no tena. Segn parece, ha estado be- OTROS TRASTORNOS MENTALES INDUCIDOS POR EL ALCO-
biendo alcohol hasta unas horas antes de llevarlo a urgencias. HOL:
Probablemente se trate de un caso de: 0 Cuadros amnsicos: amnesia en bloque (amnesia total
Sndrome de abstinencia complicado. entre dos momentos cronolgicos), amnesia lacunar (sin
Enfermedad de Korsakoff. lmites bien definidos, con retazos de memoria).
Encefalopata de Wernicke.* o Conductas de fuga y estados crepusculares: el suieto efecta
AIucinosiS alcohlica. desplazamientos sin saber, posteriormente, cmo ni por qu
.UPPJNT Intoxicacin etlico aguda. ha llegado a su destino.
Trastornos depresivos.
0 Suicidio (alcohol 2 causa de suicidio).
CURSO INTENSIVO MIR ASTURIAS

o Trastornos de ansiedad generalizada, trastorno por angus o Suele comenzar 3-4 das despus de interrumpir o disminuir
tia, fobia social. La fobia social y la agorafobia suelen pre- la ingesta y puede evolucionar a la remisin completa o a la
ceder al alcoholismo mientras el trastorno de pnico y el muerte (en 2% de los tratados y en un 20% de los no trata-
trastorno de ansiedad generalizada suelen ser secundarios dos).
a esta adiccin. o Sntomas:
o Trastornos del sueo (insomnio, pesadillas). o Hiperactividad autonmica: sudacin, taquicardia, hi-
o Otras adicciones: nicotina, tranquilizantes, cocaina. pertermia.
o Temblor en manos, lengua y prpados.
MIR 95 FAMILIA (4098): Un varn de 48 aos con una larga Ansiedad.
historia de dependencia crnica del alcohol es ingresado en un o Insomnio.
servicio de Psiquiatra porque me persiguen homosexuales, o A medida que avanza se produce deterioro del nivel de
que me llaman perverso (sexual), al pasar delante de los bares conciencia.
oye comentarios en su interior que dicen que es marica, traves-
o Alucinaciones visuales tctiles (zoopsiasll.
tido y que van a ir a por l. Todo comenz despus de haber
o Delirio ocupacional: el paciente acta y gesticula como
estado bebiendo mas alcohol del habitual durante unos das. A
si realizase su propio trabaio.
la exploracin esta tembloroso y muy asustadizo, bien orienta
o Crisis comiciales: en 5-1 5%.
do, con una memoria normal y sin alteraciones de la capacidad
o La privacin sensorial (oscuridad) acta como desencade-
de uicio. El diagnstico mas probable:
Intoxicacin patolgica alcohlica. nante de las alucinaciones.
Consumo periudicial de alcohol. o El aumento de la temperatura puede estar presente en cual-
Trastorno psictico alcohlico.* quier cuadro de privacin de sustancias, en el caso del al-
Esquizofrenia paranoide. cohol, tiene especial mal pronstico.
019 0307- Sndrome amnsico alcohlico (de Korsakoff). Durante la intoxicacin o abstinencia del alcohol concurren con
frecuencia ciertas circunstancias que pueden producir deterioro
MIR O'I (7138) Las alucnosis son esencialmente tpicas: del nivel de conciencia y que deben tratarse especficamente.
De la esquizofrenia hebefrnica. Estas son:
De los cuadros afectivos bipolares. - Hipoglucemia.
Del alcoholismo.F Traumatismo craneal.
De la anorexia mental. Hematoma subdural.
.UPFNT' De la infancia. Hemorragia digestiva.
Encefalopata heptica.
MIR 00 (6121) Cual de los siguientes trastornos se asocia con Neumona.
MENOR frecuencia al consumo de alcohol? Dficit de tiamina.
l. Trastorno antisocial de la personalidad.
2. Episodio depresivo.
3. Trastorno obsesivo-compulsivo. *
4. Fobia social.
5. Trastorno de ansiedad.

9.7. Cuadros asociados a la


disminucin/cese de ingesta de alcohol
o Los antecedentes de elevada tolerancia al alcohol predicen la
aparicin de sintomas de abstinencia graves.
ALUCINOSIS ALCOHLICA:
o Vvidas alucinaciones (auditivos) tras abandono / disminucin
de ingesta alcohlica en personas con dependencia (aunque
tambin puede aparecer en fases de aumento de consumo).
o Suelen ser voces de contenido amenazante y existe respuesta
apropiada ante el contenido de las alucinaciones (respuesta
ansiosa, comportamientos defensivos").
o Suele haber claridad del sensorio (a diferencia del delirium).
o Duracin menor de meses.
SNDROME DE ABSTINENCIA:
o Interrupcin (o disminucin) del consumo de alcohol des
pus de su consumo prolongado y en grandes cantidades,
Curso Intensivo MIR Asturias
o Dos o mas de los siguientes sntomas desarrollados horas o
dias despus de cumplirse el criterio A:
o Hiperactividad autonmica (p.e. sudacin o taquicar- TRATAMIENTO DEL SINDROME DE ABSTINECIA / DELIRIUM
dia superior a IOO pulsaciones/min.). TREMENS:
Temblor distal de las manos.
o El tratamiento de eleccin son las benzodiacepinas de vida
o Insomnio.
media larga (clorazepato, diazepam). Tienen un elevado
Nuseas o vmitos.
margen de seguridad en estos enfermos.
o Alucinaciones visuales (zoopsias: bichos), tctiles o
o Clormetiazol o tiapride, tambin son tiles.
auditivos transitorias (3-1 0%), o ilusiones.
o Debe de adiministrarse tambin Tiamina (o preparados
o Agitacin psicomotriz.
multivitamnicos con vit. BI) para una adecuada prevencin
o Ansiedad.
de sndrome de WernickeKorsakov.
o Crisis comiciales de gran mal.
Las manifestaciones de la abstinencia, especialmente el temblor,
suelen hacerse patentes 412 horas despus de interrumpir la
ingesta. Los sntomas alcanzan su maxima intensidad entre 24-
48 horas ms tarde y tienden a desaparecer al 4 5 da.
DELIRIUM TREMENS:
VIII. SUSTANCIAS PSICOTROPAS

MIR 98 (5837): Un mdico de 40 aos ingres como conse- destacar en las visitas las reas de la vida del paciente que han
cuencia de una fractura de fmur producida en un accidente de meiorado al dejar el alcohol. Despus de la rehabilitacin el
trafico. Se practic ciruga ortopdica sin complicaciones y, du 60% de los alcohlicos de clase media mantiene la abstinencia
rante los dos primeros das del postoperatorio, el paciente estuvo al menos l ao y muchos de por vida.
inquieto pero bien orientado. Al tercer da, el paciente se mostr
de repente confuso y temeroso y refiri visin de serpientes y TRATAMIENTO FARMACOLGICO:
escorpiones que se movan por las sbanas. En la exploracin se o INTERDICTQRES (MIR): DISULFIRAM (ANTABS) Y CIA-
observ midriasis, temblor de oscilaciones amplias de las ma- NAMIDA CALCICA
nos y los prpados, sudoracin profusa, taquicardia con latido o Ambos producen un acumulo de ACETAL-
hiperquintico y temperatura de 39 C. Los antecedentes perso DEHIDO al inhibir la aldehido deshidrogenasa
nales carecan de inters. Se consideraba un bebedor social (disulfiram produce inhibicin irreversible y cia-
moderado y negaba el abuso de otras drogas. De los siguientes, namida clcica reversible). Al ingerir alcohol, se
cul es el diagnstico que mas sugiere esta situacin? : produce importantes efectos histaminrgicos
l. Delirio por deprivacin alcohlica.* (efecto antabs): vasodilatacin, nuseas,
Delirio postanestesia. vmitos, enroiecimiento, disnea, palpitaciones,
Septicemia. hipotensin, ansiedad y visin borrosa. En ca-
Hematoma subdural. sos extremos: arritmias, lAM, o convulsiones.
PPP!" Uremia. No es conveniente la prescripcin sistemtica.
o Cuando se administra no debe haber habido
MIR O2 (7400): Cul de las siguientes respuestas sobre el Deli- ingesta alcohlica en al menos las 24 horas
rium Tremens es la correctaZ: previas.
l. Se trata del estado peculiar de intoxicacin producido por el o La cianamida tiene un efecto ms rpido y ms
consumo de alcohol. breve (l da) que el disulfiram ( das).
2. Se caracteriza por la aparicin de conductas impulsivas tras o Tratamiento efecto antabus: antihistamnicos /
el consumo de una pequea cantidad de alcohol. corticoides
3. Es un cuadro clnico con elevado ndice de complicaciones o Contraindicaciones absolutas:
psiquitricas y con pocas complicaciones orgnicas. o Disulram: Enfermedad cardiovascular
4. Aparece con mayor frecuencia entre los hombres alcohli- grave, embarazo, psicosis.
cos a partir de los 50 aos. o Cianamida: Enfermedad cardiovascular
5. Su tratamiento de eleccin son las benzodiacepinas.* grave.
o Contraindicaciones relativas:
9.8. Tratamiento del alcoholismo o Disulfiram: Cirrosis, bronquitis crnica, di-
abetes mellitus, hipotiroidismo, epilepsia,
DESINTOXICACIN insuficiencia renal.
Es el primer paso para la deshabituacin de un paciente alcoh- o Cianamida: Insuficiencia renal, insuficien-
lico. cia respiratoria.
. Frmacos que pueden producir efecto antabus
No es necesaria la hospitalizacin. solo en casos de: (al consumir alcohol de forma concomitan te):
o Historia de convulsiones antibiticos (cefalosporinas, cloranfenicol, gri-
o Historia de delirium tremens. seotulvina, ketaconazol, isoniacida, metronicla-
o Historia de tolerancia y consumo muy elevados. zol, tetraciclinas).
o AbLiso de otras sustancias psicoactivas.
A
o Complicaciones somticas graves.
0 Comorbilidad con otros trastornos mentales o riesgo de
suicidio. Vaso dilatacin
o Fracasos repetidos en rgimen ambulatorio.
/ Enroecmiento
La desintoxicacin consiste en la retirada brusca y total del alco-
Ansiedad
hol, por lo que debeprevenirse el sndrome de abstinencia con
psicotrmacos sedantes a dosis altas como: Benzodiacepnas
(diacepam, cloraZepato), clometiazol o tiaprideQ
o Estos frmacos tambin protegen de las convulsiones y
tambin seran tiles para el tratamiento
V del delirium tre-
mens establecido. ' A
g
o Si aparecen alucinaciones o delirios: haloperidol o similar
(cuidado porque los neurolpticos disminuyen el umbral Palpitacones
convulsivo).
o En caso de necesidad de antidepresivos, los ISRS estn ms , . \C
Vomitos
indicados que los tricclicos.

En el tratamiento de la abstinencia alcohlica y en su prevencin - FRMACOS QUE REDUCEN EFECTOS PIACENTEROS Y


es necesario una alimentacin adecuada que incluya suplemen- CRAVING:
tos de tiamina al menos l semana. 0 NALTREXONA (antagonista opiceol:
o Bloquea receptores opioides evitando los
DESHABITUACtN Y REHABILITACIN efectos placenteros del alcohol.
No se recomienda su uso s:
Se comienza esta fase una vez superada la de desintoxicacin.
Transaminasas elevadas x3.
Es prolongada. El riesgo de recada persiste mucho tiempo des-
Bilirrubina total elevada x2.
pus de interrumpir el consumo.

TRATAMlENTO PSICOSOCIAL o ACAMPROSTATO: bloquea al sistema gluta-


matrgico y disminuye el deseo de beber
Terapias: cognitivo-conductuales, de pareia y familiar, de grupo, cuando el alcohlico se expone a estmulos
grupos de autoayuda (alcohlicos annimos). No hay una que aumentan su deseo.
frmula que sea claramente mejor.
El contacto del paciente con el mdico o grupos de apoyo debe o FARMACOS QUE REDUCEN IA IMPULSIVIDAD: TOPI-
establecerse al menos 6 meses aunque es preferible un ao. Se RAMATO.

Wi
CURSO INTENSIVO MIRASTURIAS

9.9. Urgencias psiquitricas en


alcoholismo
o Enfermedad de Wernicke-Korsakov.
o Alucnosis alcohlica.
o Celotipia alcohlica.
o Sndrome de abstinencia (delirium tremens, crisis convulsi-
vas).
o Embragueces patolgicos.
o Encefalopata heptica.
o Tentativas de suicidio.

MIR 06 (8441): Una mujer viuda de 75 aos que vive sola acude
a consulta. Padece de hipertensin arlerial e hiponatremia leve
por diurticos. Refiere encontrarse bien y nicamente se queia de
pirosis. Preguntada sobre la ingesta de alcohol, dice tomar unas 4
copas al da. La tensin arterial esta mas elevada que hace l ao.
La puntuacin en el cuestionario "CAGE'" es 1/4 pero no tiene
criterios de dependencia de alcohol segn el DSM-IV. Afirma que
desea disminuir la ingesta de alcohol. Tres meses ms tarde dice
haber meiorado algo pero anhela el alcohol. Solicita alguna me-
dicacin que la ayude Cul de los siguientes es el tratamiento
mas apropiado?:
Busprona.
Disulfrarn.
Fluoxetina.
Litio.
.UPPNT' Naltrexona. *

MIR 11 (9763) Cmo actan Ia Cianamida o el Disulfiram en un


paciente alcohlic03:
1. Reducen el efecto del OH en el organismo, disminuyendo el
posible efecto positivo de un nuevo consumo.
2. Incrementan el efecto txico del OH, desmotivando al nacien-
te de cara a nuevos consumos*.
3. Anulan el efecto del OH en el organismo, actuando como
antagonistas.
4. Por su efecto estabilizador del animo posibilitan un mayor
control del consumo.
5. Ocupan los receptores GABA, reduciendo el "craving".

MlR 12 (9935): Cmo actan la Cianamida o el Disulfiram en un


paciente alcohlico?:
l. Reducen el efecto del alcohol en el organismo, disminuyendo
el posible efecto positivo de un nuevo consumo.
2. Incrementan el efecto txico del alcohol, desmotivando al Da-
ciente de cara a nuevos consumos*.
3. Anulan el efecto del alcohol en el organismo, actuando como
antagonistas.
4. Por su efecto estabilizador del nimo posibili-tan un mayor
control del consumo.
5. Ocupan los receptores GABA, reduciendo el ansia de consu-
mo.

en
VIII. SUSTANCIAS PSICOTROPAS

RESUMEN DE T. MENTALES DEBIDO AL CONSUMO DE SUSTANCIAS

Droga: sustancia que produce alteracin de la conducta o del estado psquico.


Intoxicacin: sndrome reversible que se caracteriza fundamentalmente por la aparicin de cambios psicolgicos o compor-
tamentales desadaptativos y fisiolgicos. (MIR)
Tolerancia: disminucin de los efectos de una sustancia tras la administracin repetida de Ia misma (MIR).
Dependencia (criterios del DSM-IV-TR): necesaria Ia presencia de 3 o ms tems en algn momento de un perodo continuado
de 12 meses.
o Tolerancia
Abstinencia
La sustancia se toma en cantidades mayores o durante mas tiempo del que se haba previsto.
Deseo persistente o esfuerzos infructuosos de controlar o disminuir el consumo.
Se emplea mucho tiempo en actividades relacionadas con Ia obtencin de dicha sustancia.
Reduccin importante de las actividades sociales, laborales o recreativas.
OOOOOO Se contina tomando Ia sustancia a pesar de Ios problemas psicolgicos o fsicos causados por la misma.
EI abuso o consumo periudicial se define como un patrn desadaptativo de consumo que no llega a cumplir los criterios dia-
gnsticos de la dependencia (MIR).
Sndrome de abstinencia: sndrome especfico para cada sustancia que aparece despus de interrumpir o disminuir el consu-
mo tras una ingesta crnica (MIR). Causa un malestar clnicamente significativo y deterioro de Ia actividad social o laboral.
o Agudo: sntomas y signos orgnicos y psquicos que aparecen inmediatamente despus de interrumpir el consumo. Dura
4-10 das.
o Tardo: disregulacin del SNA y de las funciones psquicas, persiste largo tiempo, contribuye a las recadas.
o Craving: deseo imperioso de consumir la sustancia psicoactiva (dopamina)
Sndrome amotivacional: astenia, apata, falta de inters, reduccin generalizada de cualquier actividad y dficit de funciones
psquicas basicas. En relacin con cualquier droga. Dificulta Ia deshabtuacin y la reinsercin social.
Tratamiento de Ias drogodependencias:
o Desintoxicagin: conseguir que el paciente deie de consumir droga. Se consigue entre 7 y I4 das.
o Deshabituacin: proceso largo y compleio en el que se pretende que desaparezca el sndrome de abstinencia tardo. EI
paciente aprende una serie de estrategias que le permitirn no recaer en el consumo.
o Reinsercin: se solapa con el anterior, integracin del individuo en el medio social que le corresponde.

I. OPICEOS
Agonistas puros: morfina, herona, metadona, meperidina, fentanilo, codena, dextropropoxifeno.
Agonstas parciales: buprenorfina, tramadol
Agonistalantagonista (MIR): nalorfina, pentazocina, levalorfano, ciclorfano
Antagonistas (MIR): naloxona, naltrexona
OI... El fentanilo, tiene Ia vida media ms corta y la mayor potencia. La metadona tiene 3 veces ms potencia que la morfina y una
vida media larga (24 horas).
Usos mdicos: Morfina: edema agudo de pulmn (MIR) y dolores intensos incluyendo etiologa cancerosa (MIR); Codena y Lo-
peramida: antidiarricos; Codena y Dextropropoxineno: antitusgenos. No se deben usar en clicos biliares pues contraen el
esfnter de Oddi (MIR).

1.1. lNTOXICACIN AGUDA POR OPICEOS (SOBREDOSIS)


Causa ms frecuente de muerte en ADVP.
Hipoactividad simptica: Hipo / Bradi + Miosis, deterioro del nivel de conciencia que puede llegar al coma (MIR9) y depresin
respiratoria por edema agudo de pulmn (MIR).
Tratamiento: naloxona i.v. (MIR) (tiene menor vida media que la mayora de los opiceos).

1.2. TRASTORNOS POR CONSUMO CRNICO


Por el opiceo: estreimiento, depresin de Ia funcin inmunitario, sndrome nefrtico.
Por los adulterantes: neuropata perifrica, amiopa, mielopata.
Por la va de administracin (iv): Infeccin de partes blandas por estafilococo, hepatitis B, C, delta (MIR), SIDA, endocarditis de-
recha por estafilococo aureus (MIR) + valvula tricspide mas frecuentemente afectada y embolismos pulmonares frecuentes
(MIR), flebitis sptca, abscesos cerebrales, artritis por pseudomona (MIR).

1.3. SNDROME DE ABSTINENCIA A OPICEOS: INOCUO SI NO EXISTE ENFERMEDAD GRAVE SUBYACENTE.

Inicio: 6-12 horas despus de Ia ltima toma, intenso: 2448 horas y desaparece: 8-10 das despus.
Hiperactividad simptica: Hiper / Taqui + Midriasis, rinorrea, lagrimeo, insomnio, sudacin, bostezos, temblor, dolor abdo-
minal, inquietud, piloereccin, agitacin, vmitos.
Tratamiento: desintoxicacin.

1.4. TRATAMIENTO DE IA DEPENDENCIA A OPICEOS

Embarazadas siempre metadona


Desintoxicacin:
o Tratamientos inespeccos: BZD, analgsicos, antidiarricos, antiemticos....
o
.
Agonstas opiceos: dosis decrecientes de metadona o detropropoxifeno (menor margen teraputico).
CURSO INTENSIVO MIR ASTURIAS

o Agonistas alfa-Q-adrenrgicos: clonidina (MIR). Su efecto secundario ms frecuente es la hipotensin. Es preciso el ingreso
hospitalario.
o Pautas cortas / ultracortas: naloxona / naltrexona + clonidina + frmacos de tratamientos inespecficos.
o Deshabituacin:
o Proqramas con antaaonistas de opiceos (Naltrexona). Consisten en tomar de forma crnica un antagonista opiceo pa
ra no notar efectos placenteros en caso de ingesta de opiceos. Hay que asegurarse una buena desintoxicacin antes del
inicio del tratamiento para no producir un sndrome de abstinencia (MIR).
o Proqramas de mantenimiento con aaonistas opiceos
o Cuando no es posible la abstinencia a opiceos, se incluye a los pacientes en estos programas para evitar las com-
plicaciones legales y mdicas del uso de opiceos.
o La metadona es la sustancia de eleccin, se administra nicamente por va oral (MtR). Se indica cuando ha habido
fracasos teraputicos repetidos en tratamientos de desintoxicacin o tras estancias en comunidades teraputicas, en
embarazadas y pacientes con complicaciones orgnicas y/o psquicas graves.
o Ante un heroinmano que exige al mdico general un opiceo de sntesis para paliar su sndrome de abstinencia, la
actitud ser la de no prescribir ningn frmaco que no entre dentro de un pacto teraputico y enviar a un centro es-
pecializado (MIR).
o Programas de reinsercin social

2. COCANA
. Psicoestmulante que produce intensa dependencia psguica.
o Vida media corta (l hora) con mucho potencial de adiccin. Administracin: esnifada, fumada o inhalada (crack) (MIR), inyec-
tada (las tres ltimas formas TTefectos psicoactivos y tienen un intenso potencial de adiccin).
o AI desaparecer los efectos aparece efecto rebote o crash: disforia, fatiga, ansiedad, que lleva al suieto a consumir ms droga.

2.1. MECANISMO DE ACCIN


o lnhibe la recaptacin de catecolaminas y aumenta su liberacin a nvel sinptico. Produce efectos simptico-mimticos. Su uso
crnico lleva a la deplecin cerebral de dopamina y noradrenalina.

2.2. INTOXICACIN AGUDA POR COCANA


o Estado de euforia, agitacin psicomotriz y sntomas maniacos. Deterioro de la capacidad de iuicio, agresividad y sensacin de
grandiosidad.
o Efectos fsicos: midriasis reactiva, taguicardia, hipertensin arterial, sudacin, hpertermia, hiperreflexia, nuseas. (MIR) .
o Efectos psquicos: cuadros psicticos (delirios autorreferenciales o persecutorios) (MIR9), alucinaciones tctiles (sentir insectos
debaio de la piel) formicacin o sndrome de Magnam-, dependen de la susceptibilidad individual.
o Tratamiento: Sntomtico. Convulsiones: diacepam. Taquicardia: propranolol. Agitacin: clorpromacina...

2.3. uso CRNICO


o El uso crnico puede producir trastorno paranoide (ideas delirantes persecutorias), formicacin, perforacin del tabique nasal,
disminucin del fluio sanguneo cerebral, aumento de la incidencia de cardiopata isqumica o ACV, hiperprolactinemia que
causa impotencia, ginecomastia, galactorrea (MIR)
0 Cuando se establece la dependencia se producen cambios conductuales: depresin, irritabilidad, anhedonia, falta de energ-
IG...
o Tratamiento: no existe un tratamiento claramente definido. Se han usado antidepresivos triciclicos como la imipramina (MIR),
ISRS, agonistas dopaminrgicos (bromocriptina, amantadina), anticraving...

3. OTROS PSlCOESTlMULANTES

3.1. ANFETAMINAS
o Mecanismo de accin: Liberacin de dopamina y noradrenalina presinpticas e inhibicin de su recaptacin (MIR).
o Intoxicacin aguda: efecto similar a cocaina (MIR). Tambin pueden producir cuadros psicticos paranoides, esquizofrenifor-
mes... Tratamiento sintomtico.
o Efectos:
o A dosis baas: Relaiacin, energa, disminucin de la fatiga, sueo y hambre.
o Dosis mayores: Pueden producirse cuadros desadaptativos y sntomas psicticos (segn la susceptibilidad individual).
o Su uso por va subcutnea se asocia a infecciones por Ekenella corrodens (MIR).
o Uso crnico: alteracin de la memoria, depresin, irritabilidad, anhedonia, falta de energa...
- Tratamiento abstinencia: BZD.

3.2.XTASlS
o Efectos psicoestimulantes (ansiedad, taquicardia, aumento de la TA, hpertermia, midriasis, hipercinesia, euforia...), pseudoa-
lucingenos y empatgenos.
o Posibilidad de cuadros psicticos crnicos por el uso prolongado.

4. ALUCINGENOS
o No produce dependencia fsica. Las alucinaciones producidas por estas sustancias reciben el nombre de psicodlicas y se
acompaan de agitacin psicomotora.
o Intoxicacin aguda: midriasis, taquicardia, hipertensin, temblor, alucinaciones, crisis de pnico (efecto indeseable ms fre-
VIII. SUSTANCIAS PSICOTROPAS

cuente), psicosis... Tratamiento sintomtico.


o Efectos:
o Sntomas somticos: midriasis, vrtigo, debilidad, temblor, nuseas, febrcula, taquicardia, sudacin, parestesias (MIR).
o Sntomas psquicos: Angustia extrema, trastornos de la percepcin y fenmenos alucinatorios mltiples (MIR), despersona-
lizacin, desrealizacin...
o Posibilidad de flashbacks: reexperimentacin breve de la sintomatologa psquica mucho tiempo despus del consumo. Apa-
rece en el 20% de los consumidores.
o Puede producir psicosis crnica.

5. CANNABIS
No produce dependencia fsica.
Intoxicacin aguda: pupilas isocricas, irritacin coniuntival, hipertensin, taquicardia, cuadros psicticos, flashbacks... Tra-
tamiento sintomtico (BZD).
. Efectos:
o Fsicos: irritacin coniuntival, taquicardia, T apetito, sequedad de boca...
o Psguicos: euforia leve, sensacin de bienestar, relaiacin, aumento de la sociabilidad, mayor sensibilidad a estmulos
sensoriales, distorsiones perceptivas...
o Uso crnico: ansiedad, crisis de pnico, cuadros psicticos, sndrome amotivacional...

6. BENZODIACEPINAS
o Tolerancia cruzada con el alcohol y con los barbitricos.
o Intoxicacin aguda: somnolencia, dsartria, depresin SNC, ataxia... Tratamiento: flumacenl v
o Desintoxicgcin: diacepam (dosis decrecientes).

7. BARBITRICOS
o lntoxicacin aguda: pupilas medias ias, somnolencia, dsartria, confusin mental, coma barbitrico, posible crisis de porfira
aguda intermitente... Tratamiento: interrupcin progresiva del barbitrico y sustitucin por BZD. Provocar diuresis y alcalinizar
orina.
o Desintoxicacin: fenobarbital (dosis decrecientes).

8. ALCOHOLISMO
o El alcohol de abuso es el etanol. Absorcin: en duodeno y yexuno proximal. Eliminacin: urinaria, pulmonar, sudor y la ms
importante: heptica 90% (sobre todo por oxidacin no mcrosomal).

8.1. INTOXICACIN AGUDA

o Prueba mas especfica para medir consumo reciente de alcohol: alcoholemia (concentracin de alcohol en sangre).
o Sean la cantidad de alcohol en sangre:
o Embriaguez subclnica (< 0.5 qr/I): alteracin del tiempo de reaccin, menor precisin.
o Embriaguez ligera o excitacin (0.5-1 .5 grl): euforia, locuacidad, agresividad
o Embriaguez grave ("l .5-3.0 grl(: aumento de las anteriores, ataxia, nistagmo, lenguaie farfullante, deterioro de la aten-
cin y memoria, clnica vegetativo (hipotensin, nuseas, sudor).
o Coma (3.0-5 grll): evolucin a estupor o coma.
o Zona letal (> 4.0 grl(: puede ser mortal, por depresin del centro respiratorio o aspiracin del vmito (MIR).
. Tratamiento: sintomtico + suero glucosado + TIAMlNA (Vit Bi) (prevencin de Wernicke-Korsakov).
o Borrachera patolgica: Desproporcn dosis-respuesta. Tras la ingesta de poca cantidad de alcohol cuadro muy aparatoso
(agitacin y estado confusional), frecuente en abstemos. Cesa con el sueo. Amnesia del episodio.
o Black out o galimpesto: Amnesia respecto a un periodo en el que el sujeto estaba bebiendo alcohol y no mostraba signos de
intoxicacin alcohlica (MIR).

8.2. ETIOLOGA DEL ALCOHOLISMO

0 Multifactorial. Factores genticos, sociales y psquicos. El riesgo de padecer alcoholismo en familiares de pacientes alcohlicos
est multiplicado x 4 respecto a la poblacin general (MIR).

8.3. PATRONES DE CONSUMO

o Mediterrneo: Consumo diario, se emborracha raramente.


o Anglosain: Grandes intoxicaciones separadas por das de abstinencia.

8.4. DlAGNSTICO DE ALCOHOLISMO

El diagnstico se hace en base a la historia clnica. El resto de pruebas son una ayuda al diagnstico.
Unidad de Bebida Estndar (UBE) = lO g de alcohol puro ( 12.5 ml de alcohol).
Cuestionarios de deteccin: CAGE (4 preguntas. Probable diagnstico: 2 tems positivos), AUDlT, MALT...
Marcadores bioqumicos:
o Elevacin de enzimas heptcos
o GOT (AST) y GPT (ALT). GOT/GPT > 2 muy sugestivo de consumo crnico de alcohol.
o Aumento de GGT (MIR). Se normaliza tras 3-4 sem de abstinencia (su cada en 5 das es patognomnica). Ms sensi-
ble que VCM.
PSIQUIATRA o Alteracin VCM: macrocitosis (con anemia o sin ella) (MIR). Tarda ms en normalizarse que GGT.

CURSO INTENSIVO MIR ASTURIAS

o Aumento GGT + Aumento VCNI: especificidad deI 75%.


o Aumento CDT. Se normaliza tras 2 semanas de abstinencia. El ms especfico.
Cuando Ia cantidad de caloras que eI alcohol aporta a Ia dieta supera el 20% se habla de bebedor excesivo (MIR).

8.5. ALCOHOLISMO CRNICO


COMPLICACIONES FSICAS
Aparato digestivo: Aumento de tamao de las glndulas partidas. Refluio gastroesofgico y piross. Sndrome de Mallory-
Weiss. Gastritis agudas que pueden evolucionar a gastritis crnicas. Pancreatitis agudas que pueden evolucionar a pancreati-
tis crnicas. Sndrome de malabsorcin. Esteatosis heptica. Hepatitis alcohlica (GOT/GPT >2) (MIR). EI 15% de los alcohli-
cos desarrollan cirrosis heptica, y de estos 10-50% hepatocarcinoma.
Sistema cardiovascular: EI alcohol acta como antagregante en pequeas dosis. EI consumo crnico, produce HTA por est-
mulo del sistema renina-angiotensina-aldosterona (hipotensin en consumo ocasional). Fibrilacin auricular, extrasstoles.
Miocardiopata alcohlica: hipertrofia ventricular izquierda, ICC, sint anginosos. La miocardiopata dilatada aparece tras ms
de IO aos de abuso del alcohol. EI alcohol es factor de riesgo de ACV.
Sistema hematolgico: anemia, neutropenia, trombopenia.
Sistema osteomuscular: Ia miopata aguda, con clnica de debilidad proximal de miembros, es Io ms frecuente. EI alcohol es
Ia causa ms frecuente de osteoporosis en varones.
Sistema nervioso:
I. Enfermedad de Marchiafava-Biqnami: desmielinizacin del cuerpo calloso. Se produce clnica de demencia, disartria,
hipocinesia, descontrol de esfnteres.
2. Mielinolisis central pontina: desmielinizacin de Ia base de Ia protuberancia, pronstico fatal.
3. Ambliopa alcohlica.
4 Neuropata perifrica: tipo sensitivo-motor, de predominio distal. EI trastorno es ms frecuente en alcohlicos crnicos.
Produce dolor y parestesias.
5. Deaeneracin cerebelosa de predominio vermano.
Metabolismo: EI alcohol produce hipoglucemia. Inhibe Ia gluconeognesis heptica (MIR). Aumenta Ia cetognesis, disminuye
Ia Iipolisis y Ia oxidacin de cidos grasos, produciendo hiperlipidemias. Determina adems trastornos de Ia sntesis de ami-
nocidos por dficit de vitamina B.
Complicaciones infecciosas: Efecto inmunosupresor. Ms frecuencia de hepatitis B y C. TBC, neumona por Gram(-) y anae
robios, mayor frecuencia de meningitis bacterianas.
Complicaciones oncolgicas: EI alcohol es factor de riesgo para Ca de cavidad oral, faringe, esfago, hgado, colon y recto.
Efectos hormonales: Una minora de varones alcohlicos crnicos puede presentar atrofia testicular irreversible (MIR). En mue
res puede aparecer amenorrea. Hipercortisolemia. Hipofuncn tiroidea (1 T3 y T4).
Sndrome alcohlico fetal: Abortos espontneos y efecto teratgeno. Retraso del crecimiento prenatal (CIR) y postnatal. Afec-
tacin SNC (microcefalia, retraso mental, sndrome de hiperactividad y alteraciones de Ia coordinacin). Anomalas faciales.
Alteraciones de los tabiques del corazn (CIA, CIV) (MIR). Malformaciones renales. Frecuente.
COMPLICACIONES SOCIALES
Deterioro social. Alteracin de las relaciones interpersonales. Violencia.
TRASTORNOS MENTALES CONCOMITANTES EN ALCOHOLISMO CRONICO
Sndrome de Wernicke-Korsakov: dficit de tiamina (vitamina BI). Frecuentemente coexisten ambos cuadros. Causa ms fre-
cuente alcoholismo crnico (MIR). Tratamiento: tiamina (MIR).
o Encefalopata de Wernicke (cuadro agudo) (MIR): triada sintomtica: obnubilacin / desorientacin, alteracin motrica
ocular, ataxia. Frecuentemente se asocia con polineuropata. Suele tener un inicio brusco (MIR).
o Sndrome de Korsakov (cuadro residual): amnesia antergrada, fabulacin
Demencia: eI efecto txico del alcohol puede causar demencia. La mayora de los alcohlicos tienen deterioro cognitivo leve
que meiora mucho al suspender Ia ingesta.
Trastorno psictico persistente inducido por el alcohol :
o Predomina Ia ideacin delirante. Los temas ms frecuentes del delirio son: celos (consumo crnico de alcohol es la prime-
ra causa de celotipa o celopata), persecucin... (MIR). Alucinaciones sobre todo de tipo auditivo
o Los pacientes tienen buen nivel de conciencia, estn orientados, con memoria normal y sin deterioro de Ia capacidad de
(uicio (pasa Io mismo en Ia alucinosis alcohlica).
Otros trastornos mentales inducidos por el alcohol son: cuadros amnsicos, trastornos depresivos (MIR), suicidio (alcohol 2
causa), trastornos de ansiedad generalizada, trastorno de pnico (MIROO), fobia social (ms que consecuencia es causa) (MIR),
trastornos de sueo.
Comorbilidad del alcoholismo: Trastorno antisocial de Ia personalidad (MIR), esquizofrenia (riesgo x3), trastornos afectivos
(riesgo x2).

8.6. CUADROS ASOCIADOS A LA DISMINUCIN / CESE OH EN ALCOHOLICO CRONICO

Alucinosis alcohlica: Alucinaciones sobre todo de tipo auditivo (voces amenazantes) (MIR).
Sndrome de abstinencia alcohlica:
o Comienza 4-12 horas tras ltima ingesta.
o Dos o ms de los siguientes sntomas: hiperactividad autonmica (sudacin, taquicardia), temblor distal de las manos, in-
somnio, nauseas, vmitos, alucinaciones visuales / tctiles (zoopsias: bichos), agitacin psicomotriz, ansiedad, crisis
'
comiciales.
Delirium alcohlico (Delirium tremengsh
o Comienza 3-4 das despus de suprimir / cesar Ia ingesta (MIR).
o Es el cuadro ms grave inducido por abstinencia alcohlica (puede ser mortal: riesgo fallecimiento 5%).
o Hiperactividad autonmica (sudoracin, taquicardia, hipertermia), temblor de manos, prpados, lengua, agitacin, an-
siedad, insomnio. A medida que progresa aparece deterioro del nivel de conciencia (MIR). Alucinaciones visuales / tctiles
(zoopsias) (MIR). Delirio ocupacional. Crisis comiciales entre un 51 5%.

MIR
VIII. SUSTANCIAS PSICOTROPAS

o Tratamiento: EI frmaco de eleccin son las BZD (diacepam, clordiacepxido) (MIR), tambin se utilizan otros frmacos como
clormetiazol + Tiamina (o preparados multivitaml'nicos con vit. Bi).

8.7. TRATAMIENTO DEL ALCOHOLISMO

Desintoxcacin: es el primer paso para la deshabituacin (MIR). No es necesaria la hospitalizacin (MIR) salvo que exista histo-
ria de convulsiones o delirium tremens, abuso de otras sustancias psicoactivas, otros trastornos mentales, riesgo de suicidio o
fracasos repetidos en rgimen ambulatorio.
Se procede a la supresin brusca de la ingesta alcohlica, se deben asociar frmacos
___para que no aparezca el sndrome de
abstinencia: Estos son: Benzodiaceginas (diacepam, clordiacepxido), clormetiazoll tiapride (MIR) Estos frmacos son tambin
tiles para el tratamiento del delirium tremens establecido. Si hay alucinaciones o delirios: Haloperdol. A veces tambin se
necesitan antidepresivos. En el tratamiento deben incluirse suplementos de vitaminas del grupo B (prevencin de Wernicke-
Korsakov) al menos una semana (MIR).
o Deshabituacin:
o Tratamiento psicosocial: Terapias conductistas, grupos de autoayuda.
o Tratamiento farmacolgico:
o lnterdictores: disulfiram o cianamida clcica, ambos producen acmulo de acetaldehdo y por tanto efecto an-
tabs tras la ingesta alcohlica. El efecto antabs consiste en: vasodilatacin, nuseas, vmitos, enroiecimiento, disnea,
palpitaciones, hipotensin, ansiedad... (MIR). Contraindicaciones absolutas de interdictores:
Dsulfiram: alteraciones cardiovasculares, embarazo y psicosis.
Cianamida clcica: enfermedad coronaria o miocrdica grave.
o Otros frmacos: naltrexona (MIR) o acamprostato (J, crovng y recompensa),- topiramato
o Despus de rehabilitacin 50% de los alcohlicos abstinencia al menos 'l ao y muchos de por vida (MIR).

8.8. URGENCIAS PSIQUITRICAS EN ALCOHOLISMO

Enfermedad de Wernicke-Korsakov Alucinosis alcohlica, Cglotigia alcohlica, sd abstinencia (delirium tremens, crisis con-
vulsivas), Embriaauez patolgica, encefalopata heptica v tentativa de suicidio.
CURSO INTENSIVO MIRASTURIAS

Trastornos afectivos
Nmero de preguntas: del captulo en el MIR

90 91 92 93 94 95f 95 9f 96 97f 97 98f 98 99f 99 OOf 00. Oi, 02. 03. 04. 05. 06. 07. 08. O9. iO i'l 12 13 14

Nmero de preguntas de cada tema

Clasificacin de los trastornos del humor

Epidemiologa

Trastornos depresivos l 28
Distima

Depresin doble

Diferencia depresin/distimia h 7
Trastornos depresivos puerperales no psicticos

Episodio maniaco 5

Episodio hipomanaco -r2


-
Episodio mixto

Trastorno bipolar 2'l

Suicidio 14

[C52 imprescindible
o El episodio depresivo mayor se caracteriza por un cuadro de al menos dOS semanas de nimo deprimido, anhedonia, anorexia,
sentimientos de culpa, ideacin recurrente sobre la muerte o incluso autoltica. Pueden asociarse sntomas psicticos. (9MIR)
o El tratamiento de eleccin de La depresin mayor son los antidepresivos tipo lSRS. Se mantienen durante al menos meses Si se
asocian sntomas psicticos y sobre todo si hay riesgo de suicidio puede estar indicada la terapia electroconvulsva. (3MIR).
o La distimia es una forma leve de depresin (no hay sntomas psicticos ni signos psicomotores) que se mantiene al menos dos
aos (3MIR).
I El episodio maniaco se caracteriza por un estado de animo anormal y persistentemente elevado eufrico, expansivo, o irritable,
que dura al menos una semana y que suele presentarse con: aumento de la autoestima, disminucin de la necesidad de dormir,
verborrea, fuga de ideas, comportamientos inadecuados y pueden aparecer ideas delirantes o alucinaciones (4MIR).
o Para su tratamiento se utiliza Litio, antipsicticos atpicos y frmacos para el control de impulsos como el Valproato. El Litio es el
frmaco de eleccin en la profilaxis de nuevos episodios. Niveles plasmticos 0,5-1,2, y se debe controlar funcin renal, heptica
y tiroidea (9MlR). Se debe mantener'varios aos
o Los principales factores a tener en cuenta para valorar el riesgo suicida son: trastornos mentales (sobre todo los trastornos afecti
vos, esquizofrenia y alcoholismo), varones de ms de 60 aos, y el intento previo de suicidio, sobre todo con mtodos de alta le-
taliclacl (7MIR)
IX. TRASTORNOS DEL HUMOR

i. Clasificacin de los trastornos del humor

f DEPRESIN 4 MAYOR, PSICOTICA


ENDGENA O MELANCOLA
NO PSICTICA
TRASTORNOS
DEPRESIVOS
(UNIPOLARES) DISTIMIA (DEPRESIN NEURTICA)
TRASTORNOS DEL
HUMOR
TIPO l: Episodios manacos o mixtos con / sin episodios depresivos
TIPO II: Uno o mas episodio depresivos mayores y episodios hipomanacos
TRASTORNOS
BIPO LARES
Trastorno ciclotmico: Episodios de hipomana y sntomas depresivos leves durante al
L menos 2 aos consecutivos sin periodos de mas de 2 meses sin sntomas

2. E-oidemioloa 3.1. Criterios diagnsticos


El HUMOR es el tono del animo que vivenciamos internamente e 25 durante un periodo de 2 semanas.
influencia nuestra percepcin del mundo y conducta. El humor CAMBIO respecto a la actividad previa.
puede ser normal o estar elevado o disminuido. U, sntomas debe ser nimo deprimido o anhedonia.
El AFECTO es la expresin externa de este humor. I. Animo deprimido durante la mayor parte del da casi cada da.
(En nios puede ser irritable).
PREVALENCIA DE LOS TRASTORNOS DEL HUMOR: 2. Disminucin acusada del inters o de la capacidad para el
Los trastornos del humor son frecuentes. placer: anhedonia.
o El trastorno depresivo mayor tiene la mayor prevalencia a lo 9 Prdida de peso (anorexia) sin hacer rgimen (mas frecuente) o
largo de la vida de cualquier trastorno psiquitrico: 12% aumento de peso.
(rango 5-'I 7%). Insomnio (mas frecuente) o hipersomnia cada da.
o Incidencia anual: 1,6% (1,9% en muieres y Agitacin o enlentecimiento psicomotores casi cada da.
1,1% en varones). Ms frecuente en muieres. Fatiga o prdida de energa casi cada da.
o Distimia: 5%. Sentimientos de inutilidad o culpa excesivos o inapropiados.
o Trastorno bipolar: 1%. PNPPF Disminucin de la capacidad para pensar, concentrarse o
tomar decisiones.
POR SEXO:
. Pensamientos recurrentes de muerte, ideacin, planes o inten-
o La depresin y la distimia son mas frecuentes en muieres tos suicidas.
(aproximadamente x2).
o EI trastorno bipolar tiene la misma incidencia en varones
Los sntomas deben acompaarse de malestar clnico significati-
que en muieres. Pero los episodios maniacos son ms fre-
vo y deterioro social, laboral o de otras areas importantes de Ia
cuentes en hombres y los depresivos en muieres. Ademas,
actividad del individuo. Hay aislamientos social y prdida de las
cuando las muieres tienen episodios maniacos son ms fre-
aficiones.
cuentemente de tipo mixto (sntomas de mana y depresin
Los sntomas psicticos refleian enfermedad grave y pueden ser:
conjuntamente).
o Congruentes con el estado de animo: ideas delirantes o
POR EDAD DE INICIO: alucinaciones con contenido compatible con temas depresi-
o Trastorno depresivo: 40 aos. vos (culpa, enfermedad, muerte, ruina, delirio de Cotard).
o Trastorno bipolar: 30 aos. Comienza antes. Es lo mas frecuente.
o lncongruentes con el estado de nimo: ideas delirantes o
MIR 09 (9206): (147) Seale la respuesta FALSA en relacin a alucinaciones con contenido no compatible con temas de-
los trastornos del estado de nimo: presivos (persecucin, control, insercin o transmisin del
l. El trastorno depresivo mayor tiene el doble de prevalencia pensamiento...)
en muieres que en varones.
2. El trastorno bipolar I tiene doble prevalencia en varones aue
en mu'eres.*
3. Los episodios maniacos son ms frecuentes en los varones.
4. El trastorno bipolar | es menos frecuente que el trastorno
depresivo mayor.
5. El trastorno bipolar I comienza mas precozmente que el
trastorno depresivo mayor.

3. Trastornos de . resivos
Episodio depresivo mayor: el primer episodio de un paciente.
Trastorno depresivo recurrente: al menos otro episodio depresi-
vo previo. No antecedentes de episodios manacos o hipomana-
cos. Intervalo libre de sntomas entre un episodio y otro de >
meses.

Distimia: menor gravedad sintomtico que en un episodio de-


presivo pero duracin >2 aos. Pensamientos recurrentes de muerte o ideas de suicidio en el episodio
depresivo mayor,
CURSO INTENSIVO MIR ASTURIAS

> MODE-
LEVE
RADO
GRAVE
3.3. Curso del trastorno depresivo mayor
TPICO: o Suele iniciarse con prdromos de sntomas ansiosos y
Humor depresivo sntomas depresivos leves.
Prdida inters 2 2 Sin tratamiento, lo habitual es que dure mas de 4 meses,
Anhedonia Alrededor del 20-30% de los casos persisten sntomas de-
Fotigobilida presivos durante meses o aos (remisin parcial).
Menor vitalidad o Los episodios son ms frecuentes a medida que avanza la
FRECUENTE edad.
atencin/concentracin o TRASTORNO DEPRESIVO RECURRENTE: trastorno depresi-
Prdida confianza vo mayor recidivante.
Ideas culpa / inutilidad o 21 episodios depresivos mayores.
o sin historia de episodios maniacos, mixtos o
Ideocin / actos suicidas 2
hipomanacos.
Trastornos sueo
o Periodo de meses libres de alteraciones en-
lapetito
tre episodios.
INTENSIDAD + ++ +++
MIR FAMILIA 00 (6629) Atendemos a una chico de 'I aos que
nos cuenta que lleva un tiempo con menos apetito y durmiendo
DURACIN SINTOMAS 2 2 SEMANAS menos yo que le cuesta poder dormirse. Es una buena
estudiante, pero en la ltimo evaluacin le han quedado cuatro
DETERIORO PSICOSOCIAL + ++ +++ asignaturas y le cuesta concentrarse en los estudios. Los dos
ltimos tines de semana no ha salido con sus amigos porque no
La depresin melanclico o endgeno implica gravedad. Se le apeteca y se siente irritada con su tamilia, aunque no
asocia con cambios del sistema nervioso autnomo y endocrino. entiende por qu, tiene dolores de cabeza y a veces le viene la
Ocurre en ausencia de desencadenantes estresantes. Adems, idea de la muerte o la imaginacin aunque piensa que no lo
puede acompaarse del llamado SINDROME SOMTICO ca- hara por sus sentimientos religiosos. Nunca le haba sucedido
racterizado por la presencia de al menos 4 de los siguientes algo parecido. El diagnstico:
sntomas: Trastorno depresivo mayor.
Trastorno ciclotmico.
Trastorno distimico.
SINDROMEISOMTICO
Anorexia.
(depresin melanclico o. endgeno)
.0... Episodio depresivo maxor.*
Prdida inters / anhedonia i
Descenso de la reactividad emocional
>4
"
l MIR 02 (7398) : Respecto al denominado "trastorno depresivo
i ' '
Despertar precoz (2 horas antes) mayor una de las siguientes afirmaciones N_O_es cierto:
Empeoramiento matutino (variacin circadiona del estado de l. Equivale en cierta medida al concepto de melancola.
nimo) 2. En l se incluyen tanto el trastorno depresivo mayor unipolar
o Inhibicin / agitacin psicomotriz como la fase depresivo del trastorno afectivo bipolar
Prdida de peso (5% en ltimo mes) (depresin bipolar).
o Descenso de la libido 3. Presenta unos criterios diagnstico claramente diferenciados
de los propuesto para otros trastornos depresivos como Ia
3.2. Sntomas asociados ol episodio Distimia.
Al margen del nimo decodo; estos individuos suelen
depresivo mayor presentar ideas, rumiaciones e T impulsos suicidas que
conducen a gestos suicidas o incluso al suicidio consumado
AFECTIVOS: .
en el 75% de los casos.
Anhedonia: Incapacidad para experimentar placer.
5. Resulta excepcional su complicacin con sintomatologa
o Disforia: Sensacin de malestar general, especialmente psictica."E
estado de nimo depresivo con ansiedad e inquietud.
o Alexitimia: Incapacidad para expresar los afectos a travs MIR 03 (7660):, La anhedonio total se relaciona
de las palabras. fundamentalmente con:
. Otros sntomas afectivos: Angustio, irritabilidad, tristeza, Mania.

rumiaciones obsesivas, preocupacin excesivo por la salud Degresinf
fsica y queios de dolor, ansiedad, tobios. Fobias sociales.
PENSAMIENTO: Ataques de pnico.
Prdida de lo autoestima, ideas de culpa o de ruina. .UJPNT Personalidad ontsocial.
CONDUCTA: F

Abandono personal, intentos de suicidio (hasta en el 15% de los MIR 'Ii (9687): Una estudiante universitaria de"'l9 aos acude
casos: 1 causo de suicidio). o la consulta acompaada por sus padres refiriendo sentirse en
Aislamiento social, prdida de la libido. los ltimos dos meses progresivamente ms ostnica, con
Ritmos biolgicos: prdida de apetito y de peso y con mayores dicultades para
Ms frecuente en primaveraotoo. concentrarse en los estudios. En la anomnesis tambin destaca
Meiora vespertino y despertar precoz en depresin endgeno. que ha perdido inters en salir con los amigas, presento ideas
de muerte sin ideacin outolitica y cogniciones pesimistas de
ALTERACIONES SOMTICAS: futuro. Su peso es el 90% del considerado ideal por edad y
Trastornos digestivos, estreimiento, astenia, amenorrea, gnero. No presento fobia ponderal ni distorsin de la imagen
sequedad de boca. corporal. El diagnstico ms adecuado es:
SNTOMAS MOTORES: agitacin o enlentencimiento psico- Anorexia nerviosa.
motor.
Trastorno de ansiedad.
Trastorno lmite de lo personalidad.
Distimia.
.UPPN. Trastorno depresivo mayor.
IX. TRASTORNOS DEL HUMOR

MIR 12 (9933): Muchacha de 19 aos estudiante, sin anteceden- 3.5. Etiopatogenia


tes personales somticos ni psiquitricos relevantes que es lleva-
da a urgencias hospitalarias por su familia por ingesta masiva Tiene diferentes orgenes (multfactorial): susceptibilidad genti-
de pastillas. El intento autoltico no haba tenido providencia de ca, factores biolgicos y factores externos.
rescate, siendo la paciente encontrada por su madre casualmen-
te al volver a su casa antes de lo que tena previsto. Una vez FACTORES BtOLGlCOS:
estabilizada orgnicamente la paciente refera presentar desde o Son bsicos en las depresiones endgenas.
unas 4 semanas atrs un cuadro de tristeza, anhedonia, deses- o Monoaminas: sobre todo disminucin de actividad de NA,
peranza, sentimientos de culpa y deseos de muerte, inhibicin 5HT y DA.
psicomotriz y enlentecimiento del pensamiento, dificultad de o Otros NTs: ACh (los agonistas colinrgicos pueden inducir
concentracin y rendimiento en los estudios, aislamiento social, depresin), GABA (parece haber una disminucin), gluta-
hipersomnia diurna y empeoramiento matutino de los sntomas. mato, glicina.
As mismo Ia paciente referia tener la sensacin de que sus veci- 0 Segundos mensaieros y cascadas intracelulares: muchos
nos Ia espaban, hablaban de ella cuando sala y se rean de frmacos actan sobre ellos.
ella Io que haba incrementado su angustia. No saba el motivo o Alteraciones hormonales: disminucin de la actividad del
por el que la gente se fiaba en ella pero estaba convencida de gen de BDNF, alteraciones del eie hipotlamo-hipfiss-
que no eran imaginaciones suyas. Seale el diagnstico: adrenal (actividad elevada en el 50% 'de los deprimidos).
Esquizofrenia. a Eie tiroideo: 5-10%tenen disfuncin tiroidea con TSH basal
Trastorno de ideas delirantes persistentes. elevada o sobrerrespuesta a Ia administracin de TRH. 30%
Trastorno depresivo con sntomas Dsicticos". no tiene una respuesta adecuada a TRH.
Trastorno de personalidad esquizoide. o Hormona del crecimiento: su secrecin est estimulado por
PPPNf Trastorno distmico. DA y NA e inhibida por CRH y somatostatina (la somatosta-
tina est elevada en mana y disminuida en depresin).
MIR 13 (10219) (220): Muier de 50 aos de edad que desde - Prolactina: estimulada por 5HT e inhibida por DA.
hace 1 mes presenta un nimo deprimido, anhedonia, prdida
de memoria, llanto fcil, prdida de energa, sensacin de
FACTORES GENTICOS:
inutilidad y culpa, prdida de peso marcada y despertar precoz, o Linkage gentico:
as como incapacidad para realizar sus tareas habituales en el o CREBT en Cromosoma 2: asociado a depre-
hogar. En tramites de separacin desde hace 3 meses. Seale el sin unipolar.
o Cromosomas 18q y 22a: asociados a trastor-
diagnstico ms apropiado:
Distimia. no bipolar.
o Cromosoma 21q: asociado a esquizofrenia y
Seudodemencia.
Trastorno adaptativo depresivo. trastorno bipolar.
Episodio de depresin mayor. * o Antecedentes familiares:
PPPNT Depresin menor. o Si padre o madre afecto: riesgo 10-25%.
o Ambos: 20-50%.
o Concordancia de monocigotos: 70-90%. Por tanto los
genes no lo explican todo.
Concordancia dicgotos: 1635%.
o La presencia de antecedentes familiares de trastorno
bipolar aumenta el riesgo para los trastornos del humor
en general, sobre todo para el trastorno bipolar.
o El trastorno depresivo es la forma de trastorno del
humor ms frecuente en familias de probandos bipola-
res.

3.6. Laboratorio
BtOQUMtCA:
DlSMlNUCIN DE LA NORADRENALINA:
EI MHPG (3-metoxi-4-hidroxi-fenilglicol) es un metabolito
de Ia noradrenalina que est disminuido en la orina de los
WM-
Datos frecuentes pruebas de hipofuncin noradrenrgica:
hipersecrecin de cortisol, disminucin de la respuesta de
GH a la hipoglucemia insulnica, disminucin de LH en mu-
ieres depresivos posmenopusicas, disminucin de la res-
puesta de TSH a la estimulacin con TRH.
- DlSMlNUCIN DE LA SEROTONINA:
Disminucin de cido 5-hidroxi-indolactico (5HlAA)
en LCR de pacientes depresivos. Ocurre en 3040%
de pacientes con depresin endgeno con ms con-
G uso Intensivo MIR Asmrils 2003 ducta suicida. ,
Depresin mayor
o ACTIVIDAD COLlNERGICA;
3.4. Factores de riesgo Descompensacin del equilibrio permanente colinrgico-
adrenrgico a favor del primero en depresivos y del segun-
Separados /divorciados. do (predominio adrenrgico) en manacos.
Historia familiar de alcohol o depresin. . DISFUNCIN DOPAMINRGICA:
Prdidas parentales en la infancia. Hipofuncin dopaminrgica en la depresin e hiperactivi-
Parto en los meses precedentes dad en la mana.
0 Contexto ambiental conflictivo. ENDOCRINOLOGA:
o Acontecimientos vitales desfavorables. Evidencias de alteracin del eie hipotlamo-hipofisario en las
Escasa relacin interpersonal. depresiones endgenas.
o Ancianos, muieres, varones venes. . Test de supresin con Dexametasona:
o Clase social baia. o En 25-40% de depresivos NO hay frenacin del corti-
gar-ss
CURSO INTENSIVO MIR ASTURIAS

sol tras administrar 0,5-2 mg de dexametasona: corti- DEPRESIN ATPlCA


sol >5ug/100ml.
o Cambios funcionales invertidos respecto a los tpicamente
o Especificidad de 60%: tambin alterado en mana, es-
endgenos: hiperfagia, hipersomnia, parlisis plmbea (pe-
quizofrenia, demencia, alcoholismo, TOC, crisis de an-
sadez intensa en brazos y piernas), marcada sensibilidad al
gustia, trastornos de la alimentacin, dolor crnico, to-
rechazo interpersonal.
ma de benzodiacepinas, estrs, embarazo, prdida de
o Pueden estar indicados los' IMAOS.
peso y en hasta 20% de muieres normales.
o Los pacientes con test de Supresin patolgico tienen DEPRESIN REACTiVA
con ms frecuencia antecedentes familiares afectivos.
o El test no permite predecir la respuesta al tratamiento Neurtica o situacional.
pero se sabe que cuando persiste alterado la recupera- Rasgos neurticos de personalidad, ansiedad.
cin clnica es incompleta y hay mas riesgo de recadas. Empeoramiento vespertino.
Menor riesgo de suicidio.
Test de estimulacin de TSH por TRH: .0... Ms sensible a psicoterapia y peor respuesta a antidepresi-
En el 30% de depresivos endgenos unipolares no hay au- vos. -
mento de la TSH tras la estimulacin con TRH. Cuando la
meiora clnica no se relaciona con la normalizacin del test, TRASTORNO AFECTlVO ESTAClONAL
hay mayor riesgo de recadas.
o Episodios depresivos en otoo-invierno con recuperacin en
o 540% tienen niveles de TSH basales elevados o una res-
primavera-verano.
puesta de TSH excesiva a TRH. Estos casos se asocian a
o Sntomas atpicos: hipersomnia, hiperfagia.
presencia de anticuerpos antitiroideos y requieren de trata-
miento sustitutivo con hormona para una adecuada res-
Apetencia por hidratos de carbono.
puesta al tratamiento. Terapia Iumnica es eficaz.
o Es inespecfico y est alterado en: tratamiento con litio, TRASTORNO DEPRESlVO BREVE RECURRENTE
trastornos endocrinos, renales o hepticos, alcoholismo,_
anorexia, dolor crnico o trastorno lmite de la personali- o Sntomas similares a episodio depresivo mayor.
dad. Duracin de 2-13 dias.
NEUROIMAGEN: TAC, RMN, PET o l episodio al mes durante 12 meses .
o Aumento de ventrculos. o No relacin con ciclo menstrual.
o Atrofia cortical, surcos amplios. DEPRESIN lNVOLUTlVA
o Disminucin de hipocampo o coudado o ambos.
o PET: metabolismo disminuido en cerebro anterior, sobre todo o Cuadro depresivo grave en ancianos.
en el izquierdo. En mana, sin embargo, hipofrontalidad de Intensa ansiedad, agitacin, ideas de culpa, ruina e hipo-
predominio derecho. condra.
o Hiperntensidad anormal en regiones subcorticales: regin . Alto riesgo de suicidio.
periventricular, ganglios basales y tlamo. o Ms frecuente en varones mayores de 50 aos (depresin
o Ms frecuente en bipolares tipo I y ancianos. endgeno de inicio tardo).
0 Por efecto deleterio neurodegenerativo de los episodios o Base hereditaria.
depresivos recurrentes. Responde a antidepresivos y TEC.

POLISOMNOGRAFA: PSEUDODEMENClA DEPRESIVA


o Alteraciones de la continuidad del sueo, con reduccin del
tiempo de sueo total (mayor frecuencia de despertares in- o Clnica similar a la demencia, con grandes trastornos de la
termitentes y despertar precoz). atencin, concentracin, memoria y orientacin.
o Mayor TCl corporal. o A diferencia de la demencia, en la pseudodemencia encon-
o Disminucin de latencia REM. Cuanto mayor disminucin, tramos:
peor pronstico en trminos de recadas. Candidatos a far- o Inicio agudo.
macoterapia (estos pacientes responden peor a psicoterapia). Progresin rpida de los sintomas.
Humor depresivo.
MIR 86 (1556): Dentro de los estudios neuroquimicos son hip- Queias cognoscitivas.
tesis ampliamente aceptadas la interpretacin de la sintomato- No empeoramiento nocturno.
loga depresiva en trminos de: Alteracin de memoria reciente y pasada.
Una hipoactividad noradrenrgca cerebral. Buena respuesta a antidepresivos.
Hipoactividad serotoninrgica.
Relacin exclusiva con la carencia de acetil-colina. DEPRESIN ANANCSTICA
DLP
.UFPNT' Aumento de la somatostatina. Depresin, habitualmente endgeno, de la que emergen ideas
obsesivas que remiten al desaparecer el episodio depresivo.
3.7. Formas especiales de depresin DEPRESION ANACLlTiCA DE SPITZ
DEPRESIN ENMASCARADA Se refiere al sndrome mostrado por los bebs en el primer ao
0 Equivalente depresivo. de vida si son privados de una figura materna adecuada.
- La depresin se manifiesta por sntomas somticos: cefale- En una primera fase hay lloros y forceieo intenso; posteriormen-
as, trastornos gastrointestinales, sintomas neurolgicos... te, si contina la deprivacn afectiva, el nio cae en fase de
o Estado de animo aparentemente normal. desesperanza (el forceieo disminuye, el llanto es ms suave y
montono). Deian de crecer, no progresan, incluso sufren una
o Muy frecuente.
regresin fisica, emocional, social y cognitiva.
o Responde bien a frmacos.
[l IX. TRASTORNOS DEL HUMOR

REACCIN DE DUELO
Reaccin ante la prdida de una persona querida. Algunos
individuos presentan sntomas caractersticos de un episodio
depresivo mayor.
EI diagnstico de trastorno depresivo mayor no est indicado
a menos que los sntomas se mantengan 2 meses despus de
la prdida.
Sntomas de duelo patolgico son:
o Culpa.
o Pensamientos de muerte distintos del sentimiento del
sobreviviente de que sera meior estar muerto o haber
muerto con la persona fallecida.
0 Preocupacin mrbida con sentimiento de inutilidad.
o Enlentecimiento motor acusado.
- Deterioro funcional acusado y prolongado.
- Experiencias alucinatorias distintas de escuchar la voz o
ver fugazmente al fallecido.

i
Cursn Iniciuim MIR Asturias

3.8. Causas secundarias de depresin


ENEERMEDDES SOMHCAS ' FACTORES TXICOS
Endocrinopatas y trastornos metablicos Frmacos
- Hipotiroidismo - Acetazolamida
- Hipertiroidismo - Asparaginasa
- Enfermedad de Addison Aureomicina
- Enfermedad de Cushing - Azatioprina
- Diabetes Bleomicina
- Hipo e hiperparatiroidismo - Bloqueantes beta
- Hipoglucemia - Carbamacepina
- Amenia - Cicloserina
- Sd adiposo-genital - Clonidna
Infecciones - Clotrimazol
- Vricas (gripe, hepatitis, neumona vrica) - Contraceptivos orales
Tuberculosis - Corticoides
- Fiebre tifoidea - Difenoxilato
- Mononucleosis infecciosa - Digital
- Infeccin por colibacilos - Disulfiram
- Infecciones urogenitales - Escopolamino
- Encefalitis - Estrgenos
Enfermedades autoinmunes - Etionamida
- Artritis reumatoidea - Etosuximida
- Lupus eritematoso - Fenacetina
Cncer - Fenilbutazona
Carcinoma de cabeza de pncreas - Griseofulvina
Trastornos del SNC - Guanetidina
- Enfermedad de Parkinson - Hidralacina
- Arteriosclerosis cerebral - Indometacina
- Demencia senil - L-dopa
- Hdrocefalia normotensiva - Metildopa
- Lesiones focales del hemisferio no dominante - Metisergida
- Miastenia - Metronidazol
- Hemorragia subaracnoidea - Mitramicina
- Esclerosis mltiple - Morfina
- Afecciones degenerativas - Propranolol
Intoxicaciones - Primidona
- Medicamentosa (ver factores txicos) - Procainamida
- Plomo - Progesterona
- Benceno - Psicolpticos
- Mercurio - Reserpina
- Bismuto - Wncristina
- Quinina - Vinblastino
- Monoxido de carbono - Vitamina D (altas dosis)
Trastornos incapacitantes de cualquier tipo Drogas
- Intoxicacin por etanol o sedantes / hpnticos
- Abstinencia de nicotina, alcohol, sedantes, hpnticos,
cocana, anfetaminas
CURSO INTENSIVO MIR ASTURIAS

3.9. Tratamiento
- PSICOTERAPIA:
o Cognitivo-conductual,psicodinmica. 4.1 . Definicin
0 Se debe concienciar al paciente del carcter temporal de su
trastorno, destacando los logros conseguidos con el trata- o Forma crnica y leve de depresin.
miento. Estado de nimo crnicamente triste y desanimado (o
o En estos perodos, se debe evitar la toma de decisiones irritable en nios y adolescentes) que est presente durante
importantes. la mayor parte del da la mayora de los das durante al
0 NO debe sustituir a los frmacos. menos 2 aos.
o I ao en nios y adolescentes.
TERAPIAS BIOLGICAS: ANTIDEPRESIVOS: 0 Durante esos dos aos, los intervalos libres de sntomas no
Eleccin: ISRS. han sido ms largos de dos meses.
Los efectos indeseables de los antidepresivos suelen aparecer
antes de la meiora clinica por lo que si no son muy incapacitan- 4.2. Clnica
tes se contina el tratamiento.
Ventana terapeutico: 2-6 semanas. Estado de nimo crnicamente triste y desanimado.
Duracin: al menos meses con misma dosis. o Tristes, pesimistas, cansados, apticos, irritables, con falta de
Se asocian antipsicticos en depresiones psicticas y si existe ilusin, incapaces de divertirse, pasivos, introvertidos, escp-
riesgo de suicidio. ticos hipercrticos, queiosos. Disminucin del rendimiento.
o Empeoramiento vespertino.
TERAPIA ELECTROCONVULSIVA:
o Estos sntomas producen deterioro socio-laboral y
La indicacin fundamental de la TEC es la depresin delirante
disminucin del rendimiento.
con peligro de suicidio.
Otras: fracaso farmacolgico, depresiones muy inhibidas, agita-
Menor riesgo de suicidio.
das o con ideas delirantes, situaciones somticas crticas (desnu- Mayor riesgo de depresin que poblacin general.
tricin, deshidratacin), reacciones adversas o enfermedades El cuadro clnico acaba hacindose similar a Ia ansiedad
fsicas que desaconseian la utilizacin de frmacos y anteceden- generalizada, con Ia que no es fcil el diagnstico diferen
tes de fracaso farmacolgico y buena respuesta a la TEC. cial.
No indicada en depresin neurtica. No hay manifestaciones psicticas.
o Presencia de 22:
DEPRESIONES RESISTENTES: Prdida o aumento del apetito.
Insomnio o hipersomnia.
o Aumentar dosis de antidepresivo. Falta de energa o fatiga.
o Cambio de antidepresivo. Baia autoestima.
Asociacin de antidepresivos. Dificultades para concentrarse o tomar decisiones.
Aadir litio.
Sentimientos de desesperanza.
o TEC.
Otros: triptfano, S-hidroxitrptfano, metilfenidato, tryodotiro- 4.3. Tratamiento
nina, terapia lumnica.
o Psicoterapia.Frmacos: ISRS de eleccin.
MIR 99 FAMILIA (6105): Si un paciente es tratado de un cuadro
depresivo leve por su mdico de Atencin Primaria con inhibido-
res selectivos de la recaptacin de serotonina (ISRS) a dosis te-
raputicas (por eemplo: 20 mg de fluoxetina) y, a la semana de
iniciar el tratamiento, no obtiene la respuesta teraputica desea-
da, z'qu deber hacer?:
l. Aumentar la dosis.
2. Cambiar de antidepresivo por otro del mismo grupo terap-
utico.
3. Cambiar de antidepresivo por otro del grupo de los tricicli-
cos.
4. Esperar, ya que la respuesta teraputica tarda aproxima-
damente 2 semanas.*
5. Derivarlo al especialista.

MIR OO (6870): El uso de la terapia electroconvulsiva en la de-


presin endgeno puede indicarse como meior opcin que los
frmacos en una de las siguientes situaciones. Selela:
Pacientes venes.
Pacientes sin apoyo familiar.
Depresin delirante.*
Depresiones endgenas unipolares.
915:p Depresiones endgenas bipolares. 5. De- resin doble
o Se superponen episodios depresivos mayores sobre un tras-
MIR 04 (7918): En relacin al tratamiento de la depresin, una
torno distmico.
de las siguientes afirmaciones es INCORRECTA?
La terapia electroconvulsiva no tiene indicacin en las depre- I Las personas distmicas que tienen episodios de depresin
mayor se recuperan a la distimia, no a la normalidad (euti-
siones neurticas.
mia).
El litio se utiliza en los cuadros bpolares.
Los lMAOS estn indicados en depresiones atpicas. n Los pacientes que presentan trastorno distimico previo a la
el-40%3 Los inhibidores de la recaptacin de serotonina son mejor depresin mayor, tienen mas posibilidades de presentar
tolerados que los antidepresivos triciclicos. otros episodios depresivos mayores y tienen peor recupera-
5. El tiempo medio de mantenimiento de la medicacin anti- cin interepisdica.
depresiva es de semanas.* o Pueden necesitar tratamiento ms intenso en la fase aguda y
mayor duracin del tratamiento.
PSIQUIATRA

g
IX. TRASTORNOS DEL HUMOR

. Diferencia de-resin end-ena / distimia



, "DEPRESIONENDGENA DISTIMtA
o Inicio brusco o Inicio variable
o Curso fsico 0 Curso continuo con fluctuaciones
o Tristeza vital o Tristeza-ansiedad
o El humor mejora por la tarde o Empeoramiento vespertino
uma/r 0 Despertar precoz
o Inhibicin-agitacin psicomotora
o
o
Insomnio inicial con pesadillas
Sin alteraciones psicomotoras importantes
o Posibles sntomas psicticos o Sin sntomas psicticos
o Riesgo de suicidio o Suicidio excepcional
o Falta de contacto o Bsqueda de contactos
o Ruptura biogrfico o Continuidad biogrfico
o Respuesta a antidepresivos o Combinado: frmacos + psicoterapia
TRATAMIENTO. " 0 TEC efectiva o No TEC
- Psicoterapia menos operante o Psicoterapia indicada
o Posible hipomana farmacgena - Sin hipomana farmacgena

MIR 00 (6725) Cual de estos sntomas es ms especifico de la


depresin mayor? : DEPRESIN PUERPERAL:
Meiora vespertina.* o Episodio depresivo mayor que aparece entre 2-4 semanas
Ansiedad despus del parto, evolucionando en meses.
Palpitaciones. o Prevalencia: 10-15% de las primparas.
- Recurrencia del 50%, puede estar en asociacin a trastorno
Anorexia.
S-"FWNT' Alivio tras el llanto. bipolar.
o Predominio de sntomas somtcos y atpcos.
IR 04 (7919) Hablamos de depresin doble" cuando: n Tratamiento: antidepresivos.
7; El paciente presenta un trastorno orgnico severo asociado a
su depresin.
2, Existe una comorbldad del cuadro depresivo con un trastor- 8. E . sodio maniaco
no por dependencia a sustancias txicas.
3. Se sobreimponen episodios depresivos mayores sobre un Estado de nimo anormal y persistentemente elevado eufrico,
trastorno distmico.* expansiva, o irritable, que dura al menos una semana (o cual-
4. Existe un riesgo de viraie a fase maniaco ya que en realidad quier duracin si es necesaria la hospitalizacin).
se trata de un trastorno afectivo bipolar. Durante el perodo de alteracin del estado de nimo han persis-
5. La depresin incide sobre un paciente esquizofrnico tras la tido 23 de los siguientes sntomas (cuatro si el estado de nimo es
remisin de los sntomas psicticos propios de la fase aguda solo irritable):
de su enfermedad.
o A__utoestima exagerada o grandiosidad
o Disminucin de la necesidad de dormir. Es uno de los prime-
MIR 05 (8181): Una adolescente de l aos acude a la consulta
ros sntomas deinicio del episodio.
contando que lleva cerca de un ao sintindose ms cansada,
Ms hablador de lo habitual: MQ
con poco apetito y dificultades para concentrarse en los estudios.
Fuga de ideas o experiencia subietiva de que el pensamiento
Cuando se le pregunta, comenta tambin que sale menos con las
est acelerado.
amigas y se muestra pesimista respecto de su futuro. El dia-
gnstico ms probable sera: a Distrabildad: atencin se desva demasiado fcilmente a
Anorexia. estmulos externos banales o irrelevantes.
Distimia. o Aumento de la actividad intencionada en el trabajo, los estu-
Depresin mayor. dios o sexualmente, o agitacin pscomotora.
Trastorno de ansiedad. o Implicacin excesiva en actividades placenteras que tienen un
91:?!97 Agorafobia. alto potencial para producir consecuencias graves: compras
irrefrenables, indiscreciones sexuales o inversiones econmi-
MIR 06 (8435): Cul puede encuadrarse como un trastorno cas alocadas.
distmco?: o Los sntomas no cumplen criterios de un episodio mixto.
l. Un cuadro crnico de la hipocondria. o Produce un deterioro socio-laboral o de las relaciones inter-
2. La persistencia de una depresin mayor a lo largo de ms de personales o necesita hospitalizacin o hay sntomas psicti-
un ao. cos.
3. Un trastorno bipolar tipo ll atpco.. o Los sntomas no son debidos a la toma de sustancias o a
4. Un humor crnicamente deprimido (o irritable en nios y alguna enfermedad mdica.
adolescentesl.* o Prdida de inhibicones sociales normales que da lugar a
5. Un sndrome de fatiga crnica, asociado a fibromialgia. comportamientos inadecuados para las circunstancias y para
el caracter del individuo.
o Pueden aparecer ideas delirantes o alucinaciones (en general
7. Trastornos depresivos auditivos).

. uer . erales 8.1. Sntomas asociados al episodio mania-


MATERNITY BLUES:
co
o Adaptativo. 0 Nula o escasa conciencia de enfermedad.
o Tristeza de Ia maternidad: trastorno que aparece entre el da o Pueden viajar impulsivamente a otras ciudades, cambian su
3 y TO del puerperio. aspecto personal a otro mas llamativo, realizan actividades
Prevalencia: hasta 50-80% de las muieres. "raras: regalan dinero, uegan excesivamente, drogas, al-
Sntomas afectivos: llanto, irritabilidad, cambios de humor. cohol...
o Se resuelve espontneamente o precisa solo apoyo psi O Puede haber agresividad.
coteraputico.
CURSO INTENSIVO MIR ASTURIAS

o Son ms frecuentes en jvenes y en mayores de 60 aos con


8.2. Evolucin
trastorno bipolar.
o La edad media de inicio son los primeros aos de la tercera Ms frecuente en hombres.
dcada de la vida. . Los episodios maniacos de las muieres suelen ser de carac-
o Los episodios maniacos comienzan de forma brusca con un tersticas mixtas.
aumento rpido de los sntomas en pocos das y frecuente- o Duracin de semanas a meses y pueden remitir hasta un
mente despus de un estrs psicosocial. periodo con pocos o ningn sntoma o evolucionar a un epi-
o La disminucin de la necesidad de dormir suele ser el primer sodio depresivo mayor, Es menos frecuente que un episodio
sntoma en las recadas. mixto evolucione a un episodio maniaco.
o Los episodios duran de varias semanas a varios meses y son
ms breves y con un final ms brusco que los episodios de- l l . Trastorno biolar
presiv05 mayores.
o TRASTORNO AFECTIVO BIPOLAR.
MIR 00 (6867): Acude a su consulta una muier de' 35 aos o Antes: PSICOSIS MANIACO-DEPRESIVA.
acompaada de su marido. Ella dice encontrarse muy bien, o Crnico.
meior que nunca, pudiendo hacer ms cosas, inclusose siente o Puede haber trastornos comrbidos: sobre todo trastorno de
capaz de comprender cuestiones muy complicadas. EI marido personalidad.
dice que ella lleva una semana durmiendo poco y que no para,
I CURSO FSICO con recadas de diverso signo:
se levanta temprano, sale a la calle, vuelve, se cambia, vuelve a
Mana (TAB tipo I) o hipomana (TAB ll).
salir, habla sin parar con cualquier persona que se encuentra y
Depresin.
no controla lo que gasta. Nunca le haba ocurrido algo parecido
Estados mixtos.
y ella no acepta tener ningn problema. El diagnstico sera:
Periodos asintomticos.
Episodio maniaco.*
Trastorno bipolar. Misma prevalencia en hombres y muieres, pero:
Trastorno psictico leve. o Hombres: ms episodios maniacos.
Trastorno de ansiedad generalizada. o Muieres: ms episodios depresivos
PFPNT' Trastorno de identidad disociativo.

MIR O3 (7658): De entre los siguientes sntomas, seale cul


sera ms IMPROBABLE encontrar en un episodio maniaco:
Aumento de las necesidades del sueo.*
Distrabilidad.
Verborrea.
Grandiosidad.
.UPSNT' nimo irritable.

MIR O (8434): La distraibilidad acusada suele darse en:


I . Esquizofrenias paranoides.
2. Hipocondracos.
3 Alucinosis alcohlica.
4 Manias.*
5 Depresiones ansiosas.

9. E-isodio hi comaniaco
nsivo MIR Asturias

o Un episodio hpomanaco es un periodo de tiempo en el que


l 'l .1 . Etiopatogenia
el estado de animo es persistentemente elevado, expansvo o . FACTORES BIOLGICOS
irritable durante al menos 4 das y es claramente distinto del
estado a nimo habitual. . FACTORES GENTICOS: Un 25 - 40% de familiares de pa-
o Durante este periodo hay tres o mas de los sntomas estudia- cientes bipolares presentaban trastornos afectivos uni o bipo-
dos en la definicin de episodio maniaco. lares.
o El episodio NO es Io sucientemente grave para provocar o Factores psicosociales: Los acontecimientos vitales estresan-
deterioro laboral o social importante o para necesitar hospi- tes nfluyen desencadenando el primer episodio y primeras
talzacin. recadas en sujetos predispuestos. Posteriormente las reca-
o NO hay sntomas psicticos. das se independizan de las situaciones de estrs.
o Entre un 545% de los pacientes con episodio hipomanaco
acabarn presentando un episodio manaco. o Factores estacionales: existe un aumento de los ingresos por
depresin en otoo e invierno. Las fases manacas se con-
centran en primavera y verano.
MANA HPOMANA
nimo elevado, expansivo o isemana 4dias MIR lO (9436): Cul de las siguientes afirmaciones N_O es
irritable cierta para el trastorno bipolar3:
Deterioro sociolaboral importante l. Se presenta con la sucesin de fases depresivasmanacas,
hospitalizacin sntomas SI NO aunque pueden presentarse intervalos de normalidad.
psicticos 2. La TEC (terapia electroconvulsiva) es indicacin teraputica
en el trastorno bipolar en casos de mana grave o resistente
al tratamiento.
IO. E-isodio mixto 3. La herencia admitida en el trastorno bipolar es de tipo
autosmico dominante con penetrancia incompleta.*
4. El tratamiento del trastorno bipolar con carbonato de litio
o Periodo de al menos una semana de duracin en el que casi est indicado tanto en fase maniaca como para una
cada da se cumplen los criterios tanto para un episodio teraputica profilctica.
manaco como para un episodio depresivo mayor. 5. La presencia de delirios excluye el diagnstico de trastorno
o Los sntomas de presentacin suelen incluir agitacin, in- bipolar.*
somnio, alteracin del apetito, sntomas psicticos e ideacin ANULADA
suicida.
lX. TRASTORNOS DEL HUMOR

l 1.2. Formas clnicas


l 1.4. Causas orgnicas de mana
o Bipolar tipo I: 21 episodio manaco o mixto. Con o sin epi-
Estos cuadros se consideran episodios maniacos secundarios y
sodios depresivos. Igual frecuencia hombres y muieres.
no deben llevar al diagnstico de trastorno bipolar.
o Bipolar tipo ll: 21 episodio depresivo mayor y al menos un
episodio de hipomana. Si hay un episodio manaco o mixto . Corticoides, levodopa, cocana, anfetam-
no se puede hacer el diagnstico de bipolar ll. Ms fre- Farmacolgicas . . . .
nas, metilfenidato, antidepreswos
cuente en muieres.
Slis terciaria, gripe, encefalitis de San
o Trastorno bipolar no especificado: Se incluyen trastornos Infecciosas
Louis, SIDA, fiebre tifoidea, legionella
con caractersticas bpolares que no cumplen criterios para
Hipertiroidismo, Addison, enfermedad de
ningn trastorno bipolar especfico. Los eiemplos incluyen: Endocrinolgicas
Cushing
o Episodios hipomanacos recdivantes sin sntomas
depresivos. Autoinmunes LES, corea reumtica
O Un episodio manaco o mixto superpuesto a un EM, Huntington, Parkinson, traumatismo
trastorno delirante, una esquizofrenia residual o un Neurolgicas craneal, crisis parciales compleias (lbulo
trastorno psictico no especificado. temporal), tumor dienceflico, ACV
o Situaciones en las que el mdico ha llegado a la
MIR 08 (8959): Alberto acompaa a su padre, de 55 aos, a Ia
conclusin de que hay un trastorno bipolar, pero es
conSUIta del psiquiatra. El padre no vea necesaria la consulta por-
incapaz de determinar si es primario, debido a en-
que dice sentirse estupendamente. Sin embargo el hio cuenta que
fermedad mdica o inducido por sustancias.
le ve cambiado desde hace algn tiempo. Todas las tardes se va a
o Trastorno ciclotmico: presencia durante al menos 2 aos un bingo, algo que antes rechazaba, gasta en l mas de Ia cuenta y
de numerosos periodos hipomana y numerosos periodos de cuando llega a casa, ya bastante tarde, da vueltas por la casa sin
sntomas depresivos que no cumplen los criterios de un epi- centrarse en nada y no hay manera de que se vaya a dormir. Habla
sodio depresivo mayor (en nios y adolescentes la duracin de las personas que ha conocido en el bingo e incluso cuenta que
debe ser al menos l ao). La persona no ha deiado de pre- hay una muier con la que esta pensando casarse. Ha hecho un
sentar sntomas por un tiempo superior a dos meses. Los amigo con el que piensa emprender un negocio seguro y no tolera
sntomas producen un malestar clnicamente significativo o que le contradigan. Al psiquiatra le cuenta que en SU vida se ha
deterioro social, laboral o de otras reas de la actividad del encontrado meior y que en todo caso ser su hiio, que siempre est
individuo. preocupado, quien necesite tratamiento. El hiio dice que nunca
antes haba visto a su padre de esta manera. Cual es su diagnsti-
l 1 .3. Formas especiales co?:
Trastorno esquizotpico.
o Cicladores rapidos: 24 episodios afectivos al ao. 10% de Episodio hipomanaco.
los bpolares. 80% muieres. Predisponentes: hipotiroidismo Distimia.
subclnico, tricclicos, esteroides. Peor pronstico (responden Trastorno lmite de Ia personalidad.
peor al tratamiento). , 915:.p Trastorno psictico breve.
o Trastorno bipolar con variante estacional: relacionado con
los ciclos fotoperidicos de Ia melatonino. Se usa como tra- MIR 09 (9294): (235) Sealar cual de los siguientes trastornos co-
tamiento estimulacin luminosa intensa. Ms en muieres. mrbidos es mas frecuente en el trastorno bipolar:
Trastornos somatomrfos.
Trastornos de la personalidad.*
Esquizofrenia.
Estados confusionales.
.UPPNT' Trastorno obsesivo-compulsivo.

111.5. Depresin bipolar Vs depresin unipolar


o Las depresiones bipolares son ms frecuentemente inhibidas y se acompaan de hpersomnia y aumento de peso (sntomas atpi-
cos). AI to riesgo suicida.
o Entre 1/3 y 1/5 de los casos que comienzan como depresin mayor evolucionar o trastorno bipolar.
Factores para predecir que una depresin mayor evolucionar a trastorno bipolar (90% de fiabilidad):
o Inicio antes de los 25 aos.
o Historia familiar de trastorno bipolar.
o Arbol genealgico con elevado nmero de enfermos afectivos.
o Depresin postparto.
o Hpersomnia y retardo psicomotor.
o Induccin de hipomana farmacolgica con antidepresivos.
DEPRESIN BIPOLAR DEPRESIN UNIPOLAR
Edad de inicio 25-35 2555
Sexo Distribucin similar Predominio femenino
Personalidad premrbda Cicloide Melanclica
Nivel cultural Ms alto Menor
Morbilidad familiar para
Trastornos afectivos Alta Ms baia
Trastorno bipolar Aumentada No aumentada
Trastorno unipolar Alta Alta
Clnica
Psicomotricidad Inhibicin Agitacin, ansiedad
Riesgo de suicidio Ms elevado Menor
Mortalidad Ms elevada Menor
Duracin de episodios 36 meses 6-9 meses
Recidivas Cada 2 aos Cada 5 aos
Hpersomnia Ms elevada Menor
Profilaxs Sales de Litio Antidepresivos
gw,
MIR
m
CURSO INTENSIVO MIR ASTURIAS

MIR 94 (3787): CuI puede ser el cuadro psicopatolgico 0 En bipolares l puede ser el preludio de una fase manaca
aqueiado por una paciente afecta de lupus sistmico en trata grave por lo que puede estar indicada Ia adiccin de anti-
miento con corticoides que presenta repentinamente una euforia psictico atpico.
desconocida en ella frecuentando discotecas y vistiendo de ma-
nera extravagante mostrndose ofensiva y a la vez seductora con EPISODIOS MIXTOS:
el personal masculino de la clnica que la atiende? Los antidepresivos deben evitarse.
Fuga psicgena. . Tratamiento de eleccin: atpicos, carbamacepina o val-
Estado crepuscular. proato.
Trastorno del estado de nimo por esteroides.* El litio es menos ecaz que en pacientes manacos.
Psicosis por lupus sistmico. Por su elevado riesgo autoltico los estados mixtos deben ser
9175 9.? Trastorno bipolar por lupus sistmico. tratados en instituciones hospitalarias.

l l .6. Tratamiento CICLADORES RPIDOS:


o El tratamiento de eleccin en ciclacin rapida i es: Litio /
EPISODIO DEPRESIVO: Valproato / Lamotrigina.
No se recomienda la monoterapia con antidepresivos, ya que el o En algunos casos puede ser necesaria la combinacin de
uso de antidepresivos supone un riesgo de viraie a la fase ma- tratamientos de primera lnea entre si o con un antipsictico
niaca. El uso concomitante de eutimizantes o antipsicticos hace atpico.
menos problable el viraie a mana. o Retirada gradual de antidepresivos (si los estaba tomando).
o Episodio leve o moderado: Litio.
o Episodio grave: Antidepresivos (lSRS) + Litio / Valproato / l l .7. Profilaxis
Antipsictico Atpico.
o Lamotrigina, como eutimizante, si existen antecedentes de Litio: el frmaco de eleccin en la prolaxis.
depresiones. Es el ms especfico. Se deben controlar los niveles plasmticos:
O,5-l,2 mEq/l.
EPISODIO MANIACO: Se debe mantener entre 3-5 aos.
En casos de episodios manacos esta contraindicado la terapia Contraindicaciones:
con antidepresivos, ya que empeoran los sntomas manacos. o Enfermedad cardaca.
Ingreso necesario en el 75% de los casos. Enfermedad renal.
. Episodio leve o moderado: Litio / Valproato / Antpsicticos o Enfermedad tiroidea.
Atpicos o Embarazo: debe planificarse tras un largo periodo de
o Episodio grave: Antipsicticos Atpicos + Litio / Valproato. remisin de Ia enfermedad. Riesgo de recada en las
o TEC. primeras semanas de puerperio y debe realizarse tra-
tamiento preventivo desde las primeras horas tras el
MIR 13 (10220): Hombre de 43 aos fumador de 20 cigarrillos parto usando un eutimizante y evitando la lactancia.
al dia, que acude a su consulta llevado por un familiar, debido a Si fracaso de monoterapia con Litio:
que lleva lO das durmiendo poco, menos de 3 horas al da sin Si mana: valproato, antipsictico atpico.
referir cansancio por ello. Aade que ha empezado a gastar Si depresin: lamotrigina.
grandes cantidades de dinero, comprometiendo las finanzas
familiares. Se encuentra verborreico, con aceleracin del MIR 94 (3783): Cul de las siguientes afirmaciones, sobre el
pensamiento y con apariencia megalomanaca. Como Litio y su utilizacin en el tratamiento farmacolgico de la fase
antecedentes personales no hay enfermedades de inters salvo manaca de la enfermedad manaco-depresiva, NO es cierta?:
un episodio depresivo hace 5 aos. No cree que le pase nada,
pero ha aceptado acudir a la consulta con la intencin de buscar Tiene un mayor grado de especificidad que los neurolpticos.
ayuda para deiar de fumar. No hay consumo de otros txicos y Tiene un fcil control mediante los niveles en plasma.
Ia analtica y exploracin neurolgica no aportan datos Tiene un inicio de accin mas rapido que los neurolpticos.*
anormales. En este paciente teniendo en cuenta su diagnstico re-ON. Los niveles inferiores a 0,4 mEq./l. habitualmente no se han
mas probable, qu tratamiento sera el MENOS indicado: asociado a respuesta teraputica.
Bugrogion. * .U Los niveles superiores a 7,5 mEq./I. habitualmente se han
cido valproico. asociado a efectos secundarios.
Carbonato de litio.
Risperidonc. MIR 95 FAMILIA (4104): Los mrgenes de litemia que se aconse-
PPPN. Olanzapna. an con carcter general en la profilaxis del trastorno afectivo
bipolar son:
Antipsicticos atpicos 0,5- 0,8 mEq/L.
o Se consideran la terapia de eleccin en los episodios manacos 0,5 l mEq/L.
graves porque controlan la agitacin del paciente. 0,5- 1,2 mEgZL.*
o Los antipsicticos atpicos tienen menor tendencia a inducir 0,5- 1,7 mEq/L.
sintomatologa depresivo que los tpicos. .UPPJN." 0,5- 1,6 mEq/L.
MQ
c Particularmente indicado en trastornos bipolares. MIR 95 (4364): El acceso manaco grave puede tener por tera-
o Tarda ms en actuar que los antipsicticos, pero su accin es pia fundamental:
ms especfica sobre el trastorno afectivo. M"
o Ms eficaz para controlar el humor y la ideacin autoltica. Relaiacin con biofeedback.
Terapia electroconvulsiva Litio.
0 Es quizs el tratamiento ms eficaz para la mana pero se usa Ansiolticos.
poco. Se prescriben habitualmente tres sesiones semanales, y la P1159359? Psicoterapia cognitivo-conductual.
eficacia suele notarse a partir de la 6 sesin.
Es de primera eleccin en: delirium manaco, mana catatnica MIR 95 (4369): Las sales de litio estn particularmente indicadas
y en embarazadas. en el tratamiento de:
La esquizofrenia paranoide.
La esquizofrenia catatnica.
H IPOMAN lA: El alcoholismo crnico. S
Tratamiento de eleccin: Litio. ,05
La psicosis manaco-depresiva o trastorno bipolar. <_<
D
.UFWNT' La neurosis obsesivo-compulsiva. Q
(I)
D.

que

ME
IX. TRASTORNOS DEL HUMOR

MIR 96 (4773): Uno de los medicamentos siguientes &se


considera til para prevenir ulteriores episodios en los trastornos 12. Suicidio
afectivos de carcter bipolar:
El Clonacepam.
El carbonato de litio. 12.1. Epidemiologa
El cido Valproico.
La Levomepromacina.* 8 por 100.000 habitantes/ao en Espaa.
PPPN. La Carbamacepina. - M;
Suicidios consumados: 3 veces ms frecuentes en varones.
MIR 96 (4775): Qu medicamento elegira en primer lugar Intentos de suicidio: 2-4 veces ms frecuentes en muieres.
para prevenir nuevos episodios manacos en un paciente diag- o Edad:
nosticado de trastorno maniaco-depresivo y sin otros trastornos 0 Suicidios consumados: Ia incidencia aumenta progresi-
somticos acompaantes? vamente con Ia edad, hacindolo bruscamente a partir
Carbamacepina. de los 55-60 aos (especialmente en varones).
Propanolol. v 70% de suicidios en mayores de 40 aos.
Carbonato de Litio.* o Intentos de suicidio: ms frecuentes en qente oven.
Diacepam.
.UPPN. Clonidina. 12.2. Factores causales
MIR 98 FAMILIA (5587): En el tratamiento de un episodio man- TRASTORNOS MENTALES:
aco, Cul de los frmacos siguientes est CONTRAINDICA- Casi el 95% de los pacientes que se suicidan tienen un trastorno
E? mental diagnosticado:
Fluoxetina.* o Trastornos afectivos 60%. Sobre todo depresin psictica.
Levomepromacina. o Consumo de alcohol 30% y otras sustancias.
Haloperidol. o Esquizofrenia 10% .
Carbamacepina. o Gran riesgo en los meses posteriores al alta hospi-
WFPNT' Carbonato de litio. talaria: depresin postesquizofrnica.
o Sndromes orgnicos cerebrales 5%.
MIR 99 FAMILIA (6107): Cul de los siguientes frmacos se o Trastornos de personalidad, sobre todo el lmite.
considera como teraputica de eleccin para los trastornos bipo-
Riesgo de suicidio en pacientes depresivos:
Iares, especialmente en el caso de los ciclodores rpidos?:
Bipolar > unipolar > distimia.
I. Olonzapina.
2. Sertralina. 12.3. Prediccin
3. Inhibidores selectivos de Ia recaptacin de serotonina (ISRS).
4. Carbamacepina.*
SUICI'DIO
5. Diacepam. INTENTOS
CONSUMADO
Muieres < 350 Varones > 60 aos
MIR 01 (7134) Respecto del tratamiento del trastorno afectivo
bipolar es FALSO que: Trastornos de la personalidad Depresin, alcoholismo, esqui-
7. El uso de frmacos antipsicticos en las fases manacos (Iimite) zofrenia
conlleva el riesgo de inducir un viraie a la fase depresivo. Baia letaldad, en pblico, Alta Ietalidad (armas de fuego,
2. El uso de frmacos antidepresivos en la fase depresivo con- deSCUbrible horca)
lleva el riesgo de inducir un viraje a la fase manoca. En privado, aislado
En el tratamiento de mantenimiento de los pacientes bipola-
res se recomienda el uso de uno o ms frmacos eutimizan- 12.4. Factores de riesgo
tes.
Los frmacos anticomiciales no deben utilizarse en estos o Sexo: varn mayor de 55 aos.
pacientes por el riesao de que desarrollen una discinesia o Estado civil: separacin reciente, divorciado o viudo.
tarda.* o Otros factores sociodemogrficos: raza blanca, religin (ate-
En los casos de mana disfrica (mana mixta) o de ciclacin os > budistas > cristianos > hindes > musulmanes), am-
rpida la respuesta a las sales de litio parece ser inferior a lo bos extremos sociales, profesionales (mdicos, militares).
normal. o Trastorno mental: trastornos afectivos (depresin), consumo
de alcohol y/o otras sustancias, esquizofrenia.
MIR 04 (7915) CuI de los siguientes aspectos N_O es carac- o Ideacn de suicido frecuente y prolongada.
terstico del tratamiento con sales de Iitio?: o Advertencias previas de que va a suicidarse, hacer testamen-
Precisa para su control de determinaciones de litemia. to.
Es eficaz en el tratamiento del episodio manaco. . Intento previo de suicidio, sobre todo, con mtodos de alta
Eleva los niveles de uricemia.* letaldad: Ia ms importante.
Es eficaz en la profilaxis del trastorno bipolar. o Aislamiento social, con malas relaciones personales con
S-"PPNT' Puede aparecer, a largo plazo, toxicidad tiroidea. familiares y amigos. Vida afectiva o de relacin pobre.
o Sentimiento de culpa, desesperanza.
MIR O7 (8699): Sealar cul de las siguientes afirmaciones es
o Agitacin motora importante, ansiedad.
FALSA en relacin a los efectos adversos de las sales de litio en o Historia familiar de suicidio (factores genticos).
el tratamiento del trastorno bipolar:
o Desempleo o dificultades econmicas especialmente si hay
i. El litio puede ocasionar alteraciones de la funcin renal.
cada del status.
2. El litio puede causar alteraciones de la electrofisiologia car-
o Problemas maritales.
diaca.
o Estrs o acontecimientos vitales.
3. EI litio puede provocar exacerbacin de la psoriasis.
o Agresividad, irritabilidad.
4. El litio puede provocar insuficiencia respiratoria.*
o Baios niveles de 5-HIAA (metobolito de serotonina).
5 El litio puede producir hipotiroidismo.
o Enfermedad fsica crnica y / o discapacitante.

6147

MIR
CURSO INTENSIVO MIR ASTURIAS

MIR 96 (4769): Cul de los siguientes trastornos afectivos con-


12.5. Tratamiento lleva un mayor riesgo de suicida?
El trastorno distmico.
o Ante un paciente con ideas suicidas se deben administrar El duelo no complicado.
antipsicticos sedantes. Los trastornos esquizoafectivos.
o En pacientes con potencial riesgo suicida est indicada la La depresin psictica.*
hospitalizacin. Especialmente pacientes con depresin ma- 9999.? La hipomana.
yor e ideacin autoltica acusada.
. En el caso de los intentos de suicidio se debe retener al pa- MIR 96 FAMILIA (4543): De los siguientes pacientes con ideacin
ciente en el hospital entre 24-48 horas para que la angustia, suicida, quin piensa que tiene mas riesgo objetivo de tener
la tensin y la situacin que han provocado el intento dismi- una conducta suicida letal32
nuyan. Posteriormente se tratar la enfermedad psiquitrica I. Varn de 50 aos que acude a urgencias acompaado de
que provoc la situacin y tratar de contarse con la colabo- su esposa por trastorno de panico.
racin de la familia (vigilancia dsimulada). Tambin se tra- 2. Muier de 18 aos con reaccin depresivo breve tras ruptura
tarn la ansiedad con benzodiacepinas o antipsicticos se- afectiva.
dantes. 3. Varn de 70 aos viudo desde hace 2 meses, con reaccin
timos"
MIR 82 (506): Las tentativas de suicidio son ms frecuentes en: 4. Muier de 35 aos con distimia y tratamiento antidepresivo
I. Nios. desde hace 3 meses.
2. Adultos. 5. Varn de 25 aos drogodependiente.
3. Jvenes y adolescentes.*
4. Vieios. MIR 98 (5840): Seale cul de estas proposiciones es FALSA
5. En la involucin. respecto a la conducta suicida:
I. El nmero de intentos de suicidio es mayor en los hombres
MIR 88 (2164): Un paciente con importantes ideas de suicidio que en las muieresj mientras que los suicidios consumados
debe ser tratado por su mdico con: se dan ms en las muieres.*
Neurolpticos sedantes.* 2. El riesgo de suicidio puede surgir en cualquier categora de
Benzodiacepinas. diagnstico psiquitrico.
Antidepresivos, exclusivamente. 3. EI riesgo de suicidio aumenta con Ia edad.
Psicoterapia de apoyo. 4. Las tentativas "chantaiistas" de suicidio son menos graves
.UFPN Barbitricos. pero no exentas de riesgo.
5. Aumenta el riesgo cuando existen antecedentes familiares de
MIR 89 (2383): Qu tratamiento farmacolgico de primera suicidio.
eleccin se debe utilizar en un paciente con contumaces ideas de
suicidio?: MIR 99 FAMILIA (6112): Se considera que hay gran riesgo de
Benzodiacepinas. repeticin de conducta suicida, cuando se dan todas estas cir-
Antidepresivos tricclicos. cunstancias EXCEPTO una selela:
Sulpiride. Ideacin suicida frecuente y prolongada.
Neurolgticos sedantes.* Uso de mtodo de alta Ietalidod.
9p? Antidepresivos IMAOs. Viudedad o separacin conyugal.
Vida afectiva y de relacin pobre.
MIR 93 (3525): Cul de estos 5 pacientes presenta mayor ries- PFF'JNT Edad inferior a 25 aos.*
go para efectuar un acto suicida3:
'I. Varn de 35 aos, soltero, vive con sus padres. MIR 00 FAMILIA (6627): Cul de los siguientes problemas
2. Muier de 55 aos, casada, internada en centro psiquitrico. psiquitricos ve aumentada su frecuencia con la edad?
3. Varn de 30 aos, soltero, alcohlico conducta violenta Depresin.
reciente, Trastornos pscticos
4. Varn de 55 aos, alcohlico, viudo, vive slo.* Trastornos fbicos.
5. Muier de 70 aos, viuda, vive con hiia. Suicidio.*
.UFPJNT Trastorno de ansiedad generalizada.
MIR 93 (3579): Un paciente alcohlico crnico de 46 aos que
vive slo, fue diagnosticado hace 2 aos de depresin severa. MIR 05 (8178): Cul de las siguientes afirmaciones con respec-
Actualmente consume unos 1809 de alcohol puro al dia y ha to al suicidio es INCORRECTA?:
presentado una recada de su enfermedad depresivo. Cual es I. El riesgo de suicidio es mayor en los familiares de pacientes
la actitud mas adecuada en este paciente? depresivos que en familiares de pacientes con mana o es-
I. Debe valorarse seriamente el riesqo de suicidio de este quizofrenia. ,
paciente. El alcoholismo, la depresin v el vivir slo son fac- 2. Las tasas de suicidio son ms altas en las zonas urbanas
tores de riesgo.* que en las rurales.
2. Debe iniciarse urgentemente un tratamiento con disulfiram. 3. Un periodo de menor riesgo al suicidio en pacientes esqui
3. Un tratamiento con antidepresivos tricclicos durante tres zofrnicos son los meses siguientes al alta hospitalaria.
semanas y valorar la situacin al cabo este tiempo es la ac- 4. En los ancianos, los intentos de suicidio son menos frecuen-
titud ms correcta. tes, pero el xito es mayor.
4. El ingreso hospitalario no est indicado, pues el paciente 5. Los varones deprimidos son ms proclives al suicidio que las
probablemente se negar a l. muieres.
5. La asociacin de antidepresivos tricclicos con benzodiace- AN U LADA
pinas puede ser til. Por una parte tratamos la depresin y
por otra mitigamos los sntomas de deprivacin alcohlica. MIR 06 (8438): Cul de los siguientes factores NQ se asocia a
un incremento del riesgo de muerte por suicidio?:
MIR 95 (4367): La prevalencia de suicidio consumado en el No ser creyente.
mundo industrializado es mayor entre: Ser militar.
Las muieres ivenes Haber sido diagnosticado de depresin.
Los hombres ancianos.* Ser muier.*
Los hombres venes. wewwr Ser mdico
Las muieres ancianas.
PPPN. Los hombres y las muieres de edad intermedia

fue

MES.
IX. TRASTORNOS DEL HUMOR

MIR 07 (8697): Cul de los siguientes enunciados respecto a la


depresin mayor o episodio depresivo mayor es INCORREC-
T03:
l. A lo largo de la vida, aproximadamente un 15% de la
poblacin sufre un episodio depresivo mayor.
2. Cerca de un 8% de los pacientes que atiende un mdico de
Atencin Primaria podra recibir un diagnstico de depresin
mayor.
3. Hasta un 75% de los casos de depresin mayor son
secundarios a una enfermedad mdica somtica o al abuso
de substancias.
4. Cuando se sospecha que un paciente Sufre un episodio
depresivo mayor, debe evitarse hacer prequntas directas al
naciente sobre ideas o proyectos suicidas, porque tal
pregunta aumenta el riesqo de suicidio un 35%.*
5. Cerca del 15% de los pacientes con depresin mayor que no
reciben tratamiento se suicidan.

MIR 'IO (9437): Seale cul de las siguientes aseveraciones es


CORRECTA en relacin con el suicidio en la esquizofrenia:
l. El porcentaie de personas con esquizofrenia que fallece por
suicidio alcanza el 5%.*
2. La presencia de deterioro cognoscitivo se asocia con un
menor riesgo de suicidio.*
3. El riesgo de suicidio es mayor en las personas de edad
media con varios aos de evolucin de la enfermedad.
El riesgo de suicidio es mayor en mujeres.
91:" El riesgo de suicidio es mayor en las personas procedentes
de un estatus socio-econmico baio.
AN U LADA

MlR 12 (9936): Lo ms probable es que las preguntas


especficas directas acerca del suicidio planteadas al comienzo
de una entrevista con un paciente deprimido:
'l. Alarmen al paciente.
2 Refuercen una ganancia secundaria.
3. Aumenten la resistencia al cambio teraputico.
4 Sean ms eficaces si se realizan de manera prudente e
indirecta.
5. Carezcan de influencia en laprobabilidad de un intento de
suicidio.

{44
CURSO INTENSIVO MIR ASTURIAS

RESUMEN DE TRASTORNOS DEL HUMOR (AFECTIVOS)


. Trastorno depresivo: alteracin del estado de nimo en sentido de tristeza.
0 Mana: exaltacin del estado nimo.
o Trastorno bipolar: combinacin de ambos tipos en el mismo paciente.

I. EPIDEMIOLOGA
o La depresin y la distimia son ms trecuentes en muieres (aproximadamente x2).
o El trastorno bipolar tiene la misma incidencia en varones que en muieres.
o Edad media de inicio: trastorno bipolar sobre los 30 aos, trastorno depresivo sobre los 40 aos.

2. SNTOMAS AFECTIVOS
o Anhedonia: Incapacidad para experimentar placer: propia de las depresiones (MIR).
o Distoria: sensacin de malestar general, especialmente estado de nimo depresivo con ansiedad e inquietud.
o Alextimia: incapacidad para expresar los afectos a travs de las palabras.

3. Trastornos depresivos
Episodio depresivo: el primer episodio de un paciente.
Trastorno depresivo recurrente: al menos otro episodio depresivo previo. No antecedentes de episodios manacos o hipoman-
acos. Intervalo libre de sntomas entre un episodio y otro de al menos meses.
o Distimia (trastorno del humor persistente): Menor gravedad que episodio depresivo y al menos 2 aos.

4. DEPRESIN MAYOR
QBITERIQs DIAQNSIICOQ:
o 5 ms de los siguientes sntomas durante al menos 2 semanas: (uno de ellos debe ser el I el 2)
Estado de animo deprimido durante la mavor parte del da, casi todos los das
Disminucin acusada del inters o de la capacidad para el placer (anhedonia)
Prdida de peso (+++) o aumento de peso (MIR)
Insomnio (+++) o hipersomnia (MIR)
Agitacin o enlentecimiento psicomotor (MIR)
Fatiga o prdida de energa
Sentimientos de inutilidad o culpa excesiva
Disminucin de la capacidad para pensar o concentrarse
9.\'9S": .9! :" Pensamientos recurrentes de muerte, ideacin suicida, plan suicida o tentativa suicida
o EI pisodio depresivo puede acompaarse de:
Sindrome somtico (hablamos de depresin melanclica, endgeno o vital):
o Al menos 4 de los siauientes sntomas:
o Prdida de inters / anhedona
Descensdo de la reactividad emocional
Despertar precoz (2 horas antes)
Empeoramiento matutino (variacin circadiana del estado de nimo) (MIR).
Inhibicin / agitacin psicomotriz
Prdida de peso (5% en ltimo mes)
00000
o Descenso de Ia libido
Sntomas psicticos: retlean enfermedad grave(MIR).
o Congruentes con el estado de nimo: ideas delirantes o alucinaciones con contenido compatible con temas depresivos habi-
tuales (culpa, enfermedad, muerte, ruina, nihilismo...) (MIR). Ms frecuente.
- Incongruentes con el estado de nimo: ideas delirantes o alucinaciones con contenido no compatible con temas depresivos
habituales (persecucin, control, insercin o transmisin del pensamiento..).

4.1 . SNTOMAS ASOCIADOS

o AFECTIVIDAD: angustia (MIR), irritabilidad, tristeza, ansiedad, fobias.


PENSAMIENTO: prdida de la autoestima (MIR), ideas de culpa o de ruina (MIR).
o CONDUCTA: abandono personal, 15% intentos suicidio. 1 causa suicidio, aislamiento, prdida de libido (MIR)
o RITMOS BIOLOGICOS: inicio en primavera/otoo , meiora vespertino (MIR), despertar precoz,
o TRASTORNOS SOMTICOS: trastornos digestivos, estreimiento, astenia, amenorrea, sequedad de boca.
o SINTOMAS CATATONICOS.

4.2. ETIOPATOGENIA

o Factores biolaicos: bsicos en las depresiones endgenas.


o Factores genticos: 25-40% de familiares de primer grado afectos en pacientes con trastorno bipolar, 20% en unipolar.

4.3.HALIAZGOS DE LABORATORIO
<_(
A. BIOQUMICA: hipotunciln: noradrenrgica, serotoninrgica y dopaminrgica. 9.4
<_(
B. NEUROENDOCRINILOGIA 3
g
o Test de supresin condexametasona (MIR): en 25-40% de los depresivos no hay trenacin del cortisol plasmtico a las l horas nu.
de administrar 1 mg de dexametasona.

Iiili
IX. TRASTORNOS DEL HUMOR

o Test de estimulacin de TSH por TRH (MIR): en un 30% de los depresivos endgenos unipolares no hay aumento de Ia TSH tros
Ia administracin de TRH.
o Cuando estos test estn alterados, no se debe suspender el tratamiento hasta que no se normalicen por el alto riesgo de re-
cadas.
C. SUEO: latencia del sueo prolongada, despertar precoz, despertares intermitentes. Acortamiento de Ia latencia del REM (se
relaciona con peor pronstico).

4.4.FORMAS ESPECIALES DE DEPRESIN

o Depresin enmascarado o equivalente depresivo: sntomas somticos que encubren un cuadro depresivo.
Degresiones atpicas: depresiones con caractersticas invertidas: hiperfagia, hiperinsomnia, parlisis plmbea (pesadez inten-
sa en brazos y piernas), marcado sensibilidad al rechazo interpersonal... Pueden estar indicados los IMAOS (MIR).
o Depresin reactiva (neurtica, situacional): rasgos neurticos de personalidad, ansiedad, empeoramiento vespertino... Menor
riesgo de suicidio que en depresin endgeno (MIR). Ms sensible o psicoterapia y peor respuesta a antidepresivos que depre
sin endgeno.
o Trastorno afectivo estacional: aparicin repetida del trastorno depresivo en Ia misma estacin del ao (ms frecuente en oto-
o / invierno, con recuperacin en primavera o verano). Clnicamente suele presentar sntomas atpicos: hipersomnia, hiper-
fagia (apetencio por HC). En relacin con ciclo de q soIar?.
o Trastorno depresivo breve recurrente: sntomas similares o episodio depresivo mayor. Duracin de 2-13 das. z I episodio /
mes durante 12 meses consecutivos. No relacin con ciclo menstrual.
o Depresin involutiva: intensa ansiedad con agitacin, ideas de culpa o de ruina, alto riesgo de suicidio. No aparece en DSM-
IV-TR ni en CIE-I 0. Ms frecuente en varones mayores de 50 aos.
o Pseudodemencia depresivo: dficit de atencin y memoria de origen depresivo, que simula una demencia. Diferencias con
demencia: inicio agudo, progresin rpida de sntomas, humor depresivo, queias cognoscitivas, no empeoramiento nocturno,
alteracin de memoria reciente y pasado, buena respuesta a antidepresivos.
a Depresin anancstica: depresin endgeno en Ia que son muy importantes los ideas obsesivas.
o Depresin anacltica de Sgizt: sndrome que ocurre en bebes menores de I ao, privados de Ia figura de Ia madre (MIR).

4.5. CIRCUNSTANCIAS QUE PUEDEN DESENCADENAR UN CUADRO DEPRESIVO (DEPRESIONES SOMATGENAS)

o FRMACOS: (MIR) entre otros: corticoides, estrgenos, progesterona, L-dopa, MetiI-dopa.


o ENFERMEDADES: Parkinson (MIR), demencia senil, ACV, hipotiroidismo, Adisson, Cushing, diabetes...

4.6. TRATAMIENTO

o PSICOTERAPIA: Cognitiva e Interpersonal. Actitud inexcusable ante cualquier depresin, aunque el tratamiento debe ser sobre
todo farmacolgico. En estos periodos se debe evitar Ia tomo de decisiones importantes (MIR).
o ANTIDEPRESIVOS:
o (Citalopram, Escitalopram, Fluoxetina (ms desinhibidor), Fluvoxamina (sedante), Paroxetna (sedante), Sertralina).
En Ia actualidad de primera eleccin, meior tolerados (MIR), menos efectos secundarios colinrgicos que los tricIcIicos y
ausencia de cardiotoxicidad. Eficacia similar a triciclicos.
o No contraindicacionesabsolutas.
o IRSN: (Duloxetina, Venlafaxina). Utiles en casos refractarios a los ISRS. Contraindicaciones absolutas: cardipatas graves,
HTA de difcil control.
o TRICICLICOS: tiles en depresiones graves y refractarios a los anteriores. En depresiones endgenas (imipramino si pre-
domina Ia inhibicin, amitriptilna si predomina Io ansiedad, cIorimipramino, si hay trastornos obsesivos asociados). Con-
traindicaciones absolutas: cardipatas graves, alto riesgo de suicidio (gran toxicidad en sobredosis. EI mas txico: ami-
triptilina)
o Para todos los antidepresivos Ia duracin mInima del tratamiento antes de uzgar el efecto antidepresivo es de 2-6 semanas y
el tratamiento de mantenimiento no menos de meses a Ia dosis a Ia que cedieron los sntomas (MIR).
. Cuando un paciente ha respondido bien a un frmaco suele volver ha hacerlo en posteriores recadas (MIR).
o IE (Terapia electroconvulsiva): Indicaciones: fracaso farmacolgico, depresiones muy inhibidas, agitadas o con ideas deli-
roides, situacin somtica crtica, grave riesgo de suicidio, reacciones adversas o enfermedades fsicas que contraindiquen
frmacos. Indicacin principal: depresin endgeno delirante con gran agitacin y peligro de suicidio (MIR). No indicado en
depresin neurtica (MIR).

5. DISTIMIA
Forma crnica y Ieve de depresin.
Estado de nimo crnicamente triste y desonimado (o irritable en nios y adolescentes (MIR), durante al menos 2 aos, (en ni-
os y adolescentes I ao (MIR) no hay intervalos libres de sntomas superiores a dos meses) con 2 o ms de los siguientes
sintomas:
I-Aumento o disminucin del apetito,
2-Insomnio o hipersomnia,
3FaIta de energa,
4-Baia autoestima,
5-Dificultad para concentrarse,
-Desesperanza.
o Tristes, pesimistos, cansados, opticos, irritables, con falta de ilusin (MIR). Disminucin del rendimiento (MIR). Empeoramiento
vespertino (MIR). Mayor riesgo de depresin mayor que en la poblacin general.
o No hay manifestaciones Dsicticas.
o EI riesgo de suicidio es menor que en Io depresin mayor (MIR).
o Tratamiento: Psicoterapia + Frmacos: ISRS (de eleccin).

MIR
CURSO INTENSIVO MIR ASTURIAS

. DEPRESIN DOBLE
Se superponen episodios depresivos mayores sobre un trastorno distmico (MIR).

7. TRASTORNO BIPOLAR
Etiopatogenia: factores biolgicos y genticos son bsicos. Los factores psicosociales pueden influir en las recadas. Los facto-
res estacionales, tambin influyen: aumento de Ias recadas por depresin en invierno-otoo y por mana en primavera-
verano. En hombres, mas frecuentes y numerosos los episodios maniacos y en muieres Ios depresivos.

7.1. EPISODIO MANIACO

Estado de animo anormal y persistentemente elevado, eufrico de aI menos I semana, (MIR) acompaado de tres mas de
Ios siguientes sntomas:
o Autoestima exagerada
Disminucin de Ia necesidad de dormir (MIR)
Verborrea (MIR)
Fuga de ideas (MIR)
Distrabilidad (MIR)
OOOOO Aumento de Ia actividad intencionada (MIR)
o Implicacin excesiva en actividades placenteras
Pueden aparecer ideas delirantes (Congruentes con el estado de animo) o alucinaciones (ms frecuentes auditivos) (MIR).
Los episodios maniacos suelen iniciarse de forma brusca, Ia disminucin de Ia necesidad de dormir suele ser el primer snto-
ma en las recadas (MIR).

7.2. EPISODIO HIPOMANIACO

Exaltacin del estado de animo (MIR). Diferencias con episodio manaco:


o Duracin temporal: al menos 4 das
o No se produce deterioro socio-laboral
o No hay sntomas psicticos.

7.3. EPISODIO MIXTO

AI menos durante I semana se cumplen diariamente los criterios tanto de episodio manaco como de episodio depresivo ma-
yor.
Frecuente ideacin suicida y sntomas psicticos.

7.4. FORMAS CLINICAS DEL TRASTORNO BIPOIAR

BIPOLAR I: se alternan episodios de depresin mayor y de mana, o episodios mltiples de mana exclusivamente (tambin
pueden aparecer episodios de hipomana o mixtos).
BIPOLAR II: 1 o ms episodios de depresin y al menos I episodio de hipomanIa (nunca episodios de mana).
TRASTORNO BIPOLAR NO ESPECIFICADO
o Episodios hipomanacos sin sntomas depresivos intercurrentes.
o Episodios maniacos o mixtos en trastornos psicticos
TRASTORNO CICLOTIMICO: Durante al menos dos aos (en nios y adolescentes I ao) numerosos episodios de hipomana
y numerosos perodos de sntomas depresivos que no cumplen criterios de episodio depresivo mayor (MIR), no mas de dos me-
ses sin sntomas.
Los episodios maniacos o hipomanacos no han sido inducidos por antidepresivos o TEC.

7.5. FORMAS ESPECIALES DEL TRASTORNO BIPOIAR

A. CICLADORES RPIDOS:
Ms de cuatro episodios aI ao de mana y/o depresin. Tratamiento: carbamacepina (MIR). Tanto en Ia fase aguda como en
Ia profilaxis. EL 80% mujeres.
TRASTORNO BIPOIAR CON PATRN ESTACIONAL:
Depresin en invierno y mana o hipomania en primavera y verano. Tratamiento: Estimulacin luminosa.

7.6. FARMACOS Y PATOLOGIAS RELACIONADAS CON CUADROS MANIFORMES

FRMACOS: entre otros: antidepresivos, corticoides (MIR), L-dopa, psicoestimulantes (cocana, anfetaminas)...
ENFERMEDADES: ACV, tumores SNC, Addison, Cushing, infecciones (sfilis terciaria, fiebre tifoidea, sepsis por IegioneIIa (MIR).

7.7. TRATAMIENTO

EPISODIO DEPRESIVO AGUDO: antidepresivos (ISRSI + Li / Vammo / Antipsictico atpico (aadir eutimzantes o anti-
psicticos hacen menos probable el giro a mana ya que el uso de antidepresivos supone un riesgo de vraie a Ia fase mana-
ca) (MIR). Considerar Ia Iamotrigina si antecedentes de inestabilidad anmca.
EPISODIO MANIACO AGUDO: NS
o Se necesita ingreso hospitalario en el 75% de los casos. .95
S
o Se emplean antigsicticos atpicos (en accesos maniacos graves porque controlan Ia agitacin del paciente) (MIR) + L_ItI__o_ 3
g
(eutimizante). EI Iitio tarda entre 9-10 das en hacer efecto (MIR). Niveles teraputicos de Iitio 0,5-I,2 mEq/I (MIR). u)
D.
IX. TRASTORNOS DEL HUMOR

Otros tratamientos tiles tanto en el tratamiento como en la prevencin de episodios manacos son: carbamacepina, valproa-
to o TEC.
En casos de mana estan contraindicados los antidepresivos, porque empeoran los sintomas manacos (MIR).
HM Lto.
EPISODIOS MIXTOS: Carbamacepina o valproato. El Litio es menos eficaz (MIR). En caso de necesitarse se usarn antipsicti-
cos atpicos. En general, se evitan los antidepresivos.
ClCLADORES RAPIDOS
EI tratamiento de eleccin en ciclacin rpida es: Litio / Valproato / Lamotrigina.
o No se recomienda el uso de frmacos con alto potencial de viraie (antidepresivos o antipsicticos). Los antidepresivos
pueden inducir viraie a fase manaca, y los antipsicticos pueden inducir viraie a fase depresivo (MIR).

7.8. PROFILAXIS DE TRASTORNOS BIPOLARES

Litio, es el frmaco de eleccin en la profilaxis (MIR). La duracin del Tratamiento sera de 3-5 aos y los niveles plasmticos de
0,5-1 ,2 mEq/l (MIR). Contraindicaciones: cardopatas, renales, tiroideas (MIR) y embarazo.
Otros frmacos tiles en profilaxis: carbamacepina, valproato, clonacepam (MIR), lamotrigina.

8. SUICIDIO
Los suicidios consumados son ms frecuentes en varones sobre todo a partir de los O aos (MIR) Los intentos de suicidio son
mas frecuentes en muieres y en gente oven (MIR). La incidencia de suicidio consumado aumenta progresivamente con Ia edad
(MIR) hacindolo bruscamente a partir de los 60 aos.
Por orden de menor a mayor incidencia de suicido segn el estado civil: casados con hiios, solteros, viudos, divorciados.
Sntomas y factores que indican riesgo de suicidio (MIR): Varn mayor de 55 aos, recientes problemas matrimoniales, aisla-
miento social, desempleo, intento de suicidio previo con mtodos de alta letalidad, deacin suicida frecuente y prolongada,
advertencias previas de que va a suicidarse, trastorno mental (depresin sobre todo endgeno y si existen sntomas psctcos
(MIR), alcoholismo, esquizofrenia gran riesgo en los meses posteriores al alta hospitalaria- (MIR), importante agitacin motora,
ansiedad, sentimientos de culpa, preocupaciones hpocondracas, enfermedad crnica dolorosa, invalidante.
La disminucin del SHIAA en LCR se considera marcador de suicidio.
Tratamiento: Si el paciente ha tenido un intento de suicidio y esta agitado, esta indicado el ingreso hospitalario y antipsicti-
cos sedantes (MIR). Posteriormente se tratar la enfermedad psiquitrica que motiv el intento de suicidio.
CURSO INTENSIVO MIR ASTURIAS

Trastornos de ansiedad, somatomorfos y


disociativos
Nmero de preguntas del captulo en el MIR

90 9192 94 9f 95 9f 96 97f 97 98f 98 99l OOf 00. 01. 02. 0. 04. 05. O. 07.08.09. 701213 14

Nmero de preguntas de cadatema

Concepto y epidemiologa

Trastornos de ansiedad fobica

Ataques de pnico (crisis de angustia)

Trastorno de panico (trastorno de angustia)

Trastorno de ansiedad generalizada

Trastorno obsesivo compulsivo (TOC)

Reacciones a estrs grave y trastorno de adaptacin

Trastorno de ansiedad secundario

Trastornos somatomorfos y trastornos disociativos

Q imprescindible
o Tema muy preguntado los ltimos aos, es fundamental diferenciar entre crisis de angustia, trastorno de angustia y trastorno de
ansiedad genearlizada. Han repetido muchas veces las caractersticas generales del Trastorno obsesivo compulsivo.
o Los ataques de pnico o crisis de angustia son episodios sbitos de miedo y malestar con una descarga neurovegatitva intensa
son sntomas fsicos y cognoscitivos, sensacin de peligro o muerte inminente (7MIR). Las benzodiacepinas meoran los sntomas.
No slo pueden aparecer en el trastorno de angustia, sino que se pueden presentar al exponerse un suieto al obieto de su fobia,
o incluso ser desencadenados por sustancias. Es importante identificar en una pregunta mir si la crisis es desencadenada por
algn agente externo o es espontnea.
o En el trastorno de angustia aparecen varias crisis de angustia sin desencadenante, con un periodo entre ellas de preocupacin
en relacin a estas crisis o de cambios en el comportamiento.
o En el trastorno de ansiedad generalizada la ansiedad es persistente, no asociada a ninguna circunstancia concreta, y es de larga
duracin. Es ms frecuente en mueres jvenes.
o En el trastorno obsesivo compulsivo el paciente experimenta obsesiones: ideas recurrentes que experimentan como intrusas que
causan malestar o ansiedad y que intenta neutralizar con pensamientos o acciones (compulsiones). No son ideas delirantes aun-
que eI sueto no las pueda controlar (reconoce que son producto de su mente). (7MlR) Las mas frecuentes son las de limpieza.
Puede ser muy grave y requerir hospitalizacin. En su tratamiento se usan lSRS.
o En los trastornos somatomorfos (hipocondra, conversin, somatizacin) no existe intencionalidad. Enel trastorno de somatiza-
cin aparecen sintomas orgnicas como cefalea, dolores articulares o abdominales, gastrointestinales, neurolgicos sin encon-
trarse evidencia de patologa organica. La hipocondra se caracteriza por una preocupacin y miedo de padecer una enferme-
dad grave a partir de la mala interpretacin de sntomas fisiolgicos.
o EI trastorno dismrco se caracteriza por una preocupacin excesiva por una caracterstica o defecto imaginario del aspecto
fsico que llega a provocar malestar o deterioro social o laboral.
o En el trastorno de conversin aparecen sintomas pseudoneurolgicos: afasia, parestesias, alteraciones del equilibrio.

en
MIR
mm
X. TRASTORNOS DE ANSIEDAD, SOMATOMORFOS Y DISOClATlVOS

i. Concepto y e-oidemioloa Trastorno esquizoafectivo.


Distimia.
o La ansiedad es un estado de activacin del SNC por el que el Trastorno de ansiedad generalizada.
organismo se prepara para hacer frente a una situacin eX- Trastorno por estrs postraumtico.
traordinaria". .UPPNI Reaccin normal frente al estrs.*
o Sensacin subjetiva de tensin, excitacin, acompaada de
activacin vegetativo. MIR 08 (8957): Respecto a los trastornos de ansiedad seale la
o Cierto grado de ansiedad, aumenta el rendimiento, pero al respuesta FALSA:
atravesar cierto umbral, el rendimiento disminuye. 1. Los pacientes que sufren un trastorno obsesivo compulsivo
tienen ideas obsesivas irracionales e intrusivas. Las ms fre-
MIEDO ANSlEDAD cuentes son las obsesiones de limpieza.
2. Son los trastornos ms comunes en Ia poblacin general.
Amenaza conocida Amenaza desconocida
Son ms frecuentes en mujeres y su incidencia disminuye
Externa lnterna con la edad.
Inmediata Futura 3. EI abuso de alcohol es una complicacin grave y frecuente.
Definida Vaga 4. El trastorno por estrs postraumtico suele aparecer en los
seis primeros meses tras el suceso traumtico.
1.1 . Ansiedad normal 5. El trastorno de ansiedad qeneralizada se caracteriza por la
presencia de crisis de anaustia, recurrentes e inesperadas y
o La ansiedad normal: respuesta a estmulos externos poten- puede cursar con o sin aaorafobia.*
cialmente peligrosos. Adaptada en intensidad y tiempo a|
estmulo que la genera y capaz de generar una respuesta
adecuada.
2. Trastornos de ansiedad fbica
1.2. Epidemiologa
2.1. Concepto
o Los trastornos de ansiedad son muy frecuentes. Prevalencia
anual en poblacin general superior al 10%. . Temores irracionales y exagerados hacia obietos, situaciones
o La prevalencia en muieres es el doble que en varones. o funciones corporales que en situacin normal no san fuen-
o Sus posibilidades de aparecer disminuyen con la edad. Mu- tes obietivas de miedo.
chos cuadros de aparicin tarda que se manifiestan con o El temor fbico tiene las siguientes caractersti-
gran ansiedad no son trastornos de angustia sino depresio- cas :desproporcionado, irracional, fuera de control volunta
nes ansiosos. rio, evitacin de la situacin temida,ansiedad anticipatoria
o El ms comn de los trastornos de ansiedad es la fobia sim- o Las fobias suelen asociarse a un aumento de los anteceden-
ple. tes familiares de ansiedad.
o El trastorno de ansiedad gue demanda tratamiento mas
frecuentemente, son Iascrisis de angustia (crisis de ansie-
2.2. Curso y pronstico
Ml.- o Suele ser crnico con exacerbaciones y remisiones.
o Comorbilidad frecuente de la ansiedad con trastornos del
o La agorafobia presenta remisiones con menor probabilidad.
estado de animo y consumo de sustancias.
o La remisin espontnea es menos frecuente en pacientes con
O Curso con frecuencia crnico con perodos de meiora y otros
sntomas estables ms de 1 ao.
de empeoramiento.
GENTICA: 2.3. Epidemiologa
Existe evidencia de transmisin gentica de los trastornos de 1. Mas frecuentes en sexo femenino.
ansiedad. Las mayores evidencias se tienen con el trastorno de 2. 7% de la poblacin general en adultos, solo 0,5-O,8% son
gnica y el TOC. incapacitantes.
3. En los nios los temores fbicos son muy frecuentes, hasta el
1.3. Sntomas de ansiedad 90% de Ia poblacin infantil los padece.
o Perifricos: por estimulacin del SN Vegetativo (Sntomas 2.4. Tipos de fobias
Somticos).
o Centrales: por estimulacin de Sistema Lmbico / Corteza Cere- Agorafobia.
bral (Sntomas Psicolgicos). Fobia social.
o Fobia especfica o simple.
o, PERIERlCAS CENTRALES? AGO RAFOBlA

. Taquicardia / Palpitaciones . Aturdimiento / Ansiedad o Aparicin de ansiedad al encontrarse en lugares o situa-


. Disnea / Sensacin ahogo . Desasosiego / lrritabilidad ciones donde escapar puede ser difcil o donde en el caso
.Temblores / Sacudidas . Miedo incoercible /Aprensin de aparecer una crisis de angustia o sntomas similares
. Mareos / Inestabilidad . Ansiedad flotante" puede no disponer de ayuda.
. Sudor/ Enroecimiento . Agobio psicolgico o Los temores agorafbicos suelen estar relacionados con un
. Escalofros / Calor . Miedo a perder el control coniunto de situaciones caractersticas como: estar solo fue-
. Nuseas / Diarrea . Deseo de escapar ra de casa, mezclarse con la gente, hacer cola pasar por un
. Parestesias / Entumecimiento . Despersonalizacin/Desrealizacin puente, viaiar en autobs, tren o automvil.
o Estas situaciones se evitan, o se resisten a costa de un
malestar o ansiedad significativos.
MIR O7 (8700): Mujer de 47 aos sin antecedentes psiquitricos o Tiene que haber al menos dos sntomas de ansiedad entre
que es hospitalizada para estudio de metrorragias y a los 5 das los siguientes al enfrentarse a Ia situacin temida (uno al
de su ingreso es informada del diagnstico de neoplasia uterina menos debe estar comprendido entre el l y el 4):
con metstasis. Se consulta a Psiquiatra porque 24 horas des- Sntomas autonmicos:
pus muestra tristeza y llanto frecuente, refiere ideas de muerte y o Palpitacones o ritmo cardiaco acelerado.
presenta insomnio. En Ia evaluacin psiquitrica no se recoge t Sudacin.
ningn antecedente psiquitrico y la exploracin detecta elevada o Temblores o sacudidas de los miembros.
ansiedad y desesperanza en relacin con las consecuencias de o Sequedad de boca.
su enfermedad neoplsica. El diagnstico ms probable es: Sntomas de pecho y abdomen:
CURSO INTENSIVO MIR ASTURIAS

Dificultad para respirar. Tratamiento:


Sensacin de ahogo. o Terapia cognitivo-conductual + ISRS.
Dolor o malestar en el pecho. o En casos restringidos puede ser til: betabloqueantes (pro-
Nuseas o malestar abdominal. pranolol) 30 minutos antes de la exposicin al estmulo fbi-
Sntomas relacionados con el estado mental: co.
o Sensacin de mareo, inestabilidad o desvanecimiento.
o Sensacin de que los objetos son irreales. MIR 93 (3527): Cul es el diagnstico ms apropiado para un
o Sensacin de perder el control (volverse loco). paciente que experimenta miedo a las tormentas?
Agorafobia.
o Miedo a morir.
Neurosis obsesiva.
Sntomas generales:
Ansiedad generalizada.
0 Sofocos o escalofros.
Trastorno adaptativo con nimo ansioso.
o Sensacin de entumecimiento u hormigueo.
.UP JEJT' Mi
El ms incapacitante de los trastorno fbicos, algunos individuos
llegan a quedar confinados en casa.
MlR 94 (3784): Cul es la entidad psicopatolgica que proba-
o Frecuentemente se asocia con trastorno de pnico. Es fre-
blemente presenta un varn de 33 aos con importante ansie-
cuente que se produzca una crisis de angustia en determina-
dad cuando en su trabaio tiene que contactar con clientes,
da situacin, y, posteriormente, el paciente evita dicha situa-
sobre todo porque tiene miedo a decir o hacer algo estpido y
cin para que no se repita (trastorno de pnico con aaorafo-
verse en ridculo y siempre le ocurre cuando se nota observado,
b_0L por eiemplo comiendo en un restaurante?
o Edad de inicio: 15-35 aos (adulto oven).
Fobia socia|.*
o Mas frecuente en muieres.
Ansiedad reactiva.
o Las actividades sociales y laborales, estn severamente limi- Agorafobia.
m y en casos extremos no puede abandonar la seguridad Trastorno obsesivo.
del hogar. WPF-NT Voyeurismo.
Tratamiento:
o Terapia cognitivo-conductual + ISRS (+/ BZD). MlR 14 (10391): Hombre de 28 aos de profesin violinista, que
o Duracin del tratamiento no inferior a 6-12 meses. consulta por haber presentado en los ltimos 3 meses crisis de
pnico durante sus actuaciones pblicas. Las crisis se acompa-
FOBlA SOClAL
an de intenso miedo a quedar bloqueado y no poder continuar
o El paciente evita situaciones sociales en las que se siente con la actuacin, algo que sera humillante para l. Este miedo
escrutado por otros por temor a la humillacin. Frente a s- le ha hecho cancelar sus prximas actuaciones. En el resto de
tuaciones como hablar, comer, beber, escribir o contestar sus actividades diarias no experimenta este temor, ni tampoco le
preguntas, en pblico, presenta una respuesta de ansiedad. sucede cuando ensaya con sus compaeros de la orquesta.
Qu diagnstico considerara ms probable para este caso?
o Temor acusado y persistente a una o ms situaciones socia-
Fobia simple.
les o actuaciones en pblico por temor a que resulten emba-
Trastorno de pnico.
razosas.
Fobia social de e'ecucin.*
o El miedo a ser eniuiciado por otras personas puede surgir
Trastorno de ansiedad generalizada.
dentro de un grupo relativamente pequeo (en agorafobia
PPP. Agorafobia.
temor a situaciones donde hay multitudes).
o Tmdo de modo patolgico" (la fobia social genera disca-
pacidad, la timidez no).
FOBlA ESPECFICA (FOBlA SIMPLE AISLADA)
o La exposicin a estos estmulos produce invariablemente una o Miedo marcado ante un obieto o situacin especfica: ani
respuesta de ansiedad, que puede tomar forma de crisis de males, fenmenos de la naturaleza, oscuridad, viaiar en
angustia. . avin, ir al dentista...
o Pueden ser restringidas (algunas actividades sociales) o difu - Para poder hacer el diagnstico es necesario que los com-
sas (todas las actividades sociales). portamientos de evitacin o el malestar provocados por la
o Estos individuos suelen presentar hipersensibilidad a la crti- situacin temida interfieran acusadamente con la rutina
ca, rechazo a las evaluaciones, baja autoestima o sentimien- normal de la persona, con las relaciones laborales o
tos de inferioridad. La situacin puede llevar a altos grados acadmicas o sociales o bien que provoque un malestar
de incapacidad social. clnicamente significativo.
0 Mayor grado de incapacidad en fobia social difusa. o Grado de incapacidad variable (depende del objeto / situa-
Edad de inicio: adolescencia. cin temida).
Prevalencia similar en ambos sexos. o La exposicin al estmulo fbico desencadena una respuesta
de ansiedad, que se acompaa de sntomas vegetativos
simpticos: hipertensin, taquicardia...
o Salvo en el caso de la hemafofobia (fobia a la sangre) don-
de los estmulos que predominan son los parasimpticos
(reaccin vagal).
o Los adultos y adolescentes, reconocen que este temor es
excesivo, esto no ocurre en los nios.
o Son los trastornos de ansiedad mcrs frecuentes (pero gene-
ran menos demanda).
o Edad inicio: infancia / adolescencia.
o Ms frecuente en muieres.
Tratamiento: ,
o Terapia cognitivo-conductual: desensibilizacin progresiva.
o Poco frecuentemente son necesarios frmacos: BZD, beta-
bloqueantes.

Quantum MtRAnmn

Fobia social
X. TRASTORNOS DE ANSIEDAD, SOMATOMORFOS Y DISOCIATIVOS

o En ocasiones se presentan como vivencias de despersonaliza-


cin con sntomas fsicos menos aparentes, desrealizacin, al
teraciones sensoperceptivas.
o No son estados psicticos ya que no hay delirio y si conciencia
de enfermedad. Son trastorno neurticos.
o Tienen carcter espontneo.
Pero pueden ser artificialmente desencadenadas por agentes
fsicos: lactato i.v., ejercicio fsico, inhalacin de C02, hiper-
ventlacin, cafena, yohimbina, soproterenol, privacin de
sueno.
TIPOS DE CRISIS DE ANGUSTIA:
o lnesperadas: surgen sin motivo aparente. Caractersticas del
trastorno de pnico.
a Situacionales: aparecen por anticipacin o tras exposicin a
estmulos o desencadenantes ambientales (visin de una araa
o serpiente). Tpicas de fobias especficas y fobia social, TOC,
estrs postraumtico...
TRATAMIENTO:
o Fase aguda: Alprazolam (BZD alta potencia).
o Tratamiento de fondo: ISRS (+/- alprazolam).

MIR 99 (6362): Un enfermo manifiesta sensacin de dicultad


Lumimensivomm 'u k ,.
respiratoria, mareo con sensacin de inestabilidad o desfalle-
Fobia simple cimiento, palpitaciones, temblor, sudoracin, sensacin de
ahogo, nuseas, despersonalizacin, parestesias, sofocacin
3. Ataque de pnico (crisis de que se alterna con escalofros, opresin precordial, miedo a
anustia) morir o a perder el control sobre s mismo. Cul es, entre los
siguientes, el diagnstico ms probable?:
Brote esquzofrnico.
o Se encuentra entre las experiencias vitales ms dolorosas.
Depresin endgena.
o Consiste en un episodio sbito diurno o nocturno, de miedo e
Trastorno obsesivo compulsivo.
intenso malestar en ausencia de peligro real que se acompa-
Crisis de angustia.*
a de descarga neurovegetativa, con sntomas fsicos y cog-
.UPPNT Sndrome de abstinencia.
noscitivos, sensacin de peligro o muerte inminente y urgente
necesidad de escapar.
MIR Ol (7T 39) Cul de estos frmacos es mas eficaz en el
o Duracin: 15-30 min. Maxima expresin en los primeros IO
tratamiento de las crisis de angustia (ataques de pnico)?:
min.
Antipsicticos atpicos.
o Al ir repitindose, la sensacin de miedo baia.
Diazepam.
. DIAGNSTICO: 24 de los siguientes Paroxetina.
o PALPlTAClONES, sacudidas del corazn o elevacin de Betabloqueadores.
la frecuencia cardiaca PPPJNT' Biperideno.
. SUDACIN AN U LADA
o TEMBLORES o sacudidas
o Sensacin de AHOGO o falta de aliento MIR Ol (7133): Ante un paciente que bruscamente ha comenza-
o Sensacin de atragantarse do con un coniunto de sntomas constituido por: sensacin de
0 OPRESIN o malestar torcico dificultad respiratoria, de ahogo (disnea) o de paro respiratorio,
o NUSEAS o molestias abdominales mareo, sensacin de inestabilidad o desfallecimiento, palpita-
o Inestabilidad, MAREO o DESMAYO cones o taquicardia y nuseas o malestar abdominal. Cul es
o Desrealizacin (sensacin de irrealidad) o el diagnstico ms probable?:
DESPERSONALIZACIN (estar separado de uno mismo) Trastornos de ansiedad generalizada.
o Miedo a perder el control o volverse loco Trastorno de pnico.
o MIEDO A MORIR Trastorno mixto ansioso-depresivo.
o PARESTESIAS Trastorno fbico.
o Escalofros o sofocaciones PPPNT Trastorno de adaptacin.
Por orden de frecuencia:
o Palpitaciones, taquicardia: 80% MIR IO (9440): Hombre 31 aos sin antecedentes psiquitricos.
o Disnea, sensacin de ahogo: 70% Acude a Urgencias refiriendo un episodio agudo de taquicardia,
o Miedo a morir: 65% sudoracin, opresin torcica, sensacin de asfixia y mareo con
o Mareo, sensacin de inestabilidad: 55% inestabilidad. Asustado cree que puede estar sufriendo un
infarto. Hace 2 das experiment otro cuadro clnico igual que se
autolimit en unos 30-40 minutos. Tambin explica que hace l
semana ha roto su relacin de pareia tras 8 aos de
convivencia. Descartada cualquier patologa orgnica qu
diagnstico psiquitrico es el ms adecuado?
Reaccin de estrs post-traumtico.
Trastorno agudo de personalidad dependiente.
Agorafobia.
Trastorno Depresivo.
PFSNT Crisis de Angustia.*

dmlmwwlmm .

Crisis de angustia
CURSO INTENSIVO MIR ASTURIAS

MIR O (8436): Con respecto al trastorno de pnico es cierto


4. Trastorno de pcInico que:
I. Las benzodiacepinas son la base del tratamiento.
trastorno de an-ustia) 2. El trmino crisis de angustia es sinnimo de trastorno de
panIco.
o Presencia de crisis de angustia recidivantes e inesperadas 3. Entre las principales enfermedades con las que habra que
(al menos 2), no asociadas a situaciones o circunstancias realizar un diaqnstico diferencial estara el hipertiroidis-
particulares, seguidas de la aparicin, durante un periodo @*
mnimo de l mes de preocupaciones persistentes por la po- 4. Una de sus variedades se caracteriza por presentarse en
sibilidad de padecer nuevas crisis de angustia (ansiedad an- personas mayores de 45 aos y acompaarse de prdida
ticipatoria) o bien de cambios en el comportamiento, rela- de conocimiento.
cionados con las crisis. 5. Su presencia es un criterio diagnstico de un episodio de
o Las crisis de angustia no se deben a ninguna sustancia ni a depresin mayor.
otra enfermedad mdica o mental.
Es bastante frecuente que se produzca la siguiente evolucin:
crisis de angustia, ansiedad anticipatoria, conductas evitativas, 5. Trastorno de ansiedad
agorafobia.
o Dependiendo si se cumplen o no criterios de agorafobia el eneralizada
diagnstico ser trastorno de angustia con agorafobia o
trastorno de angustia sin agorafobia. . Ansiedad persistente y preocupacin excesiva, no asociadas
o Cuando aparece agorafobia suele hacerlo en el primer ao,
a ninguna circunstancia concreta (ansiedad libre flotante")
pero no siempre aparece. de larga duracin (superior a meses) y curso fluctuante.
o Es necesario para el diagnstico que por lo menos al prin- o 23 de los siguientes durante al menos meses:
cipio las crisis de angustia sean espontneas. Aunque luego Inquietud o impaciencia.
puedan ser facilitadas por ciertas situaciones. Fatigabilidad fcil.
o Trastorno de panico moderado: 24 en 4 semanas. Dificultad de concentracin.
o Trastorno de pnico grave: 24 por semana durante 4 se- Irritabilidad.
manas. Tensin muscular.
En estos pacientes hay mayor incidencia de cuadros depresivos o Alteraciones del sueo.
mayores y de trastornos relacionados con el uso de sustancias o Puede haber sntomas somticos similares a los de las crisis
(alcohol u otras sustancias) y otros trastornos de ansiedad. de angustia, pero menos 'intensos
o EI curso es fluctuante con coincidentes con
EPIDEMIOLOGA: periodos de estrs. agravamientos
4
o En todas las culturas. o Con el tiempo, el paciente aprende a convivir con los snto-
o Influencia gentica en la aparicin del trastorno de angus- mas (periodo de estado).
tia. o En fase crnica pueden aparecer: tristeza, apata, desinters
o El trastorno de ansiedad que ms frecuentemente demanda (sntomas cercanos a la depresin).
tratamiento. o Estos pacientes, lloran de estrs o impotencia, y se sienten
o Edad de inicio: Final de la adolescencia y Ia mitad de Ia 3 aliviados, a diferencia del depresivo, que llora de desespe-
dcada (adulto oven). ranza y el llanto no le alivia.
o Prevalencia por sexos: o Se produce un malestar clnicamente significativo o deterioro
o Sin agorafobia: Muieres x2. social, laboral o de otras reas de la actividad
o Con agorafobia: Muieres x3. del individuo. importantes
o Alta comorbilidad con trastornos depresivos y trastornos de la
LABORATORIO: alcalosis respiratoria compensada en muchos
casos.
personalidad. 7
o Edad de inicio: adulto ioven.
TRATAMIENTO: o Ms frecuente en sexo femenino.
o FASE AGUDA:
BZD de alta potencia (alprazolam). Las BZD slo son tiles para TRATAMIENTO:
yugular las crisis agudas pero no constituyen el tratamiento de o Psicoterapia cognitivo-conductual.
fondo del trastorno de pnico. o Frmacos:
I TRATAMIENTO DE FONDO: o Antidepresivos ISRS.
o Antidepresivos: ISRS. o Otros: BZD, buspirona, antiepilpticos.
o Benzodiacepinas: pueden aadirse a los ISRS.
MIR 13 (10221): Todas las siguientes son complicaciones habi-
o Terapia cognitivo-conductual.
tuales de trastorno de ansiedad generalizada no tratado
Tras - 12 meses libre de crisis se puede intentar disminuir la
EXCEPTO:
dosis del frmaco Entre el 40- 70% recaen y se requiere enton-
Consumo de alcohol y abuso de drogas.
ces un tratamiento ms prolongado o terapia combinada de
Trastornos psicosomticos.
antidepresivos + BZD.
Trastornos depresivos.
PRONOSTICO: Trastornos psicticos. *
Tras -IO aos tratamiento 30% se han recuperado, 40-50% QPPNT' Conductas suicidas.
meioran, pero continan con los sntomas y el restante 20-30%
continan igual o peor.
. Trastorno
MIR 02 (7396) Cul delas siguientes afirmaciones E es tpica
del Trastorno de Pnico? obsesivo-com . ulsivo
l. Va asociado a agorafobia.*
2. Debe presentar al menos 3 ataques en un periodo de 3 se- DEFINICIN
manas en circunstancias distintas. o Presencia de obsesiones o compulsiones recurrentes que
3. Durante el ataque aparecen disnea, sensacin de ahogo y producen marcado malestar y prdidas de tiempo significati-
parestesias, vas (ms de I hora al da), acusado deterioro de la actividad
4. No se debe a trastorno fsico o a otro trastorno mental. general y malestar clnicamente significativo. s
o Duracin mnima: 2 semanas.
e
5. Es un trastorno crnico, episdico y recurrente. s3
Q
U3
D.
X. TRASTORNOS DE ANSIEDAD, SOMATOMORFOS Y DISOCIATIVOS

o En un reducido nmero de nios el TOC puede relacionarse


6.1 . Obsesiones con una infeccin estreptoccica betahemoltica del grupo A
Pensamientos, impulsos o imgenes recurrentes y persistentes (Pediatric Autoinmune
que se experimentan en algn momento como intrusos o in- NeuropsychatrcDisordersAssocatedwithStreptococcus
apropiadas, parsitas y egodistnicas (intrusos), causando ma- PAN DAS-).
lestar o ansiedad.
Es un pensamiento inslito y parasitario. EI suieto reconoce que
estos pensamientos son producto de su mente pero aieno a su
control: egodistnicos.
Las obsesiones generan ansiedad que se intenta neutralizar con
pensamientos o acciones (compulsiones).

6.2. Compulsiones
Conductas o actos mentales repetitivos que el individuo se ve
obligado a realizar en respuesta a una obsesin para aliviar la
ansiedad.
Las compulsiones no proporcionan placer o gratificacin.
Estos comportamientos o actos mentales no estn siempre co-
nectados de forma directa con aquello que se pretende neutra-
lizar (es decir, en muchas ocasiones, las compulsiones son in-
dependientes de la obsesin que las genera).
A menudo, aunque no siempre, este comportamiento es reco-
nocido por el enfermo como carente de sentido o ecacia, lo
que puede llevarle a sensacin de fracaso y prdida de la auto-
estima.

CURSO Y PRONSTICO:
o Crnico con periodos de empeoramiento en relacin con
situaciones de estrs.
ESTUDIOS COMPLEMENTARIOS:
o TAC, RM: disminucin del tamao del ncleo caudado
bilateral.
o PET: activacin en lbulos frontales, ganglios basales y
cngulo que se corrigen con tratamiento farmacolgico y
conductual.
TRATAMIENTO:
o Frmacos(dosis elevadas)
La CIE10 distingue entre pensamientos ideas o imgenes o Eleccin: ISRS.
(obsesiones), o actos (compulsiones). o Clomipramina
El DSM-IVTR establece esta diferencia en funcin de si el o Psicoterapia:cognitivoconductual.
pensamiento idea o imagen produce ansiedad o malestar o En casos intratables psicociruga: Ieucotoma lmbica
(obsesin) o si reduce estos sntomas (compulsin). o cingulotoma anterior.
a Cuando se retira el tratamiento las recadas son fre-
cuentes.
Contaminacin limpieza
Duda patolgica Comprobacin
6.3. Espectro obsesivo-compulsivo
Impulsos agresivos Contar Incluye impulsin (defecto de control y desinhibicin) y com-
Imagenes sexuales Repetir palabras pulsin como fenmenos opuestos en un continuum. Los si-
en silencio guientes trastornos probablemente tengan un origen o sus-
ceptibilidades similares.
Necesidad intensa de orden / Ordenar
o Impulsividad:
simetra
o Compulsiones sexuales
Religiosas Rezar Piromana
Cleptomana
Es el ms grave de todos los trastorno de ansiedad y practica- Tricotilomana
mente el nico que puede precisar hospitalizacin. Ludopatia
EPIDEMIOLOGA: Personalidad cluster B
Tics
Clases socioeconmicas elevadas.
Comportamientos autolesivos
OOOOOOO
Prevalencia del 2% de la poblacin general
o Atracones de comida
Edad inicio: infancia / adolescencia en varones, ms tardo
(20-29 aos) en muieres. 0 Compulsividad:
La distribucin por sexos es equitativa. o TOC
Hipocondra
Influencia gentica.
Dsmorfofobia
Existe una alta incidencia de TOC en individuos con sndro-
Anorexia Nerviosa
me de Guilles de laTourette.
o'ooo Despersonalizacin
CURSO INTENSIVO MIR ASTURIAS

Ideas delirantes de contaminacin.


6.4. Trastornos obsesivoides Fobia especifica.
o En situaciones de agotamiento o estrs pueden aparecer Obsesiones de contaminacin. *
fenmenos como releer un texto, repetir frases, no poderse Temores hipocondracos.
desprender de una frase, una cancin que surge reiterada- SJPPJNT' Ideas sobrevaloradas de contaminacin.
mente en el pensamiento o repetir determinados movimien-
tos. MIR 08 (8956): Sealar cul de los siguientes sntomas N_O es
o La naturaleza obsesivoide de estos actos se manifiesta por su caracterstico del trastorno obsesivo-compulsivo:
reiteracin. Pensamientos e imagenes recurrentes.
o Estos actos cesan una vez desaparece el conflicto o estrs Meticulosidad.
que los genera. Ausencia de introsgeccin.*
Lentitud.
6.5. Personalidad psicastnica de Janet PPPJNT' Compulsiones.
o Tendencia a tener automatismos psquicos. MIR 09 (9210): Respeto al trastorno obsesivo compulsivo (TOC),
o Dificultades de decisin, eiecucin, dudas, tendencia a la seale la respuesta FALSA:
fatiga, depresin, ansiedad, inadaptacin social, distancia- . La prevalencia de TOC en la poblacin general es 243%.
miento del medio ambiente, conductas poco eficaces e impre- 2. En adultos, la probabilidad de sufrir el TOC es igual en
cisas y escasa capacidad de concentracin. hombres que en mujeres.
- Rasgos propios de enfermos neurticos, especialmente de 3. La alteracin' del sistema serotoninrgico esta implicada en
obsesivo. el TOC. - e
4. La terapia conductista es escasamente eficaz en el TOC.*
MIR 98 FAMILIA (5590): Un paciente de 25 aos consulta por 5. Los frmacos de primera eleccin son los inhibidores selecti-
sintomatologa de ansiedad y profundo malestar porque en su vos de la recaptacin de la serotonina (ISRS) o la clomipra-
trabaio tiene una sensacin de inseguridad constante por la mina.
conviccin de no realizar correctamente las tareas que le compe-
ten. Esto hace que necesite permanentemente revisarlas una y MIR 13 (10224) (225): Muier de 26 aos sin antecedentes psi-
otra vez, Io que le supone prdida de tiempo y de eficacia. Esta quitricos previos, acude a Urgencias trada por sus padres que
sensacin estan dominante en su psiquismo que Ie conduce a la explican que desde hace unos das est hiperactivo, nerviosa,
idea de fracaso y a la prdida de autoestima. EI diagnstico insomne. Explican que revisa obsesivamente la instalacin elc-

inicial corresponde a trastorno: trica de casa en busca de camaras de video y micrfonos. En la
Por idea delirante. entrevista nos dice que le estan vigilando desde la Polica pues
Por ansiedad fbica. ella es una enviada galctico con poderes especiales. Cual de
De personalidad evitativo-fbica. las siguientes opciones NQ debe considerarse como diagnstico
Obsesivo-compulsivo.* diferencial?
P'PFJPT' Psictico paranoide.
Trastorno obsesivo compulsivo. *
MIR 03 (7659): Cules son los actos obsesivos mas frecuentes Esquizofrenia.
en el trastorno obsesivo-compulsivo?: Episodio maniaco.
Comprobaciones y rituales de Iimpieza.* Psicosis por consumo de sustancias txicas.
Recuentos mentales. PPPPT Tumor cerebral.
Evitar pisar las cruces en las baldosas.
Jaculatorias repetitivas de coniura. MIR 'l4 (10385): Cul de las siguientes caractersticas NO es
PPPNT Acumulacin y coleccin de obietos. tpica de los contenidos de las ideas obsesivos?
l. Pensamientos sexuales.
MIR O4 (7914): Cul es el principal tratamiento psicofarma- 2. Fonemas.*
colgicodel trastorno obsesivo-compulsivo? 3. Necesidad de simetra.
i. Antipsicticos. 4. Duda patolgica.
2. Benzodiacepinos. 5. Contaminacin.
3. Antidepresivos inhibidores de la recaptacin de noradrenaa
'
lina.
4. Antidepresivos inhibidores de la recaptacin de serotoni- 7. Reacciones a estrs grave y
Eur
5. Estimulantes.
o Consecuencia directa de estrs agudo grave o situacin
MIR 05 (8182): Sealar cul de los siguientes trastornos E traumtica sostenida. En su ausencia no hay trastorno.
parece relacionado con el espectro de la patologa obsesivo-
compulsiva: 7.1. Reaccin a estrs agudo
El trastorno dismrfico corporal.
La amnesia psicgena o Trastorno transitorio (normalmente remite en das/horas) que
La hipocondra. aparece como respuesta a un estrs fsico o psicolgico excep-
Los tics. cional (catstrofes naturales, accidentes, guerra, atracos, viola-
ShrkFNT' La cleptomana. ciones, prdida de varios seres queridos, incendio de la vivien-
da...)i
MIR 06 (8437): Hombre de 26 aos de edad que refiere que o Relacin temporal clara e inmediata entre agente estresante y
le genera mucha ansiedad tocar obietos que otras personas aparicin de sntomas.
hayan podido tocar antes (como picaportes de puertas) o que le o EI riesgo de aparicin es mayor si existe agotamiento fsico o
rocen en transportes pblicos por miedo a contaminarse. Sabe factores organicos (adultos ivenes, muieres).
que es absurdo pero mantiene una actitud continuada de vigi- o Sintomatologa: embotamiento psquico anestesia emocio-
lancia, realiza conductas de lavado de manos repetidamente y nal, disminucin del campo de conciencia, disminucin de
progresivamente ha ido restringiendo sus salidas para evitar las atencin, incapacidad para asimilar nuevos estmulos y des-
numerosas situaciones que considera de potencial riesgo de orientacin, ira, desesperacin, hiperactividad.
contaminacin. Cul es el fenmeno psicopatolgico subyacen- o Tambin pueden existir signos vegetativos de crisis de pnico
te bsico?: (taquicardia, sudacin y rubor).
o Desaparecen generalmente a los 2-3 das maximo.
o Puede existir amnesia completa o parcial del episodio.
X. TRASTORNOS DE ANSIEDAD, SOMATOMORFOS Y DISOCIATIVOS

7.2. Trastorno de estrs postraumtico o Para poder hacer el diagnstico, las alteraciones se de-
ben de prolongar mas de l mes.
Trastorno producido como respuesta tarda (periodo de latencia o El cuadro clnico provoca malestar social, laboral o de
de semanas meses) tras la exposicin a un estrs ambiental / otras reas importantes de la actividad del individuo.
psicolgico desbordante.
TRATAMIENTO:
Aparece hasta en el 30% de los personas que han experimenta-
o Terapia cognitivo conductual
do un acontecimiento traumtico (como los descritos previamen-
o Antidepresivos ISRS.
te) en su vida.
MIR FAMILIA 00 (6628): Los criterios diagnsticos para el tras-
torno por estrs postraumtico incluyen todos los siguientes
CEPTO:
La re-experimentacin del episodio.
El aumento del grado de alerta.
La evitacin de estmulos.
El estado de nimo deprimido.*
.UPPNT' Restriccin de la vida afectiva.

Cursa lnhmsivo MIR Asmri-s

TRASTORNO POR ESTRS POSTRAUMATICO


o Recuerdos y sueos recurrentes sobre el acontecimiento que provocan malestar
o El individuo acta o tiene la sensacin de que el acontecimiento traumtico esta
ocurriendo (se incluye la sensacin de revivir el acontecimiento, ilusiones, aluci-
naciones o episodios disociativos de flashback, incluso los que aparecen al des-
pertarse o al intoxicarse)
o Malestar psicolgico intenso y respuestas fisiolgicas al exponerse a estmulos
internos o externos que simbolizan o recuerdan un aspecto del acontecimiento
traumtico
o Esfuerzos por evitar pensamientos, sentimientos, conversaciones, actividades,
lugares y personas que recuerden al trauma.
o Incapacidad para recordar algn aspecto importante del trauma (amnesia
psicgena)
l. Reduccin acusada del inters por participar en actividades significativas
2. Sensacin de desapego o enaienacin frente a los dems
3. Restriccin de la vida afectiva (disminucin de la capacidad para sentir
emociones)
o Sensacin de "acortamiento de futuro" (no espera casarse, hacer una carrera...)
o Dificultades para conciliar o mantener el sueo (frecuente insomnio de manteni-
miento)
Sntomas persistentes de aumento de la activa- lrritabilidad, ataques de ra (MIR)
i arousal o -rado de alerta Dificultad de concentrarse
Hipervigilancia
Respuesta exagerada de sobresalto

7.3. Trastorno de adaptacin CRITERIOS DIAGNSTICOS:


Estrs psicosocial identificable, el cual no es de gravedad extra-
o Estado de malestar subietivo acompaado de alteraciones ordinaria ni de tipo catastrfico.
emocionales que interfieren con la actividad social. o La forma y gravedad de los sntomas puede varias de un
momento a otro.
o Aparecen durante el perodo de adaptacin a un cambio bio-
o Sintomatologa: humor depresivo, facilidad para el llanto, des-
grfico significativo o a un acontecimiento vital estresante (due-
lo, separacin, emigracin, condicin de refugiado...) esperanza, ansiedad, preocupacin, incapacidad para afrontar
o Relacin temporal clara con acontecimiento estresante (< 3 problemas.
- Subtipos clnicos:
meses) y raramente dura ms de meses.
o Predisposicin yvulnerabilidad iuega un papel importante (pero o Reaccin depresivo breve: estado depresivo leve-moderado
siempre es necesario que exista el agente estresante). y transitorio de duracin inferior a 'l mes.
o Reaccin depresivo prolongada: estado depresivo leve-
EPlDEMIOLOGA: moderado que se presenta como respuesta a la exposicin
o Adolescentes (aunque puede aparecer a cualquier edad). prolongada a una situacin estresante, pero cuya duracin
0 Muieres (2 muieres : l varn) solteras. no excede los 2 aos.
Factores precipitantes estresantes ms frecuentes en venesz pro- o Reaccin mixta de ansiedad y depresin.
blemas escolares, rechazo de los padres, divorcio y abuso de sus- TRATAMlENTO:
tancias. En adultos: problemas familiares, divorcio, cambio de o Psicoterapia cognitivo-conductual.
domicilio, emigracin y problemas econmicos. o Frmacos: ISRS, ansioliticos.
CURSO INTENSIVO MIR ASTURIAS

MIR 10 (9438): Un paciente de 36 aos, oriundo de otra cultura MlR 14 (10386): Hombre de 42 aos, casado, con tres hios
que vive en Espaa desde hace 4 aos se presenta en una menores de edad. Sin antecedentes psiquitricos. Tras un ERE en
consulta de S.N. de Salud. Refiere presentar desde hace TO el banco en que trabaiaba, es finalmente despedido y lleva 5
meses sintomatologa ansiosa y humor depresivo. Este cuadro meses de paro. No encuentra otra actividad laboral. Acude a
interfiere moderadamente en su actividad cotidiana. No tiene consulta con el siguiente cuadro clnico: desnimo general, in-
antecedentes psiquitricos previos. Dicha situacin se produce a apetencio, nerviosismo, insomnio. preocupaciones recurrentes
raz del fallecimiento, en un accidente automovilstico, de un sobre su futuro y evitacin de actividades socio-familiares. Qu
hermano mayor con quien se encontraba muy unido. Sealar opcin diagnstica de las siguientes es la ms adecuada?
cual de los siguientes diagnsticos es el apropiado: Trastorno obsesivo.
Depresin mayor. Trastorno adaptativo con sntomas ansioso-depresivos.*
Trastorno Bipolar ll. Fobia social.
Trastorno de adaptacin.* Distimia.
Distimia. .UPPNT' Ansiedad con agorafobia.
.UFPN." Ciclotimia.

8. Trastorno de ansiedad secundario


ENFERMEDADES SOMTICAS FACTORES TXICOS
Endocrinas: Frmacos:
- Hipertiroidismo(causa ms frecuente ansiedad 2) - Hormonas tiroideas
- Tirotoxicoss - Efedrina
- Hiperparatiroidismo - Levodopa
- Feocromocitoma, Sndrome carcinoide Anticolinrgicos
- Hipoglucemia - Corticosteroides
- Diabetes - Broncodilatadores
- Insulinoma - Psicoestimulantes (anfetaminas...)
- Enfermedad de Cushing - Retirada de BZD y otros sedantes (barbitricos)
Metablicas: Drogas y otras sustancias qumicas:
- Porfirio aguda intermitente - Nicotina
Cardiovasculares: Cafena
- Angina, Infarto de miocardio - Anfetaminas
- Taquicarda supraventricular paroxstica, Arritmias - Cocana
Respiratorias: - Abstinencia a depresores del SNC (alcohol, opiceos...)
- Embolismo Pulmonar, Asma, EPOC
Neurolgicas:
- Accidente cerebrovascular, Demencia

'
9. Trastornos somatomorfos y TRASTORNOS SOMATOMORFOS
Sntomas fsicos que sugieren una enfermedad mdica. No
trastornos disociativos existen hallazgos orgnicos demostrables. Es presumible una
ligazn a conflictos o factores psicolgicos: no intencionada.
9.1 . Introduccin
TRASTORNOS DISOCIATIVOS
o Trastorno mental caracterizado por una gran variedad de
sntomas somticos y sntomas mentales. Alteracin de las funciones integradoras de:
Ms frecuente en muieres: x3. o La conciencia.
Se distinguen: o La identidad.
o Trastornos somatomorfos. o La memoria.
o Trastornos disociativos. o La percepcin del entorno.

9.2. Trastornos que cursan con sntomas somticos

Sntomas somticos

Con hallazgos somticos Sin hallazgos somticos


x l

Producidos intencionalmente No producidos intencionalmente


l

Con incentivos externos Sin incentivos externos

Enfermedad Somtica Simulacin Trastorno facticio Trastorno somatomorfo


- Tr. psicosomticos

S
E
S
3
Q
U)
o.
X. TRASTORNOS DE ANSIEDAD, SOMATOMORFOS Y DISOCIATIVOS

TRASTORNO FACTICIO: o Rechazo continuado del suieto a aceptar las aclaracio-


Sndrome de Mnchausen: nes de los mdicos de que no existe causa orgnica
o Fingimiento o produccin intencionada de signos o que explique los sntomas somticos.
sntomas fsicos o psicolgicos.
TRATAMIENTO:_N0 existe tratamiento especfico (evitar yatroge-
o El suieto busca asumir el papel de enfermo.
nia mdica y exploraciones innecesarias). Psicoterapia.
o Ausencia de incentivos externos para el comportamien-
to (por ei. ganancia econmica, evitar la responsabili-
TRASTORNO POR DOLOR
dad legal o meiorar el bienestar fsico, como ocurre en
el caso de la simulacin). EPIDEMIOLOGA:
o Existe un Munchausen por poderes", en el que el en- o Se suele iniciar entre la cuarta y la quinta dcada.
fermo emplea a un nio como paciente sustituto. o Ms frecuente en mueres (relacin 2:1).
TRASTORNOS PSlCOSOMTlCOS: DIAGNSTICO:
o La enfermedad no habra llegado a producirse si no o Preocupacin por dolor en uno o ms lugares sin hallazgos
existieran factores psicosociales, y estos influyen en fa- fsicos que los expliquen.
ses de empeoramiento. o El dolor es el sntoma principal del cuadro clinico, y tiene la
o Algunas de las enfermedades influenciados por factores gravedad suficiente para merecer atencin mdica.
psicolgicos son: o En estos suietos el dolor produce incapacidad para ir a tra-
O Enfermedad coronaria. baiar o ir a la escuela; utilizacin asidua del sistema sanita-
O Hiperventlacin y ataques asmticos. rio, consumo continuado de frmacos, problemas de tipo re-
O Propensin a infecciones por disminucin de lacional (conflictos matrimoniales o alteracin de la vida fa-
las defensas. miliar) y trasformacin del dolor en el asunto central de Ia vi-
O Colon irritable, Crohn. da.
O Psoriasis, hiperhidrosis, urticaria. o Se requiere que el dolor persista por lo menos meses y no
pueda explicarse sobre la base de una alteracin fisiolgica
9.3. Trastornos somatomorfos o trastorno fsico. Se recoge el trastorno con el nombre de
Sntomas somtcos sin hallazgos mdicos. dolor somatomorfo persistente.
No hay intencionalidad. o Tipos:
o A menudo se relacionan con trastornos depresivos, trastornos o Agudo si < meses.
de ansiedad, trastornos de la personalidad y/o el abuso de o Crnico si > meses.
sustancias. TRATAMlENTO: Antidepresivos tricclicos o lSRS en dosis meno-
o TIPOS:
res que el tratamiento antidepresivo.
. TRASTORNO DE SOMATIZACIN.
TRASTORNO POR DOLOR. MIR 02 (7395): Muier de 21 aos, casada y con un hiio de T4
DISMORFOFOBIA. meses de edad. A raz de un accidente laboral sufrido hace un
HIPOCONDRA. ao (una mquina con la que trabaia le produio un arranca-
TRASTORNO DE CONVERSIN. miento de cabello y cuero cabelludo de 5 cm de dimetro en la
regin temporal izquierda), comenz a sufrir un dolor muy in-
TRASTORNO DE SOMATlZAClN
tenso localizado en la zona afectada y que persista varios meses
Llamado sndrome de Briquet. despus de la curacin y restitucin ad integrumde la herida.
EPIDEMIOLOGA: La intensidad con la que vivencia el dolor le impide realizar su
trabaio habitual e incluso ocuparse del cuidado de su ho. Cul
o Se inicia antes de los 30 aos, persiste durante varios aos
es el diagnstico ms probable segn la CIE-10?:
(al menos 2).
o Ms frecuente en muieres (relacin 5:1). o Trastorno disociativo (de conversin) (F44).
Los pacientes presentan sus sntomas de una forma dramti- Trastorno de somatizacin (F 45.0l
ca y exagerada. Reaccin a estrs grave sin especificacin (F 43.9).
o Suele desencadenarse en personas con dificultad para expre Reaccin a estrs agudo (F 43.0).
sar verbalmente sus emociones. OOO. Simulacin (Z 76.5).
o A medida que la relacin con el mdico progresa aparece
dependencia marcada e insaciable. TRASTORNO DISMRFICO (DlSMORFOFOBIA)
o Curso crnico, fluctuante. o Preocupacin excesiva (pero no delirante) por alguna carac-
DIAGNSTICO: terstica (por ei. olor corporal) o defecto imaginario en el as-
Deben cumplirse todos, aunque no simultneamente: pecto fsico.
o 4 sntomas dolorosos: dolor de 4 zonas diferentes:
No existe dficit cognitivo.
o cabeza Generalmente centrado en los rasgos faciales de un indivi-
duo aparentemente normal.
o abdomen
O espalda o Capaz de provocar marcado malestar o deterioro social o
o articulaciones... laboral.
o menstruacin, coito, miccin. . . La preocupacin es claramente excesiva]
o 2 sntomas gastrointestinales: distintos al dolor: No se debe a otro trastorno mental (ei; anorexia).
O nusecs 0 Es habitual que se asocie con:
o distensin abdominal.,. o Ideas de referencia, los dems perciben y comentan la
o T sntoma sexual: distinto al dolor: deformidad del suieto.
o indiferencia sexual o Comprobaciones delante del espeio.
o disfuncin erctil o Intentos de ocultar la deformidad con maquillaie, ropa.
o menstruacin irregular, prdidas menstruales o Evitacin de actividades sociales.
excesivas o Intentos de suicidio.
O vmitos durante el embarazo... o Se suele iniciar entre los 15 y 30 aos.
o l sntoma pseudo-neurolgico: distinto al dolor: o Ligeramente ms frecuente en muieres(relacin cercana a
o parlisis l :l).
o afona Curso continuo con pocos intervalos sin sntomas.
o ceguera La parte del cuerpo en la que se centra la preocupacin
o alucinaciones, amnesia... puede ser siempre la misma o variar.

en...
MB
CURSO INTENSIVO MIRASTURIAS

o En el 90% de los casos se observan episodios depresivos MIR 97 (5328): Seale cul de las caractersticas siguientes N_O
mayores a Io largo de Ia vida, el 70% trastornos ansiosos y el es propia del trastorno hipocondraco:
30% trastornos psicticos. l. Preocupacin persistente por tener una enfermedad grave
o EI trastorno dismrfico corporal tambin se puede asociar a| no detectada por el mdico.
TOC. 2. Abuso de medicamentos a pesar del incumplimiento de las
o La CIE-IO lo incluye en los trastornos hipocondracos. {NescriicLTclcasf
3. Solicitud de continuas exploraciones mdicas para confir-
TRATAMIENTO: mar o descartar la enfermedad que subyace a los sntomas
o ISRS a dosis altas y periodos prolongados. y molestias fsicas.
o Psicoterapia. 4. Presentacin tanto en varones como en mujeres sin acom-
paarse 'de caractersticas familiares especiales.
5. Preocupacin por los efectos adversos de los medicamentos
prescritos, hasta el punto de consultar a un segundo profe-
sional.

MIR 04 (7291): Cual de las siguientes afirmaciones sobre los


trastornos somatomorfoiQ es correcta?:
l. EI trastorno de somatizacin suele desencandenarse en per-
sonas con dificultad para expresar verbalmente sus emocio-
nes.
2 El trastorno por dolor es una enfermedad que puede llegar a
ser incapacitante.
3. La hipocondra es el miedo a contraer o padecer una enferme-
dad. Si esta idea cumple criterios de idea delirante no se diagnos
ticara de hiponcondra sino de trastorno delirante.
4. Al iaual que el trastorno facticio, los trastornos somatofor
mos se caracterizan por una produccin deliberada de uno
o un coniunto de sintomas.*
5. Los trastornos somatomorfos se relacionan con frecuencia
con los trastornos de personalidad y abuso de substancias.

MIR 09 (9207): Seale cul de los siguientes trastornos HQ es


MIR 00 FAMILIA (6626): Una muier de 25 aos es enviada al reconocido en el DSM-IVTR como trastorno somatomorfo:
psiquiatra por un cruiano plastico debido a que afirma constan- Trastorno de conversin.
temente que tiene hinchada una parte de Ia cara. El cruiano no Trastorno de despersonalizacin.*
ha podido comprobar en ningn momento esta afirmacin. El Hipocondra.
diagnstico ms probable es de. Trastorno dismrfico corporal.
Simulacin. .UPWNT' Trastorno por dolor.
Bulimia nerviosa.
Trastorno de conversin. TRASTORNO DE CONVERSIN
Trastorno de somatizacin.
PPPNT Trastorno dismrfico corporal.* o Neurosis histrica de conversin.
. Caracterzado por _>.'| sntoma pseudos-neurolgico (activi-
MIR 09 (921 I): Sealar en cual de los siguientes trastornos DL) dad motora o sensorial): afasia, ceguera, sordera, parlisis,
se observa dficit cognitivo: contracturas, diplopia, anestesias, debilidad, afona, equili-
Esquizofrenia. brio)'.
Trastorno bipolar. - El inicio o exacerbacn se precede de conflictos psicolgicos
Pseudodemencia. o acontecimientos estresantes.
Depresin unipolar. o La remisin suele ocurrir en menos de 2 semanas.
mewwe. Trastorno dismrfico corporal.* o Actitud dramtica. Sugestionables.
o Belle indiference: sin aparente repercusin.
HIPOCONDRIA o No voluntario pero ganancia secundaria.
o Edad de inicio mas entre los IO y los 35 aos.
o Preocupacin y miedo de padecer una enfermedad grave a
o Ms frecuente en muieres.
partir de la mala interpretacin de signos y sntomas fisiol-
gicos. o Niveles socioeconmicos baos y poblacin rural.
o Actitud patolgica hacia el propio funcionamiento corporal. La CIE-10 considera la conversin como un trastorno disociativo
o Negatividad de las pruebas complementarias. y distingue segn los sntomas entre: estupor disociativo, tras
o Duracin de al menos 6 meses. torno disociativo de Ia motilidad, convulsiones disociativas,
o NO delirante. anestesia y prdidas sensoriales disociativas.
o Genera malestar continuado y/o invalidez social.
TRATAMIENTO:
o EI paciente solicita continuas exploraciones mdicas, para
o No existe tratamiento especfico.
detectar la posible enfermedad que subyace a sus sntomas:
o Psicoterapia.
Doctor-shopping".
o BZD de forma aguda.
o No son partidarios de medicaciones, ya que se preocupan
mucho de los posibles efectos secundarios.
o Inicio entre los 20 y 30 aos (puede aparecer a cualquier
edad). Igual de frecuente en hombres y muieres.
Curso crnico con altibajos.
Se asocia con trastornos de ansiedad, depresivos y otros
trastornos somatomorfos.

TRATAMIENTO:
o Antidepresivos ISRS.
o Terapias de modificacin cognitiva.
X. TRASTORNOS DE ANSIEDAD, SOMATOMORFOS Y DISOCIATIVOS

MIR OI (7136): Se presenta a Ia consulta una muer de 36 aos


que cuenta, con cierta indiferencia que ha perdido visin de
forma brusca en los dos oios. Durante el interrogatorio se cons-
tata que esto ocurri hace 4 horas, cuando una amiga con Ia
que se iba a ir de vaie de placer le diio que porfin no poda ir.
La exploracin por lo dems resulta anodina y en la anamnesis
se recoge un episodio de anestesia en un brazo hace I ao.
Qu patologa, de entre las siguientes, es mcis probable que
presenta la paciente?:
Trastorno de conversin.*
Trastorno facticio.
Trastorno de somatizacin.
Simulacin.
.UPPNT' Trastorno por estrs agudo.

MIR II (9688) Muier de 52 aos que acude a urgencias con una


parlisis bilateral de las extremidades superiores que se haba
instaurado de manera brusca dos horas antes. En la entrevista
se muestra risuea y verbaliza no entender por qu sus familia-
res estn tan preocupados. La familia refiere que el da anterior
su hiio haba sido detenido por la polica por trafico de drogas y
todava no haba podido contactar con l. Cul de los siguien-
tes es el diagnstico ms probable?:
Trastorno facticio.
Trastorno delirante. o Se acompaa de confusin sobre la identidad personal y en
Hipocondrass. ocasiones nueva identidad.
Trastorno depresivo mayor. o Inicio sbito, tras acontecimientos traumticos o estresantes
.UPWN." Trastorno conversivo. (discusin).
Dura horas, das o incluso meses.
MIR 13 (i 0222): Cul de los siguientes factores NQ se asocia a Habitualmente son episodios nicos.
buen pronstico del trastorno de conversin? Tras la recuperacin no se recuerda lo sucedido en la fuga.
Inicio agudo. o Es poco frecuente. Se da mas en guerras y desastres natura-
2. Presencia de factores estresantes claramente identificables al les.
inicio de Ia enfermedad.
3. Cociente intelectual alto. AMNESIA DISOCIATIVA
4. Ausencia de otros trastornos psiquitricos y procesos
legales. o Amnesia psicgena.
5. Presencia de convulsiones y temblores. * o Es el trastorno disociativo mas frecuente.
o Incapacidad para recordar informaciones personales impor-
tantes, demasiado extensa para explicarse por olvido ordina-
9.4. Trastornos disociativos rio.
o No es debida a los efectos de una sustancia ni a una enfer-
o Prdida del sentido unitario de Ia conciencia.
medad mdica.
o Sntomas psquicos no congruentes con las enfermedades
habituales.
o Recuperacin suele ser rpida, brusca y completa.
o No producidos intencionalmente.
. Aparicin sbita y duracin breve (das, semanas).
o Carecen de causa orgnica. Se cree que son de origen
psicgeno por su relacin temporal con acontecimientos
biogrficos estresantes.
o Tratamiento: disminuir las ganancias del estado patolgico y
psicoterapia. Los frmacos son poco tiles (si acaso BZD vida
media larga).
o TIPOS:
o FUGA DISOCIATIVA.
0 AMNESIA DISOCIATIVA.
o IDENTIDAD DISOCIATIVA.
. DESPERSONALIZACIN.
FUGA DISOCIATIVA
o Prdida de la identidad personal: fuga psicgena.
o Viaie repentino e inesperado Ieios del hogar, con incapaci-
dad para recordar alguna parte o la totalidad del pasado del
individuo.
(msn inmmlm MIR mmm;

Se han descrito varios tipos:


o Amnesia localizada:
El individuo no puede recordar acontecimientos que se
han producido durante un periodo de tiempo circunscrito,
por lo general en las primeras horas que siguen a un
acontecimiento profundamente turbador.
o Amnesia selectiva:
El individuo puede recordar algunos (aunque no todos)
los acontecimientos que se han presentado durante un
periodo de tiempo circunscrito.
gw.
MIR
CURSO INTENSIVO MIR ASTURIAS

o Amnesia generalizada: TRASTORNO POR DESPERSONALIZACIN


La imposibilidad de recordar abarca toda la vida del in-
dividuo. Sensacin de extraeza o distanciamiento de uno mismo, el
. Amnesia continua: paciente se siente como un autmata.
lmposibilidad para recordar acontecimientos que han te- Sensacin de ser un observador externo de si mismo, con
nido lugar desde un acontecimiento determinado hasta la sensacin de prdida de control de sus actos incluyendo el
actualidad. habla.
o Amnesia sistematizada: Juicio de realidad preservado.
Es una prdida de memoria para ciertos tipos de nfor- No existe amnesia.
macin, como los recuerdos relacionados con la propia Para constituir un trastorno por despersonalizacin, ha de
familia o con alguien en particular. producirse de forma repetida.
Pueden aparecer en otros trastornos mentales: crisis de an-
TRASTORNO DE lDENTlDAD DlSOClATlVO gustia, esquizofrenia, uso de sustancias, epilepsia temporal,
traumatismo craneal...
o La llamada personalidad mltiple. Suele acompaarse de vivencias de desrealizacin, que
o Muy raro. consiste en: alteracin de la percepcin del ambiente que
o Caracterizado por la presencia de dos o ms identidades o rodea al individuo, de forma que se pierde el sentido de la
estados de personalidad en la misma persona de forma que realidad.
cada una controla de forma recurrente y sbita la conducta
del suieto un cierto tiempo.
o Amnesia disociativa: incapacidad para recordar informacin
personal importante.
o Cada personalidad se vive como una historia personal, una
identidad e incluso un nombre distintos.
Ms frecuente en muieres.
o Una personalidad no tiene conciencia de la otra. Salto re-
pentino de una a otra.
o Con frecuencia se desencadena por un acontecimiento estre-
sante. Curso fluctuante que tiende a ser crnico.
o El ms grave de los trastornos disociatvos.

MIR 91 (3063): Un ioven de 21 aos experimenta episodios en


los que se ve como observador de su propio cuerpo, un aut-
mata que hubiera perdido el control sobre las propias funcio-
nes, unto a una intensa ansiedad. El cuadro se repite sin otros
sntomas, por lo que se debe diagnosticar de:
Trastorno de despersonalizacin.*
Esquizofrenia de tipo hebefrnico.
Crisis de panico histeritormes.
Dismortia.
WPF-N. Trastorno esquizoatectivo.
X. TRASTORNOS DE ANSIEDAD, SOMATOMORFOS Y DISOCIATIVOS

Ansiedad: estado de activacin del SNC por el que el organismo se prepara para hacer frente a una situacin extraordina-
ria.
Ansiedad normal: respuesta a estmulos externos potencialmente peligrosos. Adaptada en intensidad y tiempo y genera una
respuesta adecuada.
Los trastornos de ansiedad son muy frecuentes (prevalencia-vida en poblacin general superior al 10%). Ms frecuentes en el
sexo femenino. Las posibilidades de aparicin disminuyen con la edad (MIR).
Comorbilidad frecuente con trastornos del estado de nimo, otros trastornos de ansiedad y consumo de sustancias.
Con frecuencia curso crnico con periodos de meiora y otros de empeoramiento.
Sntomas de ansiedad:
o Fsicos: hiperactivacin simptica (taquicardia, palpitaciones, disnea, sensacin de ahogo, mareos, inestabilidad, sudor,
enroiecimiento, escalofros, calor, nauseas, diarreas, parestesias...)
o Psquicos: activacin locus coeruleus / corteza cerebral (nerviosismo, inquietud, miedo a perder el control, aprensin, an-
siedad flotante", despersonalizacin, desrealizacin...).

I. TRASTORNOS DE ANSIEDAD FOBICA


Ansiedad exagerada frente a situaciones / obietos bien definidos y externos, generalmente no peligrosos.
Conductas de evitacin hacia el obieto / situacin que produce ansiedad (MIR).
Ansiedad anticipatoria.
Ms frecuentes en sexo femenino.
Agorafobia: DSM-IV-TR: ansiedad al encontrarse en lugares o situaciones de las que puede ser difcil escapar, o en las que
quiza no se disponga de ayuda en el caso de sufrir un ataque de panico. Las actividades socio-laborales estan severamente
limitadas CIE-IO: Miedo manifiesto o conducta de evitacin ante por lo menos dos de las siguientes situaciones: multitudes,
lugares pblicos, viaiar solo, viaiar leios de casa. Tiene que haber al menos dos de los siguientes sntomas al enfrentarse a Ia
situacin temida: palpitaciones, sudoracin, temblor, sequedad de boca, dificultad para respirar, sensacin de ahogo, dolor o
malestar e el pecho, nuseas, mareo, inestabilidad o desvanecimiento, sensacin de perder el control, miedo a morir, sofocos
o escalofros y sensacin de entumecimiento u hormigueo.
'I. Es el ms incapacitante de todos los trastornos fbicos.
2. Frecuentemente se asocia con trastorno de pnico.
3. Inicio en adulto joven.
4. Mas frecuente en muieres.
Tratamiento: Terapia cognitivo-conductual + ISRS (+/- BZD), un tiempo no inferior a 6-12 meses.
Fobia social: Temor acusado y persistente a una o mas situaciones sociales o actuaciones en pblico (MIR).
EI miedo a ser eniuiciado por otras personas puede surgir dentro de un grupo relativamente pequeo (en agorafobia temor a
situaciones donde hay multitudes). Tmido de modo patolgico (la fobia social genera discapacidad, la timidez no). La expo
sicin a estas situaciones sociales puede provocar una crisis de pnico. Pueden ser restringidas (algunas actividades sociales)
o difusas (todas las actividades sociales). Las actividades tpicamente temidas son: comer, beber, hablar, escribir en pblico.
o Mayor grado de incapacidad en la fobia social difusa.
o Inicio en adolescencia.
o Hombres z Muieres.
Tratamiento: Terapia cognitivo-conductual + ISRS
Fobia especifica lsimglel: Miedo marcado ante un obieto o situacin especfica (MIR).
o Grado de incapacidad variable (depende del obieto / situacin temida).
o Los trastornos de ansiedad ms frecuentes (pero generan una menor demanda).
o Inicio infancia / adolescencia.
o Mas frecuente en muieres.
Tratamiento: Terapia cognitivoconductual (BZD, Betabloqueantes)

2. ATAQUES DE PANICO (CRISIS DE ANGUSTIA)


Episodios sbitos de descarga neurovegetativa de 15-30 min. de duracin, con intenso agobio y sensacin de muerte inminente. Para
hacer el diagnstico se necesitan al menos 4 sntomas entre los siguientes: (MIR)
Palpitaciones, sudoracin, temblores, boca seca, sensacin de ahogo, sensacin de atragantarse, opresin o malestar torcico, nuse-
as, inestabilidad, despersonalizacin o desrealizacin, miedo a perder el control, miedo a morir, parestesias, escalofros o sofocacio-
nes.
o Tipos de atagues de pnico:
o Inesperados: surgen sin motivo desencadenante. Caractersticas del trastorno de pnico.
o Situacionales: aparecen por anticipacin o tras exposicin a estmulos o desencadenantes ambientales. Caractersticas de
fobias especficas y fobia social, TOC, trastorno de estrs postraumtico...
o Ms o menos relacionados: tienen ms probabilidades de aparecer si la persona se expone a ciertos desencadenantes,
aunque no siempre existe asociacin con el estmulo, ni siempre aparece tras exponerse al mismo
Tratamiento: En la fase aguda: alprazolam. Como tratamiento de fondo: ISRS (+/ alprazolam).

3. TRASTORNO DE PANICO (TRASTORNO DE ANGUSTIA) O ANSIEDAD PAROXlSTICA EPISODICA


Crisis imprevisibles de ansiedad grave pnico- (para diagnstico de trastorno de pnico, deben de producirse varios ataques
durante el perodo de un mes; si frecuencia menor: ataques de panico (MIR) no asociadas a situaciones o circunstancias parti-
culares.
Es necesario para el diagnstico que al menos al principio sean espontneas (MIR), aunque luego puedan ser facilitadas por
ciertas situaciones.
Es muy frecuente la aparicin de preocupaciones persistentes por la posibilidad de padecer nuevas crisis de panico (ansiedad
anticipatoria) o bien de cambios en el comportamiento relacionados con las crisis (MIR).

M
CURSO INTENSIVO MIR ASTURIAS
u

o Duracin limitada (generalmente 15-30 minutos).


o Pueden aparecer conductas de evitacin (= agorafobia). Cuando aparece agorafobia, suele hacerlo el primer ao, pero no
siempre aparece (MIR).
Existe influencia gentica en la aparicin del trastorno de angustia.
Es el trastorno de ansiedad que ms frecuentemente demanda tratamiento.
lnicio en adulto oven (MIR).
Ms frecuente en muieres.
Tratamiento: En la fase aguda: alprazolam (slo yugula las crisis). Como tratamiento de fondo: ISRS (+/- alprazolam) (MIR).
(I

4. TRASTORNO DE ANSlEDAD GENERALIZADA


o Ansiedad persistente y preocupacin excesiva, no asociadas a ninguna circunstancia concreta (ansiedad libre flotante) de
larga duracin y curso fluctuante.
o Para hacer el diagnstico se precisan 3 de los siguientes criterios durante al menos meses: inquietud o impaciencia, fatiga-
bilidad fcil, dificultad de concentracin, irritabilidad, tensin muscular y alteraciones del sueo (dificultad para conciliar o
mantener el sueo o sueo no reparador). Puede haber sntomas somticos similares a las crisis de angustia pero menos in-
tensos.
o EI individuo se encuentra limitado en los planos: social, laboral... En la fase crnica puede aparecer: tristeza, apata, desin-
ters.... (sntomas cercanos a la depresin).
o Inicio en adulto joven.
0 Ms frecuente en muieres.
o Tratamiento: Psicoterapia cognitivo-conductual + Frmacos (ISRS, BZD, buspirona, nuevos antiepilpticos).

5. TRASTORNO OBSESIVO-COMPULSIVO (TOC)


o Obsesiones: Pensamientos, impulsos o imgenes recurrentes y persistentes que se experimentan en algn momento como in-
trusas o inapropiadas y causan malestar o ansiedad (MIR, MIR). EI suieto reconoce esos pensamientos como propios (en los tras-
tornos esquizofrnicos seran: aienos, impuestos) e irracionales / absurdos (MIR, MIR). Es un pensamiento aieno al control del
sujeto (MIR, MIR).
o Compulsiones: Conductas o actos mentales repetitivos que el individuo se ve obligado a realizar en respuesta a una obsesin
para aliviar la ansiedad (MIR). En muchas ocasiones las compulsiones son independientes de la obsesin que las genera (MIR).
o EI CIE-lO denomina obsesiones a todos los actos mentales y compulsiones a las conductas. Los actos compulsivos ms fre-
cuentes son los rituales de limpieza y las comprobaciones (MIR). Espordicamente presentan episodios angustiosos de gran in-
tensidad (MIR).
o Para poder diagnosticar TOC debe haber obsesiones, compulsiones o ambas durante al menos 2 semanas. Este binomio ob-
sesin-compulsin, causa prdidas de tiempo significativas (ms de i hora al da) lleva al suieto a la sensacin de fracaso y
prdida de Ia autoestima (MIR).
Es el ms grave de todos los trastornos de ansiedad.
Inicio en adolescencia.
Hombres z Muieres.
Tratamiento: Frmacos en dosis elevadas (1 eleccin: ISRS (MIR), 2 eleccin: Clorimipramina + Psicoterapia cognitivo-
conductual. Psicociruga si fracasan todas las pautas teraputicas.
o Espectro obsesivo-compulsivo: incluye impulsin y compulsn como fenmenos opuestos en un continuum(MIR)
o Impulsividad: compulsiones sexuales, piromana, cleptomana, tricotilomana, ludopata, personalidad cluster B, tics,
comportamientos autolesivos, atracones de comida
o Compulsividad: TOC, hipocondra, dismorfofobia, AN, despersonalizacin
o Personalidad psicastnica: Caracterizada por Ia tendencia a los automatismos psquicos. Estos pacientes presentan: Dificultad
de decisin, de eiecucn, ucios inestables, dudas... Son rasgos propios de enfermos neurticos, especialmente obsesivos
(MIR).

. REACCIONES A ESTRS GRAVE Y TRASTORNOS DE ADAPTACIN


o Consecuencia directa de estrs agudo grave o situacin traumtica sostenida. En su ausencia no hay trastorno.
o La vulnerabilidad y predisposicin personal uega un papel importante.
o inicio: cualquier edad.
0 Reaccin a estrs agudo: Trastorno transitorio respuesta a un estrs fisico / psicolgico excepcional catstrofes naturales, ac-
cidentes, guerra, atracos, violaciones...) (en las fobias el desencadenante no es en principio peligroso).
Relacin temporal clara e inmediata entre agente estresante y aparicin de sntomas;
Mayor riesgo de aparicin en adultos ivenes y muieres.
Embotamiento psquico o anestesia emocional.
Desaparicin en 2/3 das.
Puede existir amnesia total / parcial del suceso.
OOOOO
o Trastorno de estrs gostraumtico: Trastorno producido como respuesta tarda (perodo de latencia de se-manas / meses) tras
la exposicin a un estrs ambiental desbordante. Se caracteriza por:
o Re-experimentacn del acontecimientoIMIR)
o Evitacin de estmulos asociados al trauma y embotamiento de la actividad general del individuo: amnesia psicgena de
algn aspecto del trauma (MIR), sensacin de acortamiento de futuro (MIR), embotamiento psquico o anestesia emocional
(MIR), restriccin de la vida afectiva (MIR).
o Sntomas persistentes de aumento de la activacin (arousal) o arado de alerta: ataques de ira (MIR), dificultad de concen-
tracin, dificultad para conciliar el sueo o insomnio de mantenimiento (MIR).
o Tratamiento: Psicoterapia cognitivo-conductual + ISRS.
o Trastorno de adaptacin: Malestar subietivo + alteraciones emocionales en periodo de adaptacin a cambio biogrfico o
acontecimiento vital estresante (duelo, separacin, emigracin...).
o Relacin temporal clara con desencadenante y duracin del trastorno menor de meses. S
.24
o Humor depresivo, ansiedad, incapacidad para afrontar problemas, afrontar rutina, para planificar futuro. S
. . , . :)
o lnlClO mas frecuente en la adolescenCIa. g
o Ms frecuente en muieres. a.


X. TRASTORNOS DE ANSIEDAD, SOMATOMORFOS Y DISOCIATiVOS

o Tratamiento: Psicoterapia cognitivo-conductual +/ Frmacos (ISRS, BZD)

7. TRASTORNO DE ANSIEDAD SECUNDARlO


Causa ms frecuente de ansiedad secundaria: hipertiroidismo (MIR).
Todas las drogas pueden producir ansiedad:
T. Los psicoestimulantes mientras se consumen
2. Las depresoras en Ia abstinencia

8. TRASTORNOS SOMATOMORFOS
Aparecen sntomas somticos sin hallazgos mdicos que los iustifiquen. No hay intencionalidad (MIR). A menudo se relacionan con los
trastornos de la personalidad y / o de abuso de sustancias (MIR).
Trastorno por somatizacin (Sindrome de Briquet): Se inicia antes de los 30 aos, se caracteriza por una combinacin de
sntomas gastrointestinales, sexuales, pseudoneurolgicos y dolor. Sigue un curso crnico y fluctuante. Suele desencadenarse
en personas con dificultad para expresar verbalmente sus emociones (MIR). Ms en mueres. La CIE-10 exige duracin tempo-
ral de al menos 2 aos.
Trastorno por dolor: Preocupacin por dolor en uno o ms lugares sin hallazgos fsicos que expliquen su presencia o intensi-
dad (MIR). Puede llegar a ser incapacitante (MIR). Ms frecuente en muieres.
Trastorno dismrfico: Preocupacin claramente excesiva por algn defecto imaginario en el aspecto fisico de un individuo
aparentemente normal. Aproximadamente misma frecuencia entre varones y muieres (MIR). Provoca un marcado malestar o
deterioro social o laboral.
Hipocondra: Temor o creencia de padecer una enfermedad grave a partir de la interpretacin personal de signos y sntomas
fsicos como prueba de enfermedad somtica(MlR). Si la idea de padecer una enfermedad cumple criterios de idea delirante se
debe de hacer el diagnstico de trastorno delirante (MIR). Se considera una actitud patolgica hacia el propio funcionamiento
corporal (MIR). Dura 6 meses por lo menos. Persiste a pesar de las explicaciones mdicas. Estos pacientes solicitan continuas
exploraciones mdicas para descartar distintas afecciones. Son reacios a tomar frmacos por sus posibles efectos secundarios
(MIR). Misma frecuencia entre varones y muieres. Existe disforia asociada.
Trastorno de conversin: Presencia de uno o ms sntomas pseudoneurolgicos (parlisis, ceguera, mutismo, parestesias...)
que aparecen en relacin temporal con un estrs psicosocial (MIR). "Belle indiference".

9. TRASTORNOS DISOCIATIVOS
Alteracin de las funciones integradoras de identidad, memoria, percepcin del entorno o la conciencia.
Fuga disociativa: viaie inesperado y repentino fuero del hogar asumiendo una nueva identidad. Amnesia de lo ocurrido tras
la recuperacin.
Amnesia disociativa: es el trastorno disociativo ms frecuente, consiste en imposibilidad para recordar infor-
macin personal importante tras acontecimientos traumticos o estresantes.
Trastorno de identidad disociativo (personalidad mltiple). Es el ms grave.
Trastorno por desnersonalizacn: Experiencias recurrentes despersonalizacin manteniendo intacto el co-
rrecto eniuiciamiento de la realidad (no amnesia como en el resto de disociativos), habitualmente se acompaa de desrealiza-
cin.
o Despersonalizacin: Los pacientes se ven a si mismos como observadores de su propio cuerpo, con la sensacin de que
han perdido el control sobre sus funciones: movimientos, |enguaie.., Io que genera mucha ansiedad (MlR).
o Desrealizacin: Se muestra cambiado el sentido de la realidad externa, no se reconoce el lugar donde est.
Tratamiento: Disminuir las ganancias del estado patolgico, en fases agudas: tcnicas sugestivas, en fases crnicas: psicote-
rapia. Escasa eficacia de los frmacos (BZD).
CURSO INTENSIVO MIR ASTURIAS

Trastornos de la conducta alimentaria


Nmero de preguntas del captulo en JelMlR

'l l l 1 'l

l l I I

90 91 92 93 94 95f 95 9f 96 97t 97 98f 98 99f 99 OOf OO. 01. 02. O3. O4. 05. O. O7. 08. O9. 10 11 12 13 14

Nmero de preguntas de cada tema

Trastornos cualitativos

F...
Transtornos cuantitativas

Potomania

i
@ Imprescindible
En la anorexia nerviosa el sntoma ms importante es la distorsin de su propia imagen corporal. Las complicaciones que pue-
den aparecer estn principalmente la amenorrea (por hipogonadismo hipogonadotropo por alteracin hipotalmica), descenso
de la actividad del eie tiroideo y aumento de cortisol y GH, y osteopenia. Si hay vmitos asociados pueden presentarse de forma
secundaria alcalosis metablica hipopotasmica (5MIR).
La principal alteracin de la bulimia nerviosa es la prdida del control de la ingesta con atracones de gran cantidad de alimento,
asociado a conductas compensatorias inadecuadas con el fin de no ganar peso: vmitos, laxantes, diurticos, dietas...

l . Trastornos cualitativos 2 . Trastornos cuantitativos


1.1. Pica o aliotriofagia 2.1 . Anorexia nerviosa
lngesta persistente de sustancias no nutritivas: tierra, tiza, Rechazo a mantener peso corporal en valores mnimos
pintura, escayola, cabellos, cuerdas, trapos, papel. normales (85% del que corresponde a sexo, edad y talla, o
22 veces por semana durante al menos l mes. indice Quetelet (IMC) < 17.5).
Ausencia de otros trastornos psiquitricos (excepto retraso Miedo intenso a ganar peso o convertirse en obeso.
mental, con el que se asocia). ALTERACIN DE LA PERCEPCIN DE LA FORMA O EL
Edad cronolgica y mental 2 2 aos. TAMANO DEL CUERPO.
Amenorrea en muieres postpuberales (puede aparecer cuan
1.2. Mericismo o rumiacin do la prdida de peso es todava poco importante).
Regurgitaciones y masticacin repetitiva de alimento >1 mes Puede no haber prdida de apetito hasta fases avanzadas.
despus de un perodo de funcionamiento normal. ETlOPATOGENIA: Etiologia multifactorial:
No est provocada por una enfermedad somtica (por eiem- Factores genticos:
plo refluio esofgico) ni ha de aparecer en el transcurso de o Antecedentes familiares (trastornos afectivos, tras-
anorexia bulimia. tornos alimentarios, alcohol).
> 3 meses de edad. Factores biolgicos:
Se asocia con retraso mental. 0 Anomalas neurotransmisores cerebrales (5-HT,
neuropptidos Y, YY). PSIQUIATRA

Q
XI. TRASTORNOS DE LA LA CONDUCTA ALIMENTARIA

o
Disfuncin de los eies hipotlamo-hipfiso- Otras caractersticas:
adrenal/gonadal. o Inicio en la pubertad.
Factores psicolgicos: o Procuran comer solas, esconden la comida que no comen,
o Personalidad: hiperresponsables, ecaces, introver- minimizan el hambre y su delgadez (llevan ropas amplias).
tidos, dificultad para establecer relaciones, perfec- o Adems de dietas rigurosas pueden utilizar Iaxantes, diurti
cionistas. cos, anorexgenos o se provocan el vmito.
Factores culturales: o Hiperactividad, tanto en el terreno acadmico, laboral o
o Bsqueda del estereotipo social de xito femenino. fsico.
o Profesiones con necesidad de mantener bajo peso o o Hasta 50% presentan episodios de bulimia.
relacionadas con la esttica: ballet, gimnastas, mo- o Niegan la enfermedad o Ia reconocen con vivencia de inca-
delos. pacidad para superarla.
EPIDEMIOLOGA:
o Afecta fundamental aunque m exclusivamente a muieres
entre los 10 y 30 aos.
o Muieres: 85% de los casos. Varones: 15%.
o Inicio ms frecuente est entre los 12-16 aos.
Prevalencia: O.5%-l% de las adolescentes.

ALTERACIONES PSlCOPATOLGlCAS:
o Alteracin de la percepcin de la imaqen corporal (dismor-
fofobia). Percepcin delirante de su propio cuerpo. Es el sn-
toma ms significativo.

Curso Intensivo MIR Aslur'rasZODB


Visten ropas amplias para ocultar su delgadez

COMPLICACIONES MDICAS:
o Amenorrea: si prdida de peso superior al 15%. Si es supe-
rior a meses puede dar lugar a osteopenia irreversible y
fracturas (si amenorrea secundaria de mas de meses rea
lizar densitometra sea). Es necesaria para el diagnstico.
o Descenso de la actividad simptica y del eie tiroideo (dis-
minucin del gasto energtico total):
Disminucin del metabolismo basal.
Bradicardia (< O lpm).
Disminucin frecuencia respiratoria.
Hipotensin.
OOOOO Hipotermia e intolerancia al fro.
. Alteraciones endocrinas:
La distorsin de Ia imagen corporal es la alteracin psicopatolgica ms o Hipogonadismo hipogonadotrofo por disfuncin hipo-
significativa de anorexia nerviosa. talmica (disminucin de |a FSH y LH con respuesta a
la LHRH exgeno). Niveles baios de estrgenos y de
o Miedo morboso a engordar. gonadotrofinas basales (sobre todo LH).
o Preocupacin persistente por ingesta, peso e imagen. o En varones testosterona disminuida.
o Sntomas de trastorno obsesivo-compulsivo ya sea en rela- O Incremento de niveles de GH.
cin con la comida o no. o Hipofuncin tiroideo (TSH normal, T4 y T3 disminuidos,
T3 inversa aumentada, T4 libre normal).
CRITERlOS DIAGNSTICOS: o Aumento de cortisol por aumento de CRH. EI test de
o Rechazo a mantener un peso corporal igual o por encima supresin con dexametasona puede ser anormal en el
del valor mnimo normal considerando la edad y Ia talla. 50% de los casos.
o Miedo intenso a ganar peso o a convertirse en obeso, inclu- o Alteraciones gastrointestinales:
so estando por debaio del peso normal. Dilatacin gstrica (si hay atracones).
o Alteracin de la percepcin del peso o silueta corporales! Retraso del vaciamiento gstrico.
exageracin de su importancia en la autoevaluacin o nega Sensacin de plenitud postprandial.
cin del peligro que comporta el baio peso corporal. OOOO Estreimiento pertinoz.
o En las muierespostpuberales, presencia de amenorrea. o Otros hallazgos frecuentes:
o Tipos: O Color de piel amarillento (hpercarotinemia).
o Tipo compulsivo purgativo: recurre regularmente a o Sequedad de piel.
atracones o purgas (por e., provocacin del vmito o o Lanugo.
uso excesivo de Iaxantes, diurticos o enemas). O Edemas (sobre todo al recuperar peso o al deiar
o Tipo restrictivo: no recurre regularmente a atracones o Iaxantes y diurticos).
purgas (sino a reduccin de lo ingesta o ejercicio fsi- o Psiquitricas:
co muy intenso). o insomnio,
o Alteraciones del estado de nimo, disforia e irritabili-
dad.
o Conductas suicidas (ms frecuentes en BN).
CURSO INTENSIVO MIR ASTURIAS
- Complicaciones asociadas al vmito: MIR 99 FAMILIA (6106): Los siguientes criterios son tiles en eI
o Trastornos hidroelectrolticos (alcalosis hipoclormica, diagnstico de anorexia nervioso EXCEPTO uno. Sec'IIeIo:
hipopotasemio eI ms peligroso ya que produce i. Rechazo a mantener un peso corporal de al menos el 85%
arritmias-, hipomagnesemia). del que correspondera a su altura y edad.
Irritacin y sangrado gastro-esotgico. 2. Gran temor a ganar peso, incluso teniendo un peso inferior
Ronquera. al normal.
Erosin esmalte dental. 3. Inters porivestir tallas de ropa mas pequeas de las que
OOOO Inflamacin parotIdea e hiperamilasemia a expensas corresponderan a sus caractersticas antropomtricas.*
del isoenzima salivar. Para evaluar gravedad de v- 4. Alteraciones en la percepcin de la imagen corporal propia.
mitos Ia prueba de laboratorio ms til es el nivel 5. Amenorrea en' mujeres postmenrquicas.
plasmtico de amilasa (el nivel de amilasa srica
tambin es til para realizar seguimiento de la reduc- MIR 02 (7410) Muier de 19 aos que consulta por llevar
cin de episodios de vmitos en pacientes que niegan meses sin regla. Se maniesta preocupada por su sobrepeso,
someterse a episodios de purga. ha estado a dieta y ha perdido 5 kg. en 8 meses. Actualmente
O Signo de Russell: callosidad en dorso de mano debido pesa 47 Kg (mide 1,65). EI test de gestacin es negativo. CuI
a provocacin de vmito. es Ia causa ms probable de su amenorrea?:
o Complicaciones asociadas al uso de laxantes: Digenesia gonadal.
o Acidosis metablica. Hipoqonadismo hpoqonadotropo.*
o Complicaciones neurolgicas: Sndrome de ovario poliqustico.
o Disminucin del umbral convulsivo por los trastornos Himen imperforado.
hidroelectroIIticos. 91:59.? Adenoma 'hipotisario.
HALLAZGOS DE LABORATORIO: MIR 04 (7920) Cul de las siguientes alteraciones NQ apare-
. Hemograma: cen en Ia anorexia nerviosa?:
o Leucopenia y anemia normoctica y normocrmica Ie- 'I. Aumento de los niveles de colesterol que no se relaciona
ve. con la ingesta de grasas.
o Pancitopenia en casos graves. 2. Aceleracin del vgciado gstrico.*
o Bioqumica: 3. Disminucin del aclaromiento de creatinina secundario o
o Hiperurcemia (por deshidratacin). >hipovolemia.
o Disminucin de aclaracin de creatinina (secundario 4. Alteraciones del EKG que guardan relacin con las prdidas
a hipovolemia). de potasio. ,
o Hipercolesterolemia. 5. Aumento de niveles de hormona del crecimiento.
o Aumento de pruebas de funcin heptica.
o Alcaloss metablica (en vmitos). MIR 05 (8180): En las pacientes con anorexia nerviosa que
o Acidosis metablica (en abuso de laxantes). llevan mas de un ao con amenorrea es necesario practicar:
o Electrocardiograma: Una densitometrIa sea.*
o Bradicardia sinusal. Una determinacin de niveles sricos de cortisol.
EVOLUCIN Y PRONSTICO: Una determinacin de los niveles de prolactina.
o La mitad recuperan el peso normal. La meiorIa en trminos Un electroencetalograma.
de peso es meior que en trminos de funcin menstrual y es- 9p? Un test de supresin con dexametasona.
tado psicopatolgico.
o 20% meoran pero mantiene peso baio. MIR 'IO (9442): Cul de los siguientes trastornos EQ es necesa-
rio incluirlo en el diagnstico diferencial de los trastornos de
20% se cronitican (5% obesidad).
conducta alimentaria?
o 5-IO% fallecen (desnutricin, arritmias cardacas o suicidio
Trastorno psictico.
mas trecuente en BN-).
Consumo cIe txicos.
o Factores de buen pronstico: Inicio en Ia primera adolescen-
Diabetes mellitus.
cia (IQ-I aos), inicio temprano de tratamiento y buen
Hipotiroidismo.
apoyo familiar.
WPPN. Neoplasias de SNC.
o Factores de mal pronstico: Inicio tardo, conductas purgan-
ANULADA
tes (vmitos, Iaxantes), presencia de episodios bulmicos,
prdida de peso extrema, comorbilidad psiquitrica y curso
MIR 12 (9932): Cul de los Siguientes hallazgos biolgicos
prolongado antes de inicio de tratamiento.
NO es propio en una persona con bulimia nerviosa que pre-
TRATAMIENTO: senta episodios de atracones y vmitos autoinducidos?:
Es un tratamiento a largo plazo: Peso normal.
o Psicolgico (psicoterapia cognitivo-conductual y familiar).
Es el ms importante. Hipertrotia parotI'dea.
o Frmacos antidepresivos: ISRS. Tpicamente tluoxetina. Sobrepeso.
Obietivos del tratamiento: PPPNT Miocardiopata.
o Aumento de peso.
o Desaparicin de alteraciones pscogatolgicas. 2.2. Bulimia nerviosa
0 Aceptacin por parte de la paciente de peso idneo.
Ingesta incontrolada crnica y recurrente de gran cantidad de
o Revertir amenorrea.
alimento.
Criterios de hospitalizacin: o Asociado a conductas compensadoras: induccin
o Desnutricin severa (prdida de ms del 25-30% del peso
del vmito, Iaxantes, aumento del eiercico tsico,
correspondiente IMC < 17).
uso de diurticos, dietas estrictas.
o Graves alteraciones hidroelectrolticas o en signos vitales.
De forma aislada es muy frecuente.
o Sintomatologa depresiva grave y suicidio.
o EI sndrome bulmico es ms frecuente que Ia anorexia.
o Graves alteraciones en dinamica familiar y/o social.
o No hay disminucin de peso ni necesariamente amenorrea,
o Falta de motivacin, tracaso de tratamiento ambulatorio aunque con frecuencia, las pacientes los presentan.
o rechazo de tratamiento.

PSIQUIATRA

a
XI. TRASTORNOS DE LA LA CONDUCTA ALIMENTARIA

COMPLICACIONES MDICAS:
o 50% presentan alteraciones del equilibrio hidroelectroltico
por vmitos y abuso de diurticos y/o Iaxantes (ver apar-
tado AN).
Dilatacin gstrica.
Trastornos endocrinolgicos:
o Niveles baios de glucosa e insulina,
o Elevacin de los cidos grasos libres, cido betahi
droxibutirico y acetoacetato.
Muchas veces amenorrea, el 50% presentan ciclos anovu-
Iatrios.
PSIQUIATRA:
o Los trastornos del estado de animo constituyen la alteracin
psicopatolgica mas frecuente.
o Son muy frecuentes los trastornos de ansiedad.
o Mayor frecuencia que en la poblacin general de consumo
excesivo de alcohol y abuso de sustancias.
o Otras conductas impulsivas (cleptomana, Iudopata, conduc-
tas auto-heteroagresivas, conductas suicidas).
o Trastornos de Ia personalidad: alta incidencia de trastornos
lmite, histrinico, por evitacin y por dependencia. La pre-
sencia de un trastorno lmite de Ia personalidad es un factor
de peor pronsticos
Menos introvertidas que las anorxicas.
o Suelen reconocer su problema (a diferencia de las anorxi-
cas).
0 Tasa de mortalidad menor que en la anorexia nerviosa.
EVOLUCIN Y PRONSTICO:
o A mayor tiempo de seguimiento existe un mayor nmero de
curaciones.
Cursa masivo MIR Asturias o Frecuentemente persisten sndromes parciales que tienden a
ir disminuyendo en su sintomatologa.
EPIDEMIOLOGA: o Factores de buen pronstico: inicio y tratamiento temprano,
Predominio femenino. buen apoyo familiar, pareia estable, clase social alta.
La edad de inicio mas frecuente son los 1518 aos (ligera- o Factores de mal pronstico: desconfianza interpersonal,
mente mas tardo que la AN). trastorno de Ia personalidad u otra comorbilidad psiquitri-
Prevalencia: 2-4% de mujeres venes (prevalencia en varo- ca, vmitos frecuentes.
nes IO veces menor).
TRATAMIENTO:
CRITERIOS DIAGNSTICOS DE BULIMIA: o Psicoterapia y control nutricional. Terapia cognitivo-
Atracones recurrentes: conductual.
o Ingesta del alimento en un corto periodo de tiempo (p.e. Frmucos= Maga.
2 horas)_en cantidad superior a Ia que Ia mayora de las I La mayora de los pacientes con bulimia nerviosa de inicio
'
personas ingeriran en un periodo de tiempo similar y en reciente y sin graves trastornos de la personalidad meioran
las mismas circunstancias. en 2-4 meses con tratamientos centrados en el conseio nu-
o Sensacin de prdida de control de sobre Ia ingesta del tricional y un programa teraputico cognitivo-conductual.
alimento. . Criterios de hospitalizacin:
Conductas compensatorias inapropiadas de manera repeti- o Alteraciones orgnicas graves.
da, con eI fin de no ganar peso (vmitos, Iaxantes, diurticos, o Grave descontrol de impulsos.
enemas u otros frmacos, ayuno y eierccio excesivo). o Conducta suicida.
Los atracones y las conductas compensatorias tiene lugar o Comorbilidad psiquitrica grave.
como promedio al menos dos veces a Ia semana durante un
periodo de tres meses. MIR 02 (7401): Una paciente de 24 aos acude a Ia urgencia
La autoevaluacin esta exageradamente influida por el peso llevada por un familiar por vmitos recidivantes. Padece bulimia
y Ia silueta corporales. nerviosa. Cul de las pruebas de laboratorio es ms til para
La alteracin no aparece exclusivamente en el transcurso de evaluar la gravedad de los vmitosez
Ia anorexia nerviosa. Nvel de Hemoglobina.
Tipos: Nivel glasmtico ale Amilasa*
o Tigo purgativo: durante el episodio de bulimia nerviosa el Nivel srico de Sodio.
individuo se provoca regularmente el vmito o usa Iaxan- Nivel plasmtico de Calcio.
tes, diurticos o enemas en exceso. .UPPNT' Nivel plasmtica de Creatinina.
o Tipo no purgativo: EI individuo utiliza otras conductas
compensatorias. MIR O3 (7654): Lo que basicamente diferencia una bulimia
nerviosa de una anorexia nerviosa es:
Lajrdida de control sobre la comida.
La preocupacin excesiva por la figura y el peso.
EI eiercicio excesivo.
La autoinduccin del vmito.
.UPSNT La utilizacin de Iaxantes.
CURSO INTENSIVO MIR ASTURIAS

AN RESTRlCTIVA AN NO RESTRlCTlVA BULIMIA NERVIOSA

PESO > . MUY BAJo BAJO NORMAL


PREOCUPACION IMAGEN CORPORAL MUCHA MUCHA MUCHA
' ' ' ' 4
IMPULSIVIDAD j No SI MUCHA
INGESTAS BULIMICAs No Sl Sl
CONTROL PESO DIETA / EJERCICIO DIETA / VOMITOS DIETA / VOMITOS
PREDISPOSICION OBESIDAD No SI SI
PSICOPATOLOGIA ++ + ++ ++++

o Ingesra excesiva de lquidos, en general agua.


Suele asociarse a personalidades histricas.
o Exis're riesgo de hiponatremia (con valores de sodio por de-
baio de 120mEq/l aparece clnica de letargia, contracciones
musculares anormales, convulsiones, hosfo coma).
o Diagnstico diferencial con diaberes inspida.
XI. TRASTORNOS DE LA LA CONDUCTA ALIMENTARIA

Los trastornos cualitativos (pica x mericsmo) suelen iniciarse en nios pequeos, se asocian con frecuencia a retraso mental.
Los trastornos cuantitativos aparecen en la adolescencia e inicio de vida adulta.

l . ANOREXIA NERVIOSA

1.1. GENERALIDADES:

Etiopatogena: factores genticos, biolgicos, psicolgicos (hiperrresponsables, eficaces, introverlidos, dificultad para estable-
cer relaciones) y socioculturales (valores estticos dominantes).
Epidemiologa: Ms frecuente en muieres (slo 15% de los afectados son varones). Inicio en adolescencia (12-16 aos). Afecta
fundamentalmente a muieres entre 10 y 30 aos.
Psicopatologa (fundamental):
o Preocupacin persistente por ingesta, peso, imagen corporal (MIR).
o Miedo morboso a engordar.
o Alteraciones en la percepcin de la imagen corporal.

1.2. CRITERIOS DIAGNSTICOS:

I. Rechazo a mantener un peso corporal de al menos el 85% del que correspondera a sexo, edad y talla (MIR), o ndice de
Quetelet (Indice de Masa Corporal) < 17.5 (MIR).
2. Miedo intenso a ganar eso o convertirse en obeso (MIR).
3. Alteracin de la percepcin del peso o silueta corporales (MIR).
4 En las muieres postpuberales la presencia de amenorrea (que puede aparecer cuando la prdida de peso es todava poco
importante) (MIR).
o Para perder peso estas muieres tienen desde el inicio del cuadro hiperactividad fsica (MIR), pueden usar laxantes, diurti-
cos, anorexgenos, o se provocan el vmito (MIR). Minimizan el hambre y la delgadez (llevan ropas amplias) (MIR). Nie-
gan la enfermedad (MIR).
o La distorsin en la imagen corporal es el sntoma ms significativo (MIR). Percepcin delirante del propio cuerpo.

1.3. COMPLICACIONES:
Amenorrea (si dura ms de meses puede dar lugar a osteopenia irreversible y fracturas). Si amenorrea secundaria de ms
de meses de evolucin realizar densitometra sea (MIR).
Presentan trastornos hormonales: Hipogonadismo hipogonadotropo (MIR) por disfuncin hipotalmica, incremento de niveles
de GH (MIR), hipofuncin tiroidea (TSH normal, T4 y T3 disminuidas, T3 inversa aumentada, T4 libre normal).
Alteraciones bioqumicas: renales (hiperuricemia, disminucin del aclaramiento de creatinina secundario a hipovolemia
(MIR)), hipercolesterolemia (MIR) y aumento de las pruebas de funcin heptica.
Gastrointestinales: dilatacin gstrica (si atracones), retraso del vaciamiento gstrico (MIR), sensacin de plenitud postpran-
dal, estreimiento pertinaz.
Pancitopenia (en AN severa).
Lanugo.
Psiquitrico: insomnio, alteraciones del estado de nmo, conductas suicidas (ms frecuentes en BN).
m
o Trastornos hidroelectrolticos (alcaloss hipoclormica, hipopotasemia -e| mas peligroso ya que produce arritmias (MIR)
hipomagnesemia).
Irritacin y sangrado gastro-esofgico.
Ronquera.
Erosin esmalte dental.
OOOO Inflamacin parotdea e hiperamilasemia. Para evaluar la gravedad de los vmitos la prueba de laboratorio ms til es
el nivel plasmtico de amilasa (MIR).
o Signo de Russell (callosidad dorso de la mano).

1.4. PRONSTICO:

50% recuperan su peso normal. 20% meioran pero mantienen baio peso, 20% se cronican. 540% fallecimiento (complica-
ciones mdicas o suicidio).
Factores de mal pronstico: inicio tardo, conductas purgantes, presencia de episodios bulmicos, prdida de peso extrema,
depresin y curso prolongado antes del inicio del tratamiento.

1.5. TRATAMIENTO:

Meior pronstico cuanto antes se inicie el tratamiento. Es sobre todo psicolgico: Psicoterapia cognitivo-conductual y familiar,
antidepresivos (ISRS). Son tratamientos a largo plazo.
Obietivos del tratamiento: aumentar peso, revertir la amenorrea, desaparicin de las alteraciones gsicopatolgigas, acepta-
cin por parte del paciente del peso idneo.
Hospitalizacin: Desnutricin severa (prdida de ms del 25-30% del peso correspondiente), graves alteraciones hidroelectro-
lticas o en signos vitales, grave sintomatologa depresivo asociada o riesgo de suicidio.

ws-,44
M111
CURSO INTENSIVO MIR ASTURIAS

2. BULIMIA NERVIOSA

2.1. GENERALIDADES:

Etiopatogenia: factores genticos, biolgicos (baios niveles de serotonina), psicolgicos y socioculturales (valores estticos do-
minantes).
Epidemiologa: Mas frecuente en muieres. Comienzo en inicio de edad adulta (15-18 aos). Ms frecuente que la AN.

2.2. CRITERIOS DIAGNSTICOS:

Preocupacin persistente por comida y deseos irresistibles de comer.


Crisis bulimicas o atracones: episodios de ingesta "incontrolada" de gran cantidad de alimentos (sentimiento de culpa poste-
rior), que van seguidos de: provocacin de vmitos, uso de laxantes o diurticos, dietas estrictas, mucho eiercicio.
Mucha preocupacin por la imagen corporal y miedo morboso a ganar peso.
Para hacer el diagnstico es necesario que los atracones y las conductas compensadoras (exagerado eiercicio fsico, uso de
laxantes o diurticos) se produzcan al menos 2 veces por semana durante 3 meses.
Diferencias basicas con AN: prdida de control sobre comida (MIR) y peso corporal (en BN normal).

2.3. COMPLICACIONES:

Gastrointestinales: dilatacin gstrica.


Vmitos: las mismas que en el apartado de AN.
Psiquitricas (comorbilidad): insomnio, trastornos afectivos (depresin), trastornos de ansiedad, conductas impulsivas (abuso
de alcohol y otras sustancias, cleptomana, ludopata, conductas auto-heteroagresivas), mayor incidencia de trastorno de la
personalidad limite, histrinico, por dependencia o por evitacin:

2.4. PRONSTICO:
Factores de mal pronstico: desconfianza interpersonal, trastornos de la personalidad u otra comorbilidad psiquitrica, vmi-
tos frecuentes.

2.5. TRATAMIENTO:

Antidepresivos ( ISRS de eleccin) + psicoterapia cognitivo-conductual.


Hospitalizacin: alteraciones orgnicas severas, grave descontrol de impulsos y conductas suicidas, comorbilidad psiquitrica.
REPASO RELAClONAL

RE PASO RE LACIONAL

Es imposible memorizar sin esfuerzo por parte del opositor"

'
'LEsymS'=eCUente.;. . en
i. Eooidemioloa El inicio de la esquizofrenia
Varones: 15-25 aos
Muieres: 25-35 aos
San mstrcetes'en..VARNES-.;. . 7 _' ._| El inicio de Ia esquizofrenia Despus de los 45 aos
Abuso de sustancias tarda
Suicidio consumado El inicio brusco del Trastorno Torno a los 20 aos
Apnea del sueo esquizoafectivo
Trastorno de la personalidad paranoide, antisocial, obsesivo- Aumenta con la edad, 20% en
. . . Alz heimer
compUlStVO y naraSIsta mayores de 80anos
-

Edad avanzada, muieres,


y
__'1. ', El insomnio estatus socioeconmico baio,
V-S'onms ,fnecuentes en. lili-MUJERES .. .
presencia de t psiquitricos
Trastornos de ansiedad
Tentativa suicida Muieres y adolescentes
Trastornos somatomorfos
Trastornos disociatvos
Mayores de 55 aos (sobre
todo 65); vivir solos; aisla-
Depresin (excepto involutiva > en varones)
miento social; trastornos men-
Trastorno de ideas delirantes
tales: trastornos afectivos,
TCA: anorexia nerviosa y bulimia dependencia alcohol, esquizo-
Insomnio Suicidio
frenia (meses posteriores al
Intento de suicidio alta hospitalaria); enfermedad
Trastorno de la personalidad lmite, histrinico y por dependen- fsica crnica, dolorosa, inca-
cia pacitante; tentativas suicidas
previas
eni .
Hiperresponsables, perfeccio-
nistas, eficaces, introvertidas,
Trastorno bi r
Anorexia nerviosa dificultades para relaciones
Trastorno obsesivo-com lsivo
interpersonales, rasgos obse
Hi ra
sivos y/o tbicos.
Trastorno dismrtico co al Suele asociarse con mayor
uizofrenia frecuencia de consumo de
alcohol y/o drogas y con otras
Bulimia nerviosa
. . (Y ,Essmstrecuenteug 1: y r . 4- I-' conductas impulsivas: clepto-
J;
. .en. -, ue.
Terrores nocturnos Nios 5-12 aos mana, ludopata, conductas
auto-heteroagresivas
La evolucin a drogadiccin,
personalidad antisocial y alco- Nios hiperquinticos
holismo
Alta letalidad Sd. neurolptico maligno
Depresin mayor unipolar Edades entre 30-40 aos

El inicio de trastorno bipolar Varones: 15-25 aos


Mujeres: 25-35 aos

41.49
MIR
CURSO INTENSIVO MIR ASTURIAS

Lo ms frecuente de es... Se ha: encontrado asociacin y; g.


Trastorno de ansiedad Fobia simple entre
Trastorno de ansiedad que Trastorno de pnico Fase l y ll del sueo Bruxismo
buscan tratamiento Fase lll y lV del sueo Sonambulismo y terrores noc-
Trastorno obsesivo- Rituales de limpieza y compro- turnos
compulsivo bacin Pesadillas y Sueos. Fenmenos
Trastorno disociativo Amnesia psicgena Sueo REM vegetativos y atona muscular.
Suicidio en trastorno afectivo Fase depresiva de trastorno Arritmias
bipolar Onicofagia Neurticos
Insomnio sin repercusiones Mana Pica Trastornos psicticos
negativas sobre la vigila Retraso mental Autismo, mericismo y pica
Hospitalizacin en psiquiatra Riesgo auto y/o heteroagresivo - Tartamudez Anorexia, terrores nocturnos y
(Esquizofrenia, Trastornos de la enuresis
personalidad, depresin grave) Neurosis Focfores psicosociales
Alucinacin auditiva Esquizofrenia Psicosis Factores biolgicos
Alucinacin visiual Dao cerebral orgnico (Deli- Represin Trastorno de conversin
rium, delirium tremens) Desplazamiento Fobia
Demenca Alzheimer Proyeccin T de ideas delirantes, t paranoi-
Delirium Enfermedades del SNC, sist- de de la personalidad
micas mefab|icas, infeccio- Anulacin y formacin reac- Obsesivos
sas-, y txicos tiva
Cuadro orgnico psquico Delirium Educacin rgida Obsesivos
reversible Trastornos afectivos, alcoholis-
Cuadros de privacin de sus- mo, trastorno obsesivo-
Delirium tremens tancias (alcohol), rara vez en Disminucin de latencia REM compulsivo, interrupcin del
intoxicaciones agudas consumo de anfetaminas y
Wernicke-Korsakov Dficit de Tiamina (B 'l) antidepresivos tricclicos
Trastorno psquico en las Pelagra No supresin del Test de la Trastornos afectivos
enfermedades carenciales Dexametasona
Adiccin por opiceos Herona Disminucin a estimulacin Trastornos afectivos
Celotipia Dependencia alcohlica con CRF
Demenca por toxicomana Alcohol Hipersecrecin de Prolactina Consumo crnico de cocana y
Muerte por sobredosis con Afectacin cardiovascular, tto con antipsicticos tpicos
antidepresivos tricclicos arritmias malignas Disminucin de neurotrans- Trastornos afectivos unipolares
Intoxicacin por Litio Dietas sosas, hiponatremia. misores (excepto la acetilco-
lina que esta elevada)
Aumento de noradrenalina y Trastorno bipolar
dopamina, disminucin de
2. Etio Hato enia acetilcolina
Pcnico Trastorno bipolar
Se ha involucrado a la herencia en la etiopatogenia de...
Oligofrenia Leptosomtico Esquizofrnico
Dislexia Atltico Trastorno depresivo
Sonambulismo Trastorno de personalidad Esquizofrenia
esquizoide
Trastorno de pnico
Hiperactividad dopaminrgi- Esquizofrenia
Trastorno obsesivo-compulsivo
ca
Trastornos afectivos (sobretodo bipolar)
Hipoperfusin en regin Esquizofrenia
Suicidio
prefrontal en SPECT
Trastorno esquizoafectivo
Atrofia cortical y dilatacin Esquizofrenia, trastornos afecti-
Esquizofrenia
ventricular en TAC vos (bipolar | y depresin mayor
Anorexia nerviosa y bulimia nerviosa psictica)
Alcoholismo Disminucin de la actividad Esquizofrenia
Personalidad antisocial y narcisista a en el EEG
Alzheimer (APOE4)
Disminucin colinrgica, Alzheimer
Se ha involucrado la disminucin de los niveles de serotonina somatostatina , noradrenali-
en la etiopatogenia de... na y serotonina
Ansiedad Epilepsia del lbulo temporal Sintomatologa psiquitrica
y tumores frontales
Crisis de panico
Depresin Factores socioculturales AN y BN
Trastornos alimentarios
Aumento de l7-OH- Suicidio
esteroides en orina
Conductas adictivas
Disminucin de 5-HllA en Suicidio
Alzheimer
l.c.r (l serotonina)
Autismo
Menor nmero de receptores Suicidio
Trastorno del control de impulsos (trastorno explosivo intermi
imipramnicos plaquetarios
tente, cleptomana, piromania, ludopata, tricotilomana)
(lr serotonina)
Suicidio
Alteracin del control de Suicidio
impulsos (lv serotonina)
REPASO RELACIONAL

Es Caracterstica de
3. Fisio catolo . a Depresin involutiva Agitacin, ideas delirantes de
culpa / ruina, alto riesgo de
La de suicidio
A las 4 horas Hipomana en primavera y vera-
Herona-Morfina Alas 6-8 horas Trastorno afectivo estacional no. Depresin moderada en
Metadona A los 2 das invierno
Alcohol De 4-12 horas. Elevada ansiedad, sntomas
Depresiones atpicas fbicos, hiperfagia con aumento
de la ingesta de HC, hipersom-
4. Anatoma catolgica nia
Superposicin de un episodio
Depresin doble depresivo mayor a una distimia
La lesin caracterstica de es...
Depresin anoncstico lmpregnacin de ideas obsesivos
Demencia cortical A nivel del crtex de asocia-
cin
Deterioro del nivel de concien-
cia, desorientacin. Alucinacio-
Demencia subcortical A nivel de los ganglios basa-
Sndromes organicos cere- nes visuales. Agitacin
les
brales Empeoromiento vespertino
Enfermedad de Alzheimer Atrofia cortical frontal y tem-
Inicio agudo y remisin al des-
poral, ovillos neurofibrilares
aparecer la causa.
de Alzheimer, placas seniles y
Black out o palimpesto No recuerda lo que hizo mien-
deposito de amloide
tras beba a pesar de no tener
Variante humana de le encefa- Placas de amloide rodeadas
sntomas de intoxicacin.
lopata espongiforme bobina de espongiosis "placas flori-
Nariz roiiza, anillo corneal senil,
dos"
teleangiectasias, hipertrofia
parotdea, eritema palmar, en-
Alcoholismo veiecmiento precoz, hepatome-
gola, atrofia testicular. Cirrosis
heptica, miocardiopata y neu-
Es caracterstico de ropata.
Mana Disminucin de la necesidad de Embrioguez patolgica Agitacin y violencia con consu-
dormir, fuga de ideas, pensa- mos boios de alcohol en no
miento proliio, verborrea, dis- dependiente
troibilidad, aumento de actividad Sndrome omotivacionol Cualquier sustancia que causa
intencionada dependencia
Depresin melanclico Delirio de negacin del pr0pio LSD Flash-back
cuerpo (Cotard) Opiceos Tolerancia
Sntomas positivos: Delirios, Sobredosis de herona Hipoactividad simpatica, miosis
Esquizofrenia alucinaciones, comportamiento y puntiforme, depresin respirato-
pensamiento desorganizado ria, coma
Sntomas negativos: Alogia, Sndrome de abstinencia Opiceos, alcohol, barbitricos,
abulia, apata, astenio, an- fsico BZD
hedono
Esquizofrenia Catatnica Sntomas psicomotores graves
Esquizofrenia Hebefrnica Desorganizacin del pensamien- No hay que confundir con...
to, lenguaie y comportamiento, y Neurosis: uicio de realidad Psicosis: el contacto con Ia
alteracin de Io afectividad conservado realidad esta perdido
inadecuada y superficial. Depresin con melancola: Distimia: empeoramiento
Esquizofrenia Paranoide Delirios -outorrefencia|es y de empeoramiento matutino, vespertino, insomnio de conci-
persecucin-, alucinaciones despertar precoz, anorexia, liacin, irritabilidad, ansiedad
auditivos prdida de peso, disminucin
Esquizofrenia simple Sntomas negativos sin antece- de la libido
dentes de sntomaSJoositivos Pseudodemencia depresivo: no Demencia: empeoramient
Trastornos psicticos org- Alucinaciones visuales variacin diurna de los snto- vespertino de los sntomas,
nicos mas, queias sobre los fallos NO queas sobre los fallos
Depresin melanclico Nihilismo cog nitivos cognitivos
M y M Dismorlofobia Demencia: deterioro global Delirium: deterioro global con
con conciencia normal disminucin del nivel de con-
Delirium Tremens Microzoopsias, alucinaciones
ciencia
inducidas, delirio ocupacional,
convulsiones (51 5%)
Demencia corticol: prdida de Demencia subconical: Bra-
Cocainmanos Alucinaciones tctiles (insectos
las funciones superiores: abs- dipsiqua. Afectacn precoz
baio la piel) traccin, orientacin. Mol ren- del lenguaie. Meior rendi-
Trastorno de la Personali- Rigidez, perfeccionismo dimiento en actividades intelec- miento de actividades intelec-
dad obsesivo
tuales tuales.
TP lmite Impulsividad y/o inestabilidad
Miosis: contraccin pupilar. Midriasis: dilatacin pupilar.
emocional
Sobredosis de opiceos, intoxi- Intoxicacin por cocana,
TP histrinico Excesiva emotividad y bsqueda cacin por alcohol, benzodia- anfetaminas y derivados,
de atencin cepinas, fenotiacinas, organo- alucingenos, anticolinrgi-
Consumo de cocaina y Psicosis poranoide fosforados. Lesin en protube- cos. Abstinencia a opiceos.
anfetaminas crnico rancia. En la primera fase de Segunda fase de anoxia cere-
Depresiones infanto- Equivalentes depresivos (rabie-
uveniles tas, dificultad escolar) L anoxia cerebral bral

Depresin en la senectud inquietud, queias somticas y


tintes poranoides
CURSO INTENSIVO MIR ASTURIAS

No hay que confundir con... Duracin mnima de los Para diagnosticar...... .


Despersonalizacin: el sentido Desrealizacin: se pierde el sntomas ' '
de la propia realidad se en- sentido de la realidad del Depresivo mayor 2 semanas
cuentra cambiado mundo exterior Episodio manaco i semana
Catapleia: prdida sbita de Catalepsa: presencia durante Episodio mixto l semana
tono muscular y de movimien- largo tiempo en posiciones o Episodio hipomanaco 4 das
tos voluntarios con manteni- actitudes fiias (Esquizofrenia Distimia Adulto: 2 aos
miento de la conciencia (Nar- catatnica). Movimientos Nio / adolescente: 1 ao
colepsia) voluntarios abolidos Cclomia Adulto: 2 aos
T de ideas delirantes: forma Esquizofrenia paranoide: Nio / adolescente: 1 ao
progresiva e insidiosa sin rom- inicio ms agudo con deses- Esquizofrenia 6 meses (fase activa de snto-
per con la personalidad previa. tructuracin del Yo: Ruptura mas caractersticos l mes)
Delirio, interpretativo, bien biogrfico: Delirio mal siste- T de ideas delirantes DSM-lVl mes
sistematizado y con lgica. No matizado e incomprensible. CIE-10 3 meses
alucinaciones (o muy poco Alucinaciones auditivos.
Tr esquizofreniforme Entre i y meses
relevantes)
Tr psictico agudo y breve Entre ida y i mes
Trastorno facticio: sntomas Simulacin: sntomas volunta-
voluntarios con fin involuntario. rios con fin voluntario.
S. de Briquet o trastorno por Trastorno de conversin:
somatizacin: mltiples snto- alterada una funcin fsica . Metodos complementarios de
mas fsicos (4 de dolor, 2 di- (psuedoneurolgico), sin con-
gestivos, l sexual y i pseudo- trol voluntario. El sntoma dia . nstico
neurolgico), no voluntarios fsico parece ser una expre-
que aparecen antes de los 30 sin de un conflicto psquico. 6.1 . Pruebas psicolgicas
aos , con un curso crnico. Personalidad histricas y nar-
cisistas. Retraso mental Coeficiente Intelectual
Estrs agudo (CIE-10): inicio Estrs postraumtco (CIE-10): Lmite 70-84
tras exposicin a agente estre- perodo de latencia en el Leve 50-69
sante masivo y desaparece en inicio (semanas / meses, ge- Media 35-49
2-3 das. neralmente en los primeros 6 Grave 20-34
meses) y dura ms de 1 mes. Profundo < 20

6.2. Analtica

.
La enfermedad/ El Sndrome consuste en.
de
Desintegrado el pensamiento Aumento: GGT, VCM, CDT,
abstracto; amnesia inicialmen- CPK, tiempo de protrombina,
Alzheimer te antergrada, afasia, Sospechar alcoholismo ante... ac. rico, colesterol y triglic-
apraxia, agnosia ridos
Alteracin precoz de las fun- Hepatitis alcohlica GOT/GPT > 2
ciones relacionadas con el Litemia > 1.5 mEq/l. Clnica:
Pick lbulo frontal: Descontrol de Sospechar intoxicacin por ataxa, disartria, temblor gro-
impulsos, agresividad, alt del Litio ante sero, nistagmo, letargia o
uicio, desinhibicin, mutismo, excitacin
hiperfagia, hipersomnia, h-
persexualidad. Afectacin ms
tarda de la memoria. 7. Tratamiento
Creutzfeldt-Jakob Demencia progresiva, rpida
+ mioclonas El tratamiento de... es.;.
Wernicke (agudo): confusin, Ansiedad generalizada ISRS +/- BZD / buspirona /
Wernicke-Korsakov oftalmopleia y nistagmo, nuevos antiepilpticos + Psi-
ataxa coterapia.
Korsakov (residual): amnesia ISRS (1) o clomipramina (2)
antergrada y de fiiacin, Trastorno obsesivo-compulsivo Si fracaso farmacolgico:
confabulacn psicociruga (capsulotoma
Ausencia de reciprocidad bilateral anterior o cingulo-
social y respuesta emocional. toma)
Kanner (Autismo infantil) Gran alteracin de la comuni- Crisis: BZD de alta potencia,
cacin. Frecuente retraso
Trastorno de pnico Tratamiento de fondo: SRS
mental +/- BZD
Trastorno facticio: produccin Deshabituacin a alcohol Interdctores (disulfirn, cia-
intencionada de sntomas en namda) / anticraving (nal-
Mnchausen el paciente o en persona de trexona, acamprosato)
corta edad relacionada con el Fobia social ISRS + Psicoterapia
paciente (Mnchausen por
Briquet Psicoterapia
poderes)
Dolor psicggno Antide presivos
Prisioneros: respuestas extra-
Esquizofrenia y Trastorno de Antipsicticos
vagantes y conductas infanti-
ideas delirantes
Ganser les. Desvaro extremo. Dia-
gnstico diferencial con otros Terapia Electroconvulsiva
trastornos disociativos y tras- Esquizofrenia catatnica (TEC)
torno de conversin Episodio depresivo ISRS (1) + Psicoterapia
Hipersomna + apnea del Cualquier cuadro que curse Antipsicticos y/o Benzodia-
Pickwick sueo. Ms frecuente el perif- con agitacin psicomotriz y cepinas
rco sntomas psicticos
PSIQUIATRA
n,
Q
REPASO RELACIONAL
J

J
'Eltt'ratamiento, de 2353-; 7' El mecanismo de accin cle es
Depresin mayor con melan- 7 ,
cola y/o delirante con riesgo TEC IMAOs Inhibir Ia accin de la Monoami-
autoltico y no respuesta a nooxidasa
psicofrmacos Antipsicticos tpicos Bloqueo dopaminrgico D2
Mana aguda Antip5cticos atpicos / litio / Antipsicticos atpicos Bloqueo del os receptores D2 y
valproato. 5-HT2
EP5Cl mXO Carbamacepina / VOIPFOO'O Metadona Agonista puro de los receptores
Ciclodores rpidos Litio / vaIproato / Iamotrgina opceos
Delirium Haloperidol, Risperidona Naloxona y Naltrexona Antagonsta puro de los recept0
Alteraciones del comporta- Antipsicticos res opiceos
miento en Demencia
Intoxicacin etlica aguda Tiamna
Delirium tremens y sndrome BZD / clormetiozol / ticpride
de abstinencia
lntoxicacieor Cannabis Sintomtico (BZD) _ . ' . .
Intoxicacin por BZD Flumacenil Ansuoltlca, hipntica, Irelaiante
Deshabituacin cocanica Antidepresivos (imipramina, BZD mUSCUlGr' onl'C"VUls'V"le Y
ISRS) / agonistas DA... _ , _ sedante _ _ _
Trastorno por dficit de aten- Estimulantes del SNC (metilfe- Clorimiprommo Anl'deplfes'vo onl"bses'v
cin Y/O hiperactividad LItlo Estabilizante del humor
nidato) . . .
Sobredosis de opiceos Naloxona Clonidma y Guanfancma Agonista presinptico (1-2 central

- Sintomtico: BZD, antipsic-


ticos, analgsicos, antidiarri- ,El'efecto/s secundario/s de es / son
Desntoxicacin de herona cos, antiemticos, agonistas BZD Sedacin, amnesia anter-
oz-2. grada, dependencia, tole-
- Sustitutivo: metadona en
rancia
pauta decreciente, -El ms frecuente: efectos
Deshabituacin del consumo - Naltrexona, previa prueba
extrapiramidales (distonia,
de herona con naloxona parknsonismo, acatsia),
Trastorno afectivo estacional Estimulacin luminosa hiperprolactinemia.
Insomnio de conciliacin BZD de vida media corta Antipsicticos tpicos -Menos frecuente: discinesia
tarda.
Raro: Sindrome neurolpti-
co maligno.
Insomnio de mantenimiento o BZD de vida media intermedia Antipsicticos atpicos - aumento de peso, dislipi-
despertar precoz demias, alteracin del meta-
Ideacin suicida grave Antipsicticos sedantes bolismo de la glucosa: sdr
Hidrocefala normotensiva Vlvula de derivacin ventr- metablico
culo-peritoneal Sntomatologa anticolinrgi-
Bulimia nerviosa ISRS + Psicoterapia ca (estreimiento, sequedad
Aumento de peso + Psicote- de boca, retencin de orina,
Anorexia nerviosa rapia +/- ATD. Casos graves: visin borrosa)
hospitalizacin. Antidepresivos cclicos Alt en el EKG : alargamiento
Enuresis Tratamiento conductual. Anti- ST, aplanamiento T ensan-
depresivos tricclicos chamiento del QRS
Guilles de la Tourette Haloperidol Hipotensin OFOSfCG. .
Psicosis hipocondriaca somti- Pimozide WLCD. Arritmias
ca moligas
Distona aguda por neurolpti- Bipirideno i.m. IMAOS CFIS' hmfirlenSIVGS . '
cos ISRS Al gastromtestinales. POSlbl-
lidad de acatsia
v , .v -; .. , . . 7 ,i T-E-C Amnesia transitoria
Eltrata en39pr9fi.lctlc_ode'
lCqL. .u'rreniigsde ._ l Litio Hpotiroidismo,
nspida nefrognca, au-
diabetes
Esquizofrenia Antipsicticos mento de peso, temblor,
Depresin Mantener el mismo antidepresi- diarreas, acn.
vo
Trastorno bipolar Litio
; :Interdccionanwconav..., a U rel./ los
Depresores del S.N.C: alcohol,
BZD, Antipsicticos, ATC e barbitricos, anestsicos, narc-
IMAOs ticos y antihstamnicos.
Benzodiacepinas Potenciar la actividad del GABA Antihipertensivos, anticoncepti-
una vez que ste se ha fiado a Antidepresivos Tricclicos vos, anticoagulantes, antiarrt-
su receptor (ATC) micos, anticolinrgicos, IMAOs
Antidepresivos cclicos Inhibicin no selectiva de la IMAOs ISRS y ATC; alimentos ricos en
recaptacin de NA y serotonina tiramina
Inhibidores selectivos de Ia Inhibicin selectiva de la recap-
recaptacin de serotonina tacin de serotonina
Inhibidores de la recapta- Inhibicin de la recaptacin de
cin de serotonina y nora- NA y serotonina,
drenalina
CURSO INTENSIVO MIR ASTURIAS

Indice Temtico
A Distonia aguda, 18

Acatisia, 18 E Obsesiones, l 14
Agonista, 72 Opiceos, 72
Agorafobia, 110 Encefalopata de Wernicke, 81
Alcoholismo crnico, 79 Enfermedad de Creutzfeldt-Jakob, 40
Alogia, l l Enfermedad de Marchiafava-Bignami,
Alteraciones de la percepcin, 10 80 Palimpesto, 79
Alteraciones del pensamiento, 1 1, 12 Episodio depresivo agudo, 105 Paralogia, ll
Alucinacin, l l Episodio hipomaniaco, 99 Paranoia, 52
Alucinosis, ll Episodio maniaco, 98 Parkinsonismo, 18
Alucinosis alcohlica, 83 Episodio mixto, 99 Personalidad mltiple, 121
Amnesia disociativo, 120 Espectro obsesivo-compulsivo, 1 14 Personalidad psicastnica de Janet,
Anfetaminas, 76 Esquizofrenia, 17, 49 1 15
Anorexia nerviosa, 125 Esquizofrenia catatnica, 51 Pica o aliotriofagia, 125
Ansiedad normal, llO Esquizofrenia hebefrnca o desorgani- Potomania, 129
Ansiolticos, 15 zada, 51 Propanolol, 16
Antagonista, 72 Esquizofrenia simple, 51 Pseudoalucinacin, ll
Antidepresivos, 21 Estrs agudo, l 15 Pseudodemencia depresiva, 95
Antidepresyos tricclicos, 21 Estrs postraumtico, l 16 Psicoanlisis, 12
Antipsicticos, i, 34 Psicosis de Korsakov, 82
Aripiprazol, 16 F Psicosis, lO
Ataque de panico, 112
Autismo infantil, 45 Fobia especfica (fobia simple aisla- R
da), l l l
B Fobia social, l l 1 Reaccin de duelo, 96
Fobias, l lO Reinsercin de las drogodependencias,
Barbitricos, 16 Forma clnicas de esquizofrenia 72
Benzodiazepinas, 15 Indiferencada, 51 Retraso mental, 44
Black out, 79 Paranoide, 50
Bulimia nerviosa, 127 Residual, 5'l S
Buspirona, i Fuga disociativo, 120
Sndrome amnsico orgnico, 35
H
Sndrome alcohlico fetal, 81
Sndrome confusional agudo, 34
Cannabis, 76 Haloperidol, l Sndrome de abstinencia, 71
Ccladores rapidos, 101 Hipocondria, 119 Sndrome de abstinencia agudo a
Clozapina, i, 17 Hipofuncin noradrenrgica, 94 opiceos, 73
Cocana, 74 Hipogonadismo hipogonadotrofo por Sndrome de abstinencia en el recin
Compulsiones, l 14 disfuncin hipotalmica, 126 nacido, 74
Crisis de angustia, H2 Sndrome de Charles Bonnet, 54
Criterios diagnsticos de bulimia, 128 Sndrome de malabsorcin, 80
Sndrome de Mallory- Weiss, 80
D I.S.RS., 21 Sndrome de Mnchausen, l 18
Ideas obsesivas, 1 1 Sndrome neurolptico maligno, 19
Delirio de los trastornos organicos, 1 i Intoxicacin aguda, 72 Sueo normal, 66
Por alcohol, 78
De'lirium tremens, 83 Suicidio, 102
Por cannabis, 77
Demencia por cuerpos de Lewy, 36, 38 Por cocana, 75
Demencia vascular, 38 Intoxicacin alcohlica idiosincrtca o
Demencias, 36 borrachera patolgica, 78
Dependencia, 71 IRSN, 22 TDAH, 46
Depresin anacltica de Spitz, 95 Terapia electroconvulsiva (TEC), 27
Depresin anancstica, 95 J Test de estimulacin de TSH por TRH,
Depresin atpica, 95 95
Depresin doble, 97 Juicio de realidad alterada, lO Tipos de fobias, llO
Depresin enmascarado, 95 Juicio de realidad conservado, 10 Tolerancia, 7l
Depresin involutiva, 95 Trastorno psictico compartido, 54
Depresin puerperal, 98 L Trastorno afectivo estacional, 95
Depresin reactiva, 95 Trastorno antisocial, 59
Depresiones resistentes, 97
Litio, 24 Trastorno bipolar, 98, 99
Deshabituacin alcohlica, 84 Trastorno de adaptacin, l 16
Deshabituacin a la cocaina, 75 M Trastorno de ansiedad, 1 1 7
Deshabituacin de las drogodepen- Trastorno de ansiedad generalizada,
dencias, 72 Maternity blues, 98 l 13
Deshabituacin de opceos, 74 Mericismo o rumiacin, 125 Trastorno de conversin, l 19
Desintoxicacin alcohlica, 84 Mielinolisis central pontna, 80 Trastorno de identidad disociativo, l2l
Desintoxicacin de las drogodepen- Trastorno de inestabilidad emocional,
dencias, 71
N 59
Desintoxicacin de opiceos, 73 Trastorno de pnico, l l 3
Discinesia tarda, 18 Neurosis, 10 Trastorno de somatizacn, 1 18
Dismorfofobia, H8 Trastorno depresivo, l
Disomnias, 67 Trastorno depresivo breve recurrente,
Distimia, 92, 97 95 PSlQUlATRA

a
NDICE TEMTICO

Trastorno depresivo recurrente, 93


Trastorno esquizode, 58
Trastorno esquizotpico, 58
Trastorno tactico, 1 18
Trastorno histrinico, 60
Trastorno narcisista, 60
Trastorno obsesivo-compulsivo
(anancstico), 61
Trastorno paranoide, 58
Trastorno pasivo-agresivo, 61
Trastorno por dependencia, 61
Trastorno por despersonalzacin, 121
Trastorno por dolor, 1 18
Trastorno por evitacin (ansioso), 61
Trastornos de ansiedad, 1 10
Trastornos depresivos, 92
Trastornos disociativos, 1 17, 120
Trastornos obsesivoides, 1 15
Trastornos psicosomticos, 1 18
Trastornos psiquitricos inducidos por
sustancias, 77
Trastornos somatomorfos, 1 17, T 18
Tratamiento de las drogodependen-
cias, 71

Potrebbero piacerti anche